mirror of
https://github.com/MartinThoma/LaTeX-examples.git
synced 2025-04-11 15:38:15 +02:00
Remove trailing spaces
The commands find . -type f -name '*.md' -exec sed --in-place 's/[[:space:]]\+$//' {} \+ and find . -type f -name '*.tex' -exec sed --in-place 's/[[:space:]]\+$//' {} \+ were used to do so.
This commit is contained in:
parent
c578b25d2f
commit
7740f0147f
538 changed files with 3496 additions and 3496 deletions
|
@ -10,6 +10,7 @@
|
|||
this should include the rendered image. For documents, there should be
|
||||
some example text explaining what it is about.
|
||||
3. Use spaces for indenting.
|
||||
4. Don't have trailing spaces.
|
||||
|
||||
|
||||
## Commit messages
|
||||
|
|
|
@ -5,7 +5,7 @@ Compiled example
|
|||
Currently, this one is not working, because of this error:
|
||||
|
||||
```
|
||||
/usr/local/texlive/2013/texmf-dist/asymptote/three.asy:
|
||||
/usr/local/texlive/2013/texmf-dist/asymptote/three.asy:
|
||||
2906.13: runtime: to support onscreen rendering, please install glut library, run ./configure, and recompile
|
||||
```
|
||||
|
||||
|
|
|
@ -1,7 +1,7 @@
|
|||
I took this from a [StackExchange Question](http://tex.stackexchange.com/a/84757/5645)
|
||||
|
||||
If you want to change it (I know you can make it better with LaTeX!)
|
||||
or if you have some important commant, feel free to submit a merge
|
||||
or if you have some important commant, feel free to submit a merge
|
||||
request or send me an email (info@martin-thoma.de)
|
||||
|
||||
|
||||
|
|
|
@ -33,7 +33,7 @@
|
|||
\item Basic path: \myCode{\textbackslash drawing-command [options] path-specification;}
|
||||
\item Path specification: \myCode{(coordinate) path-component (coordinate);}
|
||||
\item Path Reusage \myCode{postaction=\{<basic drawing commands> or <decorate>\}} When this option is given to any basic drawing commands below, the path is not immediately discarded and reused after the initial drawing command is finished. \\
|
||||
\myCode{preaction=\{<basic drawing commands> or <decorate>\}} When this option is given to any basic drawing commands below, the path is used once before the initial drawing command is executed.
|
||||
\myCode{preaction=\{<basic drawing commands> or <decorate>\}} When this option is given to any basic drawing commands below, the path is used once before the initial drawing command is executed.
|
||||
\end{itemize}
|
||||
|
||||
\section*{Basic Drawing Commands}
|
||||
|
|
|
@ -8,8 +8,8 @@
|
|||
\begin{preview}
|
||||
\begin{circuitikz}
|
||||
\draw (0,0) node[european and port] (myand){}
|
||||
(myand.in 1) node[anchor=east]{P}
|
||||
(myand.in 2) node[anchor=east]{Q}
|
||||
(myand.in 1) node[anchor=east]{P}
|
||||
(myand.in 2) node[anchor=east]{Q}
|
||||
(1.8,0) node[european not port] (mynot){}
|
||||
(mynot.out) node[anchor=west]{S}
|
||||
(myand.out) -- (mynot.in);
|
||||
|
|
|
@ -11,11 +11,11 @@
|
|||
\title{The Game '2048'}
|
||||
\author{Martin Thoma}
|
||||
|
||||
\hypersetup{
|
||||
pdfauthor = {Martin Thoma},
|
||||
pdfkeywords = {Game, combinatorics, 2048},
|
||||
pdftitle = {The Game '2048'}
|
||||
}
|
||||
\hypersetup{
|
||||
pdfauthor = {Martin Thoma},
|
||||
pdfkeywords = {Game, combinatorics, 2048},
|
||||
pdftitle = {The Game '2048'}
|
||||
}
|
||||
|
||||
%%%%%%%%%%%%%%%%%%%%%%%%%%%%%%%%%%%%%%%%%%%%%%%%%%%%%%%%%%%%%%%%%%%%%
|
||||
% Begin document %
|
||||
|
@ -36,7 +36,6 @@
|
|||
\section{Related}
|
||||
\begin{itemize}
|
||||
\item \url{http://math.stackexchange.com/q/716469/6876}
|
||||
\item
|
||||
\end{itemize}
|
||||
|
||||
\end{document}
|
||||
|
|
|
@ -33,7 +33,7 @@
|
|||
\foreach \line [count=\y] in \pixels {
|
||||
\foreach \pix [count=\x] in \line {
|
||||
\draw[fill=pixel \pix] (\x,-\y) rectangle +(1,1);
|
||||
|
||||
|
||||
\ifthenelse{\equal{0}{\pix}}
|
||||
{}
|
||||
{\node at ($(\x,-\y) + (0.5,0.5)$) {\Huge \pix};}
|
||||
|
|
|
@ -10,11 +10,11 @@
|
|||
\semester{Wintersemester 04/05}
|
||||
\scriptstate{complete}
|
||||
|
||||
\hypersetup{
|
||||
pdfauthor = {Die Mitarbeiter von http://mitschriebwiki.nomeata.de/ und GitHub-Beiträge},
|
||||
pdfkeywords = {Analysis},
|
||||
pdftitle = {Analysis I}
|
||||
}
|
||||
\hypersetup{
|
||||
pdfauthor = {Die Mitarbeiter von http://mitschriebwiki.nomeata.de/ und GitHub-Beiträge},
|
||||
pdfkeywords = {Analysis},
|
||||
pdftitle = {Analysis I}
|
||||
}
|
||||
|
||||
\author{Die Mitarbeiter von \url{http://mitschriebwiki.nomeata.de/} und GitHub-Beiträge}
|
||||
\title{Analysis I}
|
||||
|
@ -95,7 +95,7 @@ In $\MdR$ sind zwei Verknüpfungen "`+"' und "`$\cdot$"' gegeben, die jedem Paar
|
|||
\end{liste}
|
||||
\end{axiom}
|
||||
|
||||
Dabei nennt man \textbf{A1} und \textbf{A2} \begriff{Assoziativgesetze}, \textbf{A3} und \textbf{A4} \begriff{Kommutativgesetze} und \textbf{A9} \begriff{Distributivgesetz},
|
||||
Dabei nennt man \textbf{A1} und \textbf{A2} \begriff{Assoziativgesetze}, \textbf{A3} und \textbf{A4} \begriff{Kommutativgesetze} und \textbf{A9} \begriff{Distributivgesetz},
|
||||
|
||||
Alle Regeln der Grundrechenarten lassen sich aus \textbf{(A1)} bis \textbf{(A9)} herleiten. Diese Regeln seien von nun an bekannt.
|
||||
|
||||
|
@ -140,11 +140,11 @@ In \MdR\ ist eine Relation "`$\le$"' gegeben. Es sollen gelten:
|
|||
\end{schreibweisen}
|
||||
|
||||
\begin{definition}[Betrag]
|
||||
Für $a \in \MdR$ heißt $ |a| :=
|
||||
Für $a \in \MdR$ heißt $ |a| :=
|
||||
\begin{cases}
|
||||
a & \mbox{falls } a \ge 0 \\
|
||||
-a & \mbox{falls } a < 0
|
||||
\end{cases} $.
|
||||
\end{cases} $.
|
||||
|
||||
$|a|$ wird der \begriff{Betrag} von a genannt und entspricht dem "`Abstand"' von $a$ und $0$. $|a-b|$ entspricht dem "`Abstand"' von $a$ und $b$.
|
||||
\end{definition}
|
||||
|
@ -275,12 +275,12 @@ Für jedes $n \in \MdN$ sei eine Aussage $A(n)$ gemacht. Es gelte: (I) $A(1)$ is
|
|||
\item $A(n) := n \ge 1$. $A(n)\ \forall n \in \MdN$. Beweis (induktiv):\\
|
||||
Induktionsanfang (IA): $1 \ge 1$, also ist $A(1)$ wahr. \\
|
||||
Induktionsvorausseztung (IV): Sei $n \in \MdN$ und $A(n)$ wahr (also $n \ge 1$) \\
|
||||
Induktionsschritt (IS, $n \curvearrowright n + 1$): $n+1
|
||||
Induktionsschritt (IS, $n \curvearrowright n + 1$): $n+1
|
||||
\stackrel{(IV)}{\ge} 1 + 1 \ge 1$, also $A(n+1)$ wahr.
|
||||
\item Für $n \in \MdN$ sei $A_n:=(\MdN\ \cap\ [1,n])\ \cup\ [n+1, \infty)$. \\
|
||||
Behauptung: $\underbrace{A_n \text{ ist eine Induktionsmenge}}_{A(n)} \ \forall n \in \MdN$
|
||||
\item Sei $n \in \MdN, x \in \MdR$ und $n<x<n+1$. Behauptung: $x \notin \MdN$. Beweis: Annahme: $x \in \MdN$. Sei $A_m$ wie im oberen Beispiel (2) \folgt $A_m \in J \folgt \MdN \subseteq A_m \folgt x \in A_m \folgt x \le m$ oder $x\ge m+1$, Widerspruch!
|
||||
\item
|
||||
\item
|
||||
Behauptung: $\underbrace{1+2+\dots +n = \frac{n(n+1)}{2}}_{A(n)}\ \forall n\in \MdN$\\
|
||||
\textbf{Beweis:} (induktiv)\\
|
||||
IA: $\frac{1+1}{2}=1 \folgt A(1)$ ist wahr.\\
|
||||
|
@ -294,7 +294,7 @@ $1+2+\dots +n+(n+1)
|
|||
\folgt \text{A(n+1) ist wahr}$
|
||||
\end{beispiele}
|
||||
|
||||
\indexlabel{Summenzeichen}
|
||||
\indexlabel{Summenzeichen}
|
||||
\indexlabel{Produktzeichen}
|
||||
\begin{definition}[Summen- und Produktzeichen]
|
||||
\begin{liste}
|
||||
|
@ -395,7 +395,7 @@ Also $C:=\{b_1,b_2,\ldots\} \subseteq B$. $\forall x \in B \folgt x \in A \folgt
|
|||
\begin{liste}
|
||||
\item Für $a \in\MdR$ und $n\in\MdN$ gilt $a^n := a\cdot a\cdot a\cdot a\cdot \ldots \cdot a$ ($n$ Faktoren) und heißt die \indexlabel{Potenz!natürliche}\textit{$n$-te Potenz} von $a$\\
|
||||
$a^0:=1$ \\
|
||||
Für $a\ne 0$ gilt: $a^{-n}=\frac{1}{a^n}$
|
||||
Für $a\ne 0$ gilt: $a^{-n}=\frac{1}{a^n}$
|
||||
\item Für $n\in\MdN$ gilt $n! := 1\cdot 2\cdot 3\cdot \ldots \cdot n $ und heißt die \begriff{Fakultät} von $n$, $0! := 1$.
|
||||
\item Für $n\in\MdN$, $k\in\MdN_{0}$ und $k\le n$ gilt $\binom{n}{k}:=\frac{n!}{k!(n-k)!}$ ("`n über k"')
|
||||
\end{liste}
|
||||
|
@ -554,7 +554,7 @@ $$ \mathop{\inf{a_n}}_{n=1}^{\infty} := \mathop{\inf{a_n}}_{n\in \MdN} := \matho
|
|||
\begin{definition}[Konvergente Folge]
|
||||
Sei $(a_n)$ eine Folge. $(a_n)$ heißt \begriff{konvergent} $:\equizu \exists a \in \MdR$, so dass es für \textit{jedes} $\varepsilon > 0$ ein $n_0 = n_0(\varepsilon) \in \MdN$ gibt, so dass $|a_n - a| < \varepsilon \ \forall n \ge n_0$ gilt. In diesem Fall heißt $a$ der \begriff{Grenzwert} (GW) oder \begriff{Limes} von $(a_n)$ und man schreibt: $\lim_{n \to \infty}(a_n) = a$ oder $\lim{a_n} = a$ oder $a_n \to a \ (n \to \infty)$ oder $a_n \to a$. Ist $(a_n)$ nicht konvergent, so heißt $(a_n)$ \begriff{divergent}.
|
||||
\begin{eqnarray*}
|
||||
\text{Also: } a_n \to a \ (n \to \infty)
|
||||
\text{Also: } a_n \to a \ (n \to \infty)
|
||||
&\equizu& \forall\varepsilon > 0 \ \exists n_0 = n_0(\varepsilon) \in\MdN: |a_n - a| < \varepsilon \ \forall n \ge n_0 \\
|
||||
&\equizu& \forall\varepsilon > 0 \ \exists n_0 = n_0(\varepsilon) \in\MdN: a_n \in U_{\varepsilon}(a)\ \forall n \ge n_0 \\
|
||||
&\equizu& \forall\varepsilon > 0 \text{ gilt: } a_n \in U_\varepsilon(a) \text{ \ffa } n \in \MdN.
|
||||
|
@ -737,7 +737,7 @@ $a_n := \sqrt[n]{n} -1 \folgt a_n > 0 \ \forall n \in\MdN$. Zu zeigen ist: $a_n
|
|||
Sei $c>0$. Dann: $\sqrt[n]{c} \to 1 \ (n\to\infty)$.
|
||||
\end{wichtigesbeispiel}
|
||||
|
||||
\begin{beweis}
|
||||
\begin{beweis}
|
||||
Fall 1: $c\ge 1 \ \exists m \in\MdN: m \ge c \folgt 1\le c\le n \ \forall n\ge m \folgt \sqrt[n]{n} \le \underbrace{\sqrt[n]{n}}_{\to 1} \folgtnach{7.4} \sqrt[n]{c} \to 1$ \\
|
||||
Fall 2: $c<1 \folgt \frac{1}{c} > 1 \folgtnach{Fall 1} \underbrace{\sqrt[n]{\frac{1}{c}}}_{=\frac{1}{\sqrt[n]{c}}} \to 1 \folgtnach{6.2(vii)} \sqrt[n]{c} \to 1$
|
||||
\end{beweis}
|
||||
|
@ -789,12 +789,12 @@ $(a_n)$ sei eine Folge und $\alpha \in\MdR$. $\alpha$ heißt ein \begriff{Häufu
|
|||
\end{definition}
|
||||
|
||||
\begin{beispiele}
|
||||
\item $a_n = (-1)^n$. $a_{2n} = 1, a_{2n-1} = -1.$
|
||||
Sei $\ep > 0: a_{2n} \in U_{\ep}(1)\ \forall n \in \mathbb{N} \Rightarrow a_n \in U_{\ep}(1)$
|
||||
für unendlich viele $n \in \mathbb{N} \Rightarrow 1 \in H (a_n)$.
|
||||
Analog: $a_n \in U_{\ep}(-1)$ für unendlich viele $n \in \mathbb{N} \Rightarrow -1 \in \H(a_n)$.
|
||||
Sei $\alpha \in \mathbb{R}$ und 1 $\neq \alpha \neq -1$.
|
||||
Wähle $\ep > 0$ so, dass $1, -1 \not\in U_{\ep}(\alpha) \Rightarrow a_n \not\in U_{\ep}(\alpha)\ \forall n \in \mathbb{N} \Rightarrow \alpha \not\in \H(a_{n})$.
|
||||
\item $a_n = (-1)^n$. $a_{2n} = 1, a_{2n-1} = -1.$
|
||||
Sei $\ep > 0: a_{2n} \in U_{\ep}(1)\ \forall n \in \mathbb{N} \Rightarrow a_n \in U_{\ep}(1)$
|
||||
für unendlich viele $n \in \mathbb{N} \Rightarrow 1 \in H (a_n)$.
|
||||
Analog: $a_n \in U_{\ep}(-1)$ für unendlich viele $n \in \mathbb{N} \Rightarrow -1 \in \H(a_n)$.
|
||||
Sei $\alpha \in \mathbb{R}$ und 1 $\neq \alpha \neq -1$.
|
||||
Wähle $\ep > 0$ so, dass $1, -1 \not\in U_{\ep}(\alpha) \Rightarrow a_n \not\in U_{\ep}(\alpha)\ \forall n \in \mathbb{N} \Rightarrow \alpha \not\in \H(a_{n})$.
|
||||
Fazit: $\H(a_n) = \{1; -1\}$.
|
||||
\item $a_n = n$. Sei $\alpha \in \mathbb{R}$ und $\varepsilon > 0$. $\exists n_0 \in \mathbb{N}: n_0 > \alpha + \ep \Rightarrow n > \alpha + \ep\ \forall n \geq n_0 \Rightarrow a_n \not\in U_{\ep}(\alpha)\ \forall n \geq n_0 \Rightarrow a_n \in U_{\ep}(\alpha)$ für höchstens endlich viele $n \in \mathbb{N}$. $\Rightarrow \alpha \not\in \H(a_n)$. Fazit: $\H(a_n) = \emptyset$.
|
||||
\item $\mathbb{Q}$ ist abzählbar. Also: $\mathbb{Q} = \{a_1, a_2, \ldots\}$.\\
|
||||
|
@ -853,7 +853,7 @@ $(a_n)$ sei eine beschränkte Folge. Dann H$(a_n) \ne \emptyset$.
|
|||
\end{satz}
|
||||
|
||||
\begin{beweis}
|
||||
$\exists c>0: |a_n| \le c \ \forall n\in\MdN$. Hilfssatz $\folgt (a_n)$ enthält eine monotone Teilfolge $(a_{n_k})$. $|a_{n_k}| \le c \ \forall k \in\MdN$. $(a_{n_k})$ ist aber schränkt. 6.3 $\folgt (a_{n_k})$ ist konvergent. $\alpha := \lim_{k\to\infty}a_{n_k}$. 8.1(1) $\folgt \alpha \in \H(a_n)$.
|
||||
$\exists c>0: |a_n| \le c \ \forall n\in\MdN$. Hilfssatz $\folgt (a_n)$ enthält eine monotone Teilfolge $(a_{n_k})$. $|a_{n_k}| \le c \ \forall k \in\MdN$. $(a_{n_k})$ ist aber schränkt. 6.3 $\folgt (a_{n_k})$ ist konvergent. $\alpha := \lim_{k\to\infty}a_{n_k}$. 8.1(1) $\folgt \alpha \in \H(a_n)$.
|
||||
\end{beweis}
|
||||
|
||||
\chapter{Oberer und unterer Limes}
|
||||
|
@ -1075,13 +1075,13 @@ klar.
|
|||
\end{beweis}
|
||||
|
||||
\begin{definition}
|
||||
Die Reihe $\areihe$ heißt \begriff{absolut konvergent} $:\equizu \reihe{|a_n|}$ ist konvergent.
|
||||
Die Reihe $\areihe$ heißt \begriff{absolut konvergent} $:\equizu \reihe{|a_n|}$ ist konvergent.
|
||||
\end{definition}
|
||||
|
||||
\begin{satz}[Dreiecksungleichung für Reihen]
|
||||
Ist $\areihe$ absolut konvergent, so ist $\areihe$ konvergent und
|
||||
$$|\areihe| \le \reihe{|a_n|}$$
|
||||
\
|
||||
\
|
||||
\end{satz}
|
||||
|
||||
\begin{beweis}
|
||||
|
@ -1099,7 +1099,7 @@ Die \begriff{alternierende Harmonische Reihe} $\reihe{(-1)^{n+1} \frac{1}n}$.\\
|
|||
Hier: $a_n = (-1)^{n+1}\frac{1}{n}$. $|a_n| = \frac{1}{n} \folgt \reihe{a_n}$ konvergiert nicht absolut.\\
|
||||
\textbf{Behauptung:} $\reihe{a_n}$ ist konvergent. (Später: $\reihe{a_n} = \log 2$)\\
|
||||
\textbf{Beweis:} $s_n = a_1 + a_2 + \ldots + a_n$. \\
|
||||
$s_{2n+2} = s_{2n} + a_{2n+1} + a_{2n+2} = s_{2n} + \underbrace{\frac{1}{2n+1} - \frac{1}{2n+2}}_{>0} \folgt (s_{2n})$ ist monoton wachsend. Analog: $(s_{2n-1})$ ist monoton fallend.
|
||||
$s_{2n+2} = s_{2n} + a_{2n+1} + a_{2n+2} = s_{2n} + \underbrace{\frac{1}{2n+1} - \frac{1}{2n+2}}_{>0} \folgt (s_{2n})$ ist monoton wachsend. Analog: $(s_{2n-1})$ ist monoton fallend.
|
||||
$s_{2n} = s_{2n-1} + a_{2n} = s_{2n-1} - \frac{1}{2n}$ $(*)$\\
|
||||
Dann gilt $s_2 \le s_4 \le \ldots \le s_{2n} \gleichwegen{(*)} s_{2n-1} - \frac{1}{2n} < s_{2n-1} \le \ldots \le s_3 \le s_1 \folgt (s_{2n})$ und $(s_{2n-1})$ sind beschränkt. 6.3 $\folgt$ $(s_{2n})$ und $(s_{2n-1})$ sind konvergent. Aus $(*)$ folgt dann $\lim s_{2n} = \lim s_{2n-1}$. A16 $\folgt$ $(s_n)$ hat genau einen Häufungswert. 9.3 $\folgt$ $(s_n)$ ist konvergent.
|
||||
\end{beispiel}
|
||||
|
@ -1155,7 +1155,7 @@ Sei $(a_n)$ eine Folge und $\alpha := \limsup \sqrt[n]{|a_n|}$.
|
|||
|
||||
\begin{beweise}
|
||||
\item $\alpha < 1 $. Sei $\ep>0$ so, dass $x:= \alpha+\ep<1$. 9.2 $\folgt \sqrt[n]{|a_n|} < \alpha + \ep = x \ffa n\in\MdN \folgt |a_n| < x^n \ffa n\in\MdN$. $\reihe{x^n}$ ist konvergent $\folgtnach{12.1(1)}$ Behauptung.
|
||||
\item
|
||||
\item
|
||||
\begin{liste}
|
||||
\item $\alpha>1$, $\alpha<\infty$: Sei $\ep>0$ so, dass $\alpha-\ep>1$. 9.2 $\folgt \sqrt[n]{|a_n|}>\alpha-\ep>1$ für unendlich viele $n\in\MdN$ \folgt $|a_n|>1$ für unendlich viele $n\in\MdN$ \folgt $a_n \to 0 \folgtnach{11.1} \reihe{a_n}$ ist divergent.
|
||||
\item $\alpha = \infty \folgt \sqrt[n]{|a_n|} > 1$ für unendlich viele \natn \folgtnach{wie eben} $\reihe{a_n}$ ist divergent.
|
||||
|
@ -1194,7 +1194,7 @@ Sei $(a_n)$ eine Folge in $\MdR$ und $a_n \ne 0 \ffa \natn$. $\alpha_n := \frac{
|
|||
O.B.d.A.: $a_n \ne 0 \ \forall\natn$
|
||||
\begin{liste}
|
||||
\item Dann: $|a_2|\ge |a_1|>0$, $|a_3|\ge|a_2|\ge|a_1|>0$, \ldots allgemein: $|a_n|\ge|a_1|>0\ \forall\natn \folgt a_n\nrightarrow 0 \folgt$ die Behauptung.
|
||||
\item
|
||||
\item
|
||||
\begin{liste}
|
||||
\item Sei $\beta >1$, Sei $\ep>0$ so, dass $\beta-\ep>1$. 9.2 $\folgt |\alpha_n|>\beta-\ep>1 \ffa n\in\MdN \folgt$ die Behauptung.
|
||||
\item Sei $\alpha < 1$. Sei $\ep>0$ so, dass $x:=\alpha+\ep<1$. 9.2 $\folgt |\alpha_n|<\alpha+\ep=x \ffa\natn$. Dann: $|a_2|\le|a_1|x$, $|a_3|\le|a_2|x\le|a_1|x^2$,\ldots allgemein: $|a_n|\le|a_n1|x^{n-1} \ffa\natn$. $\reihe{|a_1|x^{n-1}}$ ist konvergent \folgtnach{12.2} $\reihe{a_n}$ ist absolut konvergent.
|
||||
|
@ -1499,7 +1499,7 @@ $\displaystyle{a_n := (-1)^n\cdot\frac{x^{2n}}{(2n)!} \folgt \sqrt[n]{|a_n|} = \
|
|||
|
||||
\begin{definition}[Potenzreihe]
|
||||
Sei $(a_n)_{n=0}^{\infty}$ eine Folge in \MdR. Eine Reihe der Form $\sum_{n=0}^{\infty}\
|
||||
{a_nx^n} = {\nobreak a_0 + a_1x + a_2x^2 + \ldots}$ heißt eine \begriff{Potenzreihe} (PR). Die Menge
|
||||
{a_nx^n} = {\nobreak a_0 + a_1x + a_2x^2 + \ldots}$ heißt eine \begriff{Potenzreihe} (PR). Die Menge
|
||||
$\{x \in\MdR : \reihenull{a_nx^n}$ konvergent$\}$ heißt der \begriff{Konvergenzbereich} (KB) der Potenzreihe. Klar: Die Potenzreihe konvergiert für $x=0$.
|
||||
\end{definition}
|
||||
|
||||
|
@ -1541,13 +1541,13 @@ $(\sqrt[n]{|a_nx^n|})=(\sqrt[n]{|a_n|}|x|) \folgt\ (\sqrt[n]{|a_nx^n|})$ ist nic
|
|||
n2^n&\text{n ungerade}\end{cases}$. A16 \folgt $\H(\sqrt[n]{|a_n|})=\{0, 2\}
|
||||
\folgt \rho=2 \folgt r=\frac{1}{2}$. Die Potenzreihe konvergiert absolut für $|x|<\frac{1}{2}$, sie divergiert für $|x|>\frac{1}{2}$. Sei $|x|=\frac{1}{2}$. $|a_nx^n|=|a_n|\frac{1}{2^n}=n$ falls $n$ ungerade \folgt $a_nx^n \nrightarrow 0 \folgt$ die Potenzreihe divergiert für $|x|=\frac{1}{2}$.
|
||||
\end{beispiele}
|
||||
Die folgenden Potenzreihen haben jeweils den Konvergenzradius $r=\infty:$\\
|
||||
Die folgenden Potenzreihen haben jeweils den Konvergenzradius $r=\infty:$\\
|
||||
$e^x=\reihenull{\frac{x^n}{n!},\ \sin x=\reihenull{(-1)^n\frac{x^{2n+1}}{(2n+1)!}}},\\ \cos x = \reihenull{(-1)^n\frac{x^{2n}}{(2n)!}},\ f'(x)=\reihenull{a_nnx^{n-1}}$, falls $f(x)=\reihenull{a_nx^n}$ KR $r=\infty$ hat.
|
||||
|
||||
\begin{definition}
|
||||
$\cosh x:=\frac{1}{2}(e^x+e^{-x})\ (x\in\MdR)$ (Cosinus Hyperbolikus)\\
|
||||
$\sinh x:=\frac{1}{2}(e^x-e^{-x})\ (x\in\MdR)$ (Sinus Hyperbolikus)\\
|
||||
Nachrechnen: $\cosh x=\reihenull{\frac{x^{2n}}{(2n)!}},
|
||||
Nachrechnen: $\cosh x=\reihenull{\frac{x^{2n}}{(2n)!}},
|
||||
\sinh x=\reihenull{\frac{x^{2n+1}}{(2n+1)!}} (x\in\MdR)$
|
||||
\end{definition}
|
||||
|
||||
|
@ -1571,7 +1571,7 @@ Sei $x \in (-R, R): (\reihenull{a_nx^n})(\reihenull{b_nx^n})\overset{\text{13.4}
|
|||
\end{beweis}
|
||||
|
||||
\begin{bemerkung}
|
||||
Sei $(a_n)^{\infty}_{n=0}$ eine Folge und $x_0 \in\MdR$. Eine Reihe der Form $(*) \reihenull{a_n(x-x_0)^n}$ heißt ebenfalls eine Potenzreihe ($x_0$ heißt \begriff{Entwicklungspunkt} der Potenzreihe). Substitution $t:=x-x_0$, dann erhält man die Potenzreihe $\reihenull{a_nt^n}$. Sei $r$ der Konvergenzradius dieser Potenzreihe. Dann: ist $r=0$, so konvergiert die Potenzreihe in $(*)$ \emph{nur} in $x=x_0$. Ist $r=\infty$, so konvergiert die Potenzreihe absolut $\forall\ x\in\MdR.$ Ist $0<r<\infty$, so konvergiert die Potenzreihe in $(*)$ absolut für $|x-x_0|<r$, sie divergiert für $|x-x_0|>r.$
|
||||
Sei $(a_n)^{\infty}_{n=0}$ eine Folge und $x_0 \in\MdR$. Eine Reihe der Form $(*) \reihenull{a_n(x-x_0)^n}$ heißt ebenfalls eine Potenzreihe ($x_0$ heißt \begriff{Entwicklungspunkt} der Potenzreihe). Substitution $t:=x-x_0$, dann erhält man die Potenzreihe $\reihenull{a_nt^n}$. Sei $r$ der Konvergenzradius dieser Potenzreihe. Dann: ist $r=0$, so konvergiert die Potenzreihe in $(*)$ \emph{nur} in $x=x_0$. Ist $r=\infty$, so konvergiert die Potenzreihe absolut $\forall\ x\in\MdR.$ Ist $0<r<\infty$, so konvergiert die Potenzreihe in $(*)$ absolut für $|x-x_0|<r$, sie divergiert für $|x-x_0|>r.$
|
||||
\end{bemerkung}
|
||||
|
||||
\chapter{$g$-adische Entwicklungen}
|
||||
|
@ -1619,7 +1619,7 @@ Sei $a=0,z_1z_2z_3\ldots$ ein $g$-adischer Bruch.
|
|||
|
||||
\begin{beweise}
|
||||
\item $0 \le a = \reihe{\frac{z_n}{g^n}} \stackrel{\text{($*$)}, Bem. (2)}{<} \reihe{\frac{g-1}{g^n}} \gleichnach{15.1} 1$.
|
||||
\item \textbf{Annahme:} $\exists n\in\MdN: z_n \ne w_n$. Sei $m$ der kleinste solche Index, also $z_m \ne w_m$ und $z_j = w_j$ für $j=1,\ldots ,m-1$. Etwa $z_m < w_m \folgt z_m - w_m < 0 \overset{z_m - w_m \in\MdZ}{\folgt} z_m - w_m \le -1$. $\forall n\in\MdN: z_n - w_n \le z_n \le g-1$. $\exists \nu \in\MdN$ mit $\nu \ge m+1$ und $z_\nu - w_\nu < g-1$. (andererenfalls $z_\nu - w_\nu = g-1 \ \forall \nu \ge m+1 \folgt z_\nu = w_\nu + g-1 \ \forall \nu \ge m+1 \folgt w_\nu = 0 \ \forall \nu \ge m+1 \folgt z_\nu = g-1 \ \forall \nu \ge m+1$. Widerspruch zu $(*)$). Dann:
|
||||
\item \textbf{Annahme:} $\exists n\in\MdN: z_n \ne w_n$. Sei $m$ der kleinste solche Index, also $z_m \ne w_m$ und $z_j = w_j$ für $j=1,\ldots ,m-1$. Etwa $z_m < w_m \folgt z_m - w_m < 0 \overset{z_m - w_m \in\MdZ}{\folgt} z_m - w_m \le -1$. $\forall n\in\MdN: z_n - w_n \le z_n \le g-1$. $\exists \nu \in\MdN$ mit $\nu \ge m+1$ und $z_\nu - w_\nu < g-1$. (andererenfalls $z_\nu - w_\nu = g-1 \ \forall \nu \ge m+1 \folgt z_\nu = w_\nu + g-1 \ \forall \nu \ge m+1 \folgt w_\nu = 0 \ \forall \nu \ge m+1 \folgt z_\nu = g-1 \ \forall \nu \ge m+1$. Widerspruch zu $(*)$). Dann:
|
||||
$\displaystyle{0 = a-a = \reihe{\frac{z_n}{g^n}} - \reihe{\frac{w_n}{g^n}} = \reihe{\frac{z_n - w_n}{g^n}} = \sum_{n=m}^\infty{\frac{z_n - w_n}{g^n}}}$\\
|
||||
$\displaystyle{= \underbrace{\frac{z_m - w_m}{g^m}}_{\le -\frac{1}{g^m}} + \underbrace{\sum_{n=m+1}^\infty{\frac{z_n-w_n}{g^n}}}_{< \sum_{n=m+1}^\infty{\frac{g-1}{g^n}}} < - \frac{1}{g^m} + \underbrace{\sum_{n=m+1}^\infty{\frac{g-1}{g^n}}}_{\gleichnach{15.1} \frac{1}{g^n}} = 0}$\\
|
||||
$\folgt 0<0 \text{ Widerspruch.}$
|
||||
|
@ -1651,7 +1651,7 @@ Die Menge der reellen Zahlen ist überabzählbar.
|
|||
\begin{beweis}
|
||||
Es genügt zu zeigen: $[0,1)$ ist überabzählbar.
|
||||
|
||||
\textbf{Annahme}: $[0,1)$ ist abzählbar, also $[0,1) = \{a_1,a_2,\ldots\}, a_j \ne a_k$ für $j\ne k$.
|
||||
\textbf{Annahme}: $[0,1)$ ist abzählbar, also $[0,1) = \{a_1,a_2,\ldots\}, a_j \ne a_k$ für $j\ne k$.
|
||||
Für $j \in\MdN$ sei $a_j = 0,z_1^{(j)} z_2^{(j)} z_3^{(j)}\ldots$ die 3-adische Entwicklung von $a_j$. ($z_k^{(j)} \in \{0,1,2\}$).
|
||||
$$ z_k := \begin{cases} 1 & \text{falls } z_k^{(k)} \in \{0,2\} \\0 & \text{falls } z_k^{(k)}= 1 \end{cases}$$
|
||||
Dann: $z_k \ne z_k^{(k)} \ \forall k \in\MdN$, $z_k \ne g-1 \ \forall k\in\MdN$. $a := 0,z_1z_2z_3\ldots = \reihe{\frac{z_n}{g^n}}.$ 15.2 $\folgt a \in [0,1) \folgt \exists m\in\MdN: a= a_m \folgt 0,z_1z_2z_3\ldots = 0,z_1^{(m)}z_2^{(m)}z_3^{(m)}\ldots$. 15.2 $\folgt z_j = z_j^{(m)} \ \forall j\in\MdN \folgt z_m = z_m^{(m)}$. Widerspruch!
|
||||
|
@ -1712,7 +1712,7 @@ Für $\delta >0$: $D_\delta(x_n) = D \cap U_\delta(x_0)$. $\dot D_\delta(x_0) =
|
|||
\begin{satz}[Grenzwertsätze bei Funktionen]
|
||||
\begin{liste}
|
||||
\item $\displaystyle\lim_{x\to x_0}f(x)$ existiert $\equizu$ für jede Folge $(x_n)$ in $D\backslash\{x_0\}$ mit $x_n \to x_0$ ist $f(x_n)$ konvergent.
|
||||
\item Für $a\in\MdR$ gilt: $\displaystyle\lim_{x\to x_0}f(x)$ existiert und ist gleich $a$ $\equizu$ $\forall \ep>0 \ \exists \delta(\ep) > 0$ mit $(*)$ $|f(x) - a|< \ep \ \forall x \in \dot D_\delta(x_0)$.
|
||||
\item Für $a\in\MdR$ gilt: $\displaystyle\lim_{x\to x_0}f(x)$ existiert und ist gleich $a$ $\equizu$ $\forall \ep>0 \ \exists \delta(\ep) > 0$ mit $(*)$ $|f(x) - a|< \ep \ \forall x \in \dot D_\delta(x_0)$.
|
||||
\item \textit{Cauchykriterium}\indexlabel{Cauchykriterium!bei Funktionsgrenzwerten}: $\displaystyle\lim_{x\to x_0} f(x)$ existiert $\equizu$ $\forall \ep>0 \ \exists \delta >0$: $|f(x) - f(x')|<\ep \forall x,x' \in \dot D_\delta(x_0)$
|
||||
\end{liste}
|
||||
\end{satz}
|
||||
|
@ -1744,8 +1744,8 @@ Zum Beispiel: (3) Sei $(x_n)$ Folge in $D\backslash\{x_0\}$ und $x_n \to x_0$. D
|
|||
\begin{definition}
|
||||
\begin{liste}
|
||||
\item Sei $(a_n)$ eine Folge in $\MdR$.\\
|
||||
$\lim a_n = \infty$ (oder $a_n \to \infty$) $:\equizu \forall c>0 \ \exists n_0 = n_0(c)\in\MdN: a_n > c \forall n\ge n_0$.\\
|
||||
$\lim a_n = -\infty$ (oder $a_n \to -\infty$) $:\equizu \forall c<0 \ \exists n_0 = n_0(c)\in\MdN: a_n < c \forall n\ge n_0$.
|
||||
$\lim a_n = \infty$ (oder $a_n \to \infty$) $:\equizu \forall c>0 \ \exists n_0 = n_0(c)\in\MdN: a_n > c \forall n\ge n_0$.\\
|
||||
$\lim a_n = -\infty$ (oder $a_n \to -\infty$) $:\equizu \forall c<0 \ \exists n_0 = n_0(c)\in\MdN: a_n < c \forall n\ge n_0$.
|
||||
\item $\displaystyle\lim_{x\to x_0} f(x) = \infty$ (oder $f(x) \to \infty\ (x\to x_0)$) $:\equizu$ für jede Folge $(x_n)$ in $D\backslash\{x_0\}$ und $x_n \to x_0$ gilt: $f(x_n) \to \infty$. \\
|
||||
$\displaystyle\lim_{x\to x_0} f(x) = -\infty$ (oder $f(x) \to -\infty\ (x\to x_0)$) $:\equizu$ für jede Folge $(x_n)$ in $D\backslash\{x_0\}$ und $x_n \to x_0$ gilt: $f(x_n) \to -\infty$.
|
||||
\item Sei $D$ nicht nach oben beschränkt. $\displaystyle\lim_{x\to \infty} f(x) = a$ (oder $f(x) \to a$) $:\equizu$ für jede Folge $(x_n)$ in $D$ mit $x_n\to \infty$ gilt: $f(x_n) \to a$ ($a = \pm\infty$ zugelassen). \\
|
||||
|
@ -1896,8 +1896,8 @@ $B \subseteq \MdR$ heißt \indexlabel{offene Menge}\textbf{offen} $:\equizu \for
|
|||
\begin{beweis}
|
||||
\begin{liste}
|
||||
\item Übung
|
||||
\item "`\folgt "': Sei $(x_n)$ eine konvergente Folge in $\MdR\ \backslash\ B$ und $x_0:=\lim x_n$.Annahme: $x_0 \in B$. B offen $\folgt \exists \delta>0 : U_\delta(x_0) \subseteq B$. $x_n \to x_0 \folgt x_n \in U_\delta(x_0) \subseteq B \ffa n \in\MdN$, Widerspruch! "`$\Leftarrow$ "': Sei $x \in B$. Annahme: $U_\delta(x) \nsubseteq B \forall \delta>0$.
|
||||
\folgt $U_{\frac{1}{n}}(x) \nsubseteq B \forall n \in \MdN
|
||||
\item "`\folgt "': Sei $(x_n)$ eine konvergente Folge in $\MdR\ \backslash\ B$ und $x_0:=\lim x_n$.Annahme: $x_0 \in B$. B offen $\folgt \exists \delta>0 : U_\delta(x_0) \subseteq B$. $x_n \to x_0 \folgt x_n \in U_\delta(x_0) \subseteq B \ffa n \in\MdN$, Widerspruch! "`$\Leftarrow$ "': Sei $x \in B$. Annahme: $U_\delta(x) \nsubseteq B \forall \delta>0$.
|
||||
\folgt $U_{\frac{1}{n}}(x) \nsubseteq B \forall n \in \MdN
|
||||
\folgt \forall n \in \MdN \exists x_n \in U_{\frac{1}{n}}$ mit: $x_n \in \MdR\ \backslash\ B \folgt (x_n)$ ist eine Folge in $\MdR\ \backslash\ B: x_n \to x$. $\MdR\ \backslash\ B$ abgeschlossen \folgt $x \in \MdR\ \backslash\ B$, Widerspruch!
|
||||
\item "`\folgt "': Sei $(x_n)$ Folge in $D$. $D$ beschränkt \folgt $(x_n)$ beschränkt. 8.2 \folgt $(x_n)$ enthält eine konvergente Teilfolge $(x_{n_k})$. $D$ abgeschlossen $\folgt \lim x_{n_k} \in D$. "`$\Leftarrow$ "': Übung. Sei $D$ beschränkt und abgeschlossen. Sei $s:=\sup D$. z.z.: $s \in D$ (analog zeigt man $\inf D \in D$). $\forall n \in \MdN$ ist $s-\frac{1}{n}$ keine obere Schranke von s. \folgt $\forall n \in\MdN\exists\ x_n \in D$ mit $s - \frac{1}{n} < x_n \le s \folgt x_n \to s$. D abgeschlossen \folgt $s \in D$
|
||||
\end{liste}
|
||||
|
@ -1933,7 +1933,7 @@ Sei $I \subseteq \MdR$ ein Intervall und $f \in C(I)$.
|
|||
\end{beweis}
|
||||
|
||||
\begin{satz}[Der Logarithmus]
|
||||
Sei $I=\MdR$ und $f(x)=e^x$. Bekannt: $f \in C(\MdR)$, f ist streng monoton wachsend und $f(I) = f(\MdR) = (0, \infty)$. Also existiert $f^{-1}: (0, \infty) \to \MdR$.
|
||||
Sei $I=\MdR$ und $f(x)=e^x$. Bekannt: $f \in C(\MdR)$, f ist streng monoton wachsend und $f(I) = f(\MdR) = (0, \infty)$. Also existiert $f^{-1}: (0, \infty) \to \MdR$.
|
||||
\[ \log x := \ln x := f^{-1}(x)\ (x \in (0, \infty))\ \text{\emph{Logarithmus}} \]
|
||||
\end{satz}
|
||||
|
||||
|
@ -2005,7 +2005,7 @@ $(f_n)$ konvergiert punktweise auf $D$ gegen $f$.
|
|||
Konvergiert $(f_n)$ auf $D$ punktweise gegen $f:D\to\MdR$, so bedeutet dies: Ist $\ep>0$ und $x\in D$, so existiert ein $n_0 = n_0(\ep,x)\in\MdN$: $|f_n(x)-f(x)|<\ep \ \forall n\ge n_0$
|
||||
|
||||
\begin{definition}
|
||||
$(f_n)$ heißt auf $D$ \begriff{gleichmäßig konvergent}
|
||||
$(f_n)$ heißt auf $D$ \begriff{gleichmäßig konvergent}
|
||||
$:\equizu \exists \text{ Funktion } f:D\to\MdR$ für die gilt:\\
|
||||
\hspace*{10mm} $\forall \ep>0 \ \exists n_0 = n_0(\ep) \in\MdN \ \forall n\ge n_0 \ \forall x\in D\colon |f_n(x) - f(x)|<\ep$.
|
||||
|
||||
|
@ -2475,24 +2475,24 @@ Dann: $\sum_{k=0}^\infty \frac{f^{(k)}(0)}{k!}x^k = 0 \ne f(x) \ \forall x\in\Md
|
|||
|
||||
\begin{definition}
|
||||
Sei $n\in\MdN_0$, $f\in C^n(I)$ und $x_0 \in I$.\\
|
||||
$T_n(x;x_0) := \sum_{k=0}^n \frac{f^{(k)}(x_0)}{k!}(x-x_0)^k$
|
||||
$T_n(x;x_0) := \sum_{k=0}^n \frac{f^{(k)}(x_0)}{k!}(x-x_0)^k$
|
||||
heißt das \begriff{k-te Taylorpolynom} von $f$ vom Grad $\leq k$.
|
||||
\end{definition}
|
||||
|
||||
\begin{eigenschaftenNoCounter}
|
||||
\begin{enumerate}
|
||||
\item $p$ ist ein Polynom vom Grad $\leq n$ und es gilt:
|
||||
\item $p$ ist ein Polynom vom Grad $\leq n$ und es gilt:
|
||||
$p^{(k)}(x_0) = f^{(k)}(x_0)$ für $k=0, 1, \dots, n$
|
||||
\item Ist $q$ ein Polynom vom Grad $\leq n$ und gilt $q^{(k)} (x_0) = f^{(k)} (x_0)$
|
||||
für $k=0, 1, \dots, n$, so ist $p=q$.
|
||||
\item Ist $f \in C^\infty(1)$, so ist $T_n(x, x_0)$ die n-te
|
||||
\item Ist $f \in C^\infty(1)$, so ist $T_n(x, x_0)$ die n-te
|
||||
Teilsumme der Taylorreihe $f$ (in $x_0$).
|
||||
\end{enumerate}
|
||||
\end{eigenschaftenNoCounter}
|
||||
|
||||
\begin{satz}[Satz von Taylor]
|
||||
Voraussetzungen wie in obiger Definition. Weiter sei $f\ (n+1)$-mal
|
||||
differenzierbar auf $I$ und $x\in I$. Dann existiert ein $\xi$
|
||||
Voraussetzungen wie in obiger Definition. Weiter sei $f\ (n+1)$-mal
|
||||
differenzierbar auf $I$ und $x\in I$. Dann existiert ein $\xi$
|
||||
zwischen $x$ und $x_0$ mit:
|
||||
\[ f(x) = T_n(x;x_0) + \underbrace{\frac{f^{(n+1)}(\xi)}{(n+1)!}(x-x_0)^{n+1}}_\text{Restglied nach Lagrange} \]
|
||||
\end{satz}
|
||||
|
@ -2501,7 +2501,7 @@ zwischen $x$ und $x_0$ mit:
|
|||
Ohne Beschränkung der Allgemeinheit sei $x_0 = 0$ und $x>x_0$.\\
|
||||
$\rho := (f(x) - T_n(x;0)) \frac{(n+1)!}{x^{n+1}} \folgt f(x) - T_n(x;0) = \frac{\rho}{(n+1)!}x^{n+1}$\\
|
||||
Zu zeigen ist: $\exists \xi\in[0,x]: \rho = f^{(n+1)}(\xi).$ \\
|
||||
Definiere $h: [0,x]\to \MdR$ durch $f(x) = f(x) - \sum_{k=0}^n \frac{f^{(k)}(t)}{k!} (x-t)^k - \rho\frac{(x-t)^{n+1}}{(n+1)!}$.
|
||||
Definiere $h: [0,x]\to \MdR$ durch $f(x) = f(x) - \sum_{k=0}^n \frac{f^{(k)}(t)}{k!} (x-t)^k - \rho\frac{(x-t)^{n+1}}{(n+1)!}$.
|
||||
Nachrechnen: $h(0) = h(x)$ und $h'(t) = \rho\frac{(x-t)^n}{n!} - \frac{f^{(n+1)}(t)}{n!}(x-t)^n$. \\
|
||||
$0 = \frac{h(x)-h(0)}{x-0} \gleichnach{MWS} h'(\xi) \ \xi \in (0,x) \folgt \rho\frac{(x-\xi)^n}{n!} = \frac{f^{(n+1)}(\xi)}{n!}(x-\xi)^n \folgt \rho = f^{(n+1)}(\xi)$.
|
||||
\end{beweis}
|
||||
|
@ -2684,16 +2684,16 @@ $f \in R[a,b] \equizu \forall \ep>0\ \exists Z \in \Z: S_f(Z)-s_f(Z) < \ep$.
|
|||
|
||||
\begin{beweise}
|
||||
\item O.B.d.A: $f$ ist wachsend auf $[a,b]$.\\
|
||||
Sei $n\in\MdN$ und $Z_n=\{x_0,\cdots,x_n\}$ sei die
|
||||
\begriff{äquidistante Zerlegung} von $[a,b]$ mit $n+1$
|
||||
Teilpunkten. $x_j = a+j\frac{b-a}{n}\ (j=0,\cdots,n)$.
|
||||
Dann: $|I_j|=\frac{b-a}{n}$. $m_j, M_j$ wie immer:
|
||||
Sei $n\in\MdN$ und $Z_n=\{x_0,\cdots,x_n\}$ sei die
|
||||
\begriff{äquidistante Zerlegung} von $[a,b]$ mit $n+1$
|
||||
Teilpunkten. $x_j = a+j\frac{b-a}{n}\ (j=0,\cdots,n)$.
|
||||
Dann: $|I_j|=\frac{b-a}{n}$. $m_j, M_j$ wie immer:
|
||||
$S_f(Z_n)-s_f(Z_n)=\sum^n_{j=1}(\underbrace{M_j}_{=f(x_j)}-\underbrace{m_j}_{f(x_{j-1})})|I_j|=\sum_{j=1}^n(f(x_j)_-f(x_{j-1}))\frac{b-a}{n}=\frac{b-a}{n}(f(x_1)-f(x_0)+f(x_2)-f(x_1)+\cdots+f(x_n)-f(x_{n-1}))=\frac{b-a}{n}(f(x_n)-f(x_0))=\frac{b-a}{n}(f(b)-f(a))=:\alpha_n$. Sei $\ep>0$, dann: $\exists n\in\MdN: \alpha_n <\ep\folgtnach{23.3}$Behauptung.
|
||||
\item Sei $f \in C[a, b]$ und $\ep>0$. $\exists \delta>0: (*)\ |f(t)-f(s)|<\frac{\ep}{b-a}\ \forall t,s\in[a,b]$ mit $|t-s|<\delta$. Sei $Z=\{x_0,\cdots,x_n\}\in\Z\ m_j,\ M_j,\ |I_J|$ seien wie immer; $z$ sei so gewählt, da"s $|I_j|<\delta\ (j=1,\cdots,n)$. Betrachte $I_j:$ 18.3$\folgt\ \exists s_j, t_j \in I_j: m_j=f(s_j),\ M_j=f(t_j)$. $|t_j-s_j|<\delta \folgtwegen{(*)}\underbrace{f(t_j)-f(s_j)}_{=M_j-m_j}<\frac{\ep}{b-a}\folgt S_f(Z)-s_f(Z)=\sum^n_{j=1}(\underbrace{M_j-m_j}_{\le\frac{\ep}{b-a}})|I_j|<\frac{\ep}{b-a}\sum^n_{j=1}|I_j|=\ep\folgtnach{23.3}f\in R[a,b]$
|
||||
\end{beweise}
|
||||
|
||||
\begin{definition}
|
||||
Sei $J\subseteq\MdR$ ein Intervall und $G,g: J\to\MdR$ seien Funktionen.
|
||||
Sei $J\subseteq\MdR$ ein Intervall und $G,g: J\to\MdR$ seien Funktionen.
|
||||
$G$ heißt eine \begriff{Stammfunktion} von $g$ auf $J$ :$\equizu$ G ist auf $J$ differenzierbar und $G'=g$ auf $J$.\\
|
||||
\end{definition}
|
||||
\textbf{Beachte:}
|
||||
|
@ -2715,7 +2715,7 @@ Sei $[a,b]=[-1,1]$, $f(x):=\begin{cases}
|
|||
\end{liste}
|
||||
|
||||
\begin{satz}[1. Hauptsatz der Differential- und Integralrechnung]
|
||||
Es sei $f\in R[a,b]$ und $f$ besitze auf $[a,b]$ die Stammfunktion $F$. Dann:
|
||||
Es sei $f\in R[a,b]$ und $f$ besitze auf $[a,b]$ die Stammfunktion $F$. Dann:
|
||||
$$\int_a^bf(x)dx = F(b) - F(a) =: F(x)|_a^b =: [F(x)]_a^b$$
|
||||
\end{satz}
|
||||
|
||||
|
@ -2738,19 +2738,19 @@ Aber: $\lim_{n\to\infty}\int_0^1f_ndx = 1 \ne 0 = \int_0^1fdx = \int_0^1(\lim_{n
|
|||
\end{beispiele}
|
||||
|
||||
\begin{satz}[Integrierbarkeit gleichmäßig konvergierender Funktionsfolgen]
|
||||
$(f_n)$ sei eine Folge in $R[a,b]$ und $(f_n)$ konvergiert auf $[a,b]$ \emph{gleichmäßig} gegen $f:[a,b]\to\MdR$. Dann ist $f\in R[a,b]$ und
|
||||
$(f_n)$ sei eine Folge in $R[a,b]$ und $(f_n)$ konvergiert auf $[a,b]$ \emph{gleichmäßig} gegen $f:[a,b]\to\MdR$. Dann ist $f\in R[a,b]$ und
|
||||
$$\lim_{n\to\infty}\int_a^bf_n(x)dx = \int_a^bfdx = \int_a^b(\lim_{n\to\infty} f_n)dx$$
|
||||
|
||||
$(f_n)$ sei eine Folge in $R[a,b]$ und $\sum_{n=1}^{\infty}f_n$ konvergiert auf $[a,b]$ \emph{gleichmäßig} gegen $f:[a,b]\to\MdR$. Dann ist $f\in R[a,b]$ und
|
||||
$(f_n)$ sei eine Folge in $R[a,b]$ und $\sum_{n=1}^{\infty}f_n$ konvergiert auf $[a,b]$ \emph{gleichmäßig} gegen $f:[a,b]\to\MdR$. Dann ist $f\in R[a,b]$ und
|
||||
$$\sum_{n=1}^\infty \int_a^bf_n(x)dx = \int_a^b \sum_{n=1}^\infty f_n(x)dx $$
|
||||
\end{satz}
|
||||
|
||||
\begin{beweis}
|
||||
1. Zu $\ep=1 \ \exists m \in \MdN$: $f_m-1<f<f_m+1$ auf $[a,b]$. $f_n$ beschränkt auf $[a,b]$. \\
|
||||
2. $A_n := \int_a^bf_ndx$ $(n\in\MdN)$. Sei $\ep>0$. $\exists n_0\in\MdN: f_n-\ep<f<f_n+\ep$ auf $[a,b] \ \forall n\ge n_0 \folgt$ für $n\ge n_0$ folgt (wie im Beweis von 23.2(1)):
|
||||
2. $A_n := \int_a^bf_ndx$ $(n\in\MdN)$. Sei $\ep>0$. $\exists n_0\in\MdN: f_n-\ep<f<f_n+\ep$ auf $[a,b] \ \forall n\ge n_0 \folgt$ für $n\ge n_0$ folgt (wie im Beweis von 23.2(1)):
|
||||
$$\underbrace{\uint_a^b(f_n-\ep)dx}_{=A_n-\ep(b-a)} \le \underbrace{\uint_a^bfdx}_{=: A} \le \underbrace{\oint_a^bfxds}_{=: B} \le \underbrace{\oint_a^b(f_n+\ep)dx}_{=A_n+\ep(b-a)}$$
|
||||
$\folgt |A_n - A| \le \ep(b-a)$, $|A_n -B|\le \ep (b-a)$ \\
|
||||
$\forall n\in n_0 \folgt A_n \to A, A_n \to B \ (n\to\infty) \folgt A = B $ \\
|
||||
$\folgt |A_n - A| \le \ep(b-a)$, $|A_n -B|\le \ep (b-a)$ \\
|
||||
$\forall n\in n_0 \folgt A_n \to A, A_n \to B \ (n\to\infty) \folgt A = B $ \\
|
||||
$\folgt f\in R[a,b]$ und $A_n \to \int_a^bfdx$
|
||||
\end{beweis}
|
||||
|
||||
|
@ -2833,7 +2833,7 @@ $f,g: [a,b] \to \MdR$ seien Funktionen.
|
|||
|
||||
Sei $\ep > 0.$ Wähle $\alpha \in (a,b)$ mit $2\gamma(\alpha-a) < \ep/2.$
|
||||
|
||||
$f \in C[\alpha,b] \folgt f \in R[\alpha,b] \folgtnach{23.3}$ Es gibt eine Zerlegung $Z_1$ von $[\alpha,b]$ mit:\\
|
||||
$f \in C[\alpha,b] \folgt f \in R[\alpha,b] \folgtnach{23.3}$ Es gibt eine Zerlegung $Z_1$ von $[\alpha,b]$ mit:\\
|
||||
$S_f(Z_1) - s_f(Z_1) < \ep/2.\ Z:=Z_1 \cup \{a\} \folgt Z \in \Z$ und es gilt:\\
|
||||
\begin{align*}
|
||||
S_f(Z) - s_f(Z) &= \underbrace{\sup f([a,\alpha]) - \inf f([a,\alpha]))(\alpha-1)}_{\le 2 \gamma} + \underbrace{S_f(Z_1)-s_f(Z_1)}_{< \ep/2}\\
|
||||
|
@ -2927,7 +2927,7 @@ Es ist $\frac{F(x_0+h)-F(x_0)}{h}\gleichnach{s.o.}\frac{1}{h}\int_{x_0}^{x_0+h}f
|
|||
\end{beweise}
|
||||
|
||||
\begin{satz}[Anwendung des 2. Hauptsatzes auf stetige Funktionen]
|
||||
Sei $J \subseteq \MdR$ ein beliebiges Intervall, $f \in C(J)$ und $\xi \in J$ (fest). $F:J\to\MdR$ sei definiert durch $F(x):=\int_{\xi}^xf(t)\dt$. Dann ist $F\in C^1(J)$ und $F'=f$ auf $J$.
|
||||
Sei $J \subseteq \MdR$ ein beliebiges Intervall, $f \in C(J)$ und $\xi \in J$ (fest). $F:J\to\MdR$ sei definiert durch $F(x):=\int_{\xi}^xf(t)\dt$. Dann ist $F\in C^1(J)$ und $F'=f$ auf $J$.
|
||||
\end{satz}
|
||||
|
||||
|
||||
|
@ -3216,7 +3216,7 @@ $\folgt \exists c \in (1,\infty): \frac{f(x)}{g(x)} \ge \frac{1}{2}\ \forall x \
|
|||
|
||||
\begin{definition}
|
||||
Sei $f:[a,b]\to\MdR$ und $Z=\{x_0,\ldots,x_n\} \in\Z$. $V_f(Z):=\sum_{j=1}^n|f(x_j)-f(x_{j-1})|$ ist die \begriff{Variation} von $f$ bezüglich Z.\\
|
||||
\textbf{Beachte}: Sind $Z_1,Z_2 \in \Z$ und $Z_1 \subseteq Z_2\folgt V_f(Z_1) \le V_f(Z_2)$. $M_f=\{V_f(Z):Z \in \Z\}.\ f$ heißt von \begriff{beschränkter Variation}, in Zeichen: $f\in\BV[a,b]\ :\equizu M_f$ ist nach oben beschränkt. In diesem Fall heißt $V_f[a,b]:=\sup M_f$ die \begriff{Totalvariation} von $f$ (auf $[a,b]$).
|
||||
\textbf{Beachte}: Sind $Z_1,Z_2 \in \Z$ und $Z_1 \subseteq Z_2\folgt V_f(Z_1) \le V_f(Z_2)$. $M_f=\{V_f(Z):Z \in \Z\}.\ f$ heißt von \begriff{beschränkter Variation}, in Zeichen: $f\in\BV[a,b]\ :\equizu M_f$ ist nach oben beschränkt. In diesem Fall heißt $V_f[a,b]:=\sup M_f$ die \begriff{Totalvariation} von $f$ (auf $[a,b]$).
|
||||
\end{definition}
|
||||
|
||||
\begin{beispiel}
|
||||
|
@ -3372,7 +3372,7 @@ Sei $g \in \BV[a,b]$ und $f \in R_{g}$. Dann: $${\left|\int_a^b fdg\right|}\le\g
|
|||
|
||||
\begin{beweis}
|
||||
Sei $(Z, \xi) \in \Z^*, Z = \{x_0,\ldots,x_n\},\ \xi = (\xi_1,\ldots,\xi_n)$.\\
|
||||
$|\sigma_f(Z,\xi, g)|=|\displaystyle\sum_{j=1}^nf(\xi_j)(g(x_j)-g(x_{j-1}))|\le\displaystyle\sum_{j=1}^n|f(\xi_j)||g(x_j)-g(x_{j-1})|\le\gamma V_g(Z)\le\gamma V_g[a, b]$
|
||||
$|\sigma_f(Z,\xi, g)|=|\displaystyle\sum_{j=1}^nf(\xi_j)(g(x_j)-g(x_{j-1}))|\le\displaystyle\sum_{j=1}^n|f(\xi_j)||g(x_j)-g(x_{j-1})|\le\gamma V_g(Z)\le\gamma V_g[a, b]$
|
||||
\end{beweis}
|
||||
|
||||
\begin{bezeichnungen}
|
||||
|
|
File diff suppressed because it is too large
Load diff
|
@ -7,16 +7,16 @@
|
|||
\semester{Wintersemeseter 10/11 und 12/13}
|
||||
\scriptstate{complete}
|
||||
|
||||
\author{Die Mitarbeiter von \href{http://mitschriebwiki.nomeata.de/}{mitschriebwiki.nomeata.de}
|
||||
\author{Die Mitarbeiter von \href{http://mitschriebwiki.nomeata.de/}{mitschriebwiki.nomeata.de}
|
||||
und \href{https://github.com/MartinThoma/LaTeX-examples/tree/master/documents}{GitHub}}
|
||||
\title{Analysis III - Bachelorversion}
|
||||
\makeindex
|
||||
|
||||
\hypersetup{
|
||||
pdfauthor = {Die Mitarbeiter von mitschriebwiki.nomeata.de und GitHub},
|
||||
pdfkeywords = {Analysis},
|
||||
pdftitle = {Analysis III}
|
||||
}
|
||||
\hypersetup{
|
||||
pdfauthor = {Die Mitarbeiter von mitschriebwiki.nomeata.de und GitHub},
|
||||
pdfkeywords = {Analysis},
|
||||
pdftitle = {Analysis III}
|
||||
}
|
||||
|
||||
\begin{document}
|
||||
\maketitle
|
||||
|
@ -29,10 +29,10 @@ und \href{https://github.com/MartinThoma/LaTeX-examples/tree/master/documents}{G
|
|||
\chapter*{Vorwort}
|
||||
|
||||
\section*{Über dieses Skriptum}
|
||||
Dies ist ein Mitschrieb der Vorlesung \glqq Analysis III\grqq\ von
|
||||
Herrn Schmoeger im Wintersemester 2010 an der Universität Karlsruhe
|
||||
(KIT). Die Mitschriebe der Vorlesung werden mit ausdrücklicher
|
||||
Genehmigung von Herrn Schmoeger hier veröffentlicht, Herr Schmoeger
|
||||
Dies ist ein Mitschrieb der Vorlesung \glqq Analysis III\grqq\ von
|
||||
Herrn Schmoeger im Wintersemester 2010 an der Universität Karlsruhe
|
||||
(KIT). Die Mitschriebe der Vorlesung werden mit ausdrücklicher
|
||||
Genehmigung von Herrn Schmoeger hier veröffentlicht, Herr Schmoeger
|
||||
ist für den Inhalt nicht verantwortlich.
|
||||
|
||||
Kapitel werden in Beweisen durch "`§"' abgekürzt.
|
||||
|
@ -46,13 +46,13 @@ Im September 2012 wurde das Skript mit der Revisionsnummer 7132 von
|
|||
mitschriebwiki auf \href{https://github.com/MartinThoma/LaTeX-examples/blob/master/documents/Analysis%20III}{GitHub} hochgeladen.
|
||||
|
||||
\section*{Wo}
|
||||
Alle Kapitel inklusive \LaTeX-Quellen können unter
|
||||
\href{http://mitschriebwiki.nomeata.de}{mitschriebwiki.nomeata.de}
|
||||
Alle Kapitel inklusive \LaTeX-Quellen können unter
|
||||
\href{http://mitschriebwiki.nomeata.de}{mitschriebwiki.nomeata.de}
|
||||
abgerufen werden.
|
||||
Dort ist ein \emph{Wiki} eingerichtet und von Joachim Breitner um die
|
||||
Dort ist ein \emph{Wiki} eingerichtet und von Joachim Breitner um die
|
||||
\LaTeX-Funktionen erweitert.
|
||||
Das heißt, jeder kann Fehler nachbessern und sich an der Entwicklung
|
||||
beteiligen. Auf Wunsch ist auch ein Zugang über \emph{Subversion}
|
||||
beteiligen. Auf Wunsch ist auch ein Zugang über \emph{Subversion}
|
||||
möglich.
|
||||
|
||||
Oder man geht auf \href{https://github.com/MartinThoma/LaTeX-examples/blob/master/documents/Analysis%20III/}{github},
|
||||
|
@ -158,7 +158,7 @@ erstellt einen Fork und kann direkt Änderungen umsetzen.
|
|||
\textbf{§ 10: Der Satz von Fubini}: Jan Ihrens\\
|
||||
\textbf{§ 11: Der Transformationssatz}: Jan Ihrens, Rebecca Schwerdt\\
|
||||
\textbf{§ 12: Vorbereitungen für die Integralsätze}: Rebecca Schwerdt\\
|
||||
\textbf{§ 13: Der Integralsatz von Gauß\ im \(\mdr^{2}\)}: Benjamin Unger\\
|
||||
\textbf{§ 13: Der Integralsatz von Gauß\ im \(\mdr^{2}\)}: Benjamin Unger\\
|
||||
\textbf{§ 14: Flächen im \(\mdr^{3}\)}: Benjamin Unger\\
|
||||
\textbf{§ 15: Der Integralsatz von Stokes}: Philipp Ost\\
|
||||
\textbf{§ 16: \(\fl^{p}\)-Räume und \(\mathrm{L}^{p}\)-Räume}: Philipp Ost, Rebecca Schwerdt, Peter Pan, Jan Ihrens \\
|
||||
|
|
|
@ -1,17 +1,17 @@
|
|||
In diesem Kapitel seien $X,Y,Z$ Mengen ($\ne\emptyset$) und
|
||||
In diesem Kapitel seien $X,Y,Z$ Mengen ($\ne\emptyset$) und
|
||||
$f: X\to Y,\; g:Y\to Z$ Abbildungen.
|
||||
|
||||
\begin{enumerate}
|
||||
\index{Potenzmenge}
|
||||
\index{Disjunktheit}
|
||||
\item
|
||||
\item
|
||||
\begin{enumerate}
|
||||
\item $\mathcal{P}(X):=\{A:A\subseteq X\}$ heißt
|
||||
\item $\mathcal{P}(X):=\{A:A\subseteq X\}$ heißt
|
||||
\textbf{Potenzmenge} von $X$.
|
||||
\item Sei $\fm\subseteq\mathcal{P}(X)$, so heißt $\fm$
|
||||
\textbf{disjunkt}, genau dann wenn $A\cap B=\emptyset$
|
||||
\item Sei $\fm\subseteq\mathcal{P}(X)$, so heißt $\fm$
|
||||
\textbf{disjunkt}, genau dann wenn $A\cap B=\emptyset$
|
||||
für $A,B\in\fm$ mit $A\ne B$.
|
||||
\item Sei $(A_j)$ eine Folge in $\mathcal{P}(X)$ (also
|
||||
\item Sei $(A_j)$ eine Folge in $\mathcal{P}(X)$ (also
|
||||
$A_j\subseteq X$), so heißt $(A_j)$ \textbf{disjunkt},
|
||||
genau dann wenn $\{A_1,A_2,\dots\}$ disjunkt ist.\\
|
||||
\textbf{Schreibweise}:\\
|
||||
|
@ -22,16 +22,16 @@ $f: X\to Y,\; g:Y\to Z$ Abbildungen.
|
|||
\sum_{j=1}^\infty a_j &=: \sum a_j
|
||||
\end{align*}
|
||||
\end{enumerate}
|
||||
\item Sei $A\subseteq X$, $\mathds{1}_A : X \rightarrow R$
|
||||
\item Sei $A\subseteq X$, $\mathds{1}_A : X \rightarrow R$
|
||||
definiert durch:
|
||||
\[\mathds{1}_A(x):= \begin{cases}
|
||||
1 &\text{falls } x\in A\\
|
||||
0 &\text{falls } x\in A^c
|
||||
\end{cases}\]
|
||||
wobei $A^c:=X\setminus A$. $\mathds{1}_A$ heißt die
|
||||
\textbf{charakteristische Funktion} oder
|
||||
\textbf{charakteristische Funktion} oder
|
||||
\textbf{Indikatorfunktion von A}.
|
||||
\item Sei $B\subseteq Y$ dann ist $f^{-1}(B):=\{x\in X: f(x)\in B\}$
|
||||
\item Sei $B\subseteq Y$ dann ist $f^{-1}(B):=\{x\in X: f(x)\in B\}$
|
||||
und es gelten folgende Eigenschaften:
|
||||
\begin{enumerate}
|
||||
\item $f^{-1}(B^c)=f^{-1}(B)^c$
|
||||
|
@ -47,12 +47,12 @@ $f: X\to Y,\; g:Y\to Z$ Abbildungen.
|
|||
|
||||
\begin{definition}
|
||||
\index{offen}
|
||||
Sei $n \in \mdn$ und $\emptyset \neq X \subseteq \mdr^n$ und
|
||||
Sei $n \in \mdn$ und $\emptyset \neq X \subseteq \mdr^n$ und
|
||||
$A \subseteq X$.
|
||||
|
||||
$A$ heißt $\stackrel{\text{offen}}{\text{abgeschlossen}}$ in
|
||||
$X :\Leftrightarrow \exists B \subseteq \mdr^n$.
|
||||
$B$ ist $\stackrel{\text{offen}}{\text{abgeschlossen}}$ und
|
||||
$A$ heißt $\stackrel{\text{offen}}{\text{abgeschlossen}}$ in
|
||||
$X :\Leftrightarrow \exists B \subseteq \mdr^n$.
|
||||
$B$ ist $\stackrel{\text{offen}}{\text{abgeschlossen}}$ und
|
||||
$A = B \cap X$
|
||||
\end{definition}
|
||||
|
||||
|
@ -61,18 +61,18 @@ $f: X\to Y,\; g:Y\to Z$ Abbildungen.
|
|||
$f: X \rightarrow \mdr^n$.
|
||||
|
||||
\begin{enumerate}
|
||||
\item $A$ ist offen in $X \Leftrightarrow \forall x \in A$
|
||||
\item $A$ ist offen in $X \Leftrightarrow \forall x \in A$
|
||||
ex. eine Umgebung $U$ von $x$ mit $U \cap X \subseteq A$
|
||||
\item $A$ ist abgeschlossen in $X$\\
|
||||
$\Leftrightarrow X \setminus A$ ist offen in $X$\\
|
||||
$\Leftrightarrow$ für jede konvergente Folge $(a_k)$
|
||||
$\Leftrightarrow$ für jede konvergente Folge $(a_k)$
|
||||
in $A$ mit $\lim a_k \in X$ ist $\lim a_k \in A$
|
||||
\item Die folgenden Aussagen sind äquivalent:
|
||||
\begin{enumerate}
|
||||
\item $f \in C(X, \mdr^m)$
|
||||
\item für jede offene Menge $B \subseteq \mdr^m$ ist
|
||||
\item für jede offene Menge $B \subseteq \mdr^m$ ist
|
||||
$f^{-1}(B)$ offen in $X$
|
||||
\item für jede abgeschlossene Menge $B \subseteq \mdr^m$ ist
|
||||
\item für jede abgeschlossene Menge $B \subseteq \mdr^m$ ist
|
||||
$f^{-1}(B)$ abgeschlossen in $X$
|
||||
\end{enumerate}
|
||||
\end{enumerate}
|
||||
|
|
|
@ -2,7 +2,7 @@ In diesem Kapitel sei $X \neq \emptyset$ eine Menge.
|
|||
|
||||
\begin{definition}
|
||||
\index{$\sigma$-!Algebra}
|
||||
Sei $\fa\subseteq\mathcal{P}(X)$, $\fa$ heißt eine
|
||||
Sei $\fa\subseteq\mathcal{P}(X)$, $\fa$ heißt eine
|
||||
\textbf{$\sigma$-Algebra} auf $X$, wenn gilt:
|
||||
\begin{enumerate}
|
||||
\item[($\sigma_1$)] $X\in\fa$
|
||||
|
@ -14,11 +14,11 @@ In diesem Kapitel sei $X \neq \emptyset$ eine Menge.
|
|||
|
||||
\begin{beispieleX}
|
||||
\begin{enumerate}
|
||||
\item $\Set{X,\emptyset}$ und $\mathcal{P}(X)$ sind
|
||||
\item $\Set{X,\emptyset}$ und $\mathcal{P}(X)$ sind
|
||||
$\sigma$-Algebren auf $X$.
|
||||
\item Sei $A\subseteq X$, dann ist $\Set{X,\emptyset, A, A^c}$
|
||||
\item Sei $A\subseteq X$, dann ist $\Set{X,\emptyset, A, A^c}$
|
||||
eine $\sigma$-Algebra auf $X$.
|
||||
\item $\fa:=\Set{A\subseteq X | A \text{ abzählbar oder } A^c \text{ abzählbar}}$
|
||||
\item $\fa:=\Set{A\subseteq X | A \text{ abzählbar oder } A^c \text{ abzählbar}}$
|
||||
ist eine $\sigma$-Algebra auf $X$.
|
||||
\end{enumerate}
|
||||
\end{beispieleX}
|
||||
|
@ -41,13 +41,13 @@ Sei $\fa$ eine $\sigma$-Algebra auf $X$, dann:
|
|||
\begin{beweis}
|
||||
\begin{enumerate}
|
||||
\item \folgtnach{$\sigma_2$} $\emptyset=X^c\in\fa$.
|
||||
\item $D:=\bigcap A_j$. $D^c=\bigcup A_j^c\in\fa$ (nach
|
||||
($\sigma_2$) und ($\sigma_3$)), also gilt auch
|
||||
\item $D:=\bigcap A_j$. $D^c=\bigcup A_j^c\in\fa$ (nach
|
||||
($\sigma_2$) und ($\sigma_3$)), also gilt auch
|
||||
$D=(D^c)^c\in\fa$.
|
||||
\item \begin{enumerate}
|
||||
\item \folgtnach{($\sigma_3$) mit $A_{n+j}:=\emptyset$ ($j\ge 1$)}
|
||||
\item \folgtnach{($\sigma_3$) mit $A_{n+j}:=\emptyset$ ($j\ge 1$)}
|
||||
$A_1\cup\dots\cup A_n\in\fa$.
|
||||
\item \folgtnach{(2) mit $A_{n+j}:=X$ ($j\ge 1)$}
|
||||
\item \folgtnach{(2) mit $A_{n+j}:=X$ ($j\ge 1)$}
|
||||
$A_1\cap\dots\cap A_n\in\fa$.
|
||||
\item $A_1\setminus A_2=A_1\cap A_2^c\in\fa$
|
||||
\end{enumerate}
|
||||
|
@ -56,8 +56,8 @@ Sei $\fa$ eine $\sigma$-Algebra auf $X$, dann:
|
|||
|
||||
\begin{lemma}
|
||||
\label{Lemma 1.2}
|
||||
Sei $\cf \neq \emptyset$ eine Menge von $\sigma$-Algebren auf $X$.
|
||||
Dann ist
|
||||
Sei $\cf \neq \emptyset$ eine Menge von $\sigma$-Algebren auf $X$.
|
||||
Dann ist
|
||||
\[\fa_0:=\bigcap_{\fa\in\cf}\fa\]
|
||||
eine $\sigma$-Algebra auf $X$.
|
||||
\end{lemma}
|
||||
|
@ -70,7 +70,7 @@ Sei $\fa$ eine $\sigma$-Algebra auf $X$, dann:
|
|||
\forall\fa\in\cf:A\in\fa &\implies \forall\fa\in\cf:A^c\in\fa\\
|
||||
&\implies A^c\in\fa_0
|
||||
\end{align*}
|
||||
\item[($\sigma_3$)] Sei $(A_j)$ eine Folge in $\fa_0$, dann
|
||||
\item[($\sigma_3$)] Sei $(A_j)$ eine Folge in $\fa_0$, dann
|
||||
ist $(A_j)$ Folge in $\fa$ für alle $\fa\in\cf$, dann gilt:
|
||||
\begin{align*}
|
||||
\forall\fa\in\cf:\bigcap A_j\in\fa \implies \bigcap A_j\in\fa_0
|
||||
|
@ -80,14 +80,14 @@ Sei $\fa$ eine $\sigma$-Algebra auf $X$, dann:
|
|||
|
||||
\begin{definition}
|
||||
\index{Erzeuger}
|
||||
Sei $\emptyset \neq \mathcal{E} \subseteq \mathcal{P}(X)$ und
|
||||
$\cf:=\{\fa:\fa$ ist $\sigma$-Algebra auf $X$ mit
|
||||
Sei $\emptyset \neq \mathcal{E} \subseteq \mathcal{P}(X)$ und
|
||||
$\cf:=\{\fa:\fa$ ist $\sigma$-Algebra auf $X$ mit
|
||||
$\mathcal{E}\subseteq\fa\}$. Definiere
|
||||
\[\sigma(\mathcal{E}):=\bigcap_{\fa\in\cf}\fa\]
|
||||
\folgtnach{1.2} $\sigma(\mathcal{E})$ ist eine $\sigma$-Algebra
|
||||
auf $X$. $\sigma(\mathcal{E})$ heißt die
|
||||
\textbf{von $\mathcal{E}$ erzeugte $\sigma$-Algebra}.
|
||||
$\mathcal{E}$ heißt ein \textbf{Erzeuger} von
|
||||
\folgtnach{1.2} $\sigma(\mathcal{E})$ ist eine $\sigma$-Algebra
|
||||
auf $X$. $\sigma(\mathcal{E})$ heißt die
|
||||
\textbf{von $\mathcal{E}$ erzeugte $\sigma$-Algebra}.
|
||||
$\mathcal{E}$ heißt ein \textbf{Erzeuger} von
|
||||
$\sigma(\mathcal{E})$.
|
||||
\end{definition}
|
||||
|
||||
|
@ -95,12 +95,12 @@ Sei $\fa$ eine $\sigma$-Algebra auf $X$, dann:
|
|||
\label{Lemma 1.3}
|
||||
Sei $\emptyset\ne\mathcal{E}\subseteq\mathcal{P}(X)$.
|
||||
\begin{enumerate}
|
||||
\item $\mathcal{E}\subseteq\sigma(\mathcal{E})$.
|
||||
$\sigma(\mathcal{E})$ ist die "`kleinste"'
|
||||
\item $\mathcal{E}\subseteq\sigma(\mathcal{E})$.
|
||||
$\sigma(\mathcal{E})$ ist die "`kleinste"'
|
||||
$\sigma$-Algebra auf $X$, die $\mathcal{E}$ enthält.
|
||||
\item Ist $\mathcal{E}$ eine $\sigma$-Algebra, so ist
|
||||
\item Ist $\mathcal{E}$ eine $\sigma$-Algebra, so ist
|
||||
$\sigma(\mathcal{E})=\mathcal{E}$.
|
||||
\item Ist $\mathcal{E}\subseteq\mathcal{E}'$, so ist
|
||||
\item Ist $\mathcal{E}\subseteq\mathcal{E}'$, so ist
|
||||
$\sigma(\mathcal{E})\subseteq\sigma(\mathcal{E}')$.
|
||||
\end{enumerate}
|
||||
\end{lemma}
|
||||
|
@ -108,19 +108,19 @@ Sei $\fa$ eine $\sigma$-Algebra auf $X$, dann:
|
|||
\begin{beweis}
|
||||
\begin{enumerate}
|
||||
\item Klar nach Definition.
|
||||
\item $\fa:=\mathcal{E}$, dann gilt
|
||||
\item $\fa:=\mathcal{E}$, dann gilt
|
||||
$\fa\subseteq\sigma(\mathcal{E})\subseteq\fa$.
|
||||
\item $\mathcal{E}\subseteq\mathcal{E}'\subseteq\sigma(\mathcal{E}')$,
|
||||
also folgt nach Definition
|
||||
\item $\mathcal{E}\subseteq\mathcal{E}'\subseteq\sigma(\mathcal{E}')$,
|
||||
also folgt nach Definition
|
||||
$\sigma(\mathcal{E})\subseteq\sigma(\mathcal{E}')$.
|
||||
\end{enumerate}
|
||||
\end{beweis}
|
||||
|
||||
\begin{beispiel}
|
||||
\begin{enumerate}
|
||||
\item Sei $A\subseteq X$ und $\mathcal{E}:=\{A\}$. Dann ist
|
||||
\item Sei $A\subseteq X$ und $\mathcal{E}:=\{A\}$. Dann ist
|
||||
$\sigma(\mathcal{E})=\{X,\emptyset,A,A^c\}$.
|
||||
\item $X:=\{1,2,3,4,5\}, \mathcal{E}:=\{\{1\},\{1,2\}\}$.
|
||||
\item $X:=\{1,2,3,4,5\}, \mathcal{E}:=\{\{1\},\{1,2\}\}$.
|
||||
Dann gilt:
|
||||
\[\sigma(\mathcal{E}):=\{X,\emptyset, \{1\},\{2\},\{1,2\},\{3,4,5\},\{1,3,4,5\},\{2,3,4,5\}\}\]
|
||||
\end{enumerate}
|
||||
|
@ -128,11 +128,11 @@ Sei $\fa$ eine $\sigma$-Algebra auf $X$, dann:
|
|||
|
||||
\begin{erinnerung}
|
||||
\index{Offenheit}\index{Abgeschlossenheit}
|
||||
Sei $d\in\mdn, X\subseteq\mdr^d$. $A\subseteq X$ heißt
|
||||
\textbf{offen} (\textbf{abgeschlossen}) in $X$, genau dann wenn
|
||||
ein offenes (abgeschlossenes) $G\subseteq\mdr^d$ existiert mit
|
||||
Sei $d\in\mdn, X\subseteq\mdr^d$. $A\subseteq X$ heißt
|
||||
\textbf{offen} (\textbf{abgeschlossen}) in $X$, genau dann wenn
|
||||
ein offenes (abgeschlossenes) $G\subseteq\mdr^d$ existiert mit
|
||||
$A=X\cap G$.\\
|
||||
Beachte: $A$ abgeschlossen in $X$ $\iff$ $X\setminus A$ offen in
|
||||
Beachte: $A$ abgeschlossen in $X$ $\iff$ $X\setminus A$ offen in
|
||||
$X$.
|
||||
\end{erinnerung}
|
||||
|
||||
|
@ -142,27 +142,27 @@ Sei $\fa$ eine $\sigma$-Algebra auf $X$, dann:
|
|||
Sei $X\subseteq\mdr^d$.
|
||||
\begin{enumerate}
|
||||
\item $\mathcal{O}(X):=\Set{A\subseteq X | A \text{ ist offen in } X}$
|
||||
\item $\fb(X):=\sigma(\mathcal{O}(X))$ heißt
|
||||
\item $\fb(X):=\sigma(\mathcal{O}(X))$ heißt
|
||||
\textbf{Borelsche $\sigma$-Algebra} auf $X$.
|
||||
\item $\fb_d:=\fb(\mdr^d)$. Die Elemente von $\fb_d$ heißen
|
||||
\item $\fb_d:=\fb(\mdr^d)$. Die Elemente von $\fb_d$ heißen
|
||||
\textbf{Borelsche Mengen} oder \textbf{Borel-Mengen}.
|
||||
\end{enumerate}
|
||||
\end{definition}
|
||||
|
||||
\begin{beispiel}
|
||||
\begin{enumerate}
|
||||
\item Sei $\emptyset \neq X\subseteq\mdr^d$. Ist $A\subseteq$
|
||||
\item Sei $\emptyset \neq X\subseteq\mdr^d$. Ist $A\subseteq$
|
||||
$\stackrel{\hbox{offen}}{\hbox{abgeschlossen}}$
|
||||
in $X$, so ist $A\in\fb(X)$.
|
||||
\item Ist $A\subseteq\mdr^d$
|
||||
\item Ist $A\subseteq\mdr^d$
|
||||
$\stackrel{\hbox{offen}}{\hbox{abgeschlossen}}$,
|
||||
so ist $A\in\fb_d$.
|
||||
\item Sei $d=1, A=\mdq$. $\mdq$ ist abzählbar, also
|
||||
$\mdq=\{r_1,r_2,\dots\}$ (mit $r_i\ne r_j$ für $i\ne j$).
|
||||
Also ist $\mdq=\bigcup \{r_j\}$. Sei nun $r\in\mdq$,
|
||||
dann ist $B:=(-\infty,r)\cup(r,\infty)\in\fb_1$. Daraus
|
||||
\item Sei $d=1, A=\mdq$. $\mdq$ ist abzählbar, also
|
||||
$\mdq=\{r_1,r_2,\dots\}$ (mit $r_i\ne r_j$ für $i\ne j$).
|
||||
Also ist $\mdq=\bigcup \{r_j\}$. Sei nun $r\in\mdq$,
|
||||
dann ist $B:=(-\infty,r)\cup(r,\infty)\in\fb_1$. Daraus
|
||||
folgt $\{r_j\}\in\fb_1$, also auch $\mdq\in\fb_1$.\\
|
||||
Allgemeiner lässt sich zeigen:
|
||||
Allgemeiner lässt sich zeigen:
|
||||
$\mdq^d:=\{(x_1,\dots,x_n):x_j\in\mdq (j=1,\dots,n)\}\in\fb_d$.
|
||||
\item Sei $x_0 \in \mdr^d, \Set{x_0}$ ist abgeschlossen
|
||||
$\Rightarrow \Set{x_0} \in \fb$
|
||||
|
@ -173,8 +173,8 @@ Sei $\fa$ eine $\sigma$-Algebra auf $X$, dann:
|
|||
\index{Intervall}
|
||||
\index{Halbraum}
|
||||
\begin{enumerate}
|
||||
\item Seien $I_1,\dots,I_d$ Intervalle in $\mdr$.
|
||||
Dann heißt $I_1\times\dots\times I_d$ ein \textbf{Intervall}
|
||||
\item Seien $I_1,\dots,I_d$ Intervalle in $\mdr$.
|
||||
Dann heißt $I_1\times\dots\times I_d$ ein \textbf{Intervall}
|
||||
in $\mdr^d$.
|
||||
\item Seien $a=(a_1,\dots,a_d), b=(b_1,\dots,b_d)\in\mdr^d$.
|
||||
\[a\le b:\iff a_j\le b_j \quad \forall j \in \Set{1, \dots, d}\]
|
||||
|
@ -185,9 +185,9 @@ Sei $\fa$ eine $\sigma$-Algebra auf $X$, dann:
|
|||
[a,b) &:= [a_1,b_1)\times[a_2,b_2)\times\dots\times[a_d,b_d)\\
|
||||
[a,b] &:= [a_1,b_1]\times[a_2,b_2]\times\dots\times[a_d,b_d]
|
||||
\end{align*}
|
||||
mit der Festlegung $(a,b):=(a,b]:=[a,b):=\emptyset$, falls
|
||||
mit der Festlegung $(a,b):=(a,b]:=[a,b):=\emptyset$, falls
|
||||
$a_j=b_j$ für ein $j\in\{1,\dots,d\}$.
|
||||
\item Für $k\in\{1,\dots,d\}$ und $\alpha\in\mdr$ definiere die
|
||||
\item Für $k\in\{1,\dots,d\}$ und $\alpha\in\mdr$ definiere die
|
||||
folgenden \textbf{Halbräume}:
|
||||
\begin{align*}
|
||||
H_k^-(\alpha) &:= \Set{(x_1,\dots,x_d)\in\mdr^d:x_k\le\alpha}\\
|
||||
|
@ -211,7 +211,7 @@ die beiden Halbräume:\\
|
|||
|
||||
\fill[green!15] (a) -- (b) -- (c) -- (d) -- (a);
|
||||
|
||||
% Draw lines indicating intersection with y and x axis. Here we
|
||||
% Draw lines indicating intersection with y and x axis. Here we
|
||||
% use the perpendicular coordinate system
|
||||
\draw[dotted] (yaxis |- a) node[left] {$a_2$}
|
||||
-| (xaxis -| a) node[below] {$a_1$};
|
||||
|
@ -274,10 +274,10 @@ Entsprechendes gilt für die anderen Typen von Intervallen und Halbräumen.
|
|||
\end{satz}
|
||||
|
||||
\begin{beweis}
|
||||
\[\fb_d
|
||||
\stackrel{(1)}{\subseteq} \sigma(\ce_1)
|
||||
\stackrel{(2)}{\subseteq} \sigma(\ce_2)
|
||||
\stackrel{(3)}{\subseteq} \sigma(\ce_3)
|
||||
\[\fb_d
|
||||
\stackrel{(1)}{\subseteq} \sigma(\ce_1)
|
||||
\stackrel{(2)}{\subseteq} \sigma(\ce_2)
|
||||
\stackrel{(3)}{\subseteq} \sigma(\ce_3)
|
||||
\stackrel{(4)}{\subseteq} \fb_d
|
||||
\]
|
||||
\begin{enumerate}
|
||||
|
@ -293,57 +293,57 @@ Entsprechendes gilt für die anderen Typen von Intervallen und Halbräumen.
|
|||
\[\exists N\in\mdn:\forall n\ge N: \forall j\in\{1,\dots,d\}:a_j<b_j-\frac1n\]
|
||||
Definiere $c_n:=(\frac1n,\dots,\frac1n)\in\mdq^d$. Dann gilt:
|
||||
\[(a,b)=\bigcup_{n\ge N}(a,b-c_n]\in\sigma(\ce_2)\]
|
||||
Also auch $\ce_1\subseteq\sigma(\ce_2)$ und damit
|
||||
Also auch $\ce_1\subseteq\sigma(\ce_2)$ und damit
|
||||
$\sigma(\ce_1)\subseteq\sigma(\ce_2)$.
|
||||
\item Seien $a = (a_1,\dots,a_d), b=(b_1,\dots,b_d) \in \mdq^d$
|
||||
mit $a \leq b$.
|
||||
\item Seien $a = (a_1,\dots,a_d), b=(b_1,\dots,b_d) \in \mdq^d$
|
||||
mit $a \leq b$.
|
||||
Nachrechnen:
|
||||
\[(a,b] = \bigcap_{k=1}^d (H^-_k(b_k) \cap H^-_k(a_k)^c) \in \sigma(\ce_3). \]
|
||||
Das heißt $\ce_2 \subseteq \sigma(\ce_3)$ und damit auch
|
||||
$\sigma(\ce_2) \subseteq \sigma(\ce_3)$.
|
||||
\item $H^-_k(\alpha)$ ist abgeschlossen, somit ist
|
||||
$H^-_k(\alpha)^c$ offen und damit $H^-_k(\alpha)^c \in \fb_d$,
|
||||
also auch $H^-_k(\alpha) \in \fb_d$. Damit ist
|
||||
$\ce_3 \subseteq \fb_d \implies \sigma(\ce_3) \subseteq \fb_d$.
|
||||
Das heißt $\ce_2 \subseteq \sigma(\ce_3)$ und damit auch
|
||||
$\sigma(\ce_2) \subseteq \sigma(\ce_3)$.
|
||||
\item $H^-_k(\alpha)$ ist abgeschlossen, somit ist
|
||||
$H^-_k(\alpha)^c$ offen und damit $H^-_k(\alpha)^c \in \fb_d$,
|
||||
also auch $H^-_k(\alpha) \in \fb_d$. Damit ist
|
||||
$\ce_3 \subseteq \fb_d \implies \sigma(\ce_3) \subseteq \fb_d$.
|
||||
\end{enumerate}
|
||||
\end{beweis}
|
||||
|
||||
\begin{definition}
|
||||
\index{Spur}
|
||||
Sei $\emptyset \neq \fm \subseteq \mathcal{P}(X)$ und
|
||||
$\emptyset \neq Y \subseteq X$.
|
||||
\[\fm_Y := \{A \cap Y : A \in \fm\}\]
|
||||
Sei $\emptyset \neq \fm \subseteq \mathcal{P}(X)$ und
|
||||
$\emptyset \neq Y \subseteq X$.
|
||||
\[\fm_Y := \{A \cap Y : A \in \fm\}\]
|
||||
heißt die \textbf{Spur von $\fm$ in $Y$}.
|
||||
\end{definition}
|
||||
|
||||
\begin{beispiel}
|
||||
$X = \mdr^d, \fm \subseteq \sigma(\mdr^d), \; Y \subseteq X$.
|
||||
$X = \mdr^d, \fm \subseteq \sigma(\mdr^d), \; Y \subseteq X$.
|
||||
Dann: $(\co(\mdr^d))_Y = \sigma(Y)$
|
||||
\end{beispiel}
|
||||
|
||||
\begin{satz}[Spuren und $\sigma$-Algebren]
|
||||
\label{Satz 1.5}
|
||||
Sei $\emptyset \neq Y \subseteq X$ und $\fa$ eine
|
||||
Sei $\emptyset \neq Y \subseteq X$ und $\fa$ eine
|
||||
$\sigma$-Algebra auf $X$.
|
||||
\begin{enumerate}
|
||||
\item $\fa_Y$ ist eine $\sigma$-Algebra auf $Y$.
|
||||
\item $\fa_Y \subseteq \fa \iff Y \in \fa$
|
||||
\item Ist $\emptyset \neq \ce \subseteq \mathcal{P}(X)$, so
|
||||
\item Ist $\emptyset \neq \ce \subseteq \mathcal{P}(X)$, so
|
||||
ist $\sigma(\ce_Y) = \sigma(\ce)_Y$.
|
||||
\end{enumerate}
|
||||
\end{satz}
|
||||
|
||||
\begin{beweis}
|
||||
\begin{enumerate}
|
||||
\item
|
||||
\item
|
||||
\begin{enumerate}
|
||||
\item[($\sigma_1$)] Es ist $Y=Y\cap X\in\fa_Y$, da $X\in\fa$.
|
||||
\item[($\sigma_2$)] Sei $B\in\fa_Y$, dann existiert ein
|
||||
\item[($\sigma_2$)] Sei $B\in\fa_Y$, dann existiert ein
|
||||
$A\in\fa$ mit $B=A\cap Y$.\\
|
||||
Also ist
|
||||
Also ist
|
||||
$Y\setminus B=\overbrace{(X\setminus A)}^{\in\fa} \cap Y\in\fa_Y$.
|
||||
\item[($\sigma_3$)] Sei $(B_j)$ eine Folge in $\fa_Y$, dann
|
||||
existiert eine Folge $(A_j)\in\fa^\mdn$
|
||||
\item[($\sigma_3$)] Sei $(B_j)$ eine Folge in $\fa_Y$, dann
|
||||
existiert eine Folge $(A_j)\in\fa^\mdn$
|
||||
mit $B_j=A_j\cap Y$. Es gilt:
|
||||
\[\bigcup B_j=\bigcup(A_j\cap Y)=(\bigcup A_j)\cap Y\in\fa_Y\]
|
||||
\end{enumerate}
|
||||
|
@ -392,10 +392,10 @@ Außerdem sei $[0,+\infty]:=[0,\infty)\cup\{+\infty\}$.
|
|||
\begin{cases}
|
||||
\exists n \in \mdn \text{ mit } a_n = +\infty \text{ oder }\\
|
||||
\sum a_n \text{ divergiert}
|
||||
\end{cases}
|
||||
\end{cases}
|
||||
\]
|
||||
\end{enumerate}
|
||||
Wegen Ana I, 13.1 können Reihen der obigen Form beliebig umgeordnet
|
||||
\end{enumerate}
|
||||
Wegen Ana I, 13.1 können Reihen der obigen Form beliebig umgeordnet
|
||||
werden, ohne dass sich ihr Wert verändert.
|
||||
\end{definition}
|
||||
|
||||
|
@ -405,13 +405,13 @@ werden, ohne dass sich ihr Wert verändert.
|
|||
\index{Maßraum}
|
||||
\index{Maß!endliches}
|
||||
\index{Wahrscheinlichkeitsmaß}\index{Maß!Wahrscheinlichkeits-}
|
||||
Sei $\fa$ eine $\sigma$-Algebra auf $X$ und $\mu:\fa\to[0,+\infty]$
|
||||
eine Abbildung. $\mu$ heißt ein \textbf{Maß} auf $\fa$, genau dann
|
||||
Sei $\fa$ eine $\sigma$-Algebra auf $X$ und $\mu:\fa\to[0,+\infty]$
|
||||
eine Abbildung. $\mu$ heißt ein \textbf{Maß} auf $\fa$, genau dann
|
||||
wenn gilt:
|
||||
\begin{enumerate}
|
||||
\item[$(M_1)$] $\mu(\emptyset)=0$
|
||||
\item[$(M_2)$] Ist $(A_j)$ eine disjunkte Folge in $\fa$, so ist
|
||||
$\mu(\bigcup A_j)=\sum\mu(A_j)$. Diese Eigenschaft heißt
|
||||
\item[$(M_2)$] Ist $(A_j)$ eine disjunkte Folge in $\fa$, so ist
|
||||
$\mu(\bigcup A_j)=\sum\mu(A_j)$. Diese Eigenschaft heißt
|
||||
\textbf{$\sigma$-Additivität}.
|
||||
\end{enumerate}
|
||||
In diesem Fall heißt $(X,\fa,\mu)$ ein \textbf{Maßraum}.\\
|
||||
|
@ -424,7 +424,7 @@ Ein Maß $\mu$ heißt ein \textbf{Wahrscheinlichkeitsmaß} $:\Leftrightarrow\mu(
|
|||
\index{Dirac-Maß}\index{Maß!Dirac-}
|
||||
\index{Zählmaß}\index{Maß!Zähl-}
|
||||
\begin{enumerate}
|
||||
\item Sei $\fa:=\cp(X)$ und $x_0\in X$.
|
||||
\item Sei $\fa:=\cp(X)$ und $x_0\in X$.
|
||||
$\delta_{x_0}:\fa\to[0,+\infty]$ sei definiert durch:
|
||||
\[\delta_{x_0}(A):=
|
||||
\begin{cases}
|
||||
|
@ -438,36 +438,36 @@ Ein Maß $\mu$ heißt ein \textbf{Wahrscheinlichkeitsmaß} $:\Leftrightarrow\mu(
|
|||
1,\ x_0\in\bigcup A_j\\
|
||||
0,\ x_0\not\in\bigcup A_j
|
||||
\end{cases}\right\}=\sum\delta_{x_0}(A_j)\]
|
||||
$\delta_{x_0}$ ist ein Maß auf $\fa$ und heißt
|
||||
$\delta_{x_0}$ ist ein Maß auf $\fa$ und heißt
|
||||
\textbf{Punktmaß} oder \textbf{Dirac-Maß}.
|
||||
\item Sei $X:=\mdn$, $\fa:=\cp(X)$ und $(p_j)$ eine Folge in
|
||||
\item Sei $X:=\mdn$, $\fa:=\cp(X)$ und $(p_j)$ eine Folge in
|
||||
$[0,+\infty]$. Definiere $\mu:\fa\to[0,+\infty]$ durch:
|
||||
\begin{align*}
|
||||
\text{Für } A \in \fa: \quad
|
||||
\text{Für } A \in \fa: \quad
|
||||
\mu(A):=
|
||||
\begin{cases}
|
||||
0 &\text{, falls } A=\emptyset\\
|
||||
\sum_{j\in A}p_j &\text{, falls } A\ne\emptyset
|
||||
\end{cases}
|
||||
\end{align*}
|
||||
Übung: $\mu$ ist ein Maß auf $\fa=\cp(\mdn)$ und heißt ein \textbf{Zählmaß}.
|
||||
Sind alle $p_j=1$, so ist $\mu(A)$ gerade die Anzahl der
|
||||
Übung: $\mu$ ist ein Maß auf $\fa=\cp(\mdn)$ und heißt ein \textbf{Zählmaß}.
|
||||
Sind alle $p_j=1$, so ist $\mu(A)$ gerade die Anzahl der
|
||||
Elemente von $A$.
|
||||
\item Sei $(X,\fa,\mu)$ ein Maßraum, $\emptyset\ne Y\subseteq X$
|
||||
und $\fa_0\subseteq\fa$ eine $\sigma$-Algebra auf $Y$.
|
||||
Definiere $\mu_0:\fa_0\to[0,+\infty]$ durch
|
||||
\item Sei $(X,\fa,\mu)$ ein Maßraum, $\emptyset\ne Y\subseteq X$
|
||||
und $\fa_0\subseteq\fa$ eine $\sigma$-Algebra auf $Y$.
|
||||
Definiere $\mu_0:\fa_0\to[0,+\infty]$ durch
|
||||
$\mu_0(A):=\mu(A)$ ($A\in\fa_0$).\\
|
||||
Dann ist
|
||||
Dann ist
|
||||
$(Y,\fa_0,\mu_0)$ ein Maßraum.\\
|
||||
Ist spezieller $Y\in\fa$, so ist $\fa_0:=\fa_Y\subseteq\fa$
|
||||
und man definiert $\mu_{|Y}:\fa_Y\to[0,+\infty]$ durch
|
||||
Ist spezieller $Y\in\fa$, so ist $\fa_0:=\fa_Y\subseteq\fa$
|
||||
und man definiert $\mu_{|Y}:\fa_Y\to[0,+\infty]$ durch
|
||||
$\mu_{|Y}(A):=\mu(A)$ ist ein Maß auf $\fa_Y$.
|
||||
\end{enumerate}
|
||||
\end{beispiel}
|
||||
|
||||
\begin{satz}
|
||||
\label{Satz 1.7}
|
||||
\((X,\fa,\mu)\) sei ein Maßraum, es seien \(A,B\in\fa\) und
|
||||
\((X,\fa,\mu)\) sei ein Maßraum, es seien \(A,B\in\fa\) und
|
||||
\((A_{j})\) sei eine Folge in \(\fa\). Dann:
|
||||
\begin{enumerate}
|
||||
\item \(A\subseteq B\,\implies\,\mu(A)\leq\mu(B)\)
|
||||
|
@ -481,7 +481,7 @@ Ein Maß $\mu$ heißt ein \textbf{Wahrscheinlichkeitsmaß} $:\Leftrightarrow\mu(
|
|||
\end{satz}
|
||||
\begin{beweis}
|
||||
\begin{enumerate}
|
||||
% Eigentlich muesste es in folgender Zeile statt B=(B\setminus A)\cup A korrekt
|
||||
% Eigentlich muesste es in folgender Zeile statt B=(B\setminus A)\cup A korrekt
|
||||
% heissen: B=(B\setminus A)\cupdot A -- Spaeter
|
||||
\item[(1)-(3)] \(B=(B\setminus A)\cup A\). Dann: \(\mu(B)=\underbrace{\mu(B\setminus A)}_{\geq0}+\mu(A)\geq\mu(A)\)
|
||||
\item[(4)] % Das muesste jetzt eigentlich Punkt 4 sein
|
||||
|
|
|
@ -7,7 +7,7 @@ Für \(x\in\mdr^k\):
|
|||
\[F(x):=\int_{\mdr^l}f(x,y)\,dy=\int_{\mdr^l}f^x(y)\,dy\]
|
||||
Für \(y\in\mdr^l\):
|
||||
\[G(y):=\int_{\mdr^k}f(x,y)\,dx=\int_{\mdr^k}f_y(x)\,dx\]
|
||||
Dann sind $F,G$ messbar und
|
||||
Dann sind $F,G$ messbar und
|
||||
\[\int_{\mdr^d}f(z)\,dz=\int_{\mdr^k}F(x)\,dx=\int_{\mdr^l}G(y)\,dy\]
|
||||
also
|
||||
\begin{align*}
|
||||
|
@ -26,7 +26,7 @@ Für \(x\in\mdr^k\) und \(\natn\) gilt:
|
|||
und nach Fall 2 ist \(F_n\) messbar. \\
|
||||
Aus \(0\leq f_n\leq f_{n+1}\) folgt \(0\leq F_n\leq F_{n+1}\) und \ref{Satz 4.6} liefert \(F_n\to F\) auf \(\mdr^k\). Dann gilt
|
||||
\[\int_{\mdr^d}f(z)\,dz = \lim \int_{\mdr^d}f_n(z)\,dz \overset{Fall 2}= \lim \int_{\mdr^k}F_n(x)\,dx \overset{\ref{Satz 4.6}}=\int_{\mdr^k}F(x)\,dx\]
|
||||
Genauso zeigt man
|
||||
Genauso zeigt man
|
||||
\[\int_{\mdr^d}(f(z)\,dz=\int_{\mdr^l}G(y)\,dy\]
|
||||
\end{beweis}
|
||||
|
||||
|
@ -35,7 +35,7 @@ Genauso zeigt man
|
|||
Es sei \(f\colon\mdr^d\to\imdr\) integrierbar. Dann existieren Nullmengen \(M\subseteq\mdr^k\) und \(N\subseteq\mdr^l\) mit
|
||||
\begin{align*}
|
||||
f^x\colon\mdr^l\to\imdr \text{ ist integrierbar für jedes } x\in\mdr^k\setminus M \\
|
||||
f_y\colon\mdr^k\to\imdr \text{ ist integrierbar für jedes } y\in\mdr^l\setminus N
|
||||
f_y\colon\mdr^k\to\imdr \text{ ist integrierbar für jedes } y\in\mdr^l\setminus N
|
||||
\end{align*}
|
||||
Setze
|
||||
\begin{align*}
|
||||
|
@ -60,22 +60,22 @@ Es gilt also wieder \((\ast)\) aus \ref{Satz 10.1}.
|
|||
Wir zeigen nur die Aussagen über \(f^x\), $F$ und die erste der obigen beiden Gleichungen. Genauso zeigt man die Aussagen über \(f_n, G\) und die zweite Gleichung.\\
|
||||
Aus \ref{Lemma 8.1} folgt, dass \(f^x\) messbar ist. Definiere
|
||||
\begin{align*}
|
||||
\Phi(x) := \int_{\mdr^l}\lvert f^x(y)\rvert\,dy
|
||||
= \int_{\mdr^l}\lvert f(x,y)\rvert\,dy \ \text{ für } x\in\mdr^k
|
||||
\Phi(x) := \int_{\mdr^l}\lvert f^x(y)\rvert\,dy
|
||||
= \int_{\mdr^l}\lvert f(x,y)\rvert\,dy \ \text{ für } x\in\mdr^k
|
||||
\end{align*}
|
||||
Nach \ref{Satz 10.1} ist \(\Phi\) messbar und
|
||||
Nach \ref{Satz 10.1} ist \(\Phi\) messbar und
|
||||
\begin{align*}
|
||||
\int_{\mdr^k}\Phi(x)\,dx
|
||||
= \int_{\mdr^k}\left(\int_{\mdr^l}\lvert f(x,y)\rvert\,dy\right)dx \overset{\ref{Satz 10.1}}
|
||||
= \int_{\mdr^d}\lvert f(z)\rvert\,dz
|
||||
\int_{\mdr^k}\Phi(x)\,dx
|
||||
= \int_{\mdr^k}\left(\int_{\mdr^l}\lvert f(x,y)\rvert\,dy\right)dx \overset{\ref{Satz 10.1}}
|
||||
= \int_{\mdr^d}\lvert f(z)\rvert\,dz
|
||||
< \infty
|
||||
\end{align*}
|
||||
(denn mit $f$ ist nach \ref{Satz 4.9} auch \(\lvert f\rvert\) integrierbar). Somit ist \(\Phi\) integrierbar.
|
||||
Setze \(M:=\{\Phi = \infty \}\) was nach \ref{Satz 4.10} eine Nullmenge ist.
|
||||
Setze \(M:=\{\Phi = \infty \}\) was nach \ref{Satz 4.10} eine Nullmenge ist.
|
||||
Also gilt:
|
||||
\begin{align*}
|
||||
\int_{\mdr^l}\lvert f^x(y)\rvert\,dy
|
||||
= \Phi(x) < \infty \ \text{ für jedes } x\in\mdr^k\setminus M
|
||||
\int_{\mdr^l}\lvert f^x(y)\rvert\,dy
|
||||
= \Phi(x) < \infty \ \text{ für jedes } x\in\mdr^k\setminus M
|
||||
\end{align*}
|
||||
Das heißt, \(\lvert f^x\rvert\) ist für jedes \(x\in\mdr^k\setminus M\) integrierbar und es gilt nach \ref{Satz 4.9} auch
|
||||
\begin{align*}
|
||||
|
@ -90,22 +90,22 @@ Setze
|
|||
0 &\text{, falls } z\in M\times\mdr^l
|
||||
\end{cases}
|
||||
\end{align*}
|
||||
Aus \ref{Lemma 9.3} folgt, dass \(\tilde f\) messbar ist. Klar ist, dass fast überall \(f=\tilde f\) gilt. Es ist
|
||||
Aus \ref{Lemma 9.3} folgt, dass \(\tilde f\) messbar ist. Klar ist, dass fast überall \(f=\tilde f\) gilt. Es ist
|
||||
\[\tilde f^x = \left(\mathds{1}_{(M\times\mdr^l)^C}\cdot f\right)^x\]
|
||||
Das heißt \(\tilde f^x\) ist integrierbar für jedes \(x\in\mdr^k\). Dann gilt
|
||||
\begin{align*}
|
||||
F(x) \overset{\ref{Satz 5.3}}
|
||||
= \int_{\mdr^l}\tilde f(x,y)\,dy
|
||||
= \underbrace{\int_{\mdr^l}\tilde f_+ (x,y)\,dy}_{=:F^+(x)} - \underbrace{\int_{\mdr^l}\tilde f_- (x,y)\,dy}_{=:F^-(x)}
|
||||
F(x) \overset{\ref{Satz 5.3}}
|
||||
= \int_{\mdr^l}\tilde f(x,y)\,dy
|
||||
= \underbrace{\int_{\mdr^l}\tilde f_+ (x,y)\,dy}_{=:F^+(x)} - \underbrace{\int_{\mdr^l}\tilde f_- (x,y)\,dy}_{=:F^-(x)}
|
||||
\end{align*}
|
||||
Nach \ref{Satz 10.1} sind \(F^+\) und \(F^-\) messbar. Die Dreiecksungleichung liefert nun
|
||||
\begin{align*}
|
||||
\lvert F(x)\rvert
|
||||
\leq \int_{\mdr^l}\lvert \tilde f(x,y)\rvert\,dy
|
||||
\overset{\ref{Satz 5.3}}= \int_{\mdr^l}\lvert f(x,y)\rvert\,dy
|
||||
\lvert F(x)\rvert
|
||||
\leq \int_{\mdr^l}\lvert \tilde f(x,y)\rvert\,dy
|
||||
\overset{\ref{Satz 5.3}}= \int_{\mdr^l}\lvert f(x,y)\rvert\,dy
|
||||
= \Phi(x) \ \text{ für } x\in\mdr^k
|
||||
\end{align*}
|
||||
Also ist \(\lvert F\rvert\leq\Phi\) und \(\Phi\) ist integrierbar. Aus \ref{Satz 4.9} folgt, dass $F$ und \(\lvert F\rvert\) integrierbar sind
|
||||
Also ist \(\lvert F\rvert\leq\Phi\) und \(\Phi\) ist integrierbar. Aus \ref{Satz 4.9} folgt, dass $F$ und \(\lvert F\rvert\) integrierbar sind
|
||||
und dann sind auch \(F^+\) und \(F^-\) integrierbar (zur Übung). Es folgt
|
||||
\begin{align*}
|
||||
\int_{\mdr^k}F(x)\,dx
|
||||
|
@ -138,15 +138,15 @@ Gegeben: \(\emptyset\neq D\subseteq\fb_d\) und messbares \(f\colon D\to\imdr\).
|
|||
Setze $f$ auf \(\mdr^d\) zu einer messbaren Funktion \(\tilde f\) fort (zum Beispiel wie in \ref{Lemma 9.3}).
|
||||
Aus \ref{Satz 3.8} folgt dann, dass \(\mathds{1}_{D}\tilde f\) messbar ist und \ref{Satz 10.1} liefert
|
||||
\begin{align*}
|
||||
\int_{\mdr^d}\lvert \mathds{1}_{D}\tilde f\rvert\,dz
|
||||
= \int_{\mdr^k}\left(\int_{\mdr^l}\lvert \mathds{1}_{D}\tilde f\rvert\,dy\right)dx
|
||||
\int_{\mdr^d}\lvert \mathds{1}_{D}\tilde f\rvert\,dz
|
||||
= \int_{\mdr^k}\left(\int_{\mdr^l}\lvert \mathds{1}_{D}\tilde f\rvert\,dy\right)dx
|
||||
= \int_{\mdr^l}\left(\int_{\mdr^k}\lvert \mathds{1}_{D}\tilde f\rvert\,dx\right)dy
|
||||
\end{align*}
|
||||
Ist eines der drei obigen Integrale endlich, so ist \(\lvert \mathds{1}_{D}\tilde f\rvert\) integrierbar und
|
||||
damit ist nach \ref{Satz 4.9} auch \(\mathds{1}_{D}\tilde f\) integrierbar.\\
|
||||
Dann ist $f$ integrierbar und es folgt
|
||||
\begin{align*}
|
||||
\int_Df(z)\,dz
|
||||
\int_Df(z)\,dz
|
||||
& = \int_{\mdr^d}\left(\mathds{1}_{D}\tilde f\right)(z)\,dz \\
|
||||
& \overset{\ref{Satz 10.2}}= \int_{\mdr^k}\left(\int_{\mdr^l}\left(\mathds{1}_{D}\tilde f\right)(x,y)\,dy\right)dx \\
|
||||
& = \int_{\mdr^l}\left(\int_{\mdr^k}\left(\mathds{1}_{D}\tilde f\right)(x,y)\,dx\right)dy
|
||||
|
@ -154,11 +154,11 @@ Dann ist $f$ integrierbar und es folgt
|
|||
|
||||
\begin{beispiel}
|
||||
\begin{enumerate}
|
||||
\item Sei \(D=[a_1,b_1]\times[a_2,b_2]\times\dots\times[a_d,b_d]\) mit \(a_i\leq b_i \ (i=1,\dots,d)\).
|
||||
\item Sei \(D=[a_1,b_1]\times[a_2,b_2]\times\dots\times[a_d,b_d]\) mit \(a_i\leq b_i \ (i=1,\dots,d)\).
|
||||
Es sei \(f\colon D\to\mdr\) stetig. $D$ ist kompakt, also gilt \(D\in\fb_d\).
|
||||
Nach \ref{Satz 4.12}(2) ist \(f\in\mathfrak{L}^1(D)\) und aus obiger Bemerkung folgt
|
||||
\begin{align*}
|
||||
\int_Df(x_1,\dots,x_d)\,d(x_1,\dots,x_d)
|
||||
\int_Df(x_1,\dots,x_d)\,d(x_1,\dots,x_d)
|
||||
= \int_{a_d}^{b^d} \left(\dots \left( \int_{a_2}^{b^2} \left(\int_{a_1}^{b^1}f(x_1,\dots,x_d)\,dx_1\right)dx_2\right)\dots\right)dx_d
|
||||
\end{align*}
|
||||
Die Reihenfolge der Integrationen darf beliebig vertauscht werden. Aus \ref{Satz 4.13} folgt
|
||||
|
@ -167,7 +167,7 @@ Die Reihenfolge der Integrationen darf beliebig vertauscht werden. Aus \ref{Satz
|
|||
\textbf{Konkretes Beispiel}\\
|
||||
Sei \(D:=[a,b]\times[c,d]\subseteq\mdr^2\), \(f\in C([a,b])\) und \(g\in C([c,d])\).
|
||||
\begin{align*}
|
||||
\int_Df(x)g(y)\,d(x,y)
|
||||
\int_Df(x)g(y)\,d(x,y)
|
||||
& = \int_c^d\left(\int_a^bf(x)g(y)\,dx\right)dy \\
|
||||
& = \int_c^d\left(g(y)\left(\int_a^bf(x)\,dx\right)\right)dy \\
|
||||
&= \left(\int_a^bf(x)\,dx\right) \left(\int_c^dg(y)\,dy\right)
|
||||
|
@ -175,7 +175,7 @@ Sei \(D:=[a,b]\times[c,d]\subseteq\mdr^2\), \(f\in C([a,b])\) und \(g\in C([c,d
|
|||
\item
|
||||
Wir rechtfertigen die "'Kochrezepte"' aus Analysis II, Paragraph 15.
|
||||
Seien \(a,b\in\mdr\) mit \(a<b\) und \(I:=[a,b]\). Weiter seien
|
||||
\(h_1,h_2\in C(I)\) mit \(h_1\leq h_2\) auf \(I\) und
|
||||
\(h_1,h_2\in C(I)\) mit \(h_1\leq h_2\) auf \(I\) und
|
||||
\[A:=\{(x,y)\in\mdr^2: x\in I, h_1(x)\leq y\leq h_2(x)\}\]
|
||||
Sei \(f\colon A\to\mdr\) stetig. Da \(h_1\) und \(h_2\) stetig
|
||||
sind, ist \(A\) kompakt und somit gilt \(A\in\fb_2\). Aus
|
||||
|
@ -187,15 +187,15 @@ Sei \(D:=[a,b]\times[c,d]\subseteq\mdr^2\), \(f\in C([a,b])\) und \(g\in C([c,d
|
|||
0 &\text{, falls } (x,y)\notin A
|
||||
\end{cases}
|
||||
\]
|
||||
Nach \ref{Lemma 9.3} ist \(\tilde f\) messbar. Setze
|
||||
Nach \ref{Lemma 9.3} ist \(\tilde f\) messbar. Setze
|
||||
\[M:=\max\{\lvert f(x,y)\rvert:(x,y)\in A\}\]
|
||||
Dann gilt \(\lvert\tilde f\rvert \leq M\cdot\mathds{1}_A\).
|
||||
Wegen \(\lambda_2(A)<\infty\) ist \(M\cdot\mathds{1}_A\)
|
||||
Wegen \(\lambda_2(A)<\infty\) ist \(M\cdot\mathds{1}_A\)
|
||||
integrierbar und nach \ref{Satz 4.9} ist \(\lvert\tilde f\rvert\)
|
||||
und damit auch \(\tilde f\) integrierbar. Dann ist
|
||||
\begin{align*}
|
||||
\int_A f(x,y)\,d(x,y) &= \int_{\mdr^2}\tilde f(x,y)\,d(x,y) \\
|
||||
& \overset{\ref{Satz 10.3}}=
|
||||
& \overset{\ref{Satz 10.3}}=
|
||||
\int_\mdr\left(\int_\mdr\tilde f (x,y)\,dy\right)dx \\
|
||||
&=\int_a^b\left(\int^{h_2(x)}_{h_1(x)}f(x,y)\,dy\right)dx
|
||||
\end{align*}
|
||||
|
@ -211,7 +211,7 @@ Sei \(D:=[a,b]\times[c,d]\subseteq\mdr^2\), \(f\in C([a,b])\) und \(g\in C([c,d
|
|||
\(\tilde f\) ist eine Fortsetzung von \(f\) auf \(X\times Y\).
|
||||
\(\tilde f\) ist also messbar. Es ist
|
||||
\begin{align*}
|
||||
\int_D\lvert f\rvert\,d(x,y)
|
||||
\int_D\lvert f\rvert\,d(x,y)
|
||||
&=\int_Q\mathds{1}_D\cdot\lvert\tilde f\rvert\,d(x,y) \\
|
||||
&\overset{\ref{Satz 10.1}}=
|
||||
\int_X\left(\int_Y\mathds{1}_D(x,y)\frac1x\lvert\cos(xy)\rvert
|
||||
|
@ -255,12 +255,12 @@ und daraus folgt \(\int_0^\infty e^{-xy}\,dy=\frac1x\)
|
|||
1 &\text{, falls } x=0
|
||||
\end{cases}\]
|
||||
$g$ ist stetig auf \([0,\infty)\). Aus Analysis 1 ist bekannt, dass
|
||||
\(\int_0^\infty g(x)\,dx\) konvergent, aber \textbf{ nicht }
|
||||
absolut konvergent ist. Aus \ref{Satz 4.14} folgt, dass
|
||||
\(\int_0^\infty g(x)\,dx\) konvergent, aber \textbf{ nicht }
|
||||
absolut konvergent ist. Aus \ref{Satz 4.14} folgt, dass
|
||||
\(g\notin\mathfrak{L}^1\left([0,\infty)\right)\)\\
|
||||
\textbf{Behauptung: } \(\int^\infty_0 g(x)\,dx = \frac\pi{2}\)\\
|
||||
\textbf{Beweis: } Setze \(X:=[0,R]\) mit \(R>0\), \(Y:=[0,\infty)\) und
|
||||
\(D:=X\times Y\), sowie
|
||||
\(D:=X\times Y\), sowie
|
||||
\[f(x,y):= e^{-xy}\sin x \text{ für } (x,y)\in D\]
|
||||
Es ist \(D\in\fb_2\) und $f$ stetig, also messbar. Es ist weiter
|
||||
\(f\in\mathfrak{L}^1(D)\) (warum?) und
|
||||
|
@ -301,8 +301,8 @@ und daraus folgt \(\int_0^\infty e^{-xy}\,dy=\frac1x\)
|
|||
&\leq 2\int^\infty_0 e^{-yR}\,dy \\
|
||||
&\overset{\text{Vorbemerkung}}=\frac2R
|
||||
\end{align*}
|
||||
Das heißt also \(\tilde I_R\to 0 \ (R\to\infty)\) und damit folgt
|
||||
Das heißt also \(\tilde I_R\to 0 \ (R\to\infty)\) und damit folgt
|
||||
die Behauptung durch
|
||||
\[I_R=\frac{\pi}2-\tilde I_R\to\frac{\pi}2 \ (R\to\infty)\]
|
||||
\[I_R=\frac{\pi}2-\tilde I_R\to\frac{\pi}2 \ (R\to\infty)\]
|
||||
\end{enumerate}
|
||||
\end{beispiel}
|
||||
|
|
|
@ -1,10 +1,10 @@
|
|||
|
||||
Die Sätze in diesem Kapitel geben wir \textbf{ohne} Beweis an. Es seien
|
||||
\(X,Y\subseteq\mdr^d\) nichtleer und offen.
|
||||
\(X,Y\subseteq\mdr^d\) nichtleer und offen.
|
||||
|
||||
\begin{definition}
|
||||
\index{Diffeomorphismus}
|
||||
Sei \(\Phi\colon X\to Y\) eine Abbildung. \(\Phi\) heißt
|
||||
Sei \(\Phi\colon X\to Y\) eine Abbildung. \(\Phi\) heißt
|
||||
\textbf{Diffeomorphismus} genau dann wenn \(\Phi\in C^1(X,\mdr^d)\), \(\Phi\)
|
||||
ist bijektiv und \(\Phi^{-1}\in C^{1}(Y,\mdr^d)\).\\
|
||||
Es gilt \[x=\Phi^{-1}(\Phi(x))\text{ für jedes } x\in X\]
|
||||
|
@ -35,7 +35,7 @@ Sei \(A\subseteq\mdr^d\) und \(A^\circ:=\{x\in A :\text{ es existiert ein } r=r(
|
|||
\end{erinnerung}
|
||||
|
||||
\begin{beispiel}
|
||||
Sei \(A=\mdr\setminus\mdq\). Es ist \(A^\circ=\emptyset\) und
|
||||
Sei \(A=\mdr\setminus\mdq\). Es ist \(A^\circ=\emptyset\) und
|
||||
\(A\setminus A^\circ=A\). Aus \(\mdr=A\dot\cup\mdq\) folgt
|
||||
\[\infty=\lambda_1(\mdr)=\lambda_1(A)+\lambda_1(\mdq)=\lambda_1(A)\]
|
||||
Das heißt \(A\setminus A^\circ\) ist keine Nullmenge.
|
||||
|
@ -66,7 +66,7 @@ Ist $f \in \fl^{1}(B)$ so gilt $(\ast\ast)$
|
|||
\item Sei $T\colon \MdR^d \to \MdR^d$ linear und $\det T \neq 0$. Weiter sei $A \in \fb_d$ und $v \in \MdR^d$.
|
||||
Dann ist $T(A) \in \fb_d$ und es gilt:
|
||||
\[\lambda_d(T(A)+v) = \lvert\det T\rvert \cdot\lambda_d(A)\]
|
||||
\item $\Phi\colon X \to Y$ sei ein Diffeomorphismus und $A \in \fb(X)$.
|
||||
\item $\Phi\colon X \to Y$ sei ein Diffeomorphismus und $A \in \fb(X)$.
|
||||
Dann ist $\Phi(A) \in \fb_d$ und es gilt:
|
||||
\[\lambda_d(\Phi(A)) = \int_A |\det \Phi'(X)| \, dx\]
|
||||
\item Sei $F \in C^1(X, \MdR^d)$ und $N \subseteq X$ eine Nullmenge.
|
||||
|
@ -101,9 +101,9 @@ y = r \sin(\varphi)
|
|||
\end{cases}\]
|
||||
Definiere nun für $(r,\varphi) \in [0,\infty)\times[0,2\pi]$:
|
||||
\[\Phi(r,\varphi) := (r \cos(\varphi), r \sin(\varphi))\]
|
||||
Dann ist $\Phi \in C^1(\MdR^2, \MdR^2)$ und es gilt:
|
||||
Dann ist $\Phi \in C^1(\MdR^2, \MdR^2)$ und es gilt:
|
||||
\[\Phi'(r,\varphi) = \begin{pmatrix}
|
||||
\cos(\varphi) & -r \sin(\varphi) \\
|
||||
\cos(\varphi) & -r \sin(\varphi) \\
|
||||
\sin(\varphi) & r \cos(\varphi)
|
||||
\end{pmatrix}\]
|
||||
d.h. falls $r > 0$ ist gilt:
|
||||
|
@ -119,19 +119,19 @@ Mit \ref{Satz 11.2} folgt dann:
|
|||
|
||||
\begin{beispiel}
|
||||
\begin{enumerate}
|
||||
\item Sei $0 \le \rho < R$. Definiere
|
||||
\item Sei $0 \le \rho < R$. Definiere
|
||||
\[B := \{(x,y) \in \MdR^2 : \rho^2 \le x^2 + y^2 \le R^2\} \]
|
||||
Dann gilt:
|
||||
Dann gilt:
|
||||
%% BILD: der Kreisfläche und Trafo
|
||||
\begin{align*}
|
||||
\lambda_2(B) &= \int_B 1 \text{ d}(x,y)\\
|
||||
&= \int_A 1 \cdot r \text{ d}(r,\varphi)\\
|
||||
\lambda_2(B) &= \int_B 1 \text{ d}(x,y)\\
|
||||
&= \int_A 1 \cdot r \text{ d}(r,\varphi)\\
|
||||
&\overset{\text{§\ref{Kapitel 10}}}= \int_{\rho}^{R} \left( \int_0^{2\pi} r \text{ d}\varphi \right) \text{ d}r\\
|
||||
&= \left[ 2\pi \frac{1}{2} r^2 \right]_\rho^R\\
|
||||
&= \pi (R^2 - \rho^2)
|
||||
\end{align*}
|
||||
|
||||
\item Definiere
|
||||
|
||||
\item Definiere
|
||||
\[B := \{ (x,y) \in \MdR^2 : x^2 + y^2 \le 1, y \ge 0 \}\]
|
||||
%% BILD: der (Halb)Kreisfläche und Trafo
|
||||
Dann gilt:
|
||||
|
@ -162,7 +162,7 @@ Außerdem gilt:
|
|||
&= \int_0^\rho \left( \int_0^\rho e^{-x^2} e^{-y^2} \text{ d}y \right) \text{ d}x \\
|
||||
&= \left( \int_0^\rho e^{-x^2} \text{ d}x \right)^2
|
||||
\end{align*}
|
||||
|
||||
|
||||
Wegen $ B_\rho \subseteq Q_\rho \subseteq B_{\sqrt{2} \rho} $ und $f \ge 0$ folgt:
|
||||
\begin{center}
|
||||
\begin{tabular}{cccccc}
|
||||
|
@ -172,7 +172,7 @@ $\implies$ &$h(\rho)$ &$\le$ &$\left( \int_0^\rho e^{-x^2} \text{ d}x \right)^2$
|
|||
$\implies$ &$\sqrt{h(\rho)}$ &$\le$ &$\int_0^\rho e^{-x^2} \text{ d}x$ &$\le$ &$\sqrt{h(\sqrt{2} \rho)}$\\
|
||||
\end{tabular}
|
||||
\end{center}
|
||||
Mit $\rho \to \infty$ folgt daraus
|
||||
Mit $\rho \to \infty$ folgt daraus
|
||||
\[\int_0^\infty e^{-x^2} \text{ d}x = \frac{\sqrt{\pi}}{2}\]
|
||||
und damit die Behauptung.
|
||||
\end{enumerate}
|
||||
|
|
|
@ -1,11 +1,11 @@
|
|||
In diesem Kapitel sei $(x_0,y_0)\in\MdR^2$ (fest), es sei
|
||||
$R:[0,2\pi]\to[0,\infty)$ stetig und stückweise stetig
|
||||
differenzierbar und $R(0) = R(2\pi)$. Weiter sei
|
||||
In diesem Kapitel sei $(x_0,y_0)\in\MdR^2$ (fest), es sei
|
||||
$R:[0,2\pi]\to[0,\infty)$ stetig und stückweise stetig
|
||||
differenzierbar und $R(0) = R(2\pi)$. Weiter sei
|
||||
\begin{displaymath}
|
||||
\gamma(t) := (x_0 + R(t)\cos t,y_0 + R(t)\sin t) \text{ } (t\in[0,2\pi])
|
||||
\end{displaymath}
|
||||
Dann ist $\gamma$ ein stückweise stetig differenzierbarer, geschlossener und rektifizierbarer Weg in $\MdR^2$. Es sei
|
||||
\[B:= \{(x_0+r\cos t,y_0 + r\sin t): t\in [0,2\pi ], 0\le r\le R(t)\}\]
|
||||
Dann ist $\gamma$ ein stückweise stetig differenzierbarer, geschlossener und rektifizierbarer Weg in $\MdR^2$. Es sei
|
||||
\[B:= \{(x_0+r\cos t,y_0 + r\sin t): t\in [0,2\pi ], 0\le r\le R(t)\}\]
|
||||
Dann ist $B$ kompakt, also $B\in\fb_2 $. Weiter ist $\partial B = \gamma([0,2\pi]) = \Gamma_\gamma$.\\
|
||||
Sind $B$ und $\gamma$ wie oben, so heißt $B$ \begriff{zulässig}.
|
||||
\index{zulässig}
|
||||
|
@ -48,13 +48,13 @@ Mit Polarkoordinaten, Transformations-Satz und Fubini:
|
|||
A = \int_0^{2\pi }(\int_0^{R(t)} u_x(r\cos t,r\sin t)r dr) dt
|
||||
\end{displaymath}
|
||||
\begin{enumerate}
|
||||
\item $\beta(r,t) := u(r\cos t,r\sin t)$. Nachrechnen: $r\beta_r(r,t)\cos t - \beta_t(r,t)\sin t = u_x(r\cos t,r\sin t)r$. Also:
|
||||
\item $\beta(r,t) := u(r\cos t,r\sin t)$. Nachrechnen: $r\beta_r(r,t)\cos t - \beta_t(r,t)\sin t = u_x(r\cos t,r\sin t)r$. Also:
|
||||
\begin{displaymath}
|
||||
A = \int_0^{2\pi} (\int_0^{R(t)} (r\beta_r(r,t)\cos t - \beta_t(r,t)\sin t) dr)dt
|
||||
\end{displaymath}
|
||||
\item $\int_0^{R(t)} r\beta_r(r,t) dr = r\beta(r,t)\vert_{r=0}^{r=R(t)} - \underbrace{\int_0^{R(t)} \beta(r,t) dr}_{=:\alpha(t)} = R(t)\beta(R(t),t) - \alpha(t) = R(t)u(\gamma(t)) -\alpha(t)$
|
||||
\item $\Psi(s,t) := \int_0^s \beta(r,t)dr$. Mit dem zweiten Hauptsatz aus Analysis 1 folgt: $\Psi_s(s,t) = \beta(s,t)$ \\ 7.3 \folgt $\Psi_t(s,t) = \int_0^s \beta_t(r,t) dr$.\\
|
||||
Dann: $\alpha(t) = \Psi(R(t),t)$, also
|
||||
Dann: $\alpha(t) = \Psi(R(t),t)$, also
|
||||
\begin{displaymath}
|
||||
\alpha'(t) = \Psi_s(R(t),t)\cdot R'(t) + \Psi_t(R(t),t)\cdot 1 = R'(t)\underbrace{\beta(R(t),t)}_{=u(\gamma(t))} + \int_0^{R(t)} \beta_t(r,t) dr
|
||||
\end{displaymath}
|
||||
|
|
|
@ -4,7 +4,7 @@
|
|||
\index{Parameterbereich}
|
||||
\index{Normalenvektor}
|
||||
\index{Flächeninhalt}
|
||||
Es sei $\emptyset \ne B\subseteq \MdR^2$ kompakt, $D\subseteq\MdR^2$ offen und $B\subseteq D$. Weiter sei $\varphi = (\varphi_1,\varphi_2,\varphi_3) \in C^1(D,\MdR^3)$ und $\varphi = \varphi(u,v)$. Dann heißt $\varphi_{|B}$ eine \textbf{Fläche} (im $\MdR^3$), $S:= \varphi(B)$ heißt \textbf{Flächenstück} und $B$ heißt \textbf{Parameterbereich} der Fläche. Es ist
|
||||
Es sei $\emptyset \ne B\subseteq \MdR^2$ kompakt, $D\subseteq\MdR^2$ offen und $B\subseteq D$. Weiter sei $\varphi = (\varphi_1,\varphi_2,\varphi_3) \in C^1(D,\MdR^3)$ und $\varphi = \varphi(u,v)$. Dann heißt $\varphi_{|B}$ eine \textbf{Fläche} (im $\MdR^3$), $S:= \varphi(B)$ heißt \textbf{Flächenstück} und $B$ heißt \textbf{Parameterbereich} der Fläche. Es ist
|
||||
\begin{displaymath}
|
||||
\varphi' = \begin{pmatrix}\frac{\partial \varphi_1}{\partial u} & \frac{\partial\varphi_1}{\partial v}\\
|
||||
\frac{\partial \varphi_2}{\partial u} & \frac{\partial\varphi_2}{\partial v}\\
|
||||
|
@ -16,10 +16,10 @@
|
|||
\gamma(t) &:= \varphi(t,v_0) &\gamma'(t) &= \varphi_u(t,v_0) &\gamma'(u_0) &= \varphi_u(u_0,v_0)\\
|
||||
\tilde{\gamma}(t)&:= \varphi(u_0,t) &\tilde{\gamma}'(t) &= \varphi_v(u_0,v) &\tilde{\gamma}'(v_0) &= \varphi_v(u_0,v_0)
|
||||
\end{align*}
|
||||
Definere damit den \textbf{Normalenvektor} in $\varphi(u_0,v_0)$:
|
||||
Definere damit den \textbf{Normalenvektor} in $\varphi(u_0,v_0)$:
|
||||
\[N(u_0,v_0) := \varphi_u(u_0,v_0)\times\varphi_v(u_0,v_0)\]
|
||||
Seien $\Delta u,\Delta v >0$ (aber "`klein"'). $a:= \Delta u\varphi_u(u_0,v_0)$, $b:= \Delta v\varphi_v(u_0,v_0)$.
|
||||
\[P:= \{\lambda a+\mu b: \ \lambda,\mu\in [0,1]\}\]
|
||||
\[P:= \{\lambda a+\mu b: \ \lambda,\mu\in [0,1]\}\]
|
||||
Aus der Linearen Algebra folgt, der "`Inhalt"' von $P$ ist $\|a \times b\| = \Delta u\Delta v \|N(u_0,v_0)\|$.
|
||||
\begin{displaymath}
|
||||
I(\varphi) = \int_B \|N(u,v)\| d(u,v)
|
||||
|
@ -31,12 +31,12 @@
|
|||
$B:=[0,2\pi]\times[-\frac\pi2,\frac\pi2]$, $D=\MdR^2$\\
|
||||
$\varphi(u,v) := (\cos u\cos v,\sin u\cos v,\sin v)$. Dann: $\varphi(B) = \{(x,y,z)\in\MdR^3:\ x^2+y^2+z^2 = 1\}$.\\
|
||||
Nachrechnen: $N(u,v) = \cos v\varphi(u,v)$. Dann: $\|N(u,v)\| = |\cos v|\underbrace{\|\varphi(u,v)\|}_{=1} = \cos v\ \ \ \ ((u,v)\in B)$. \\
|
||||
Damit gilt:
|
||||
Damit gilt:
|
||||
\[I(\varphi) = \int_B \cos v d(u,v) = \int_0^{2\pi} (\int_{-\frac\pi2}^{\frac\pi2}\cos v d(v)) d(u) = 4\pi\]
|
||||
\end{beispiel}
|
||||
|
||||
\section{Explizite Parameterdarstellung}
|
||||
Seien \(B\) und \(D\) wie in obiger Definition und \(f\in C^{1}(D,\,\mdr)\). Setze
|
||||
Seien \(B\) und \(D\) wie in obiger Definition und \(f\in C^{1}(D,\,\mdr)\). Setze
|
||||
\[\varphi(u,v):=(u,v,f(u,v))\quad((u,v)\in D)\]
|
||||
Damit ist \(\varphi_{|B}\) eine Fläche (in expliziter Darstellung).
|
||||
% hier Graphik einfuegen
|
||||
|
@ -45,7 +45,7 @@ Dann ist \(S=\varphi(B)\) gleich dem Graph von \(f_{|B}\).
|
|||
\[
|
||||
\varphi_{u}=(1,0,f_{u}),\quad \varphi_{v}=(0,1,f_{v}),\quad N(u,v)=(-f_{u},-f_{v},1)\quad\text{(Nachrechnen!)}
|
||||
\]
|
||||
Damit gilt:
|
||||
Damit gilt:
|
||||
\[I(\varphi)=\int_{B}{(f_{u}^{2}+f_{v}^{2}+1)^{\frac{1}{2}}\mathrm{d}(u,v)}\]
|
||||
|
||||
\begin{beispiel}
|
||||
|
|
|
@ -1,13 +1,13 @@
|
|||
In diesem Kapitel sei \(\emptyset\neq B\subseteq\mdr^{2}\), \(B\)
|
||||
In diesem Kapitel sei \(\emptyset\neq B\subseteq\mdr^{2}\), \(B\)
|
||||
kompakt, \(D\subseteq\mdr^{2}\) offen, \(B\subseteq D\)
|
||||
und \(\varphi=(\varphi_{1},\varphi_{2},\varphi_{3})\in C^{1}(D,\mdr^{3})\). Das heißt: \(\varphi_{|B}\) ist eine Fläche mit
|
||||
und \(\varphi=(\varphi_{1},\varphi_{2},\varphi_{3})\in C^{1}(D,\mdr^{3})\). Das heißt: \(\varphi_{|B}\) ist eine Fläche mit
|
||||
Parameterbereich \(B\), \(S:=\varphi(B)\)
|
||||
|
||||
\begin{definition}
|
||||
\index{Oberflächenintegral}
|
||||
Definiere die folgenden \textbf{Oberflächenintegrale}:
|
||||
\begin{enumerate}
|
||||
\item Sei \(f:\,S\to\mdr\) stetig. Dann:
|
||||
\item Sei \(f:\,S\to\mdr\) stetig. Dann:
|
||||
\[
|
||||
\int_{\varphi}{f\mathrm{d}\sigma}:=\int_{B}{f(\varphi(u,v))\lVert N(u,v)\rVert\mathrm{d}(u,v)}
|
||||
\]
|
||||
|
@ -28,7 +28,7 @@ F(\varphi(u,v))\cdot N(u,v)&=F(u,v,u^{2}+v^{2})\cdot(-2u,-2v,1)\\
|
|||
&=-(u^{2}+v^{2})
|
||||
\end{align*}
|
||||
|
||||
Also:
|
||||
Also:
|
||||
\[
|
||||
\int_{\varphi}{F\cdot n\mathrm{d}\sigma}=-\int_{B}{(u^{2}+v^{2})\mathrm{d}(u,v)}=-\frac{\pi}{2}
|
||||
\]
|
||||
|
@ -47,7 +47,7 @@ Es sei \(B\) zulässig, \(\partial B=\Gamma_{\gamma}\), wobei \(\gamma=(\gamma_{
|
|||
\begin{beispiel}
|
||||
\(D,\,B,\,f,\,F\) und \(\varphi\) seien wie in obigem Beispiel.
|
||||
% Bild einfuegen
|
||||
Hier: \(\gamma(t)=(\cos t,\sin t)\quad(t\in [0,2\pi])\).
|
||||
Hier: \(\gamma(t)=(\cos t,\sin t)\quad(t\in [0,2\pi])\).
|
||||
Dann: \((\varphi\circ\gamma)(t)=\varphi(\cos t, \sin t)=(\cos t, \sin t, 1)\quad(t\in [0,2\pi])\).
|
||||
|
||||
Es ist \(\rot F=0\), also: \(\int_{\varphi}{\rot F\cdot n\mathrm{d}\sigma}=0\)
|
||||
|
@ -61,7 +61,7 @@ Es ist \(\rot F=0\), also: \(\int_{\varphi}{\rot F\cdot n\mathrm{d}\sigma}=0\)
|
|||
\end{beispiel}
|
||||
|
||||
\begin{beweis}
|
||||
Sei \(\varphi:=\varphi\circ\gamma,\,\varphi=(\varphi_{1},\varphi_{2},\varphi_{3})\), also
|
||||
Sei \(\varphi:=\varphi\circ\gamma,\,\varphi=(\varphi_{1},\varphi_{2},\varphi_{3})\), also
|
||||
\(\varphi_{j}=\varphi_{j}\circ\gamma\quad(j=1,2,3)\).
|
||||
|
||||
Zu zeigen:
|
||||
|
|
|
@ -48,7 +48,7 @@ Für \(f\in\fl^{\infty}(X)\): \(\lVert f\rVert_{\infty}:=\esssup_{x\in X}\lVert
|
|||
\end{definition}
|
||||
|
||||
\begin{bemerkung}
|
||||
Es sei \(f\in\fl^{\infty}(X)\) und stetig. Außerdem habe jede in \(X\) offene, nichtleere Teilmenge positives Maß. Dann ist \(f\) auf \(X\) beschränkt und \(\sup_{x\in X}\lvert f(x)\rvert=\esssup_{x\in X}\lvert f(x)\rvert\).
|
||||
Es sei \(f\in\fl^{\infty}(X)\) und stetig. Außerdem habe jede in \(X\) offene, nichtleere Teilmenge positives Maß. Dann ist \(f\) auf \(X\) beschränkt und \(\sup_{x\in X}\lvert f(x)\rvert=\esssup_{x\in X}\lvert f(x)\rvert\).
|
||||
\end{bemerkung}
|
||||
\begin{beweis}
|
||||
Übung (ist \(N\subseteq X\) eine Nullmenge, so ist \(N^{\circ}=\emptyset\) und \(\overline{X\setminus N}=X\))
|
||||
|
@ -57,10 +57,10 @@ Es sei \(f\in\fl^{\infty}(X)\) und stetig. Außerdem habe jede in \(X\) offene,
|
|||
\begin{beispiel}
|
||||
Sei \(d=1,\,X=[1,\infty),\,p>1\,(p<\infty),\,\alpha,\beta>0,\,f(x)=\frac{1}{x^{\alpha}},\,g(x)=\frac{1}{x^{\beta}}\)
|
||||
\begin{enumerate}
|
||||
\item \[f\in\fl^{p}(X)\overset{\text{\ref{Satz 4.14}}}{\iff}\int_{1}^{\infty}{\frac{1}{x^{\alpha p}}}\mathrm{d}x\]
|
||||
\item \[f\in\fl^{p}(X)\overset{\text{\ref{Satz 4.14}}}{\iff}\int_{1}^{\infty}{\frac{1}{x^{\alpha p}}}\mathrm{d}x\]
|
||||
konvergiert genau dann, wenn \(\alpha p>1\Leftrightarrow \alpha>\frac{1}{p}\)
|
||||
\item
|
||||
\[fg\in\fl^{1}(X)\overset{\text{\ref{Satz 4.14}}}{\iff}\int_{1}^{\infty}{\frac{1}{x^{\alpha+\beta}}\mathrm{d}x}\]
|
||||
\[fg\in\fl^{1}(X)\overset{\text{\ref{Satz 4.14}}}{\iff}\int_{1}^{\infty}{\frac{1}{x^{\alpha+\beta}}\mathrm{d}x}\]
|
||||
konvergiert genau dann, wenn $\alpha+\beta >1$
|
||||
\end{enumerate}
|
||||
\end{beispiel}
|
||||
|
@ -92,7 +92,7 @@ Ist \(p=2\,(\implies p'=2)\), so heißt obige Ungleichung auch \textbf{Cauchy-Sc
|
|||
\begin{itemize}
|
||||
\item[Fall 1:] \(p=1\) (also \(p'=\infty\)) oder \(p=\infty\) (also \(p'=1\)). Etwa \(p=1,\,p'=\infty\).
|
||||
|
||||
Sei \(c>0\) und \(N_{c}\subseteq X\) Nullmenge mit: \(\lvert g(x)\rvert\leq c\,\forall x\in X\setminus N_{c}\).
|
||||
Sei \(c>0\) und \(N_{c}\subseteq X\) Nullmenge mit: \(\lvert g(x)\rvert\leq c\,\forall x\in X\setminus N_{c}\).
|
||||
\(\tilde{g}:=\mathds{1}_{X\setminus N_{c}}\cdot g\)
|
||||
|
||||
Dann: \(g=\tilde{g}\) fast überall und \(\lvert\tilde{g}\rvert\leq c\) auf \(X\). Weiter: \(fg=f\tilde{g}\) fast überall,
|
||||
|
@ -102,7 +102,7 @@ Dann:
|
|||
\[
|
||||
\int_{X}{\lvert fg\rvert\mathrm{d}x}=\int_{X}{\lvert f\tilde{g}\rvert\mathrm{d}x}=\int_{X}{\lvert f\rvert\underbrace{\lvert\tilde{g}\rvert}_{\leq c}\mathrm{d}x}\leq\int_{X}{\lvert f\rvert\mathrm{d}x}=c\cdot\lVert f\rVert_{1}<\infty
|
||||
\]
|
||||
Also: \(fg\in\fl^{1}(X)\) und \(\lVert fg\rVert_{1}\leq c\lVert f\rVert_{1}\). Übergang zum Infimum über alle \(c>0\)
|
||||
Also: \(fg\in\fl^{1}(X)\) und \(\lVert fg\rVert_{1}\leq c\lVert f\rVert_{1}\). Übergang zum Infimum über alle \(c>0\)
|
||||
liefert: \(\lVert fg\rVert_{1}\leq\lVert g\rVert_{\infty}\cdot\lVert f\rVert_{1}\)
|
||||
\item[Fall 2:] Sei \(1<p<\infty\). Ist \(\lVert f\rVert_{p}=0\) oder \(\lVert g\rVert_{p'}=0\), so ist \(f=0\) fast überall
|
||||
oder \(g=0\) fast überall. Daraus folgt: \(\lvert fg\rvert=0\) fast überall.
|
||||
|
@ -146,7 +146,7 @@ Nullmengen und \(\lvert f(x)\rvert\leq c_{1}\forall x\in X\setminus N_{1},\,\lve
|
|||
\item[Fall 3:] Sei \(1<p<\infty\) und \(f,\,g\in\fl^{p}(X)\). Es ist \(\lvert f+g\rvert^{p}\leq(\lvert f\rvert+\lvert g\rvert)^{p}\leq\left(2\max\{\lvert f\rvert,\,\lvert g\rvert\}\right)^{p}\leq 2^{p}\left(\lvert f\rvert^{p}+\lvert g\rvert^{p}\right)\)
|
||||
auf \(X\). Mit \ref{Satz 4.9} folgt: \(\lvert f+g\rvert^{p}\in\fl^{1}(X)\implies f+g\in\fl^{p}(X)\)\\
|
||||
|
||||
\(p'=\frac{p}{p-1};\,h:=\lvert f+g\rvert^{p-1}\), dann: \(h^{p'}=\left(\lvert f+g\rvert^{p-1}\right)^{\frac{p}{p-1}}=\lvert f+g\rvert^{p}\in\fl^{1}(X)\). Dann ist \(h\in\fl^{p'}(X)\). Also: \(h\in\fl^{p'}(X),\,f\in\fl^{p}(X)\)
|
||||
\(p'=\frac{p}{p-1};\,h:=\lvert f+g\rvert^{p-1}\), dann: \(h^{p'}=\left(\lvert f+g\rvert^{p-1}\right)^{\frac{p}{p-1}}=\lvert f+g\rvert^{p}\in\fl^{1}(X)\). Dann ist \(h\in\fl^{p'}(X)\). Also: \(h\in\fl^{p'}(X),\,f\in\fl^{p}(X)\)
|
||||
(und \(\frac{1}{p}+\frac{1}{p'}=1\)).
|
||||
|
||||
Mit der Hölderschen Ungleichung folgt:
|
||||
|
@ -201,7 +201,7 @@ Also gilt:
|
|||
|
||||
\begin {beispiel}
|
||||
\begin{enumerate}
|
||||
\item Sei $X:=(0,1]$, $1\le p<q<\infty$ (also $\frac 1q<\frac1p$) und $f(x):=\frac 1{x^\alpha}$ $(\alpha>0)$. Dann gilt nach
|
||||
\item Sei $X:=(0,1]$, $1\le p<q<\infty$ (also $\frac 1q<\frac1p$) und $f(x):=\frac 1{x^\alpha}$ $(\alpha>0)$. Dann gilt nach
|
||||
\ref{Satz 4.14} und Analysis I:
|
||||
\begin{align*}
|
||||
f\in\fl^p(X)&\iff\int_0^1\frac1{x^{\alpha p}}\text{ d}x \text{ konvergiert}\\
|
||||
|
@ -229,13 +229,13 @@ Dann ist $f\in\fl^p(X)$ und es gilt
|
|||
Aus (i) und (ii) folgt: $|f|^p \leq g$ f.ü.
|
||||
Im Kapitel 5 haben wir gesehen, dass dann gilt:
|
||||
\[ \int_X |f|^p \text{ d}x \leq \int_X g \text{ d}x < \infty \]
|
||||
(denn $g$ ist nach Voraussetzung integrierbar).
|
||||
(denn $g$ ist nach Voraussetzung integrierbar).
|
||||
Daraus folgt: $f \in \fl^p(X)$.
|
||||
|
||||
Setze $g_n := |f_n - f|^p$. Aus (i): $g_n \to 0$ f.ü. Es sind $f_n, f \in \fl^p(X)$ (ersteres nach Voraussetzung, zweiteres haben wir gerade gezeigt), und weil $\fl^p(X)$ ein reeller Vektorraum ist (\ref{Satz 16.1}(2)), folgt:
|
||||
\[ f_n - f \in \fl^p(X) \]
|
||||
Also $g_n \in \fl^1(X)$.
|
||||
Es ist
|
||||
Es ist
|
||||
\[ 0 \leq g_n \leq \left( |f_n| + |f| \right)^p \leq \left( g^{\frac{1}{p}} + g^{\frac{1}{p}} \right)^p = \left( 2g^{\frac{1}{p}} \right)^p = 2^p g \quad\text{f.ü.} \]
|
||||
Mit \ref{Satz 6.2} folgt schließlich:
|
||||
\[ \underbrace{\int_X g_n \text{ d}x}_{=\|f_n - f\|_p^p} \to 0. \]
|
||||
|
@ -256,19 +256,19 @@ und die Addition
|
|||
zu einem Vektorraum über $\mdr$ wird.
|
||||
\end{definition}
|
||||
|
||||
Setze für $\hat f \in L^1(X)$:
|
||||
Setze für $\hat f \in L^1(X)$:
|
||||
\[\int_X \hat f(x) \text{ d}x := \int_X f(x) \text{ d}x\]
|
||||
dabei ist diese Definition unabhängig von der Wahl des Repräsentanten $f \in \fl^1(X)$ von $\hat f$, denn: ist auch noch $g \in \fl^1(X)$ und $\hat g = \hat f$, so ist $f - g \in \cn$, also $f-g = 0$ f.ü. und damit: $\int_X f \text{ d}x = \int_X g \text{ d}x$.
|
||||
|
||||
Für $\hat f \in L^p(X)$ definiere
|
||||
Für $\hat f \in L^p(X)$ definiere
|
||||
\[\| \hat f \|_p := \| f \|_p\]
|
||||
wobei diese Definition unabhängig ist von der Wahl des Repräsentanten $f \in \fl^p(X)$ von $\hat f$.
|
||||
|
||||
Für $\hat f, \hat g \in L^2(X)$ setze
|
||||
Für $\hat f, \hat g \in L^2(X)$ setze
|
||||
\[( \hat f | \hat g ) := \int_X f(x)g(x) \text{ d}x\]
|
||||
(auch diese Definition ist Repräsentanten-unabhängig) (Beachte: $f\cdot g \in \fl^1(X)$ )
|
||||
|
||||
\textbf{Dann gilt:}
|
||||
\textbf{Dann gilt:}
|
||||
\index{Ungleichung!Cauchy-Schwarz}
|
||||
\begin{enumerate} \item $L^p(X)$ ist unter $\| \cdot \|_p$ ein normierter Raum (NR).
|
||||
\item Für $\hat f, \hat g \in L^2(X)$ gilt:
|
||||
|
@ -296,7 +296,7 @@ so heißt $B$ ein \textbf{Prähilbertraum}. Ist $B$ ein Banachraum mit $(*)$, so
|
|||
Seien \(f,f_n\in\fl^p(X)\)
|
||||
\begin{enumerate}
|
||||
\item \(\| f_n-f\|_p = \| \hat{f_n}-\hat f\|_p\to 0\) genau
|
||||
dann, wenn \((\hat{f_n})\) eine konvergente Folge im normierten Raum \(L^p(X)\)
|
||||
dann, wenn \((\hat{f_n})\) eine konvergente Folge im normierten Raum \(L^p(X)\)
|
||||
mit dem Grenzwert \(\hat f\) ist.
|
||||
\item \((\hat f_n)\) ist eine \textbf{Cauchyfolge} (CF) in \(L^p(X)\) genau dann, wenn für jedes $\ep>0$ ein $n_0\in\mdn$ exitiert mit:
|
||||
\begin{align*}
|
||||
|
@ -331,26 +331,26 @@ Sei \(X=[0,1]\) und \((I_n)\) sei die folgende Folge von Intervallen:
|
|||
\[I_1=\left[0,1\right], I_2=\left[0,\frac12\right], I_3=\left[\frac12,1\right], I_4=\left[0,\frac14\right],
|
||||
I_5=\left[\frac14,\frac12\right], I_6=\left[\frac12, \frac34\right], I_7=\left[\frac34,1\right], \dots\]
|
||||
Es sei \(f_n:=\mathds{1}_{I_n}\), sodass \(\int_X f_n\,dx=\int_{I_n}1\,dx=\lambda_1(I_n)\to 0\).
|
||||
Also \(\hat f_n\in L^1(X)\) und \(\| \hat f_n-\hat 0\|_1\to 0\).
|
||||
Ist \(x\in X\), so gilt: \(x\in I_n\) für unendlich viele \natn. Daraus folgt, dass eine Teilfolge
|
||||
Also \(\hat f_n\in L^1(X)\) und \(\| \hat f_n-\hat 0\|_1\to 0\).
|
||||
Ist \(x\in X\), so gilt: \(x\in I_n\) für unendlich viele \natn. Daraus folgt, dass eine Teilfolge
|
||||
\(I_{n_j}\) mit \(x\in I_{n_j}\) für jedes \(j\in\mdn\) existiert. Somit ist \(f_{n_j}(x)=1\) für jedes \(j\in\mdn\)
|
||||
und deshalb gilt fast überall \(f_n\nrightarrow 0\).
|
||||
\end{beispiel}
|
||||
|
||||
\begin{beweis}[von \ref{Satz 16.4}]
|
||||
Setze \(\ep_j:=\frac1{2^j}\ (j\in\mdn)\).
|
||||
Zu \(\ep_1\) existiert ein \(n_1\in\mdn\) mit \(\| f_l-f_{n_1}\|_p<\ep_1\)
|
||||
für alle \(l\geq n_1\).
|
||||
Zu \(\ep_2\) existiert ein \(n_2\in\mdn\) mit \(n_2>n_2\) und
|
||||
Setze \(\ep_j:=\frac1{2^j}\ (j\in\mdn)\).
|
||||
Zu \(\ep_1\) existiert ein \(n_1\in\mdn\) mit \(\| f_l-f_{n_1}\|_p<\ep_1\)
|
||||
für alle \(l\geq n_1\).
|
||||
Zu \(\ep_2\) existiert ein \(n_2\in\mdn\) mit \(n_2>n_2\) und
|
||||
\(\| f_l-f_{n_2}\|_p<\ep_2\) für alle \(l\geq n_2\).
|
||||
Etc.\\
|
||||
Wir erhalten eine Teilfolge \((f_{n_j})\) mit
|
||||
Wir erhalten eine Teilfolge \((f_{n_j})\) mit
|
||||
\[(+)\ \ \ \| f_l-f_{n_j}\|_p<\ep_j \text{ für alle } l\geq n_j \text{ mit } j\in\mdn\]
|
||||
Setze \(g_j:=f_{n_{j+1}}-f_{n_j}\ (j\in\mdn)\). Klar: \(g_l\in\fl^p(X)\).
|
||||
Für \(N\in\mdn\): \[S_N:=\int_X\left(\sum^N_{j=1}\lvert g_j(x)\rvert^p\right)^{\frac1p}\]
|
||||
Dann:
|
||||
\begin{align*}
|
||||
S_N=\left\lvert\left\lvert\sum^N_{j=1}\lvert g_j\rvert\right\rvert\right\rvert_p
|
||||
S_N=\left\lvert\left\lvert\sum^N_{j=1}\lvert g_j\rvert\right\rvert\right\rvert_p
|
||||
\leq \sum^N_{j=1}\| g_j\|_p
|
||||
\overset{\text{(+)}}\leq \sum^N_{j=1}\ep_j
|
||||
=\sum^N_{j=1}\frac1{2^j}
|
||||
|
@ -368,10 +368,10 @@ Somit ist \(g^p\) ist integrierbar. Aus \ref{Satz 5.2} folgt, dass eine Nullmeng
|
|||
existiert mit \(0\leq g^p(x)<\infty\) für alle \(x\in X\setminus N_1\). Es ist dann auch
|
||||
\(0\leq g(x)<\infty\) für alle \(x\in X\setminus N_1\) und somit folgt nach Konstruktion von $g$, dass
|
||||
\(\sum^\infty_{j=1}g_j\,dx\) konvergiert absolut in jedem \(x\in X\setminus N_1\).
|
||||
Aus Analysis I folgt, dass damit \(\sum^\infty_{j=1}g_j\,dx\) in jedem
|
||||
Aus Analysis I folgt, dass damit \(\sum^\infty_{j=1}g_j\,dx\) in jedem
|
||||
\(x\in X\setminus N_1\) konvergiert.
|
||||
|
||||
Für \(m\in\mdn\):
|
||||
Für \(m\in\mdn\):
|
||||
\[\sum^{m-1}_{j=1}g_j=f_{n_m}-f_{n_1} \implies f_{n_m}=\sum^{m-1}_{j=1}g_j + f_{n_1} \]
|
||||
Deshalb ist \((f_{n_m})\) konvergent (in \mdr) für alle \(x\in X\setminus N_1\).
|
||||
\begin{align*}
|
||||
|
@ -381,18 +381,18 @@ f(x):=
|
|||
0 &, x\in N_1
|
||||
\end{cases}
|
||||
\end{align*}
|
||||
Aus \S 3 ist bekannt, dass $f$ messbar ist. Klar: \(f_{n_m}\to f\) fast überall und
|
||||
Aus \S 3 ist bekannt, dass $f$ messbar ist. Klar: \(f_{n_m}\to f\) fast überall und
|
||||
\(f(X)\subseteq\mdr\).
|
||||
Es ist \(f_{n_m}=\sum^{m-1}_{j=1}g_j + f_{n_1}\) und somit
|
||||
Es ist \(f_{n_m}=\sum^{m-1}_{j=1}g_j + f_{n_1}\) und somit
|
||||
\[\lvert f_{n_m}\rvert = \lvert f_{n_1}\rvert + \sum^{m-1}_{j=1}g_j \leq \lvert f_{n_1}\rvert +
|
||||
\lvert g\rvert\]
|
||||
Wie im Beweis von Satz \ref{Satz 16.1} folgern wir
|
||||
\[\lvert f_{n_m}\rvert^p\leq 2^p\left(\lvert f_{n_1}\rvert^p+g^p\right)=:\tilde g \]
|
||||
\(f_{n_1}\in\fl^p(X)\), \(g^p\) ist integrierbar. Aus \ref{Satz 16.3} folgt, dass \(f\in\fl^p(X)\)
|
||||
und \[\| f_{n_m}-f\|_p\to 0 \ (m\to\infty)\]
|
||||
Sei nun \(\ep>0\). Wähle \(m\in M\) so, dass \(\frac1{2^m}<\frac\ep2\) und
|
||||
Sei nun \(\ep>0\). Wähle \(m\in M\) so, dass \(\frac1{2^m}<\frac\ep2\) und
|
||||
\(\| f-f_{n_m}\|_p<\frac\ep2\).
|
||||
Für \(l\geq n_m\) gilt:
|
||||
Für \(l\geq n_m\) gilt:
|
||||
\[\| f_l-f\|_p= \| f_l-f_{n_m}+f_{n_m}-f\|_p
|
||||
\leq \| f_l-f_{n_m}\|_p + \| f_{n_m}-f\|_p
|
||||
\overset{\text{(+)}}< \frac1{2^m}+\frac\ep2 <\ep\]
|
||||
|
@ -416,7 +416,7 @@ Dann ist fast überall \(f=g\).
|
|||
\item[\textbf{1.}]
|
||||
Aus Bemerkung (3) vor \ref{Satz 16.4} folgt, dass \((\hat f_n)\) ist eine Cachyfolge in
|
||||
\(L^p(X)\). Wegen \ref{Satz 16.4} existiert dann ein \(\varphi\in\fl^p(X)\) und eine Teilfolge
|
||||
\((f_{n_j})\) mit: \(f_{n_j}\to\varphi\) fast überall und
|
||||
\((f_{n_j})\) mit: \(f_{n_j}\to\varphi\) fast überall und
|
||||
\(\| f_n-\varphi\|_p\to0\)
|
||||
\begin{align*}
|
||||
\| f-\varphi\|_p
|
||||
|
@ -434,14 +434,14 @@ Dann ist fast überall \(f=g\).
|
|||
\[g_{j_k}(x)\to g(x) \text{ für alle } x\in X\setminus N_2\]
|
||||
\end{enumerate}
|
||||
Wir wissen, dass \(N:=N_1\cup N_2\) eine Nullmenge ist. Sei nun \(x\in X\setminus N\). Dann
|
||||
folgt aus dem ersten Schritt \(f_{n_j}(x)\to f(x)\) und daraus
|
||||
folgt aus dem ersten Schritt \(f_{n_j}(x)\to f(x)\) und daraus
|
||||
\[ \underbrace{f_{n_{j_k}}(x)}_{=g_{n_{j_k}}(x)}\to f(x) \]
|
||||
Aus dem Zweiten Schritt folgt dann, dass \(f_{n_{j_k}}(x)\to g(x)\) und somit \(f(x)=g(x)\).
|
||||
\end{beweis}
|
||||
|
||||
\begin{bemerkung}
|
||||
Seien \(f_n,f\in\fl^p(X)\) und es gelte \(\| f_n-f\|_p\to 0\ \ (n\to\infty)\). Der
|
||||
Beweis von \ref{Satz 16.5} zeigt, dass eine Teilfolge \((f_{n_j})\) von \((f_n)\) existiert mit
|
||||
Beweis von \ref{Satz 16.5} zeigt, dass eine Teilfolge \((f_{n_j})\) von \((f_n)\) existiert mit
|
||||
\(f_{n_j}\to f\) fast überall.
|
||||
\end{bemerkung}
|
||||
|
||||
|
@ -457,7 +457,7 @@ Sei \((f_n)\) wie im Beispiel vor \ref{Satz 16.4}. Also \(\| f_n-0\|_p\to 0\), a
|
|||
|
||||
\begin{beispiel}
|
||||
%Bild einfügen
|
||||
Sei \(X=[0,1]\) und \(f_n\) sei wie im Bild. \(f_n\) ist stetig, also messbar.
|
||||
Sei \(X=[0,1]\) und \(f_n\) sei wie im Bild. \(f_n\) ist stetig, also messbar.
|
||||
\[\int_X f_n\,dx=1 \text{ für alle } \natn\]
|
||||
Somit ist \(f_n\in\fl^1(X)\).
|
||||
\[f_n(x)\to
|
||||
|
@ -465,7 +465,7 @@ Somit ist \(f_n\in\fl^1(X)\).
|
|||
0, x\in(0,1]\\
|
||||
1, x=0
|
||||
\end{cases}\]
|
||||
Damit gilt fast überall \(f_n\to0\), aber
|
||||
Damit gilt fast überall \(f_n\to0\), aber
|
||||
\(\| f_n-0\|_1=1\nrightarrow0 \ \ (n\to\infty)\)
|
||||
\end{beispiel}
|
||||
|
||||
|
@ -481,9 +481,9 @@ Seien \((E,\|\cdot\|_1), (F,\|\cdot\|_2)\) normierte Räume.
|
|||
\[\sum^\infty_{n=1}x_n:=\lim_{n\to\infty}s_n\]
|
||||
\item \(\Phi\colon E\to F\) sei eine Abbildung. \(\Phi\) heißt \textbf{stetig} in \(x_0\in E\)
|
||||
genau dann, wenn für jede konvergente Folge \((x_n)\) in $E$ mit \(x_n\to x_0\)
|
||||
gilt: \[\Phi(x_n)\to\Phi(x_0)\]
|
||||
gilt: \[\Phi(x_n)\to\Phi(x_0)\]
|
||||
\(\Phi\) heißt auf $E$ stetig genau dann, wenn \(\Phi\) ist in jedem \(x\in E\) stetig.
|
||||
\item Für $(x,y)\in E\times E$ setze
|
||||
\item Für $(x,y)\in E\times E$ setze
|
||||
\[\|(x,y)\|:=\sqrt{\|x\|_1^2+\|y\|_1^2}\]
|
||||
Dann ist $\|\cdot\|$ eine Norm auf $E\times E$ (nachrechnen!). Weiter gilt, dass $E\times E$ genau dann ein Banachraum ist, wenn $E$ einer ist. Für eine Folge $((x_n,y_n))$ in $E\times E$ und $(x,y)\in E\times E$ gilt
|
||||
\[(x_n,y_n)\stackrel{\|\cdot\|}\to (x,y) \iff x_n\stackrel{\|\cdot\|}\to x \wedge y_n\stackrel{\|\cdot\|}\to y\]
|
||||
|
@ -506,7 +506,7 @@ Für den Rest dieser Vorlesung schreiben wir (meist) $f$ statt $\hat f$ und iden
|
|||
\item Die Abbildung $\Phi:L^p(X)\to\mdr$, definiert durch
|
||||
\[\Phi(f):=\|f\|_p\]
|
||||
ist stetig auf $L^p(X)$. D.h. für $f_n,f\in L^p(X)$ mit $f_n\stackrel{\|\cdot\|_p}\to f$ gilt $\|f_n\|_p\to\|f\|_p$, also
|
||||
\[\int_X|f_n|^p\text{ d}x\to\int_X|f|^p\text{ d}x\]
|
||||
\[\int_X|f_n|^p\text{ d}x\to\int_X|f|^p\text{ d}x\]
|
||||
\begin{beweis}
|
||||
Aus Analysis II §17 folgt:
|
||||
\[| \|f_n\|_p-\|f\|_p |\le \|f_n-f\|_p\stackrel{n\to\infty}\to 0\]
|
||||
|
@ -553,13 +553,13 @@ Es genügt den Fall $f\ge 0$ zu betrachten (also $f=f_+$, $f_-\equiv 0$). Sei al
|
|||
\begin{align*}
|
||||
0\le\varphi_n&\le (|f_n|+|f|)^p\\
|
||||
&=|f_n+f|^p\le (2f)^p\\
|
||||
&=2^pf^p=:g
|
||||
&=2^pf^p=:g
|
||||
\end{align*}
|
||||
Dann ist $g\in L^1(X)$ integrierbar.\\
|
||||
Aus \ref{Satz 4.9} folgt:
|
||||
\begin{align*}
|
||||
\varphi\in L^1(X)&\implies f_n-f\in L^p(X)\\
|
||||
&\implies f_n=(f_n-f)+f\in L^p(X)
|
||||
&\implies f_n=(f_n-f)+f\in L^p(X)
|
||||
\end{align*}
|
||||
Aus \ref{Satz 6.2} folgt:
|
||||
\[\int_X\varphi_n\text{ d}x\to 0 \implies \|f_n-f\|_p^p\to 0\]
|
||||
|
|
|
@ -12,7 +12,7 @@ Aus 3.2 folgt: $f$ ist messbar genau dann, wenn $u$ und $v$ messbar sind.
|
|||
\begin{definition}
|
||||
\index{integrierbar}\index{Integral}
|
||||
Sei $f$ messbar. $f$ heißt \textbf{integrierbar} (ib.) genau dann, wenn $u$ und $v$ integrierbar sind.
|
||||
In diesem Fall setze
|
||||
In diesem Fall setze
|
||||
\[ \int_X f \text{ d}x := \int_X u \text{ d}x + i\int_X v \text{ d}x \quad ( \in \MdC) \]
|
||||
\end{definition}
|
||||
|
||||
|
@ -23,22 +23,22 @@ Hieraus und aus 4.9 folgt: $f$ ist integrierbar genau dann, wenn $|f|$ integrier
|
|||
\[ \fl^p(X, \MdC) := \{ f : X \to \MdC | f \text{ ist messbar und } \int_X |f|^p \text{ d}x < \infty \} \]
|
||||
(Achtung: mit den Betragsstrichen in ob. Integral ist der komplexe Betrag gemeint!)
|
||||
\[ \cn := \{ f: X \to \MdC | f \text{ ist messbar und } f = 0 \text{ f.ü.} \} \]
|
||||
$\fl^p(X,\MdC )$ ist ein komplexer Vektorraum (siehe 17.1) und $\cn$ ist ein Untervektorraum von $\fl^p(X,\MdC )$.
|
||||
$\fl^p(X,\MdC )$ ist ein komplexer Vektorraum (siehe 17.1) und $\cn$ ist ein Untervektorraum von $\fl^p(X,\MdC )$.
|
||||
\[ L^p(X,\MdC ) := \fl^p(X,\MdC)\diagup\cn \]
|
||||
\end{definition}
|
||||
|
||||
\begin{definition}
|
||||
\index{orthogonal}
|
||||
Für $f,g \in L^2(X,\MdC )$ setze
|
||||
Für $f,g \in L^2(X,\MdC )$ setze
|
||||
\[(f | g) := \int_X f(x) \overline{g(x)} \text{ d}x\]
|
||||
sowie
|
||||
sowie
|
||||
\[f \bot g :\Longleftrightarrow (f | g) = 0 \quad \text{ ($f$ und $g$ sind \textbf{orthogonal}).} \]
|
||||
( $\overline{z}$ bezeichne hierbei die komplex Konjugierte von $z$, vgl. Lineare Algebra).
|
||||
\end{definition}
|
||||
|
||||
\textbf{Klar:} \begin{enumerate}
|
||||
\item $L^p(X,\MdC )$ ist mit $\| f \|_p := (\int_X |f|^p \text{ d}x )^{\frac{1}{p}}$ ein komplexer normierter Raum (NR).
|
||||
\item $(f | g)$ definiert ein Skalarprodukt auf $L^2(X,\MdC)$. Es ist
|
||||
\item $(f | g)$ definiert ein Skalarprodukt auf $L^2(X,\MdC)$. Es ist
|
||||
\[(f | g) = \overline{(g | f)}, \]
|
||||
\[ (f | f) = \int_X f(x) \overline{f(x)} \text{ d}x = \int_X |f(x)|^2 \text{ d}x = \| f \|_2^2 \text{, also:} \]
|
||||
\[ \| f\|_2 = \sqrt{(f|f)} \quad (f,g \in L^2(X,\MdC )) \]
|
||||
|
@ -56,16 +56,16 @@ sowie
|
|||
\begin{enumerate}
|
||||
\item Seien \(f,g\colon X\to\mdc\) integrierbar und \(\alpha,\beta\in\mdc\). Dann gelten:
|
||||
\begin{enumerate}
|
||||
\item[(i)] \(\alpha f+\beta g\) ist integrierbar und
|
||||
\item[(i)] \(\alpha f+\beta g\) ist integrierbar und
|
||||
\[\int_X(\alpha f+\beta g)\,dx = \alpha\int_Xf\,dx+\beta\int_Xg\,dx\]
|
||||
\item[(ii)] \(\text{Re}\left(\int_Xf\,dx\right) = \int_X\text{Re}(f)\,dx\ \) und
|
||||
\(\ \text{Im}\left(\int_Xf\,dx\right) = \int_X\text{Im}(f)\,dx\)
|
||||
\item[(iii)] \(\overline f\) ist integrierbar und
|
||||
\item[(iii)] \(\overline f\) ist integrierbar und
|
||||
\[\int_X\overline f\,dx=\overline{\int_Xf\,dx}\]
|
||||
\end{enumerate}
|
||||
\item Die Sätze \ref{Satz 16.1} bis \ref{Satz 16.3} und das Beispiel \ref{Beispiel 16.6} gelten in
|
||||
\item Die Sätze \ref{Satz 16.1} bis \ref{Satz 16.3} und das Beispiel \ref{Beispiel 16.6} gelten in
|
||||
\(L^p(X,\mdc)\).
|
||||
\item \(L^p(X,\mdc)\) ist ein komplexer Banachraum, \(L^2(X,\mdc)\) ist ein komplexer
|
||||
\item \(L^p(X,\mdc)\) ist ein komplexer Banachraum, \(L^2(X,\mdc)\) ist ein komplexer
|
||||
Hilbertraum.
|
||||
\end{enumerate}
|
||||
\end{satz}
|
||||
|
@ -79,12 +79,12 @@ Sei \(X=[0,2\pi]\). Für \(k\in\MdZ\) und \(t\in\mdr\) setzen wir
|
|||
Dann gilt: \(b_k,e_k\in L^2([0,2\pi],\mdc)\) und \[\int_0^{2\pi}e_0(t)\,dt=2\pi\]
|
||||
Für \(k\in\MdZ\) und \(k\neq0\) ist
|
||||
\begin{align*}
|
||||
\int_0^{2\pi}e_k(t)\,dt=\left.\frac1{ik}e^{ikt}\right\rvert_0^{2\pi}
|
||||
\int_0^{2\pi}e_k(t)\,dt=\left.\frac1{ik}e^{ikt}\right\rvert_0^{2\pi}
|
||||
= \frac1{ik}\left(e^{2\pi ki}-1\right)=0
|
||||
\intertext{Damit ist}
|
||||
(b_k\mid b_l) = \int^{2\pi}_0 b_k\overline{b_l}\,dt = \frac1{2\pi}\int_0^{2\pi}e^{ikt}e^{-ilt}\,dt
|
||||
= \frac1{2\pi}\int_0^{2\pi}e^{i(k-l)t}\,dt =
|
||||
\begin{cases}
|
||||
\begin{cases}
|
||||
1 ,\text{falls } k=l\\
|
||||
0 ,\text{falls }k\neq l
|
||||
\end{cases}
|
||||
|
@ -95,19 +95,19 @@ Zur Übung: \(\{b_k\mid k\in\MdZ\}\) ist linear unabhängig in \(L^2([0,2\pi],\m
|
|||
\end{wichtigesbeispiel}
|
||||
|
||||
\begin{definition}
|
||||
Sei \((\alpha_k)_{k\in\MdZ}\) eine Folge in \(\mdc\) und \((f_k)_{k\in\MdZ}\) eine Folge in
|
||||
Sei \((\alpha_k)_{k\in\MdZ}\) eine Folge in \(\mdc\) und \((f_k)_{k\in\MdZ}\) eine Folge in
|
||||
\(L^2(X,\mdc)\).
|
||||
\begin{enumerate}
|
||||
\item Für \(n\in\mdn_0\) setze
|
||||
\item Für \(n\in\mdn_0\) setze
|
||||
\[s_n:=\sum^n_{k=-n}\alpha_k = \sum_{\lvert k\rvert\leq n}\alpha_k
|
||||
=\alpha_{-n}+\alpha_{-(n-1)}+\dots+\alpha_0+\alpha_1+\dots+\alpha_n\]
|
||||
Existiert \(\lim_{n\to\infty}s_n\) in \(\mdc\), so schreiben wir
|
||||
\(\sum_{k\in\MdZ}\alpha_k:=\lim_{n\to\infty}s_n\)
|
||||
\item Für \(n\in\mdn_0\) setze
|
||||
\item Für \(n\in\mdn_0\) setze
|
||||
\[\sigma_n:=\sum^n_{k=-n}f_k=\sum_{\lvert k\rvert\leq n}f_k\]
|
||||
Gilt für ein \(f\in L^2(X,\mdc)\):
|
||||
Gilt für ein \(f\in L^2(X,\mdc)\):
|
||||
\(\| f-\sigma_n\|_2\overset{n\to\infty}\longrightarrow 0\), so schreiben
|
||||
wir \[f\overset{\|\cdot\|_2}=\sum_{k\in\MdZ}f_k \ \ \
|
||||
wir \[f\overset{\|\cdot\|_2}=\sum_{k\in\MdZ}f_k \ \ \
|
||||
\left(=\lim_{n\to\infty}\sigma_n \text{ im Sinne der } L^2\text{-Norm}\right)\]
|
||||
\end{enumerate}
|
||||
\end{definition}
|
||||
|
@ -119,19 +119,19 @@ Sei \(\{b_k\mid k\in\MdZ\}\) wie in \ref{Beispiel 17.2}. \(\{b_k\mid k\in\MdZ\}\
|
|||
\(f\in L^2([0,2\pi],\mdc)\) eine Folge \[(c_k)_{k\in\MdZ}=(c_k(f))_{k\in\MdZ}\] gibt, mit
|
||||
\[(\ast)\ \ \ \ \ \ \ \ \ f\overset{\|\cdot\|_2}=\sum_{k\in\MdZ}c_kb_k \]
|
||||
\textbf{Frage:} Ist \(\{b_k\mid k\in\MdZ\}\) eine ONB von \(L^2([0,2\pi],\mdc)\)?\\
|
||||
\textbf{Antwort:} Ja! In \ref{Satz 18.5} werden wir sehen, dass \((\ast)\) gilt mit
|
||||
\textbf{Antwort:} Ja! In \ref{Satz 18.5} werden wir sehen, dass \((\ast)\) gilt mit
|
||||
\(c_k=(f\mid b_k)\).
|
||||
\end{definition}
|
||||
|
||||
\chapter{Fourierreihen}
|
||||
\label{Kapitel 18}
|
||||
|
||||
In diesem Kapitel sei stets \(X=[0,2\pi]\), \(L^2:=L^2([0,2\pi],\mdc)\) und
|
||||
In diesem Kapitel sei stets \(X=[0,2\pi]\), \(L^2:=L^2([0,2\pi],\mdc)\) und
|
||||
\(L^2_\mdr:=L^2([0,2\pi],\mdr)\). Weiter sei \(\{b_k\mid k\in\MdZ\}\) wie in \ref{Beispiel 17.2}.
|
||||
|
||||
\begin{satz}
|
||||
\label{Satz 18.1}
|
||||
Ist \(f\in L^2\) und gilt mit einer Folge \((c_k)_{k\in\MdZ}\) in \(\mdc\):
|
||||
Ist \(f\in L^2\) und gilt mit einer Folge \((c_k)_{k\in\MdZ}\) in \(\mdc\):
|
||||
\(f\overset{\|\cdot\|_2}=\sum_{k\in\MdZ}c_kb_k \), so gilt:
|
||||
\[c_k=(f\mid b_k) \text{ für alle } k\in\MdZ\]
|
||||
\end{satz}
|
||||
|
@ -139,7 +139,7 @@ Ist \(f\in L^2\) und gilt mit einer Folge \((c_k)_{k\in\MdZ}\) in \(\mdc\):
|
|||
\begin{beweis}
|
||||
Für \(n\in\mdn_0\) setze \[\sigma_n:=\sum_{\lvert k\rvert\leq n}c_kb_k\] Aus der Voraussetzung folgt
|
||||
\(\| \sigma_n-f\|_2\to 0\) für \(n\to\infty\). Sei \(j\in\MdZ\) und \(n\in\mdn\) mit
|
||||
\(n\geq \lvert j\rvert\). Es gilt einerseits
|
||||
\(n\geq \lvert j\rvert\). Es gilt einerseits
|
||||
\[(\sigma_n\mid b_j) = \sum_{\lvert k\rvert\leq n}c_k(b_k\mid b_j)=c_j, \text{ da gilt: }
|
||||
(b_k\mid b_j)=
|
||||
\begin{cases}
|
||||
|
@ -163,11 +163,11 @@ Sei \(f\in L^2\), \(n\in\mdn_0\) und \(k\in\MdZ\).
|
|||
\to0\]
|
||||
\item \((f\mid b_k)\) heißt \textbf{k-ter Fourierkoeffizient von f}.
|
||||
\item \(\sum_{k\in\MdZ}(f\mid b_k)b_k\) heißt \textbf{Fourierreihe von f}.
|
||||
\item Für \(n_0\in\mdn_0\) setze
|
||||
\(E_n:=[b_{-n},b_{-(n-1)},\dots,b_0,b_1,\dots,b_n]\)
|
||||
\item Für \(n_0\in\mdn_0\) setze
|
||||
\(E_n:=[b_{-n},b_{-(n-1)},\dots,b_0,b_1,\dots,b_n]\)
|
||||
(lineare Hülle). Es ist dann \[\dim E_n=2n+1\]
|
||||
\textbf{Beachte: } Für \(v\in E_n\) gilt \(v(0)=v(2\pi)\).
|
||||
\end{enumerate}
|
||||
\end{enumerate}
|
||||
\end{definition}
|
||||
|
||||
\begin{satz}
|
||||
|
@ -177,7 +177,7 @@ Sei \(f\in L^2\), \(n\in\mdn_0\) und \(k\in\MdZ\).
|
|||
Seien \(f_1,\dots,f_n,f\in L^2\).
|
||||
\begin{enumerate}
|
||||
\item Gilt \(f_\mu\perp f_\nu\) für \(\mu\neq\nu\) (\(\mu,\nu=1,\dots,n\)),
|
||||
so gilt der Satz des Pythagoras
|
||||
so gilt der Satz des Pythagoras
|
||||
\[\| f_1+\dots+f_n\|^2_2=
|
||||
\| f_1\|^2_2+\dots+
|
||||
\| f_n\|^2_2\]
|
||||
|
@ -186,15 +186,15 @@ Seien \(f_1,\dots,f_n,f\in L^2\).
|
|||
L^2\to E_n\\
|
||||
S_nf:=\sum_{\lvert k\rvert\leq n}(f\mid b_k)b_k
|
||||
\end{cases}\]
|
||||
ist linear und für jedes \(v\in E_n\) gilt \(S_nv=v\) und
|
||||
ist linear und für jedes \(v\in E_n\) gilt \(S_nv=v\) und
|
||||
\((f-S_nf)\perp v\) mit \(f\in L^2\).
|
||||
\item Die \textbf{Besselsche Ungleichung} lautet:
|
||||
\[\| S_nf\|^2_2
|
||||
=\sum_{\lvert k\rvert\leq n}\lvert(f\mid b_k)\rvert^2
|
||||
=\| f\|_2^2-\|(f-S_nf)\|^2_2
|
||||
\leq\| f\|^2_2\]
|
||||
\item Für alle \(v\in E_n\) gilt:
|
||||
\[\| f-S_nf\|_2\leq\| f-v\|_2
|
||||
\item Für alle \(v\in E_n\) gilt:
|
||||
\[\| f-S_nf\|_2\leq\| f-v\|_2
|
||||
\]
|
||||
\end{enumerate}
|
||||
\end{satz}
|
||||
|
@ -207,26 +207,26 @@ Seien \(f_1,\dots,f_n,f\in L^2\).
|
|||
&= (f_1+f_2\mid f_1+f_2) \\
|
||||
&= (f_1\mid f_1)+(f_1\mid f_2)+(f_2\mid f_1)+(f_2\mid f_2) \\
|
||||
&= (f_1\mid f_1)+(f_2\mid f_2) \\
|
||||
&=\| f_1\|^2_2+\| f_2\|^2_2
|
||||
&=\| f_1\|^2_2+\| f_2\|^2_2
|
||||
\end{align*}
|
||||
\item Übung!
|
||||
\item Es gilt
|
||||
\begin{align*}
|
||||
\| S_nf\|^2_2
|
||||
&= \left\lvert\left\lvert\sum_{\lvert k\rvert\leq n}(f\mid b_k)b_k\right\rvert
|
||||
\right\rvert^2_2
|
||||
\right\rvert^2_2
|
||||
\overset{(1)}=
|
||||
\sum_{\lvert k\rvert\leq n}\|(f\mid b_k)b_k\rvert
|
||||
\rvert^2_2
|
||||
\rvert^2_2
|
||||
= \sum_{\lvert k\rvert\leq n}\lvert(f\mid b_k)\rvert^2\| b_k\rvert
|
||||
\rvert^2_2
|
||||
\rvert^2_2
|
||||
= \sum_{\lvert k\rvert\leq n}\lvert(f\mid b_k)\rvert^2
|
||||
\end{align*}
|
||||
und
|
||||
\begin{align*}
|
||||
\| f\|^2_2
|
||||
\| f\|^2_2
|
||||
= \|\underbrace{(f-S_nf)}_{\underset{(2)}\perp E_n}
|
||||
+\underbrace{S_nf}_{\in E_n}\|^2_2
|
||||
+\underbrace{S_nf}_{\in E_n}\|^2_2
|
||||
= \| f-S_nf\|^2_2 + \| S_nf\|^2_2
|
||||
\end{align*}
|
||||
\item Sei \(v\in E_n\). Dann gilt:
|
||||
|
@ -244,17 +244,17 @@ Seien \(f_1,\dots,f_n,f\in L^2\).
|
|||
|
||||
\begin{wichtigebemerkung}
|
||||
\label{Bemerkung 18.3}
|
||||
Es sei \(\mdk\in\{\mdr,\mdc\},\,a,b\in\mdr,\,I:=[a,b]\,(a<b)\) und \(f_{n},\,f,\,g\in C(I,\mdk)\); es war
|
||||
Es sei \(\mdk\in\{\mdr,\mdc\},\,a,b\in\mdr,\,I:=[a,b]\,(a<b)\) und \(f_{n},\,f,\,g\in C(I,\mdk)\); es war
|
||||
\(\lVert f\rVert_{\infty}:=\max_{t\in I}\lvert f(t)\rvert\).
|
||||
\begin{enumerate}
|
||||
\item \((f_{n})\) konvergiert auf \(I\) gleichmäßig gegen \(f\) genau dann, wenn
|
||||
\item \((f_{n})\) konvergiert auf \(I\) gleichmäßig gegen \(f\) genau dann, wenn
|
||||
\(\lVert f_{n}-f\rVert_{\infty}\to 0\,(n\to\infty)\) (vgl. Analysis I/II).
|
||||
\item \(f\in\mathrm{L}^{p}(I,\mdk)\) und \(\lVert f\rVert_{p}\leq(b-a)^{\frac{1}{p}}\lVert f\rVert_{\infty}\) (siehe \ref{Satz 16.2}).
|
||||
\item Gilt \(f=g\) fast überall, so ist \(f=g\) auf \(I\).
|
||||
\begin{beweis}
|
||||
Es existiert eine Nullmenge \(N\subseteq I:\,f(x)=g(x)\,\forall x\in I\setminus N\).\\
|
||||
Sei \(x_{0}\in\mdn\). Für \(\ep>0\) gilt: \(U_{\ep}(x_{0})\cap I\not\subseteq N\) (andernfalls:
|
||||
\(\lambda_{1}(N)\geq\lambda_{1}(U_{\ep}(x_{0})\cap I)>0\)). Das heißt, es existiert ein
|
||||
Sei \(x_{0}\in\mdn\). Für \(\ep>0\) gilt: \(U_{\ep}(x_{0})\cap I\not\subseteq N\) (andernfalls:
|
||||
\(\lambda_{1}(N)\geq\lambda_{1}(U_{\ep}(x_{0})\cap I)>0\)). Das heißt, es existiert ein
|
||||
\(x_{\ep}\in U_{\ep}(x_{0})\cap I:\,x_{\ep}\not\in N\). Also:
|
||||
\(\forall n\in\mdn\,\exists x_{n}\in U_{\frac{1}{n}}(x_{0})\cap I:\, x_{n}\not\in N\). Also: \(x_{n}\to x_{0}\).\\
|
||||
Dann: \(f(x_{0})=\lim_{n\to\infty}f(x_{n})=\lim_{n\to\infty}g(x_{n})=g(x_{0})\)
|
||||
|
@ -287,9 +287,9 @@ Sei \(f\in\mathrm{L}^{2}\). Dann gilt: \(f\overset{\lVert\cdot\rVert_{2}}{=}\sum
|
|||
|
||||
\begin{beweis}
|
||||
Zu zeigen: \(\lVert f-S_{n}f\rVert_{2}\to0\,(n\to\infty)\). Die Parsevalsche Gleichung folgt dann aus \ref{Satz 18.2}.\\
|
||||
Sei \(\ep>0\). Wende \ref{Satz 16.8}(2) auf \(\Re f\) und \(\Im f\) an. Dies liefert eine stetige Funktion
|
||||
Sei \(\ep>0\). Wende \ref{Satz 16.8}(2) auf \(\Re f\) und \(\Im f\) an. Dies liefert eine stetige Funktion
|
||||
\(g:\,(0,2\pi)\to\mdc\) mit: \(K:=\supp(g)\subseteq(0,2\pi)\), \(K\) kompakt und \(\lVert f-g\rVert_{2}<\ep\).\\
|
||||
Setze \(g(0):=g(2\pi):=0\). Dann ist \(g\) stetig auf \([0,2\pi]\). Satz \ref{Satz 18.4} liefert nun:
|
||||
Setze \(g(0):=g(2\pi):=0\). Dann ist \(g\) stetig auf \([0,2\pi]\). Satz \ref{Satz 18.4} liefert nun:
|
||||
\(\exists n\in\mdn\exists v\in\mathrm{E}_{n}:\,\lVert g-v\rVert_{\infty}<\ep\).\\
|
||||
Damit: \(\lVert g-v\rVert_{2}\leq\sqrt{2\pi}\lVert g-v\rVert_{\infty}<\sqrt{2\pi}\ep\). Somit:
|
||||
\begin{align*}
|
||||
|
@ -344,9 +344,9 @@ Fourierreihe von \(f\) ist eine \textbf{Sinusreihe}.
|
|||
\begin{enumerate}
|
||||
\item \(f(t):=\begin{cases}1,&0\leq t\leq\pi\\-1,&\pi<t\leq 2\pi\end{cases}\)
|
||||
|
||||
\(f\) ist ungerade, also \(\alpha_{k}=0\,\forall k\in\mdn_{0}\). Es ist
|
||||
\(f\) ist ungerade, also \(\alpha_{k}=0\,\forall k\in\mdn_{0}\). Es ist
|
||||
\(\beta_{k}=\frac{2}{\pi}\int_{0}^{\pi}{\sin(kt)\mathrm{d}t}=\begin{cases}0,&k\text{ gerade}\\\frac{4}{k\pi},&k\text{ ungerade}\end{cases}\).\\
|
||||
Damit:
|
||||
Damit:
|
||||
\[
|
||||
f\overset{\lVert\cdot\rVert_{2}}{=}\frac{4}{\pi}\sum_{j=0}^{\infty}{\frac{\sin((2j+1)\cdot)}{2j+1}}
|
||||
\]
|
||||
|
|
|
@ -3,7 +3,7 @@
|
|||
In diesem Kapitel sei \(X\) eine Menge, \(X\neq\emptyset\).
|
||||
\begin{definition}
|
||||
\index{Ring}
|
||||
Sei \(\emptyset\neq \fr \subseteq \cp(X)\).
|
||||
Sei \(\emptyset\neq \fr \subseteq \cp(X)\).
|
||||
$\fr$ heißt ein \textbf{Ring} auf \(X\), genau dann wenn gilt:
|
||||
\begin{enumerate}
|
||||
\item[(R1)] \(\emptyset \in \fr\)
|
||||
|
@ -12,7 +12,7 @@ In diesem Kapitel sei \(X\) eine Menge, \(X\neq\emptyset\).
|
|||
\end{definition}
|
||||
|
||||
\textbf{Hinweis}: $(\fr, \cup, \setminus)$ ist kein Ring im Sinne
|
||||
der linearen Algebra, $(\fr, \cup)$ kein Inverses Element hat und
|
||||
der linearen Algebra, $(\fr, \cup)$ kein Inverses Element hat und
|
||||
$(\fr, \cup)$ nicht kommutativ ist.
|
||||
|
||||
\begin{definition}
|
||||
|
@ -21,7 +21,7 @@ $(\fr, \cup)$ nicht kommutativ ist.
|
|||
Sei \(d\in\MdN\).
|
||||
\begin{enumerate}
|
||||
\item \(\ci_d :=\Set{(a,b] | a,b \in \MdR^{d}, \, a \leq b} (\emptyset \in \ci_d)\).
|
||||
Seien \(a=(a_{1},\dots,a_{d}),\,b=(b_{1},\dots,b_{d})\in\MdR^d\)
|
||||
Seien \(a=(a_{1},\dots,a_{d}),\,b=(b_{1},\dots,b_{d})\in\MdR^d\)
|
||||
und \(I:=(a,b] \in \ci_{d}\)
|
||||
\[
|
||||
\lambda_{d}(I)= \begin{cases}
|
||||
|
@ -31,7 +31,7 @@ $(\fr, \cup)$ nicht kommutativ ist.
|
|||
\item \(\cf_d:=\Set{\bigcup_{j=1}^{n}I_{j} | n\in\MdN,\,I_{1},\dots,I_{n}\in \ci_d}\) (\textbf{Menge der Figuren})
|
||||
\end{enumerate}
|
||||
\end{definition}
|
||||
Ziel dieses Kapitels: Fortsetzung von \(\lambda_{d}\) auf \(\cf_{d}\)
|
||||
Ziel dieses Kapitels: Fortsetzung von \(\lambda_{d}\) auf \(\cf_{d}\)
|
||||
und dann auf \(\fb_d\) (\(\leadsto\) Lebesgue-Maß)
|
||||
|
||||
Beachte: \(\ci_{d}\subseteq\cf_{d}\subseteq\fb_{d}\overset{1.4}{\implies}\fb_{d}=\sigma(\ci_{d})=\sigma(\cf_{d})\)
|
||||
|
@ -40,7 +40,7 @@ Beachte: \(\ci_{d}\subseteq\cf_{d}\subseteq\fb_{d}\overset{1.4}{\implies}\fb_{d}
|
|||
Seien \(I,I'\in\ci_{d}\) und \(A\in\cf_{d}\). Dann:
|
||||
\begin{enumerate}
|
||||
\item \(I\cap I'\in\ci_{d}\)
|
||||
\item \(I\setminus I'\in\cf_{d}.\)
|
||||
\item \(I\setminus I'\in\cf_{d}.\)
|
||||
Genauer: \(\exists\left\{I_{1}',\dots,I_{l}'\right\}\subseteq\ci_{d}\) disjunkt:
|
||||
\(I\setminus I'=\bigcup_{j=1}^{l}{I_{j}'}\) % \bigcupdot
|
||||
\item \(\exists\left\{I_{1}',\dots,I_{l}'\right\}\subseteq\ci_{d}\) disjunkt: \(A=\bigcup_{j=1}^{l}{I_{j}'}\)
|
||||
|
@ -67,7 +67,7 @@ Beachte: \(\ci_{d}\subseteq\cf_{d}\subseteq\fb_{d}\overset{1.4}{\implies}\fb_{d}
|
|||
\(I=I_{1}\times I_{2},\,I'=I_{1}'\times I_{2}'\)
|
||||
% Graphik einfuegen!
|
||||
|
||||
Nachrechnen:
|
||||
Nachrechnen:
|
||||
\[
|
||||
I\setminus I'=(I_{1}\setminus I_{1}')\times I_{2}\dot \cup(I_{1}\cap I_{1}')\times(I_{2}\setminus I_{2}')
|
||||
\]
|
||||
|
@ -76,11 +76,11 @@ Beachte: \(\ci_{d}\subseteq\cf_{d}\subseteq\fb_{d}\overset{1.4}{\implies}\fb_{d}
|
|||
Daraus folgt die Behauptung für \(d+1\)
|
||||
\end{itemize}
|
||||
\item \begin{itemize}
|
||||
\item[\underline{Vor.:}] Sei $n \in \mdn$ und
|
||||
\(A=\bigcup_{j=1}^{n}{I_{j}}\) mit
|
||||
\(I_{1},\dots,I_{d}\in\ci_{d}\)
|
||||
\item[\underline{Vor.:}] Sei $n \in \mdn$ und
|
||||
\(A=\bigcup_{j=1}^{n}{I_{j}}\) mit
|
||||
\(I_{1},\dots,I_{d}\in\ci_{d}\)
|
||||
\item[\underline{Beh.:}] Es existiert
|
||||
\(\{I_{1}',\dots,I_{l}'\}\subseteq\ci_{d}\) disjunkt:
|
||||
\(\{I_{1}',\dots,I_{l}'\}\subseteq\ci_{d}\) disjunkt:
|
||||
\(A=\bigcup_{j=1}^{l}{I_{j}'}\)
|
||||
\item[\underline{Bew.:}] mit Induktion nach $n$:
|
||||
\begin{itemize}
|
||||
|
@ -93,7 +93,7 @@ Beachte: \(\ci_{d}\subseteq\cf_{d}\subseteq\fb_{d}\overset{1.4}{\implies}\fb_{d}
|
|||
|
||||
Dann: \(A=I_{n+1}\cup\bigcup_{j=1}^{l}{I_{j}'}=I_{n+1}\cup\bigcup_{j=1}^{l}{(I_{j}'\setminus I_{n+1})}\) % \cupdot
|
||||
|
||||
Wende (2) auf jedes \(I_{j}'\setminus I_{n+1}\) an \((j=1,\dots,l)\):
|
||||
Wende (2) auf jedes \(I_{j}'\setminus I_{n+1}\) an \((j=1,\dots,l)\):
|
||||
\(I_{j}'\setminus I_{n+1}=\bigcup_{j=1}^{l_{j}}{I_{j}''}\quad(I_{j}''\in\ci_{d})\)
|
||||
|
||||
Damit folgt:
|
||||
|
@ -124,14 +124,14 @@ ohne Beweis:
|
|||
\begin{lemma}[Unabhängigkeit von der Darstellung]
|
||||
\label{Lemma 2.2}
|
||||
Sei \(A\in\cf_{d}\) und \(\{I_{1},\dots,I_{n}\}\subseteq\ci_{d}\) disjunkt und
|
||||
\(\{I_{1}',\dots,I_{m}'\}\subseteq\ci_{d}\) disjunkt mit
|
||||
\(\{I_{1}',\dots,I_{m}'\}\subseteq\ci_{d}\) disjunkt mit
|
||||
\(\bigcup_{j=1}^{n}{I_{j}}=A=\bigcup_{j=1}^{m}{I_{j}'}\). Dann:
|
||||
\[
|
||||
\sum_{j=1}^{n}{\lambda_{d}(I_{j})}=\sum_{j=1}^{m}{\lambda_{d}(I_{j}')}
|
||||
\]
|
||||
\end{lemma}
|
||||
\begin{definition}
|
||||
Sei \(A\in\cf_{d}\) und \(A=\bigcup_{j=1}^{n}{I_{j}}\) mit
|
||||
Sei \(A\in\cf_{d}\) und \(A=\bigcup_{j=1}^{n}{I_{j}}\) mit
|
||||
\(\{I_{1},\dots,I_{n}\}\subseteq\ci_{d}\)
|
||||
disjunkt (beachte Lemma \ref{Lemma 2.1}, Punkt 3).
|
||||
\[
|
||||
|
@ -147,23 +147,23 @@ ohne Beweis:
|
|||
\item \(A\cap B=\emptyset\implies\lambda_{d}(A\cup B)=\lambda_{d}(A)+\lambda_{d}(B)\)
|
||||
\item \(A\subseteq B\implies\lambda_{d}(A)\leq\lambda_{d}(B)\)
|
||||
\item \(\lambda_{d}(A\cup B)\leq\lambda_{d}(A)+\lambda_{d}(B)\)
|
||||
\item Sei \(\delta>0\). Es existiert \(C\in\cf_{d}:\overline{C}\subseteq B\)
|
||||
\item Sei \(\delta>0\). Es existiert \(C\in\cf_{d}:\overline{C}\subseteq B\)
|
||||
und \(\lambda_{d}(B\setminus C)\leq\delta\).
|
||||
\item Ist \(B_{n+1}\subseteq B_{n}\forall n\in\mdn\) und
|
||||
\(\bigcap B_{n}=\emptyset\), so gilt:
|
||||
\item Ist \(B_{n+1}\subseteq B_{n}\forall n\in\mdn\) und
|
||||
\(\bigcap B_{n}=\emptyset\), so gilt:
|
||||
\(\lambda_{d}(B_{n})\to 0\,(n\to \infty)\)
|
||||
\end{enumerate}
|
||||
\end{satz}
|
||||
|
||||
\begin{beweis}
|
||||
\begin{enumerate}
|
||||
\item Aus Lemma \ref{Lemma 2.1} folgt: Es existiert
|
||||
\item Aus Lemma \ref{Lemma 2.1} folgt: Es existiert
|
||||
\(\{I_{1},\dots,I_{n}\}\subseteq\ci_{d}\)
|
||||
disjunkt und es existiert \(\{I_{1}',\dots,I_{m}'\}\subseteq\ci_{d}\) disjunkt:
|
||||
\(A=\bigcup_{j=1}^{n}{I_{j}},\,B=\bigcup_{j=1}^{m}{I_{j}'}\).
|
||||
|
||||
\(J:=\{I_{1},\dots,I_{n},I_{1}',\dots,I_{m}'\}\subseteq\ci_{d}\). Aus
|
||||
\(A\cap B=\emptyset\) folgt: \(J\) ist disjunkt. Dann:
|
||||
\(J:=\{I_{1},\dots,I_{n},I_{1}',\dots,I_{m}'\}\subseteq\ci_{d}\). Aus
|
||||
\(A\cap B=\emptyset\) folgt: \(J\) ist disjunkt. Dann:
|
||||
\(A\cup B=\bigcup_{I\in J}{I}\) % Hier auch wieder: \bigcupdot
|
||||
|
||||
Also:
|
||||
|
@ -187,7 +187,7 @@ Dann:
|
|||
Aus der Definition von Kompaktheit (Analysis II, \S 2) folgt:
|
||||
\(\exists m\in\mdn:\,\bigcup_{j=1}^{m}{\overline{C}_{j}^{c}}\supseteq\overline{B}_{1}\)
|
||||
Dann: \(\bigcap_{j=1}^{m}{\overline{C}_{j}}\subseteq\overline{B}_{1}^{c}\).
|
||||
Andererseits: \(\bigcap_{j=1}^{m}{\overline{C}_{j}}\subseteq\bigcap_{j=1}^{m}{B_{j}}\subseteq B_{1}\subseteq\overline{B}_{1}\).
|
||||
Andererseits: \(\bigcap_{j=1}^{m}{\overline{C}_{j}}\subseteq\bigcap_{j=1}^{m}{B_{j}}\subseteq B_{1}\subseteq\overline{B}_{1}\).
|
||||
|
||||
Also: \(\bigcap_{j=1}^{m}{\overline{C}_{j}}=\emptyset\). Das heißt:
|
||||
\(\bigcap_{j=1}^{n}{\overline{C}_{j}}=\emptyset \quad \forall n\geq m\)
|
||||
|
@ -198,7 +198,7 @@ Also: \(\bigcap_{j=1}^{m}{\overline{C}_{j}}=\emptyset\). Das heißt:
|
|||
\begin{beweis} (induktiv)
|
||||
\begin{itemize}
|
||||
\item[I.A.] \(\lambda_{d}(B_{1}\setminus D_{1})=\lambda_{d}(B_{1}\setminus C_{1})\overset{\eqref{eq: Abschaetzung Mass -- Beweis Satz 2.3.(5)}}{\leq}\frac{\ep}{2}=\left(1-\frac{1}{2}\right)\ep\) \checkmark
|
||||
\item[I.V.] Sei \(n\in\mdn\) und es gelte
|
||||
\item[I.V.] Sei \(n\in\mdn\) und es gelte
|
||||
$\lambda_{d}(B_{n}\setminus D_{n})\leq\left(1-\frac{1}{2^{n}}\right)\ep$
|
||||
\item[I.S.] \begin{align*}
|
||||
\lambda_{d}(B_{n+1}\setminus D_{n+1})&=\lambda_{d}\left((B_{n+1}\setminus D_{n})\cup(B_{n+1}\setminus C_{n+1})\right)\\
|
||||
|
@ -216,7 +216,7 @@ Für \(n\geq m:\,D_{n}=\emptyset\,\implies\,\lambda_{d}(B_{n})=\lambda_{d}(B_{n}
|
|||
|
||||
\begin{definition}
|
||||
\index{Prämaß}
|
||||
Es sei \(\fr\) ein Ring auf \(X\). Eine Abbildung \(\mu:\fr\to[0,\infty]\)
|
||||
Es sei \(\fr\) ein Ring auf \(X\). Eine Abbildung \(\mu:\fr\to[0,\infty]\)
|
||||
heißt ein \textbf{Prämaß} \ auf \(\fr\), wenn gilt:
|
||||
\begin{enumerate}
|
||||
\item \(\mu(\emptyset)=0\)
|
||||
|
@ -240,7 +240,7 @@ Für \(n\geq 2\):
|
|||
\[
|
||||
\lambda_{d}(A)=\lambda_{d}(A_{1}\cup\dots\cup A_{n-1}\cup B_{n})\overset{\ref{Satz 2.3}.(1)}{=}\sum_{j=1}^{n-1}{\lambda_{d}(A_{j})}+\lambda_{d}(B_{n})
|
||||
\]
|
||||
Daraus folgt:
|
||||
Daraus folgt:
|
||||
\[
|
||||
\sum_{j=1}^{n-1}{\lambda_{d}(A_{j})}=\lambda_{d}(A)-\lambda_{d}(B_{n})\quad\forall n\geq 2
|
||||
\]
|
||||
|
@ -268,7 +268,7 @@ Sei \(\emptyset\neq\ce\subseteq\cp(X)\), es seien \(\nu,\,\mu\) Maße auf
|
|||
Es gelte:
|
||||
\begin{enumerate}
|
||||
\item \(E,F\in\ce\implies E\cap F\in\ce\quad\text{(durchschnittstabil)}\)
|
||||
\item $\exists$ eine Folge \((E_{n})\) in \(\ce\): \(\bigcup{E_{n}}=X\)
|
||||
\item $\exists$ eine Folge \((E_{n})\) in \(\ce\): \(\bigcup{E_{n}}=X\)
|
||||
und \(\mu(E_{n})<\infty \quad \forall n\in\mdn\).
|
||||
\item \(\mu(E)=\nu(E) \quad \forall E\in\ce\)
|
||||
\end{enumerate}
|
||||
|
@ -286,18 +286,18 @@ und wird ebenfalls mit \(\lambda_{d}\) bezeichnet.
|
|||
\folgtnach{(\ref{Lemma 2.1}) und (\ref{Satz 2.4})}: \(\lambda_{d}\) ist ein
|
||||
Prämaß\ auf \(\fr:=\cf_{d}\); es ist \(\sigma(\fr)=\fb_{d}\).
|
||||
|
||||
\folgtnach{\ref{Satz 2.5}}: \(\lambda_{d}\) kann zu einem Maß auf
|
||||
\(\sigma(\cf_{d}) = \fb_{d}\) fortgesetzt werden. Für diese
|
||||
\folgtnach{\ref{Satz 2.5}}: \(\lambda_{d}\) kann zu einem Maß auf
|
||||
\(\sigma(\cf_{d}) = \fb_{d}\) fortgesetzt werden. Für diese
|
||||
Fortsetzung schreiben wir wieder $\lambda_d$, also
|
||||
$\lambda_d: \fb_{d} \rightarrow [0, +\infty]$
|
||||
|
||||
Sei \(\nu\) ein weiteres Maß\ auf \(\fb_{d}\) mit:
|
||||
Sei \(\nu\) ein weiteres Maß\ auf \(\fb_{d}\) mit:
|
||||
\(\nu(A)=\lambda_{d}(A)\,\forall A\in\cf_{d}\). \(\ce:=\ci_{d}\). Dann:
|
||||
\(\sigma(\ce)\overset{\ref{Satz 1.4}}{=}\fb_{d}\).
|
||||
\begin{enumerate}
|
||||
\item \(E,F\in\ce\overset{\ref{Lemma 2.1}}{\implies}E\cap F\in\ce\)
|
||||
\item \(E_{n}:=(-n,n]^{d}\)
|
||||
Klar:
|
||||
Klar:
|
||||
\begin{align*}
|
||||
\bigcup E_{n}&=\mdr^{d}\\
|
||||
\lambda_{d}(E_{n})&=(2n)^{d}<\infty
|
||||
|
@ -426,9 +426,9 @@ Also auch:
|
|||
\end{beweis}
|
||||
|
||||
\textbf{Auswahlaxiom:}\\
|
||||
Sei $\emptyset\ne\Omega$ Indexmenge, es sei $\Set{X_\omega | \omega\in\Omega}$
|
||||
ein disjunktes System von nichtleeren Mengen $X_\omega$. Dann
|
||||
existiert ein $C\subseteq\bigcup_{\omega\in\Omega}X_\omega$, sodass
|
||||
Sei $\emptyset\ne\Omega$ Indexmenge, es sei $\Set{X_\omega | \omega\in\Omega}$
|
||||
ein disjunktes System von nichtleeren Mengen $X_\omega$. Dann
|
||||
existiert ein $C\subseteq\bigcup_{\omega\in\Omega}X_\omega$, sodass
|
||||
$C$ mit jedem $X_j$ genau ein Element gemeinsam hat.
|
||||
|
||||
\begin{satz}[Satz von Vitali]
|
||||
|
@ -450,7 +450,7 @@ Es ist $\mdq^d\cap[-1,1]^d=\{q_1,q_2,\dots\}$ mit $q_i\ne q_j$ für $(i\ne j)$.
|
|||
\end{align*}
|
||||
\begin{beweis}
|
||||
Sei $x\in[0,1]^d$. Wähle $y\in C$ mit $y\in[x]$, dann ist $x\sim y$, also $x-y\in\mdq^d\cap[-1,1]^d$. D.h.:
|
||||
\[\exists n\in\mdn: x-y=q_n\implies x=q_n+y\in q_n+C\]
|
||||
\[\exists n\in\mdn: x-y=q_n\implies x=q_n+y\in q_n+C\]
|
||||
\end{beweis}
|
||||
Außerdem ist $\Set{q_n+C | n\in\mdn}$ disjunkt.
|
||||
\begin{beweis}
|
||||
|
|
|
@ -18,8 +18,8 @@ Seien die Bezeichnungen wie in obiger Definition, dann gilt:
|
|||
\begin{enumerate}
|
||||
\item $f$ sei $\fa$-$\fb$-messbar, $\fa'$ eine weitere $\sigma$-Algebra auf $X$ mit $\fa\subseteq\fa'$ und $\fb'$ sei eine $\sigma$-Algebra auf $Y$ mit $\fb'\subseteq\fb$.\\
|
||||
Dann ist $f$ $\fa'$-$\fb'$-messbar.
|
||||
\item Sei $X_0\in\fa$, dann gilt $\fa_{X_0}\subseteq\fa$ nach
|
||||
\ref{Satz 1.5}. Nun sei $f:X\to Y$ $\fa$-$\fb$-messbar, dann ist
|
||||
\item Sei $X_0\in\fa$, dann gilt $\fa_{X_0}\subseteq\fa$ nach
|
||||
\ref{Satz 1.5}. Nun sei $f:X\to Y$ $\fa$-$\fb$-messbar, dann ist
|
||||
$f_{\mid X_0}:X_0\to Y$ $\fa_{X_0}$-$\fb$-messbar.
|
||||
\end{enumerate}
|
||||
\end{bemerkung}
|
||||
|
@ -66,7 +66,7 @@ Dann: \(\fb=\sigma(\ce)\subseteq\fd\). Ist \(B\in\fb\), so ist \(B\in\fd\), also
|
|||
\index{messbar!Borel}\index{messbar}
|
||||
Sei \(X\in\fb_{d}\). Ist \(f:\,X\to\mdr^{k}\) \(\fb(X)-\fb_{k}-\)messbar, so heißt \(f\) \textbf{(Borel-)messbar}.
|
||||
\end{definition}
|
||||
Ab jetzt sei stets \(\emptyset \neq X\in\fb_{d}\).
|
||||
Ab jetzt sei stets \(\emptyset \neq X\in\fb_{d}\).
|
||||
(Erinnerung: \(\fb(X)=\Set{A\in\fb_{d} | A\subseteq X}\))
|
||||
|
||||
\begin{satz}
|
||||
|
@ -80,7 +80,7 @@ Seien \(f,\,g:\,X\to\mdr^{k}\) Abbildungen und \(\alpha,\beta\in\mdr\).
|
|||
\item Sei \(k=1\) und \(f\) und \(g\) seien messbar. Dann:
|
||||
\begin{enumerate}
|
||||
\item \(f \cdot g\) ist messbar
|
||||
\item Ist \(f(x)\neq 0 \quad \forall x\in X\), so ist
|
||||
\item Ist \(f(x)\neq 0 \quad \forall x\in X\), so ist
|
||||
\(\frac{1}{f}\) messbar
|
||||
\item \(\Set{x\in X | f(x)\stackrel{>}{\geq} g(x)} \in \fb(X)\)
|
||||
\end{enumerate}
|
||||
|
@ -96,13 +96,13 @@ Seien \(f,\,g:\,X\to\mdr^{k}\) Abbildungen und \(\alpha,\beta\in\mdr\).
|
|||
stetig, also messbar.
|
||||
|
||||
Es ist \(g=\vp\circ f\). \folgtnach{\ref{Satz 3.1}.(1)} \(g\) ist messbar.
|
||||
\item
|
||||
\item
|
||||
\begin{itemize}
|
||||
\item["`\(\Rightarrow:\)"'] Für \(j=1, \dots,k\) sei
|
||||
\(p_{j}:\mdr^{k}\to\mdr\) definiert durch
|
||||
\item["`\(\Rightarrow:\)"'] Für \(j=1, \dots,k\) sei
|
||||
\(p_{j}:\mdr^{k}\to\mdr\) definiert durch
|
||||
\(p_{j}(x_{1},\dots,x_{k}):=x_{j}\)
|
||||
\(p_{j}\) ist stetig, also messbar. Es ist
|
||||
\(f_{j}=p_{j}\circ f\) \folgtnach{\ref{Satz 3.1}.(1)}
|
||||
\(p_{j}\) ist stetig, also messbar. Es ist
|
||||
\(f_{j}=p_{j}\circ f\) \folgtnach{\ref{Satz 3.1}.(1)}
|
||||
\(f_{j}\) ist messbar.
|
||||
\item["`\(\Leftarrow:\)"'] Sei \(I=(a,b]=\prod_{j=1}^{k}{(a_{j},b_{j}]}\in I_{k}\quad (a=(a_{1},\dots,a_{k}),\,b=(b_{1},\dots,b_{k}),\,a\leq b)\)\\
|
||||
Dann: \(f^{-1}(I)=\bigcap_{j=1}^{k}{\underbrace{f_{j}^{-1}(\underbrace{(a_{j},b_{j}]}_{\in\fb_{1}}}_{\in\fb(X)}}\in\fb(X)\)
|
||||
|
@ -114,7 +114,7 @@ Es ist \(g=\vp\circ f\). \folgtnach{\ref{Satz 3.1}.(1)} \(g\) ist messbar.
|
|||
|
||||
\(\vp\) ist stetig, also messbar. Es ist \(\alpha f+\beta g=\vp\circ h\)
|
||||
\folgtnach{\ref{Satz 3.1}.(1)} \(\alpha f+\beta g\) ist messbar.
|
||||
\item
|
||||
\item
|
||||
\begin{enumerate}
|
||||
\item \(h:=(f,g):\,X\to\mdr^{2k}\) ist messbar (nach (2)); \(\vp(x,y):=xy\), \(\vp\) ist stetig, also messbar.
|
||||
|
||||
|
@ -130,10 +130,10 @@ Es ist \(fg=\vp\circ h\) \folgtnach{\ref{Satz 3.1}.(1)} \(fg\) ist messbar.
|
|||
|
||||
\begin{folgerungen}
|
||||
\label{Lemma 3.3}
|
||||
Seien \(A,\,B\in\fb(X),\,A\cap B=\emptyset\) und \(X=A\cup B\).
|
||||
Seien \(A,\,B\in\fb(X),\,A\cap B=\emptyset\) und \(X=A\cup B\).
|
||||
Weiter seien \(f:A\to\mdr^{k}\) und
|
||||
\(g:B\to\mdr^{k}\) messbar.\\
|
||||
Dann ist \(h:X\to\mdr^{k}\), definiert durch
|
||||
Dann ist \(h:X\to\mdr^{k}\), definiert durch
|
||||
\[
|
||||
h(x):=\begin{cases}f(x)&x\in A\\g(x)&x\in B\end{cases},
|
||||
\]
|
||||
|
@ -184,7 +184,7 @@ In \(\imdr\) gelten folgende Regeln, wobei \(a\in\mdr\):
|
|||
|
||||
\begin{definition}
|
||||
\begin{enumerate}
|
||||
\item Sei \((x_{n})\) eine Folge in
|
||||
\item Sei \((x_{n})\) eine Folge in
|
||||
\(\imdr\). \(x_{n}\rightarrow+\infty:\Leftrightarrow\forall c\in\mdr\,\exists n_{c}\in\mdn:x_{n}\geq c\quad\forall n\geq n_{c}\)\\
|
||||
Analog für \(-\infty\).
|
||||
\item Seien \(f,g: X\to\imdr\) Funktionen. Dann:
|
||||
|
@ -208,7 +208,7 @@ Analog für \(-\infty\).
|
|||
|
||||
\begin{definition}
|
||||
\index{Borel!$\sigma$-Algebra}\index{messbar}
|
||||
\(\ifb_{1}:=\Set{B\cup E | B\in\fb_{1},\,E\subseteq\Set{-\infty,+\infty}}\).
|
||||
\(\ifb_{1}:=\Set{B\cup E | B\in\fb_{1},\,E\subseteq\Set{-\infty,+\infty}}\).
|
||||
Dann: \(\fb_{1}\subseteq\ifb_{1}\)\\
|
||||
Übung: \(\ifb_{1}\) ist eine \(\sigma\)-Algebra auf \(\imdr\).\\
|
||||
Klar: \(\fb_{1} \subseteq \ifb_{1}\)
|
||||
|
@ -258,11 +258,11 @@ Die folgenden Beweise erfolgen exemplarisch für einen der Unterpunkte und funkt
|
|||
\item Es gilt:
|
||||
\[\forall a \in \mdq\colon \{f\le a\}=\Set{x\in X | f(x)\le a}=f^{-1}(\underbrace{[-\infty,a]}_{\ce_1}) (*)\]
|
||||
Die Äquivalenz folgt dann aus (1) und \ref{Satz 3.1}.
|
||||
\item Die Funktion $f:X\to\imdr$ kann aufgefasst werden als Funktion $\overline{f}:X\to\imdr$. Es ist $f$ genau dann $\fb(X)$-$\fb_1$-messbar wenn $\overline{f}$ $\fb(X)$-$\overline{\fb_1}$-messbar ist.
|
||||
\item Die Funktion $f:X\to\imdr$ kann aufgefasst werden als Funktion $\overline{f}:X\to\imdr$. Es ist $f$ genau dann $\fb(X)$-$\fb_1$-messbar wenn $\overline{f}$ $\fb(X)$-$\overline{\fb_1}$-messbar ist.
|
||||
\end{enumerate}
|
||||
\end{beweis}
|
||||
|
||||
\begin{bemerkung}\
|
||||
\begin{bemerkung}\
|
||||
\begin{enumerate}
|
||||
\item Ist $X \subseteq \mdr$ ein Intervall und $f: \bar X \rightarrow \mdr$ monoton, so ist
|
||||
$f$ messbar (vgl. 3. ÜB)
|
||||
|
@ -284,11 +284,11 @@ Es ist $|f(x)|=1 \quad \forall x \in \mdr^d$, also $|f| = \mathds{1}_{\mdr^d}$.
|
|||
\begin{definition}
|
||||
Sei $M\subseteq\imdr$.
|
||||
\begin{enumerate}
|
||||
\item Ist $M=\emptyset$ oder $M=\{-\infty\}$, so sei
|
||||
\item Ist $M=\emptyset$ oder $M=\{-\infty\}$, so sei
|
||||
\[\sup M:=-\infty\]
|
||||
\item Ist $M\setminus\{-\infty\}\ne\emptyset$ und nach oben beschränkt (also insbesondere $\infty\not\in M$), so sei
|
||||
\item Ist $M\setminus\{-\infty\}\ne\emptyset$ und nach oben beschränkt (also insbesondere $\infty\not\in M$), so sei
|
||||
\[\sup M:= \sup (M\setminus\{-\infty\})\]
|
||||
\item Ist $M\setminus\{-\infty\}$ nicht nach oben beschränkt oder $\infty\in M$, so sei
|
||||
\item Ist $M\setminus\{-\infty\}$ nicht nach oben beschränkt oder $\infty\in M$, so sei
|
||||
\[\sup M:=\infty\]
|
||||
\item Es sei $\inf M:=-\sup(-M)$, wobei $-M:=\Set{-m | m\in M}$.
|
||||
\end{enumerate}
|
||||
|
@ -343,7 +343,7 @@ Also ist $\sup_{n\in\mdn} f_n$ messbar. Analog lässt sich die Messbarkeit von $
|
|||
Sei $X=I$ ein Intervall in $\mdr$ und $f:I\to\mdr$ sei auf $I$ differenzierbar.\\
|
||||
Für $x\in I,n\in\mdn$ sei $f_n:= n(f(x-\frac1n)-f(x))$. Da $f$ stetig ist, ist auch jedes $f_n$ stetig, also insbesondere messbar und es gilt:
|
||||
\[f_n(x)=\frac{f(x-\frac1n)-f(x)}{\frac1n}\stackrel{n\to\infty}{\to}f'(x)\]
|
||||
Aus \ref{Satz 3.5}(2) folgt, dass $f'$ messbar ist.
|
||||
Aus \ref{Satz 3.5}(2) folgt, dass $f'$ messbar ist.
|
||||
\end{beispiel}
|
||||
|
||||
\begin{definition}
|
||||
|
@ -435,13 +435,13 @@ Sei $f:X\to\imdr$ eine Funktion, dann ist $f$ genau dann messbar, wenn eine Folg
|
|||
Dann ist $\varphi_n$ $(\fb_1)_{[0,\infty]}$-$\fb_1$-messbar, außerdem gilt:
|
||||
\begin{align*}
|
||||
\forall t\in[0,\infty]\forall n\in\mdn&: 0\le\varphi_1\le\dots\le t\\
|
||||
\forall t\in[0,n]\forall n\in\mdn&: t-\frac1{2^n}\le\varphi_n(t)\le t
|
||||
\forall t\in[0,n]\forall n\in\mdn&: t-\frac1{2^n}\le\varphi_n(t)\le t
|
||||
\end{align*}
|
||||
und es ist $\varphi_n(t)\stackrel{n\to\infty}\to t$ für alle $t\in[0\infty]$. Setze $f_n:=\varphi_n\circ f$. Dann leistet $(f_n)$ das gewünschte.
|
||||
\item Es ist $f=f_+-f_-$ und $f_+,f_-\ge0$ auf $X$. Seien $(g_n),(h_n)$ zulässige Folgen für $f_+$ bzw. $f_-$. Definiere $f_n:=g_n-h_n$. Dann ist klar, dass gilt:
|
||||
\[\forall x\in X: f_n(x)=g_n(x)-h_n(x)\stackrel{n\to\infty}\to f_+(x)-f_-(x)=f(x)\]
|
||||
Weiter gilt:
|
||||
\[|f_n|\le g_n+h_n\le f_++f_-=|f|\]
|
||||
\item Ohne Beweis.
|
||||
\item Ohne Beweis.
|
||||
\end{enumerate}
|
||||
\end{beweis}
|
||||
|
|
|
@ -53,7 +53,7 @@ Sei $f:X\to[0,\infty]$ messbar. $(f_n)$ sei eine für $f$ zulässige Folge. Das
|
|||
\end{align*}
|
||||
\end{definition}
|
||||
|
||||
\begin{bemerkung}\
|
||||
\begin{bemerkung}\
|
||||
\begin{enumerate}
|
||||
\item In \ref{Satz 4.3} werden wir sehen, dass $(*)$ unabhängig ist von der Wahl der für $f$ zulässigen Folge $(f_n)$.
|
||||
\item $(f_n(x))$ ist wachsend für alle $x\in X$, d.h.:
|
||||
|
@ -92,8 +92,8 @@ Es folgt \(x\in B_n\) für jedes \(n\geq n(x)\).\\
|
|||
\textbf{Fazit:} \(X=\bigcup B_n\). \[A_j=A_j\cap X=A_j\cap\left(\bigcup B_n\right) = \bigcup(A_j\cap B_n) \text{ und } A_j\cap B_n\subseteq A_j\cap B_{n+1} \]
|
||||
Aus \ref{Satz 1.7} folgt \(\lambda(A_j)=\lim\limits_{n\to\infty}\lambda(A_j\cap B_n)\). Das liefert:
|
||||
\begin{align*}
|
||||
\int\limits_Xg\,dx &= \sum\limits_{j=1}^m y_j\lambda(A_j)
|
||||
= \sum\limits_{j=1}^m y_j\lim\limits_{n\to\infty}\lambda(A_j\cap B_n)\\
|
||||
\int\limits_Xg\,dx &= \sum\limits_{j=1}^m y_j\lambda(A_j)
|
||||
= \sum\limits_{j=1}^m y_j\lim\limits_{n\to\infty}\lambda(A_j\cap B_n)\\
|
||||
&=\lim\limits_{n\to\infty}\sum\limits_{j=1}^m y_j\lambda(A_j\cap B_n)
|
||||
\overset{\ref{Satz 4.1}}= \lim\limits_{n\to\infty} \int\limits_X \mathds{1}_{B_n}g\,dx\\
|
||||
&\leq \lim\limits_{n\to\infty} \int\limits_X \alpha f_n\,dx
|
||||
|
@ -127,7 +127,7 @@ Dann ist wegen \ref{Satz 3.7} und \(\alpha , \beta \geq 0\), dass \((h_n)\) zul
|
|||
\begin{enumerate}
|
||||
\item["'$\implies$"'] Sei \(\int_Xf\,dx=0\) und \(A_n:=\{f>\frac{1}{n}\}\). Dann ist \(A=\bigcup A_n\) und \(f\geq\frac{1}{n}\mathds{1}_{A_n}\). Damit folgt:
|
||||
\begin{align*}
|
||||
0 = \int_Xf\,dx
|
||||
0 = \int_Xf\,dx
|
||||
\overset{\text{(2)}}\geq \int_X\frac1{n}\mathds{1}_{A_n}\,dx
|
||||
=\frac1{n}\lambda(A_n)
|
||||
\intertext{Es ist also \(\lambda(A_n)=0\) und damit gilt weiter}
|
||||
|
@ -212,7 +212,7 @@ Sei $X \in \fb_1$, $f(x) := \begin{cases} 1&,x\in X\cap\MdQ\\ 0&,x\in X\setminus
|
|||
$X, \MdQ \in \fb_1 \implies X \cap \MdQ \in \fb_1 \implies f$ ist messbar.
|
||||
\[0 \leq \int_X f(x) \text{ d}x = \int_X \mathds{1}_{X\cap\MdQ} \text{ d}x = \lambda(X\cap\MdQ) \leq \lambda(\MdQ) = 0\]
|
||||
\textbf{Das heißt:} $f \in \fl^1(X)$, $\int_X f \text{ d}x = 0$.
|
||||
Ist speziell $X = [a,b]\quad (a<b)$, so gilt: $f \in \fl^1([a,b])$, aber $f \not\in R([a,b])$.
|
||||
Ist speziell $X = [a,b]\quad (a<b)$, so gilt: $f \in \fl^1([a,b])$, aber $f \not\in R([a,b])$.
|
||||
\end{beispiel}
|
||||
|
||||
\begin{satz}[Charakterisierung der Integrierbarkeit]
|
||||
|
@ -258,9 +258,9 @@ Sei $f:X\to\imdr$ integrierbar und $N := \{\lvert f \rvert = +\infty\} = \{x\in
|
|||
\end{folgerungen}
|
||||
|
||||
\begin{beweis}
|
||||
$\ref{Satz 3.4} \implies N \in \fb(X).$ $n\mathds{1}_N \leq \lvert f \rvert$ für alle $n\in \MdN$. Dann:
|
||||
$\ref{Satz 3.4} \implies N \in \fb(X).$ $n\mathds{1}_N \leq \lvert f \rvert$ für alle $n\in \MdN$. Dann:
|
||||
\[n \cdot \lambda(N) = \int_X n\mathds{1}_N \text{ d}x \stackrel{4.5}{\leq} \int_X \lvert f \rvert \text{ d}x \stackrel{4.9}{<} \infty \text{ für alle } n \in \mdn\]
|
||||
Also: $0 \leq n\lambda(N) \leq \int_X \lvert f \rvert \text{ d}x \quad \forall n \in \mdn \implies \lambda(N) = 0$
|
||||
Also: $0 \leq n\lambda(N) \leq \int_X \lvert f \rvert \text{ d}x \quad \forall n \in \mdn \implies \lambda(N) = 0$
|
||||
\end{beweis}
|
||||
|
||||
\begin{satz}
|
||||
|
@ -277,13 +277,13 @@ $f, g: X \to \imdr$ seien integrierbar und es sei $\alpha \in \mdr$.
|
|||
\item $\lvert \int_X f \text{ d}x \rvert \leq \int_X \lvert f \rvert \text{ d}x$. (Dreiecksungleichung für Integrale)
|
||||
\item Sei $\emptyset\ne Y \in \fb(X)$. Dann sind die Funktionen $f_{|Y}: Y \to \imdr$ und $\mathds{1}_Y\cdot f: X \to \imdr$ integrierbar und
|
||||
\[\int_Y f(x) \text{ d}x := \int_Y f_{|Y} (x) \text{ d}x = \int_X(\mathds{1}_Y \cdot f)(x) \text{ d}x\]
|
||||
\item Sei $\lambda(X) < \infty$ und $h: X \to \mdr$ sei messbar und beschränkt. Dann: $h \in \fl^1(X)$ und $\lvert \int_X h \text{ d}x\rvert \leq \|h\|_\infty \lambda(X) \quad$ (mit $\|h\|_\infty := \sup\{|h(x)| : x\in X\}$)
|
||||
\item Sei $\lambda(X) < \infty$ und $h: X \to \mdr$ sei messbar und beschränkt. Dann: $h \in \fl^1(X)$ und $\lvert \int_X h \text{ d}x\rvert \leq \|h\|_\infty \lambda(X) \quad$ (mit $\|h\|_\infty := \sup\{|h(x)| : x\in X\}$)
|
||||
\end{enumerate}
|
||||
\end{satz}
|
||||
|
||||
\begin{beweis}
|
||||
\begin{enumerate}
|
||||
\item folgt aus \(\alpha f)_{\pm}=\alpha f_{\pm}\), falls \(\alpha\geq0\) und \(\alpha f)_{\pm}=-\alpha f_{\mp}\), falls
|
||||
\begin{enumerate}
|
||||
\item folgt aus \(\alpha f)_{\pm}=\alpha f_{\pm}\), falls \(\alpha\geq0\) und \(\alpha f)_{\pm}=-\alpha f_{\mp}\), falls
|
||||
\(\alpha<0\).
|
||||
\item Es gilt \(f+g=\underbrace{f_{+}+g_{+}}_{=:u}-\underbrace{(f_{-}+g_{-})}_{=:v}=u-v\). Dann:
|
||||
\[
|
||||
|
@ -309,11 +309,11 @@ Es folgt:
|
|||
\[
|
||||
\int_{X}{f\mathrm{d}x}=\int_{X}{f_{+}\mathrm{d}x}-\int_{X}{f_{-}\mathrm{d}x}\overset{\ref{Satz 4.5}}{\leq}\int_{X}{g_{+}\mathrm{d}x}-\int_{X}{g_{-}\mathrm{d}x}=\int_{X}{g\mathrm{d}x}
|
||||
\]
|
||||
\item Es ist \(\pm f\leq\lvert f\rvert\). Mit Aussage (1) und (5) folgt:
|
||||
\item Es ist \(\pm f\leq\lvert f\rvert\). Mit Aussage (1) und (5) folgt:
|
||||
\(\pm\int_{X}{f\mathrm{d}x}=\int_{X}{(\pm f)\mathrm{d}x}\leq\int_{X}{\lvert f\rvert\mathrm{d}x}\).\\
|
||||
Es ist \(\int_{X}{f\mathrm{d}x}=\lvert\int_{X}{f\mathrm{d}x}\rvert\) oder \(-\int_{X}{f\mathrm{d}x}=\lvert\int_{X}{f\mathrm{d}x}\rvert\)
|
||||
\item Mit Bemerkung (2) vor \ref{Satz 3.1} und Satz \ref{Satz 3.6}.(2) folgt: \(f_{|Y}\) und \(\mathds{1}_{Y}\cdot f\) sind
|
||||
messbar. Es gilt: \((f_{|Y})_{\pm}=(f_{\pm})_{|Y}\) und \((\mathds{1}_{Y}\cdot f)_{\pm}=\mathds{1}\cdot f_{\pm}\). Weiterhin
|
||||
messbar. Es gilt: \((f_{|Y})_{\pm}=(f_{\pm})_{|Y}\) und \((\mathds{1}_{Y}\cdot f)_{\pm}=\mathds{1}\cdot f_{\pm}\). Weiterhin
|
||||
gilt \(0\leq\mathds{1}_{Y}f_{\pm}\leq f_{\pm}\). Mit \ref{Satz 4.9} folgt dann, daß\ \(\mathds{1}_{Y}f_{\pm}\) integrierbar
|
||||
ist. Dann:
|
||||
\begin{align*}
|
||||
|
@ -325,7 +325,7 @@ Es folgt: \(f_{|Y}\) ist integrierbar und \(\int_{Y}{f_{|Y}\mathrm{d}x}=\int_{Y}
|
|||
\[
|
||||
\int_{X}{\lvert h\rvert\mathrm{d}x}\leq\int_{X}{\lVert h\rVert_{\infty}\mathds{1}_{X}\mathrm{d}x}=\lVert h\rVert_{\infty}\lambda(X)<\infty
|
||||
\]
|
||||
Damit: \(\lvert h\rvert\) ist integrierbar und mit \ref{Satz 4.9} auch \(h\). Da \(h\) beschränkt ist, folgt:
|
||||
Damit: \(\lvert h\rvert\) ist integrierbar und mit \ref{Satz 4.9} auch \(h\). Da \(h\) beschränkt ist, folgt:
|
||||
\(h\in\fl^{1}(X)\). Schließlich:
|
||||
\[
|
||||
\left\lvert\int_{X}{h\mathrm{d}x}\right\rvert\leq\int_{X}{\lvert h\rvert\mathrm{d}x}\leq\lVert h\lVert_{\infty}\lambda(X)
|
||||
|
@ -345,7 +345,7 @@ Damit: \(\lvert h\rvert\) ist integrierbar und mit \ref{Satz 4.9} auch \(h\). Da
|
|||
|
||||
\begin{beweis}
|
||||
\begin{enumerate}
|
||||
\item Aus \ref{Satz 4.11}(7) folgt: $f$ ist integrierbar über $A$ und integrierbar über $B$. Es ist
|
||||
\item Aus \ref{Satz 4.11}(7) folgt: $f$ ist integrierbar über $A$ und integrierbar über $B$. Es ist
|
||||
\[ \int_X f(x) \text{ d}x = \int_X \left( \mathds{1}_{A\cup B} \cdot f \right)(x) \text{ d}x = \int_X \left( \left( \mathds{1}_A + \mathds{1}_B \right) f\right)(x) \text{ d}x \]
|
||||
\[= \int_X \left(\mathds{1}_A f + \mathds{1}_B f \right)(x) \text{ d}x \stackrel{4.11(2)}{=} \int_X \mathds{1}_A f \text{ d}x + \int_X \mathds{1}_B f \text{ d}x \stackrel{4.11(7)}{=} \int_A f \text{ d}x + \int_B f \text{ d}x.\]
|
||||
|
||||
|
@ -364,8 +364,8 @@ Sei $\natn$, $t_j^{(n)}:=a+j\frac{b-a}{n}$ ($j=0,\dots,n$) und $I_j^{(n)}:=\left
|
|||
\begin{align*}
|
||||
S_n:=\sum^n_{j=1} f \left(t_j^{(n)}\right) \underbrace{ \frac{b-a}{n}}_{= \lambda_1 \left(I_j^{(n)}\right)} \text{ ist Riemannsche Zwischensumme für R-} \int_a^bf(x)\,dx.
|
||||
\end{align*}
|
||||
Aus Analysis I folgt $S_n\to\text{R-}\int_a^bf(x)\,dx$ ($n\to\infty$).
|
||||
Definiere $f_n:=\sum^n_{j=1}f \left(t_j^{(n)} \right) \mathds{1}_{I_j^{(n)}} $. Dann ist $f_n$ einfach und
|
||||
Aus Analysis I folgt $S_n\to\text{R-}\int_a^bf(x)\,dx$ ($n\to\infty$).
|
||||
Definiere $f_n:=\sum^n_{j=1}f \left(t_j^{(n)} \right) \mathds{1}_{I_j^{(n)}} $. Dann ist $f_n$ einfach und
|
||||
\[\int_X f_n(x)\,dx=\sum_{j=1}^n f \left(t_j^{(n)} \right) \lambda_1 \left(I_j^{(n)}\right)=S_n\]
|
||||
$f$ ist auf $X$ gleichmäßig stetig also konvergiert $f_n$ auf $X$ gleichmäßig gegen $f$ (Übung!), also gilt:
|
||||
\[\lVert f_n-f \rVert_{\infty}=\text{sup} \left \{ \lvert f_n(x)-f(x) \rvert : x\in X \right\} \to 0 \ (n\to \infty)\]
|
||||
|
|
|
@ -56,7 +56,7 @@ Seien $f:X\to\imdr$ messbare Funktionen.
|
|||
\item Ist $f$ integrierbar, so ist $f$ fast überall endlich.
|
||||
\item Ist $f \ge0$ auf $X$, so ist $\int_X f(x)\text{ d}x=0$ genau dann wenn fast überall $f=0$.
|
||||
\item Ist $f$ integrierbar und $N\subseteq X$ eine Nullmenge, so gilt:
|
||||
\[\int_N f(x)\text{ d}x=0\]
|
||||
\[\int_N f(x)\text{ d}x=0\]
|
||||
\end{enumerate}
|
||||
\end{satz}
|
||||
|
||||
|
@ -64,9 +64,9 @@ Seien $f:X\to\imdr$ messbare Funktionen.
|
|||
\begin{enumerate}
|
||||
\item ist gerade \ref{Folgerung 4.10}.
|
||||
\item ist gerade \ref{Satz 4.5}(3)
|
||||
\item Setze $g:=\mathds{1}_N f$. Aus \ref{Satz 4.11} folgt, dass g integrierbar ist, also ist nach \ref{Satz 4.9} auch $\lvert g \rvert$ integrierbar. Für $x\in X\setminus N$ gilt:
|
||||
\item Setze $g:=\mathds{1}_N f$. Aus \ref{Satz 4.11} folgt, dass g integrierbar ist, also ist nach \ref{Satz 4.9} auch $\lvert g \rvert$ integrierbar. Für $x\in X\setminus N$ gilt:
|
||||
\[g(x)=\lvert g(x) \rvert =0\]
|
||||
D.h. $\lvert g \rvert =0$ fast überall. Aus (2) folgt damit $\int_X \lvert g \rvert \,dx = 0$. Dann ist mit \ref{Satz 4.11}: \[\left\lvert\int_X g\,dx \right\rvert \leq \int_X \lvert g \rvert \,dx =0\]
|
||||
D.h. $\lvert g \rvert =0$ fast überall. Aus (2) folgt damit $\int_X \lvert g \rvert \,dx = 0$. Dann ist mit \ref{Satz 4.11}: \[\left\lvert\int_X g\,dx \right\rvert \leq \int_X \lvert g \rvert \,dx =0\]
|
||||
und somit $\int_X g\,dx=0$.
|
||||
\end{enumerate}
|
||||
\end{beweis}
|
||||
|
@ -78,7 +78,7 @@ $f,g:X\to\imdr$ seien messbar.
|
|||
\item Ist $f$ integrierbar und gilt fast überall $f=g$, so ist $g$ integrierbar und es gilt:
|
||||
\[\int_Xf\,dx=\int_Xg\,dx\]
|
||||
\item Ist $f$ integrierbar und $g:=\mathds{1}_{\{ \lvert f \rvert <\infty \}}\cdot f$, so ist $g$ integrierbar und es gilt: \[\int_Xf\,dx=\int_Xg\,dx\]
|
||||
\item Sind $f$ und $g$ beide $\geq0$ auf $X$, und ist fast überall $f=g$, so ist
|
||||
\item Sind $f$ und $g$ beide $\geq0$ auf $X$, und ist fast überall $f=g$, so ist
|
||||
\[\int_Xf\,dx=\int_Xg\,dx\]
|
||||
\end{enumerate}
|
||||
\end{satz}
|
||||
|
@ -86,14 +86,14 @@ $f,g:X\to\imdr$ seien messbar.
|
|||
\begin{beweis}
|
||||
\begin{enumerate}
|
||||
\item Nach Voraussetzung existiert eine Nullmenge $N\subseteq X$, sodass gilt:
|
||||
\[\forall x\in X\setminus N:f(x)=g(x)\]
|
||||
\[\forall x\in X\setminus N:f(x)=g(x)\]
|
||||
Aus \ref{Satz 5.2}(3) folgt dann $\int_N f\,dx=0$.
|
||||
Sei $x\in X\setminus N$ Dann gilt:
|
||||
\[\left( \mathds{1}_N \lvert g \rvert \right)(x)=\mathds{1}_N(x)\cdot \lvert g(x) \rvert=0\]
|
||||
Sei $x\in X\setminus N$ Dann gilt:
|
||||
\[\left( \mathds{1}_N \lvert g \rvert \right)(x)=\mathds{1}_N(x)\cdot \lvert g(x) \rvert=0\]
|
||||
D.h.: Fast überall ist $\mathds{1}_N \lvert g \rvert =0$. Aus \ref{Satz 5.2}(2) folgt $\int_N \lvert g \rvert\,dx=\int_X\mathds{1}_N\cdot \lvert g \rvert\,dx=0$.
|
||||
Dann gilt:
|
||||
\begin{align*}
|
||||
\int_X \lvert g\rvert\,dx & = \int_X \left(\mathds{1}_N \lvert g\rvert + \mathds{1}_{X\setminus N} \lvert g\rvert \right)\,dx\\
|
||||
\int_X \lvert g\rvert\,dx & = \int_X \left(\mathds{1}_N \lvert g\rvert + \mathds{1}_{X\setminus N} \lvert g\rvert \right)\,dx\\
|
||||
&= \int_X\mathds{1}_N \lvert g\rvert\,dx + \int _X\mathds{1}_{X\setminus N} \lvert g\rvert\,dx\\
|
||||
&= \int_X \mathds{1}_{X\setminus N} \lvert g \rvert\,dx\\
|
||||
& \leq\int_X \lvert f\rvert\,dx \overset{\ref{Satz 4.9}}< \infty
|
||||
|
@ -141,15 +141,15 @@ Ist \(g\) wie in (2), so muss \(g\) nicht messbar sein (ein Beispiel gibt es in
|
|||
|
||||
\begin{beweis}
|
||||
\begin{enumerate}
|
||||
\item Es existiert eine Nullmenge \(N_{1}\subseteq X:\,(f_{n}(x))\) konvergiert in \(\imdr\) für alle
|
||||
\item Es existiert eine Nullmenge \(N_{1}\subseteq X:\,(f_{n}(x))\) konvergiert in \(\imdr\) für alle
|
||||
\(x\in X\setminus N_{1}\).
|
||||
\[
|
||||
f(x)=\begin{cases}0&x\in N_{1}\\\lim_{n\to\infty}{f_{n}(x)}&x\in X\setminus N_{1}\end{cases}
|
||||
\]
|
||||
\(g_{n}:=\mathds{1}_{X\setminus N}\cdot f_{n}\), \(g_{n}\) ist messbar und \(g_{n}(x)\to f(x)\) für alle \(x\in X\).
|
||||
Mit \ref{Satz 3.5} folgt: \(f\) ist messbar.
|
||||
\item Es existiert eine Nullmenge \(N_{2}\subseteq X:\,f_{n}(x)\to g(x)\,\forall x\in X\setminus N_{2}\).
|
||||
\(N=N_{1}\cup N_{2}\). Aus \ref{Lemma 5.1} folgt: \(N\) ist eine Nullmenge.
|
||||
\item Es existiert eine Nullmenge \(N_{2}\subseteq X:\,f_{n}(x)\to g(x)\,\forall x\in X\setminus N_{2}\).
|
||||
\(N=N_{1}\cup N_{2}\). Aus \ref{Lemma 5.1} folgt: \(N\) ist eine Nullmenge.
|
||||
|
||||
Für \(x\in X\setminus N:\,f(x)=g(x)\).
|
||||
\end{enumerate}
|
||||
|
@ -159,19 +159,19 @@ Für \(x\in X\setminus N:\,f(x)=g(x)\).
|
|||
\label{Satz 5.5}
|
||||
Sei \((f_{n})\) eine Folge messbarer Funktionen \(f_{n}:\,X\to[0,+\infty]\) und für jedes \(n\in\mdn\) gelte:
|
||||
\(f_{n}\leq f_{n+1}\) fast überall. Dann existiert eine messbare Funktion
|
||||
\(f:X\to[0,+\infty]\) mit: \(f_{n}\to f\) fast überall und
|
||||
\(f:X\to[0,+\infty]\) mit: \(f_{n}\to f\) fast überall und
|
||||
\[\int_{X}{f\mathrm{d}x}=\lim_{n\to\infty}{\int_{X}{f_{n}\mathrm{d}x}}\]
|
||||
\end{satz}
|
||||
|
||||
\begin{beweis}
|
||||
Zu jedem \(n\in\mdn\) existiert eine Nullmenge
|
||||
\(N_{n}:\,f_{n}(x)\leq f_{n+1}(x)\;\forall x\in X\setminus N_{n}\).\\
|
||||
Zu jedem \(n\in\mdn\) existiert eine Nullmenge
|
||||
\(N_{n}:\,f_{n}(x)\leq f_{n+1}(x)\;\forall x\in X\setminus N_{n}\).\\
|
||||
\(N:=\bigcup_{n=1}^{\infty}{N_{n}}\) \folgtnach{\ref{Lemma 5.1}} \(N\) ist eine
|
||||
Nullmenge.
|
||||
|
||||
Dann: \(f_{n}(x)\leq f_{n+1}(x)\forall x\in X\setminus N\forall n\in\mdn\).
|
||||
|
||||
\(\hat{f}_{n}:=\mathds{1}_{X\setminus N}\cdot f_{n}\), \(\hat{f}_{n}\) ist
|
||||
\(\hat{f}_{n}:=\mathds{1}_{X\setminus N}\cdot f_{n}\), \(\hat{f}_{n}\) ist
|
||||
messbar, \(\forall n\in\mdn: \hat{f}_{n}\leq\hat{f}_{n+1}\) auf $X$.
|
||||
|
||||
\(f(x):=\lim_{n\to\infty}{\hat{f}_{n}(x)}\,(x\in X)\) \folgtnach{\ref{Satz 3.5}}
|
||||
|
|
|
@ -41,7 +41,7 @@ Dann gilt \(f=\mathds{1}_{X\setminus N}\cdot f\) fast überall.
|
|||
&\overset{(1)}{\leq}\liminf_{n\to\infty}\int_{X}{\mathds{1}_{X\setminus N}f_{n}\mathrm{d}x}\\
|
||||
&\overset{\text{\ref{Satz 5.3}.(3)}}{=}\liminf_{n\to\infty}\int_{X}{f_{n}\mathrm{d}x}
|
||||
\end{align*}
|
||||
\item folgt aus (2). Nach Voraussetzung gilt
|
||||
\item folgt aus (2). Nach Voraussetzung gilt
|
||||
\[
|
||||
0\leq\int_{X}{f\mathrm{d}x}\overset{\text{(2)}}{\leq}\liminf_{n\to\infty}\int_{X}{f_{n}\mathrm{d}x}<\infty
|
||||
\]
|
||||
|
@ -67,8 +67,8 @@ Dann sind alle \(f_{n}\) integrierbar und es existiert ein \(f\in\fl^{1}(X)\) mi
|
|||
\[
|
||||
\int_{X}{f_{n}\mathrm{d}x}=n\cdot\lambda_{1}\left(\left(0,\frac{1}{n}\right)\right)=n\cdot\frac{1}{n}=1\quad\forall n\in\mdn
|
||||
\]
|
||||
Es gilt \(f_{n}\to f:=0\) punktweise und \(\int_{X}{f\mathrm{d}x}=0 \neq 1 = \int_{X}{f_{n}\mathrm{d}x}\).
|
||||
$\Rightarrow$ \ref{Satz 6.2} ist ohne die integrierbare Majorante
|
||||
Es gilt \(f_{n}\to f:=0\) punktweise und \(\int_{X}{f\mathrm{d}x}=0 \neq 1 = \int_{X}{f_{n}\mathrm{d}x}\).
|
||||
$\Rightarrow$ \ref{Satz 6.2} ist ohne die integrierbare Majorante
|
||||
$g$ im allgemeinen falsch.
|
||||
\item Sei $X = [1, \infty), \alpha > 1, f_n(x) := \frac{1}{x^\alpha} \sin{\frac{x}{n}} (x \in X, n \in \mathbb{N})$.\\
|
||||
Berechne $\lim_{n \rightarrow \infty} \int_X f_n(x) \mathrm{d}x$\\
|
||||
|
@ -132,9 +132,9 @@ Also gilt auch:
|
|||
|
||||
\begin{beispiel}
|
||||
Sei \(X:=[1,\infty)\) und \(f_n(x):=\frac1{x^\frac32}\sin\left(\frac xn \right) \) für alle \(x\in X, n\in\mdn\) mit \(f_n(x)\to f(x)\equiv 0\) für jedes \(x\in X\).
|
||||
Dann ist \(\lvert f_n(x) \rvert\leq \frac1{x^\frac32}\) für jedes \(x\in X\) und $\natn$.
|
||||
Dann ist \(\lvert f_n(x) \rvert\leq \frac1{x^\frac32}\) für jedes \(x\in X\) und $\natn$.
|
||||
Definiere nun \[g(x):=\frac1{x^\frac32}\]
|
||||
Aus Analysis I ist bekannt, dass \(\int^\infty_1 g(x)\,dx\) (absolut) konvergent ist
|
||||
Aus Analysis I ist bekannt, dass \(\int^\infty_1 g(x)\,dx\) (absolut) konvergent ist
|
||||
und aus \ref{Satz 4.14} folgt \[g\in\mathfrak{L}^1(X) \text{ sowie } \int_X g(x)\,dx = \text{R-}\int^\infty_1 g(x)\,dx\]
|
||||
Weiter folgen aus \ref{Satz 6.2}:
|
||||
\[\int_X f_n\,dx\to 0 \text{ und } \int_X\lvert f_n\rvert\,dx\to 0 \ (n\to\infty) \]
|
||||
|
@ -155,17 +155,17 @@ Weiter folgen aus \ref{Satz 6.2}:
|
|||
|
||||
\begin{beweis}
|
||||
\begin{enumerate}
|
||||
\item Sei \(x\in X\). Es exisitert ein $m\in\mdn$, für das \(x\in A_m\) ist und somit auch \(x\in A_n \) für jedes \(n\geq m\). Nach der Definition von $f_n$ gilt dann \(f_n(x)=f(x)\) für jedes \(n\geq m\) und somit \(f_n\to f\) auf $X$. Damit gilt auch \[\lvert f_n\rvert\to\lvert f\rvert \text{ auf } X\] Durch die Konstruktion der $f_n$ ergibt sich:
|
||||
\item Sei \(x\in X\). Es exisitert ein $m\in\mdn$, für das \(x\in A_m\) ist und somit auch \(x\in A_n \) für jedes \(n\geq m\). Nach der Definition von $f_n$ gilt dann \(f_n(x)=f(x)\) für jedes \(n\geq m\) und somit \(f_n\to f\) auf $X$. Damit gilt auch \[\lvert f_n\rvert\to\lvert f\rvert \text{ auf } X\] Durch die Konstruktion der $f_n$ ergibt sich:
|
||||
\[ \lvert f_n\rvert=\lvert \mathds{1}_{A_n}f\rvert=\mathds{1}_{A_n}\lvert f\rvert \leq \mathds{1}_{A_{n+1}}\lvert f\rvert=\lvert f_{n+1}\rvert \]
|
||||
Dann gilt:
|
||||
\[ \int_X \lvert f\rvert\,dx \overset{\ref{Satz 4.6}}=\lim\int_X \lvert f_n\rvert\,dx = \lim\int_{A_n} \lvert f\rvert\,dx \overset{Vor.}<\infty \]
|
||||
Es folgt, dass \(\lvert f\rvert\) integrierbar ist und somit ist nach \ref{Satz 4.9} auch $f$ integrierbar. Da \(\lvert f_n\rvert \leq \lvert f\rvert\) auf $X$ für jedes \(\natn\) gilt, ist $f$ eine
|
||||
Es folgt, dass \(\lvert f\rvert\) integrierbar ist und somit ist nach \ref{Satz 4.9} auch $f$ integrierbar. Da \(\lvert f_n\rvert \leq \lvert f\rvert\) auf $X$ für jedes \(\natn\) gilt, ist $f$ eine
|
||||
integrierbare Majorante und es folgt mit \ref{Satz 6.2}:
|
||||
\[ \int_Xf\,dx = \lim\int_Xf_n\,dx = \lim\int_{A_n}f\,dx \]
|
||||
\item Setze \(A_n:=[a,n]\ (\natn)\) und es gelte o.B.d.A.: \(a\leq 1\). Dann gilt:
|
||||
\[ \int_{A_n}\lvert f\rvert\,dx \overset{\ref{Satz 4.13}}= \text{R-}\int^n_a \lvert f\rvert\,dx \overset{Vor.}\longrightarrow \text{R-}\int^\infty_a \lvert f\rvert\,dx \]
|
||||
D.h.\(\left(\int_{A_n}\lvert f\rvert\,dx\right)\) ist beschränkt. Definiere \(f_n:=\mathds{1}_{A_n}f\) mit \ref{Satz 4.13} folgt daraus, dass $f_n$ integrierbar ist. Weiter folgt
|
||||
aus (1) \(f\in\mathfrak{L}^1(X)\) (denn es ist \(f(X)\subseteq\mdr\)) und
|
||||
aus (1) \(f\in\mathfrak{L}^1(X)\) (denn es ist \(f(X)\subseteq\mdr\)) und
|
||||
\[ \text{L-}\int_Xf\,dx = \lim\int_{A_n}f\,dx \overset{\ref{Satz 4.13}}= \lim\left(\text{R-}\int^n_a f\,dx \right) = \text{R-}\int^\infty_a f\,dx. \]
|
||||
\end{enumerate}
|
||||
\end{beweis}
|
||||
|
@ -177,12 +177,12 @@ Weiter folgen aus \ref{Satz 6.2}:
|
|||
\begin{folgerung}
|
||||
\label{Folgerung 6.4}
|
||||
\begin{enumerate}
|
||||
\item \((f_n)\) sei eine Folge integrierbarer Funktionen \(f_n\colon X\to\imdr\), \(g\colon X\to[0,+\infty]\) sei ebenfalls integrierbar und
|
||||
\item \((f_n)\) sei eine Folge integrierbarer Funktionen \(f_n\colon X\to\imdr\), \(g\colon X\to[0,+\infty]\) sei ebenfalls integrierbar und
|
||||
\[g_n:=f_1+f_2+\dots+f_n \ (\natn)\]
|
||||
Weiter sei $N$ eine Nullmenge in $X$ so, dass \((g_n(x))\) für jedes \(x\in X\setminus N\) in $\imdr$ konvergiert und
|
||||
Weiter sei $N$ eine Nullmenge in $X$ so, dass \((g_n(x))\) für jedes \(x\in X\setminus N\) in $\imdr$ konvergiert und
|
||||
\[\lvert g_n(x)\rvert \leq g(x) \text{ für jedes } \natn \text{ und } x\in X\setminus N\]
|
||||
Setzt man
|
||||
\[f(x):=\sum^\infty_{j=1}f_j(x):=
|
||||
\[f(x):=\sum^\infty_{j=1}f_j(x):=
|
||||
\begin{cases}
|
||||
0, & \text{falls } x\in N \\
|
||||
\lim\limits_{n\to\infty}g_n(x), & \text{falls } x\in X\setminus N
|
||||
|
@ -198,7 +198,7 @@ Weiter folgen aus \ref{Satz 6.2}:
|
|||
\begin{enumerate}
|
||||
\item Fast überall gelten \(g_n\to f\) und für jedes \(\natn\) auch \(\lvert g_n\rvert \leq g\). Aus \ref{Satz 6.2} folgt
|
||||
\begin{align*}
|
||||
\int_X \left(\sum^\infty_{j=1}f_j(x)\right) \,dx
|
||||
\int_X \left(\sum^\infty_{j=1}f_j(x)\right) \,dx
|
||||
&= \int_Xf\,dx \\
|
||||
&\overset{\ref{Satz 6.2}}= \lim\int_Xg_n\,dx \\
|
||||
&= \lim\int_X\left(\sum^n_{j=1}f_j\right)\,dx \\
|
||||
|
|
|
@ -6,7 +6,7 @@ Sei \(U\in\fb_k, t_0\in U\) und es sei \(f\colon U\times X\to \mdr\) eine Funkti
|
|||
\begin{enumerate}
|
||||
\item Für jedes \(t\in U\) ist \(x\mapsto f(t,x)\) messbar.
|
||||
\item Es existiert eine Nullmenge \(N\subseteq X\) so, dass \(t\mapsto f(t,x)\) für jedes \(x\in X\setminus N\) stetig in $t_0$ ist.
|
||||
\item Es existiert eine integrierbare Funktion \(g\colon X\to [0,\infty]\) und zu jedem \(t\in U\) existiert eine Nullmenge \(N_t\subseteq X\) so, dass für
|
||||
\item Es existiert eine integrierbare Funktion \(g\colon X\to [0,\infty]\) und zu jedem \(t\in U\) existiert eine Nullmenge \(N_t\subseteq X\) so, dass für
|
||||
jedes \(t\in U\) und jedes \(x\in X\setminus N_t\) gilt: \[ \lvert f(t,x)\rvert \leq g(x) \]
|
||||
\end{enumerate}
|
||||
Dann ist \(x\mapsto f(t,x)\) für jedes \(t\in U\) integrierbar. Ist \(F\colon U\to\mdr\) definiert durch
|
||||
|
@ -17,31 +17,31 @@ so ist $F$ stetig in $t_0$.
|
|||
Also: \[ \lim\limits_{t\to t_0}\int_X f(t,x)\,dx = \lim\limits_{t\to t_0}F(t)=F(t_0) = \int_X f(t_0,x)\,dx =\int_X\lim\limits_{t\to t_0} f(t,x)\,dx \]
|
||||
|
||||
\begin{beweis}
|
||||
Aus (1) und (3) folgt, dass \(x\mapsto f(t,x)\) für jedes \(t\in U\) integrierbar ist (zur Übung). Sei \((t_n)\) eine Folge in $U$ mit \(t_n\to t_0\) und
|
||||
Aus (1) und (3) folgt, dass \(x\mapsto f(t,x)\) für jedes \(t\in U\) integrierbar ist (zur Übung). Sei \((t_n)\) eine Folge in $U$ mit \(t_n\to t_0\) und
|
||||
\[g_n(x):=f(t_n,x) \ (\natn, x\in X) \]
|
||||
Setze \[ \tilde N := N\cup \left(\bigcup^\infty_{n=1}N_{t_n} \right) \]
|
||||
Aus \ref{Lemma 5.1} folgt, dass \(\tilde N\) eine Nullmenge ist. Voraussetzung (2) liefert \(g_n(x)\to f(t_0,x)\) für jedes \(x\in X\setminus\tilde N\), also gilt
|
||||
Aus \ref{Lemma 5.1} folgt, dass \(\tilde N\) eine Nullmenge ist. Voraussetzung (2) liefert \(g_n(x)\to f(t_0,x)\) für jedes \(x\in X\setminus\tilde N\), also gilt
|
||||
\[g_n(x)\to f(t_0,x) \text{ fast überall auf } X\]
|
||||
Voraussetzung (3) liefert \(\lvert g_n(x)\rvert = \lvert f(t_n,x)\rvert \leq g(x) \) für jedes \(\natn\) und \(x\in X\setminus\tilde N\). Aus \ref{Satz 6.2} folgt
|
||||
\[ F(t_n) = \int_X f(t_n,x)\,dx = \int_Xg_n\,dx \longrightarrow \int_X f(t_0,x)\,dx = F(t_0) \]
|
||||
\end{beweis}
|
||||
|
||||
\textbf{Bezeichnung}\\
|
||||
Sei \(I\subseteq\mdr\) ein Intervall, \(a:=\inf I\) und \(b:=\sup I\), wobei \(a=-\infty\) oder \(b=+\infty\) zugelassen sind. Weiter sei \(f\colon I\to\imdr\) integrierbar
|
||||
(oder $f$ ist messbar und \(\geq 0\)) und
|
||||
Sei \(I\subseteq\mdr\) ein Intervall, \(a:=\inf I\) und \(b:=\sup I\), wobei \(a=-\infty\) oder \(b=+\infty\) zugelassen sind. Weiter sei \(f\colon I\to\imdr\) integrierbar
|
||||
(oder $f$ ist messbar und \(\geq 0\)) und
|
||||
\[\int\limits^b_af(x)\,dx:=\int\limits_{(a,b)}f_{|(a,b)}(x)\,dx \]
|
||||
Dann ist
|
||||
Dann ist
|
||||
\[ \int_I f(x) dx = \int_{(a,b)} f(x) dx\]
|
||||
Ist z.B. \(I=[a,b)\), dann gilt, da \(\{a\}\) eine Nullmenge ist: \[\int_If\,dx=\int_{\{a\}}f\,dx + \int_{(a,b)}f\,dx= \int_{(a,b)}f\,dx \]
|
||||
Ist z.B. \(I=[a,b)\), dann gilt, da \(\{a\}\) eine Nullmenge ist: \[\int_If\,dx=\int_{\{a\}}f\,dx + \int_{(a,b)}f\,dx= \int_{(a,b)}f\,dx \]
|
||||
|
||||
\begin{folgerung}
|
||||
\label{Folgerung 7.2}
|
||||
Sei \(I\subseteq\mdr\) ein Intervall, \(a=\inf I\) und \(f\colon I\to\mdr\) sei integrierbar. Definiert man \(F\colon I\to\mdr\) durch
|
||||
Sei \(I\subseteq\mdr\) ein Intervall, \(a=\inf I\) und \(f\colon I\to\mdr\) sei integrierbar. Definiert man \(F\colon I\to\mdr\) durch
|
||||
\[F(t):=\int^t_a f(x)\,dx,\] so ist \(F\in C(I)\).
|
||||
\end{folgerung}
|
||||
|
||||
\begin{beweis}
|
||||
Für \(x,t\in I\) definiere \(h(t,x):=\mathds{1}_{(a,t)}f(x)\). Dann ist \(F(t)=\int_I h(t,x)\,dx\) und
|
||||
Für \(x,t\in I\) definiere \(h(t,x):=\mathds{1}_{(a,t)}f(x)\). Dann ist \(F(t)=\int_I h(t,x)\,dx\) und
|
||||
\[\lvert h(t,x)\rvert = \mathds{1}_{(a,t)}\cdot \lvert f(x)\rvert \leq \lvert f(x)\rvert \text{ für alle } t,x\in I\]
|
||||
Aus \ref{Satz 4.9} folgt, dass \(\lvert f\rvert\) integrierbar ist. Sei \(t_0\in I\) und \(N:=\{t_0\}\), also eine Nullmenge.
|
||||
Dann ist \(t\mapsto h(t,x)\) für jedes \(x\in I\setminus N\) stetig in \(t_0\) (zur Übung). Die Behauptung folgt aus \ref{Satz 7.1}.
|
||||
|
|
|
@ -44,7 +44,7 @@ C_y= \begin{cases}
|
|||
{\emptyset, \text{falls } x\notin A}\\
|
||||
{B, \text{falls } x\in A}
|
||||
\end{cases}
|
||||
\end{align*}
|
||||
\end{align*}
|
||||
|
||||
\begin{lemma}
|
||||
\label{Lemma 8.3}
|
||||
|
@ -74,11 +74,11 @@ folgt aus \ref{Lemma 8.1} und \ref{Lemma 8.3}.
|
|||
\end{beweis}
|
||||
|
||||
%vielleicht funktioniert die nummerierung jetzt
|
||||
\begin{defusatz}[ohne Beweis]
|
||||
\begin{defusatz}[ohne Beweis]
|
||||
\label{Satz 8.5}
|
||||
Sei \(C\in\fb_d\). Die Funktionen \(\varphi_C\) und \(\psi_C\) seien unter Beachtung von \ref{Lemma 8.2} definiert durch:
|
||||
\begin{align*}
|
||||
\varphi_C(x):=\lambda_l(C^x) \ \ (x\in\mdr^k) & & \psi_C(x):=\lambda_k(C_y) \ \ (y\in\mdr^l)
|
||||
\varphi_C(x):=\lambda_l(C^x) \ \ (x\in\mdr^k) & & \psi_C(x):=\lambda_k(C_y) \ \ (y\in\mdr^l)
|
||||
\end{align*}
|
||||
Dann sind \(\varphi_C\) und \(\psi_C\) messbar.
|
||||
\end{defusatz}
|
||||
|
|
|
@ -26,7 +26,7 @@ Das heißt:
|
|||
&\overset{Ana I}= \pi r^2
|
||||
\end{align*}
|
||||
\item Sei \(\emptyset\neq X\subseteq\mdr^d\). $X$ sei kompakt, also \(X\in\fb_d\). Weiter sei \(f\colon X\to[0,\infty)\) stetig, woraus mit \ref{Satz 4.11} \(f\in\mathfrak{L}^1(X)\) folgt.
|
||||
Setze \[C:=\{(x,y):x\in X, 0\leq y\leq f(x)\}\]
|
||||
Setze \[C:=\{(x,y):x\in X, 0\leq y\leq f(x)\}\]
|
||||
$C$ ist kompakt und somit gilt: \(C\in\fb_{d+1}\).\\
|
||||
Ist \(x\notin X\), so ist \(C^x=\emptyset\), also \(\lambda_1(C^x)=0\).\\
|
||||
Ist \(x\in X\), so ist \(C^x=[0,f(x)]\), also \(\lambda_1(C^x)=f(x)\). Damit gilt
|
||||
|
@ -35,7 +35,7 @@ Das heißt:
|
|||
\[C:=\{(x,y)\in\mdr^2:x\in I, 0\leq y\leq f(x)\}\]
|
||||
Aus Beispiel (2) und \ref{Satz 4.13} folgt \[\lambda_2(C)=\text{R-}\int_a^bf(x)\,dx \]
|
||||
\item $X$ und $f$ seien wie in Beispiel (2). Setze \[G:=\{(x,f(x)):x\in X\}\]
|
||||
$G$ ist kompakt, also ist \(G\in\fb_2\).
|
||||
$G$ ist kompakt, also ist \(G\in\fb_2\).
|
||||
Ist \(x\notin X\), so ist \(G^x=\emptyset\), also \(\lambda_1(G^x)=0\).
|
||||
Ist \(x\in X\), so ist \(G^x=\{f(x)\}\), also \(\lambda_1(G^x)=0\).
|
||||
Aus \ref{Satz 9.1} folgt \[\lambda_2(G)=\int_\mdr\lambda_1(G^x)\,dx=0\]
|
||||
|
@ -58,7 +58,7 @@ Sei $(A_j)$ eine disjunkte Folge in $\fb_d$. Dann ist $(A_j^x)$ ebenfalls disjun
|
|||
D.h. $\mu$ ist ein Maß auf $\fb_d$. Analog lässt sich zeigen, dass $\nu$ ein Maß auf $\fb_d$ ist.\\
|
||||
Sei nun $I\in\ci_d$, dann existieren $I'\in\ci_k, I''\in\ci_l$ mit $I=I'\times I''$. Aus §\ref{Kapitel 8} folgt:
|
||||
\begin{align*}
|
||||
I^x=\begin{cases} I''&,x\in I'\\
|
||||
I^x=\begin{cases} I''&,x\in I'\\
|
||||
\emptyset &,x\not\in I'\end{cases}
|
||||
\end{align*}
|
||||
Also ist $\lambda_l(I^x)=\lambda_l(I'')\cdot\mathds{1}_{I'}(x)$ und damit:
|
||||
|
@ -111,7 +111,7 @@ Also folgt aus \ref{Satz 3.4} die Messbarkeit von $\tilde f$.
|
|||
\begin{beispiel}
|
||||
\index{Rotationskörper}
|
||||
\begin{enumerate}
|
||||
\item Sei $r>0$ und
|
||||
\item Sei $r>0$ und
|
||||
\[K:=\{(x,y)\in\mdr^2\mid x^2+y^2<r^2\}\]
|
||||
Dann ist $K$ offen, also $K\in\fb_2$ und es gilt:
|
||||
\[\partial K=\overline{K}\setminus K=\{(x,y)\in\mdr^2\mid x^2+y^2=r^2\}\in\fb_2\]
|
||||
|
@ -120,7 +120,7 @@ Damit enthält die Menge $(\partial K)_y$ für alle $x\in\mdr$ höchstens zwei E
|
|||
Mit $\overline K=(\partial K) \dot\cup K$ folgt dann
|
||||
\[\lambda_2(K)=\lambda_2(\partial K)+\lambda_2(\overline K)=\lambda_2(\overline K)=\pi r^2\]
|
||||
Sei nun $A\in\fb_2$ mit $K\subseteq A\subseteq\overline K$, dann ist $\lambda_2(A)=\pi r^2$.
|
||||
\item Sei $r>0$ und
|
||||
\item Sei $r>0$ und
|
||||
\[K:=\{(x,y,z)\in\mdr^3\mid x^2+y^2+z^2\le r^2\}\]
|
||||
Dann ist $K$ abgeschlossen, also $K\in\fb_3$.\\
|
||||
\textbf{Fall $|z|>r$:} Es ist $K_z=\emptyset$, also $\lambda_2(K_z)=0$.\\
|
||||
|
@ -146,7 +146,7 @@ und damit $\lambda_2(V_z)=\pi f(z)^2$.\\
|
|||
Aus \ref{Satz 9.1} folgt dann:
|
||||
\begin{align*}
|
||||
\lambda_3(V)&=\int_\mdr \lambda_2(V_z)\text{ d}z\\
|
||||
&= \pi\int_a^b f(z)^2\text{ d}z
|
||||
&= \pi\int_a^b f(z)^2\text{ d}z
|
||||
\end{align*}
|
||||
\item Sei $h>0$, $I=[0,h]$ und $f(z)=\frac rhz$. Definiere den Kegel
|
||||
\[V:=\{(x,y,z)\in\mdr^3\mid x^2+y^2\le \frac{r^2}{h^2}z^2\}\]
|
||||
|
|
|
@ -15,7 +15,7 @@
|
|||
\fancyhead{}
|
||||
\renewcommand{\headrulewidth}{0pt}
|
||||
}
|
||||
|
||||
|
||||
\setlength{\headheight}{15pt} % fixes \headheight warning
|
||||
\lhead{\Vorname{}~\Nachname{}, \Strasse{}, \PLZ{}~\Ort}
|
||||
\rhead{Id-Nr. \Idnr}
|
||||
|
|
|
@ -5,7 +5,7 @@
|
|||
\usepackage[ngerman, num]{isodate} % get DD.MM.YYYY dates
|
||||
\usepackage{pdfpages} % Signatureinbingung und includepdf
|
||||
\usepackage{myInformation}
|
||||
|
||||
|
||||
% pdfinfo
|
||||
\pdfinfo{
|
||||
/Author (\Nachname, \Vorname)
|
||||
|
@ -18,7 +18,7 @@
|
|||
\signature{\Vorname~\Nachname}
|
||||
\setkomavar{customer}[Steuernummer (Id-Nr.)]{\Idnr}
|
||||
\backaddress{\Vorname~\Nachname, \Strasse~\Hausnummer, \PLZ~\Ort}
|
||||
|
||||
|
||||
% Begin document %%%%%%%%%%%%%%%%%%%%%%%%%%%%%%%%%%%%%%%%%%%%%%%%%%%%
|
||||
\begin{document}
|
||||
\begin{letter}{\Empfaenger \\ \EStrasse \\ \EPLZ~\EOrt}
|
||||
|
@ -26,7 +26,7 @@
|
|||
\subject{Einnahmenüberschussrechnung \Year}
|
||||
\opening{Sehr geehrte Damen und Herren,}
|
||||
|
||||
Im Anhang befindet sich die Überschussrechnung von \Year.
|
||||
Im Anhang befindet sich die Überschussrechnung von \Year.
|
||||
|
||||
\closing{Mit freundlichen Grüßen,}
|
||||
\end{letter}
|
||||
|
|
|
@ -1,5 +1,5 @@
|
|||
* Einmalig müssen Parameter in der `myInformation.tex` bearbeitet werden.
|
||||
* Dann nur noch jedes Jahr die vier `.csv`-Dateien und gegebenenfalls
|
||||
* Dann nur noch jedes Jahr die vier `.csv`-Dateien und gegebenenfalls
|
||||
unter `Einnahmenueberschussrechnung.tex` Anmerkungen machen
|
||||
|
||||
Tags: Steuer, Steuererklärung, LaTeX, Finanzen, EÜR
|
||||
|
|
|
@ -19,10 +19,10 @@
|
|||
\newcommand\yourTutorial{10}
|
||||
%%%%%%%%%%%%%%%%%%%%%%%%%%%%%%%%%%%%%%%%%%%%%%%%%%%%%%%%%%%%%%%%%%%%%
|
||||
|
||||
\hypersetup {
|
||||
pdfauthor = {Martin Thoma},
|
||||
pdfkeywords = {Feedback},
|
||||
pdftitle = {Feedback}
|
||||
\hypersetup {
|
||||
pdfauthor = {Martin Thoma},
|
||||
pdfkeywords = {Feedback},
|
||||
pdftitle = {Feedback}
|
||||
}
|
||||
|
||||
\pagestyle{fancy}% eigenen Seitestil aktivieren}
|
||||
|
@ -74,7 +74,7 @@ Diese Themen sollten wiederholt werden (z.B. Arrays, for/while Schleifen, Ausdr
|
|||
\hfill\vspace{3cm}
|
||||
\end{framed}
|
||||
|
||||
\noindent \textbf{Andere Kommentare}: z.B. Was ist unklar?
|
||||
\noindent \textbf{Andere Kommentare}: z.B. Was ist unklar?
|
||||
Was war heute gut / schlecht?
|
||||
\begin{framed}
|
||||
\hfill\vspace{3cm}
|
||||
|
|
|
@ -18,7 +18,7 @@ Folgende Definition wurde dem Skript von Herrn Prof.~Dr.~Leuzinger für
|
|||
Lineare Algebra entnommen:
|
||||
|
||||
\begin{definition}\xindex{Abbildung!affine}%
|
||||
Es seien $V$ und $W$ $\mdk$-Vektorräume und $\mda(V)$ und $\mda(W)$ die
|
||||
Es seien $V$ und $W$ $\mdk$-Vektorräume und $\mda(V)$ und $\mda(W)$ die
|
||||
zugehörigen affinen Räume. Eine Abbildung $f:V \rightarrow W$ heißt \textbf{affin},
|
||||
falls für alle $a, b \in V$ und alle $\lambda, \mu \in \mdk$ mit $\lambda + \mu = 1$ gilt:
|
||||
\[f(\lambda a + \mu b) = \lambda f(a) + \mu f(b)\]
|
||||
|
@ -36,13 +36,13 @@ Lineare Algebra entnommen:
|
|||
\end{definition}
|
||||
|
||||
\begin{satz*}[Zwischenwertsatz]\xindex{Zwischenwertsatz}%
|
||||
Sei $a<b$ und $f \in\ C[a, b]:=C([a, b])$, weiter sei $y_0 \in \mdr$ und
|
||||
$f(a) < y_0 < f(b)$ oder $f(b) < y_0 < f(a)$. Dann existiert ein
|
||||
Sei $a<b$ und $f \in\ C[a, b]:=C([a, b])$, weiter sei $y_0 \in \mdr$ und
|
||||
$f(a) < y_0 < f(b)$ oder $f(b) < y_0 < f(a)$. Dann existiert ein
|
||||
$x_0 \in [a, b]$ mit $f(x_0) = y_0$.
|
||||
\end{satz*}
|
||||
|
||||
\begin{definition}\xindex{Eigenwert}\xindex{Eigenvektor}%
|
||||
Sei $V$ ein Vektorraum über einem Körper $\mdk$ und $f: V \rightarrow V$ eine
|
||||
Sei $V$ ein Vektorraum über einem Körper $\mdk$ und $f: V \rightarrow V$ eine
|
||||
lineare Abbildung.
|
||||
|
||||
$v \in V \setminus \Set{0}$ heißt \textbf{Eigenvektor} $:\Leftrightarrow \exists \lambda \in \mdk: f(v) = \lambda v$.
|
||||
|
|
|
@ -39,10 +39,10 @@
|
|||
\usepackage[left=10mm,right=10mm, top=2mm, bottom=10mm]{geometry}
|
||||
\usepackage{../shortcuts}
|
||||
|
||||
\hypersetup{
|
||||
pdfauthor = {Martin Thoma},
|
||||
pdfkeywords = {Geometrie und Topologie},
|
||||
pdftitle = {Fragen zu Definitionen}
|
||||
\hypersetup{
|
||||
pdfauthor = {Martin Thoma},
|
||||
pdfkeywords = {Geometrie und Topologie},
|
||||
pdftitle = {Fragen zu Definitionen}
|
||||
}
|
||||
\allowdisplaybreaks
|
||||
|
||||
|
@ -73,7 +73,7 @@ $f:|K| \rightarrow |L|$
|
|||
mit $f(\Delta) \notin L$?}
|
||||
|
||||
\section*{18.) ÜB 1, Aufgabe 2}
|
||||
\underline{Vor.:} Es sei $(X, d)$ ein metrischer Raum, $A \subseteq X$.
|
||||
\underline{Vor.:} Es sei $(X, d)$ ein metrischer Raum, $A \subseteq X$.
|
||||
Weiter bezeichne $\fT$ die von $d$ auf $X$ erzeugte Topologie $\fT'$, die von
|
||||
der auf $A \times A$ eingeschränkten Metrik $d|_{A \times A}$ erzeugte Topologie.
|
||||
|
||||
|
@ -87,7 +87,7 @@ Sei $U \in \fT|_A = \Set{V \cap A | V \in \fT}$.\\
|
|||
Dann ex. also $V \in \fT$ mit
|
||||
$U = V \cap A$.\\
|
||||
Sei $x \in U$.\\
|
||||
Da $V \in \fT$, ex. nach Bemerkung~3 ein $r > 0$ mit
|
||||
Da $V \in \fT$, ex. nach Bemerkung~3 ein $r > 0$ mit
|
||||
|
||||
\begin{align*}
|
||||
\fB_r(x) := \Set{y \in X | d(x,y) < r} &\subseteq V\\
|
||||
|
@ -123,7 +123,7 @@ Da $x \in U$ beliebig gewählt war gilt: $\fT|_A \subseteq \fT'$
|
|||
\[m_g: X \rightarrow X, x \mapsto g \circ x\]
|
||||
ein Homöomorphismus ist.
|
||||
\item Ist $G$ eine topologische Gruppe, so heißt die Gruppenoperation $\circ$
|
||||
\textbf{stetig}\xindex{Gruppenoperation!stetige}, wenn
|
||||
\textbf{stetig}\xindex{Gruppenoperation!stetige}, wenn
|
||||
$\circ: G \times X \rightarrow X$ stetig ist.
|
||||
\end{defenum}
|
||||
\end{definition}
|
||||
|
@ -172,7 +172,7 @@ $\Rightarrow$ Widerspruch
|
|||
Da $r > 0$ ist $H_1$ nicht leer, da $r \in \mdr$ ist $H_2$ nicht leer.
|
||||
|
||||
\underline{Zu zeigen:} $\forall A \in H_i$, $B \in H_j$ mit
|
||||
$i,j \in \Set{1,2}$ gilt:
|
||||
$i,j \in \Set{1,2}$ gilt:
|
||||
$\overline{AB} \cap g \neq \emptyset \Leftrightarrow i \neq j$\\
|
||||
\enquote{$\Leftarrow$}: Da $d_\mdh$ stetig ist, folgt diese Richtung
|
||||
direkt. Alle Punkte in $H_1$ haben einen Abstand von $m$ der kleiner
|
||||
|
@ -189,7 +189,7 @@ $\Rightarrow$ Widerspruch
|
|||
\[\mdh = \underbrace{\Set{z \in \mdh | \Re(z) < x}}_{=: H_1 \text{ (Links)}} \dcup \underbrace{\Set{z \in \mdh | \Re(z) > x}}_{=: H_2 \text{ (Rechts)}}\]
|
||||
|
||||
\underline{Zu zeigen:} $\forall A \in H_i$, $B \in H_j$ mit
|
||||
$i,j \in \Set{1,2}$ gilt:
|
||||
$i,j \in \Set{1,2}$ gilt:
|
||||
$\overline{AB} \cap g \neq \emptyset \Leftrightarrow i \neq j$\\
|
||||
\enquote{$\Leftarrow$}: Wie zuvor mit dem Zwischenwertsatz.
|
||||
|
||||
|
@ -207,7 +207,7 @@ $\Rightarrow$ Widerspruch
|
|||
\begin{enumerate}
|
||||
\item Deformationsretrakt: Das hatten wir nicht in der Vorlesung, oder? Ich meine mich zwar an das Wort zu erinnern (aus einem Übungsblatt? Einem Tutorium?) Könntest du bitte nochmals erklären was das ist?
|
||||
Das ist zwar auf Blatt 7 und 8 vorgekommen, aber sonst nie.
|
||||
\item Damit verbunden: Was genau ist eine "Einbettung"?
|
||||
\item Damit verbunden: Was genau ist eine "Einbettung"?
|
||||
\item Was bedeutet der Pfeil: $f:S^1 \hookrightarrow \mdr^2\;\;\;$ Einbettung der Kreislinie in die Ebene
|
||||
\item Was ist eine Inklusionsabbildung?
|
||||
\item Was ist ein Homotopietyp? (Ist das eventuell die Anzahl der Homotopieklassen?)
|
||||
|
|
|
@ -11,7 +11,7 @@
|
|||
\item Ist $I$ eine Menge und $U_i \in \fT$ für jedes $i \in I$,
|
||||
so ist $\displaystyle \bigcup_{i \in I} U_i \in \fT$
|
||||
\end{defenumprops}
|
||||
Die Elemente von $\fT$ heißen \textbf{offene Teilmengen} von $X$.
|
||||
Die Elemente von $\fT$ heißen \textbf{offene Teilmengen} von $X$.
|
||||
|
||||
$A \subseteq X$ heißt \textbf{abgeschlossen}, wenn $X \setminus A$ offen ist.
|
||||
\end{definition}
|
||||
|
@ -44,7 +44,7 @@ Auch gibt es Mengen, die sowohl abgeschlossen als auch offen sind.
|
|||
\item Jeder metrische Raum $(X, d)$ ist auch ein topologischer Raum.
|
||||
\item Für eine Menge $X$ heißt $\fT_{\ts{Diskret}} = \powerset{X}$ \textbf{diskrete Topologie}\xindex{Topologie!diskrete}.
|
||||
\item $X :=\mdr, \fT_Z := \Set{U \subseteq \mdr | \mdr \setminus U \text{ endlich}} \cup \Set{\emptyset}$ heißt \textbf{Zariski-Topologie} \xindex{Topologie!Zariski}\\
|
||||
Beobachtungen:
|
||||
Beobachtungen:
|
||||
\begin{itemize}
|
||||
\item $U \in \fT_Z \gdw \exists f \in \mdr[X]$, sodass $\mdr \setminus U = V(f) = \Set{x \in \mdr | f(x) = 0}$
|
||||
\item Es gibt keine disjunkten offenen Mengen in $\fT_Z$.
|
||||
|
@ -77,10 +77,10 @@ Auch gibt es Mengen, die sowohl abgeschlossen als auch offen sind.
|
|||
|
||||
\begin{beispiel}
|
||||
\begin{bspenum}
|
||||
\item Sei $X = \mdr$ mit euklidischer Topologie und
|
||||
$M = \mdq$. Dann gilt: $\overline{M} = \mdr$ und
|
||||
\item Sei $X = \mdr$ mit euklidischer Topologie und
|
||||
$M = \mdq$. Dann gilt: $\overline{M} = \mdr$ und
|
||||
$M^\circ = \emptyset$
|
||||
\item Sei $X = \mdr$ und $M=(a,b)$. Dann gilt:
|
||||
\item Sei $X = \mdr$ und $M=(a,b)$. Dann gilt:
|
||||
$\overline{M} = [a,b]$
|
||||
\item Sei $X = \mdr, \fT = \fT_Z$ und $M = (a,b)$. Dann gilt:
|
||||
$\overline{M} = \mdr$
|
||||
|
@ -102,14 +102,14 @@ Auch gibt es Mengen, die sowohl abgeschlossen als auch offen sind.
|
|||
\begin{beispiel}[Basis und Subbasis]
|
||||
\begin{bspenum}
|
||||
\item Jede Basis ist auch eine Subbasis, z.B.\\
|
||||
$S=\Set{ (a,b) | a,b \in \mdr, a<b }$ ist für $\mdr$ mit der
|
||||
$S=\Set{ (a,b) | a,b \in \mdr, a<b }$ ist für $\mdr$ mit der
|
||||
Standardtopologie sowohl Basis als auch Subbasis.
|
||||
\item Gegeben sei $X = \mdr^n$ mit euklidischer Topologie $\fT$. Dann ist
|
||||
\[\fB = \Set{B_r(x) | r \in \mdq_{> 0}, x \in \mdq^n}\]
|
||||
ist eine abzählbare Basis von $\fT$.
|
||||
\item Sei $(X, \fT)$ ein topologischer Raum mit
|
||||
\item Sei $(X, \fT)$ ein topologischer Raum mit
|
||||
$X = \Set{0,1,2}$ und $\fT = \Set{\emptyset, \Set{0}, \Set{0,1}, \Set{0,2}, X}$.\\
|
||||
Dann ist $\calS = \Set{\emptyset, \Set{0,1}, \Set{0,2}}$ eine Subbasis von
|
||||
Dann ist $\calS = \Set{\emptyset, \Set{0,1}, \Set{0,2}}$ eine Subbasis von
|
||||
$\fT$, da gilt:
|
||||
\begin{itemize}
|
||||
\item $\calS \subseteq \fT$
|
||||
|
@ -132,11 +132,11 @@ Auch gibt es Mengen, die sowohl abgeschlossen als auch offen sind.
|
|||
Sei $(X, \fT)$ ein topologischer Raum und $Y \subseteq X$.\\
|
||||
$\fT_Y := \Set{U \cap Y | U \in \fT}$ ist eine Topologie auf $Y$.
|
||||
|
||||
$\fT_Y$ heißt \textbf{Teilraumtopologie} und $(Y, \fT_Y)$ heißt ein
|
||||
$\fT_Y$ heißt \textbf{Teilraumtopologie} und $(Y, \fT_Y)$ heißt ein
|
||||
\textbf{Teilraum} von $(X, \fT)$.
|
||||
\end{definition}
|
||||
|
||||
Die Teilraumtopologie wird auch \textit{Spurtopologie} oder
|
||||
Die Teilraumtopologie wird auch \textit{Spurtopologie} oder
|
||||
\textit{Unterraumtopologie} genannt.
|
||||
|
||||
%%%%%%%%%%%%%%%%%%%%%%%%%%%%%%%%%%%%%%%%%%%%%%%%%%%%%%%%%%%%%%%%%%%%%
|
||||
|
@ -190,14 +190,14 @@ Die Teilraumtopologie wird auch \textit{Spurtopologie} oder
|
|||
|
||||
\begin{beispiel}
|
||||
$X = \mdr, a \sim b :\Leftrightarrow a-b \in \mdz$
|
||||
|
||||
|
||||
\input{figures/number-ray-circle-topology}
|
||||
|
||||
$0 \sim 1$, d.~h. $[0] = [1]$
|
||||
\end{beispiel}
|
||||
|
||||
\begin{beispiel}\xindex{Torus}%
|
||||
Sei $X = \mdr^2$ und $(x_1, y_1) \sim (x_2, y_2) \gdw x_1 - x_2 \in \mdz$
|
||||
Sei $X = \mdr^2$ und $(x_1, y_1) \sim (x_2, y_2) \gdw x_1 - x_2 \in \mdz$
|
||||
und $y_1 - y_2 \in \mdz$. Dann ist $X /_\sim$ ein Torus.
|
||||
\end{beispiel}
|
||||
|
||||
|
@ -234,7 +234,7 @@ Die Teilraumtopologie wird auch \textit{Spurtopologie} oder
|
|||
|
||||
\begin{definition}\xindex{Isometrie}\label{def:Isometrie}%
|
||||
Seien $(X, d_X)$ und $(Y, d_Y)$ metrische Räume und $\varphi: X \rightarrow Y$
|
||||
eine Abbildung mit
|
||||
eine Abbildung mit
|
||||
\[\forall x_1, x_2 \in X: d_X(x_1, x_2) = d_Y(\varphi(x_1), \varphi(x_2)) \]
|
||||
|
||||
Dann heißt $\varphi$ eine \textbf{Isometrie} von $X$ nach $Y$.
|
||||
|
@ -252,7 +252,7 @@ Die Teilraumtopologie wird auch \textit{Spurtopologie} oder
|
|||
0 & \text{falls } x=y\\
|
||||
1 & \text{falls } x \neq y
|
||||
\end{cases}\]
|
||||
die \textbf{diskrete Metrik}. Die Metrik $d$ induziert die
|
||||
die \textbf{diskrete Metrik}. Die Metrik $d$ induziert die
|
||||
\textbf{diskrete Topologie}.
|
||||
\end{beispiel}
|
||||
\clearpage
|
||||
|
@ -280,7 +280,7 @@ Die Teilraumtopologie wird auch \textit{Spurtopologie} oder
|
|||
\end{beispiel}
|
||||
\clearpage
|
||||
\begin{beispiel}[SNCF-Metrik\footnotemark]\xindex{Metrik!SNCF}
|
||||
$X = \mdr^2$
|
||||
$X = \mdr^2$
|
||||
|
||||
\input{figures/sncf-metrik}
|
||||
\end{beispiel}
|
||||
|
@ -293,7 +293,7 @@ Die Teilraumtopologie wird auch \textit{Spurtopologie} oder
|
|||
\end{definition}
|
||||
|
||||
\begin{bemerkung}[Trennungseigenschaft]\label{Trennungseigenschaft}
|
||||
Metrische Räume sind hausdorffsch, wegen
|
||||
Metrische Räume sind hausdorffsch, wegen
|
||||
\[d(x, y) > 0 \Rightarrow \exists \varepsilon > 0: \fB_\varepsilon(x) \cap \fB_\varepsilon(y) = \emptyset\]
|
||||
\end{bemerkung}
|
||||
|
||||
|
@ -337,7 +337,7 @@ Die Teilraumtopologie wird auch \textit{Spurtopologie} oder
|
|||
Sei $(x_n)$ eine konvergierende Folge und $x$ und $y$ Grenzwerte der Folge.
|
||||
|
||||
Da $X$ hausdorffsch ist, gibt es Umgebungen $U_x$ von $x$ und $U_y$
|
||||
von $y$ mit $U_x \cap U_y = \emptyset$ falls $x \neq y$. Da
|
||||
von $y$ mit $U_x \cap U_y = \emptyset$ falls $x \neq y$. Da
|
||||
$(x_n)$ gegen $x$ und $y$ konvergiert, existiert ein
|
||||
$n_0$ mit $x_n \in U_x \cap U_y$ für alle $n \geq n_0$
|
||||
$\Rightarrow x = y \qed$
|
||||
|
@ -345,14 +345,14 @@ Die Teilraumtopologie wird auch \textit{Spurtopologie} oder
|
|||
|
||||
\section{Stetigkeit}\index{Stetigkeit|(}
|
||||
\begin{definition}
|
||||
Seien $(X, \fT_X), (Y, \fT_Y)$ topologische Räume und
|
||||
Seien $(X, \fT_X), (Y, \fT_Y)$ topologische Räume und
|
||||
$f:X \rightarrow Y$ eine Abbildung.
|
||||
|
||||
\begin{defenum}
|
||||
\item \label{def:stetigkeit} $f$ heißt \textbf{stetig}\xindex{Abbildung!stetige}
|
||||
$:\gdw \forall U \in \fT_Y: f^{-1} (U) \in \fT_X$.
|
||||
\item \label{def:homoeomorphismus} $f$ heißt \textbf{Homöomorphismus}\xindex{Homöomorphismus}, wenn $f$ stetig ist
|
||||
und es eine
|
||||
und es eine
|
||||
stetige Abbildung $g: Y \rightarrow X$ gibt, sodass
|
||||
$g \circ f = \id_X$ und $f \circ g = \id_Y$.
|
||||
\end{defenum}
|
||||
|
@ -378,9 +378,9 @@ Die Teilraumtopologie wird auch \textit{Spurtopologie} oder
|
|||
\enquote{$\Rightarrow$}: Sei $x \in X, \varepsilon > 0$ gegeben
|
||||
und $U := \fB_\varepsilon(f(x))$.\\
|
||||
Dann ist $U$ offen in $Y$.\\
|
||||
$\xRightarrow{\crefabbr{def:stetigkeit}} f^{-1}(U)$ ist
|
||||
$\xRightarrow{\crefabbr{def:stetigkeit}} f^{-1}(U)$ ist
|
||||
offen in $X$. Dann ist $x \in f^{-1}(U)$.\\
|
||||
$\Rightarrow \exists \delta > 0$, sodass
|
||||
$\Rightarrow \exists \delta > 0$, sodass
|
||||
$\fB_\delta(x) \subseteq f^{-1} (U)$\\
|
||||
$\Rightarrow f(\fB_\delta(x)) \subseteq U$\\
|
||||
$\Rightarrow \Set{y \in X | d_X(x,y) < \delta} \Rightarrow$ Beh.
|
||||
|
@ -415,18 +415,18 @@ Die Teilraumtopologie wird auch \textit{Spurtopologie} oder
|
|||
\begin{figure}[htp]
|
||||
\centering
|
||||
\input{figures/topology-continuous-mapping}
|
||||
\caption{Beispiel einer stetigen Funktion $f$, deren
|
||||
\caption{Beispiel einer stetigen Funktion $f$, deren
|
||||
Umkehrabbildung $g$ nicht stetig ist.}
|
||||
\label{fig:nicht-stetige-umkehrabbildung}
|
||||
\end{figure}
|
||||
|
||||
|
||||
Die Umkehrabbildung $g$ ist nicht stetig, da $g^{-1}(U)$
|
||||
nicht offen ist (vgl. \cref{fig:nicht-stetige-umkehrabbildung}).
|
||||
\end{bspenum}
|
||||
\end{beispiel}
|
||||
|
||||
\begin{bemerkung}[Verkettungen stetiger Abbildungen sind stetig]
|
||||
Seien $X, Y, Z$ topologische Räume, $f:X \rightarrow Y$ und
|
||||
Seien $X, Y, Z$ topologische Räume, $f:X \rightarrow Y$ und
|
||||
$g:Y \rightarrow Z$ stetige Abbildungen.
|
||||
|
||||
Dann ist $g \circ f: X \rightarrow Z$ stetig.
|
||||
|
@ -449,10 +449,10 @@ Die Teilraumtopologie wird auch \textit{Spurtopologie} oder
|
|||
|
||||
\begin{bemerkung}
|
||||
\begin{bemenum}
|
||||
\item \xindex{Homöomorphismengruppe}Für jeden topologischen Raum $X$ ist
|
||||
\item \xindex{Homöomorphismengruppe}Für jeden topologischen Raum $X$ ist
|
||||
\[\Homoo(X) := \Set{f: X \rightarrow X | f \text{ ist Homöomorphismus}}\]
|
||||
eine Gruppe.
|
||||
\item \xindex{Isometrie}Jede Isometrie $f:X \rightarrow Y$ zwischen metrischen
|
||||
\item \xindex{Isometrie}Jede Isometrie $f:X \rightarrow Y$ zwischen metrischen
|
||||
Räumen ist ein Homöomorphismus.
|
||||
\item \xindex{Isometriegruppe}$\Iso(X) := \Set{f:X \rightarrow X | f \text{ ist Isometrie}}$ ist
|
||||
eine Untergruppe von $\Homoo(X)$ für jeden
|
||||
|
@ -462,7 +462,7 @@ Die Teilraumtopologie wird auch \textit{Spurtopologie} oder
|
|||
|
||||
\begin{bemerkung}[Projektionen sind stetig]
|
||||
Seien $X, Y$ topologische Räume. $\pi_X: X \times Y \rightarrow X$
|
||||
und $\pi_Y: X \times Y \rightarrow Y$ die Projektionen
|
||||
und $\pi_Y: X \times Y \rightarrow Y$ die Projektionen
|
||||
\[\pi_X: (x,y) \mapsto x \text{ und } \pi_Y: (x,y) \mapsto y\]
|
||||
Wird $X \times Y$ mit der Produkttopologie versehen, so sind $\pi_X$
|
||||
und $\pi_Y$ stetig.
|
||||
|
@ -482,8 +482,8 @@ Die Teilraumtopologie wird auch \textit{Spurtopologie} oder
|
|||
\end{bemerkung}
|
||||
|
||||
\begin{beweis}
|
||||
Nach Definition ist
|
||||
$U \subseteq \overline{X}$ offen $\gdw \pi^{-1}(U) \subseteq X$
|
||||
Nach Definition ist
|
||||
$U \subseteq \overline{X}$ offen $\gdw \pi^{-1}(U) \subseteq X$
|
||||
offen. $\qed$
|
||||
\end{beweis}
|
||||
|
||||
|
@ -498,7 +498,7 @@ sodass $\pi$ stetig wird.
|
|||
S^n &= \Set{x \in \mdr^{n+1} | \|x\| = 1}\\
|
||||
&= \Set{x \in \mdr^{n+1} | \sum_{i=1}^{n+1} x_i^2 = 1}
|
||||
\end{align*}
|
||||
|
||||
|
||||
\Obda sei $N = \begin{pmatrix}0\\ \vdots\\ 0\\1\end{pmatrix}$. Die
|
||||
Gerade durch $N$ und $P$ schneidet die Ebene $H$ in genau einem
|
||||
Punkt $\hat{P}$. $P$ wird auf $\hat{P}$ abgebildet.
|
||||
|
@ -523,7 +523,7 @@ sodass $\pi$ stetig wird.
|
|||
schneiden sich $L_P$ und $H$ in genau einem Punkt $\hat{P}$.
|
||||
|
||||
Es gilt: $f$ ist bijektiv und die Umkehrabbildung ist ebenfalls
|
||||
stetig.
|
||||
stetig.
|
||||
\end{beispiel}
|
||||
\index{Stetigkeit|)}
|
||||
%%%%%%%%%%%%%%%%%%%%%%%%%%%%%%%%%%%%%%%%%%%%%%%%%%%%%%%%%%%%%%%%%%%%%
|
||||
|
@ -533,7 +533,7 @@ sodass $\pi$ stetig wird.
|
|||
\begin{definition}\xindex{Raum!zusammenhaengender@zusammenhängender}\xindex{Menge!zusammenhaengende@zusammenhängende}%
|
||||
\begin{defenum}
|
||||
\item Ein Raum $X$ heißt \textbf{zusammenhängend}, wenn es keine offenen,
|
||||
nichtleeren Teilmengen $U_1, U_2$ von $X$ gibt mit
|
||||
nichtleeren Teilmengen $U_1, U_2$ von $X$ gibt mit
|
||||
$U_1 \cap U_2 = \emptyset$ und $U_1 \cup U_2 = X$.
|
||||
\item Eine Teilmenge $Y \subseteq X$ heißt zusammenhängend, wenn $Y$
|
||||
als topologischer Raum mit der Teilraumtopologie zusammenhängend ist.
|
||||
|
@ -542,7 +542,7 @@ sodass $\pi$ stetig wird.
|
|||
|
||||
\begin{bemerkung}
|
||||
$X$ ist zusammenhängend $\gdw$ Es gibt keine abgeschlossenen,
|
||||
nichtleeren Teilmengen $A_1, A_2$ mit $A_1 \cap A_2 = \emptyset$
|
||||
nichtleeren Teilmengen $A_1, A_2$ mit $A_1 \cap A_2 = \emptyset$
|
||||
und $A_1 \cup A_2 = X$.
|
||||
\end{bemerkung}
|
||||
|
||||
|
@ -553,20 +553,20 @@ sodass $\pi$ stetig wird.
|
|||
\underline{Annahme}: $\mdr^n = U_1 \dcup U_2$ mit $\emptyset \neq U_1, U_2 \in \fT_{\ts{Euklid}}$ existieren.
|
||||
|
||||
Sei $x \in U_1, y \in U_2$ und $[x,y]$ die Strecke zwischen $x$
|
||||
und $y$. Sei $V = [x,y]$. Nun betrachten wir $V \subsetneq \mdr^n$ als
|
||||
und $y$. Sei $V = [x,y]$. Nun betrachten wir $V \subsetneq \mdr^n$ als
|
||||
(metrischen) Teilraum mit der Teilraumtopologie $\fT_V$.
|
||||
Somit gilt $U_1 \cap [x,y] \in \fT_V$ wegen der Definition der
|
||||
Somit gilt $U_1 \cap [x,y] \in \fT_V$ wegen der Definition der
|
||||
Teilraumtopologie.
|
||||
|
||||
Dann gibt es $z \in [x,y]$ mit $z \in \partial (U_1 \cap [x,y])$,
|
||||
aber $z \notin U_1 \Rightarrow z \in U_2$. In jeder Umgebung von
|
||||
aber $z \notin U_1 \Rightarrow z \in U_2$. In jeder Umgebung von
|
||||
$z$ liegt ein Punkt von $U_1 \Rightarrow$ Widerspruch zu $U_2$ offen.
|
||||
\item $\mdr \setminus \Set{0}$ ist nicht zusammenhängend, denn
|
||||
$\mdr \setminus \Set{0} = \mdr_{< 0} \cup \mdr_{> 0}$
|
||||
\item $\mdr^2 \setminus \Set{0}$ ist zusammenhängend.
|
||||
\item $\mdq \subsetneq \mdr$ ist nicht zusammenhängend, da
|
||||
\item $\mdq \subsetneq \mdr$ ist nicht zusammenhängend, da
|
||||
$(\mdq \cap \mdr_{< \sqrt{2}}) \cup (\mdq \cap \mdr_{> \sqrt{2}}) = \mdq$
|
||||
\item $\Set{x}$ ist zusammenhängend für jedes $x \in X$,
|
||||
\item $\Set{x}$ ist zusammenhängend für jedes $x \in X$,
|
||||
wobei $X$ ein topologischer Raum ist.
|
||||
\item $\mdr$ mit Zariski-Topologie ist zusammenhängend.\xindex{Topologie!Zariski}
|
||||
\end{bspenum}
|
||||
|
@ -590,7 +590,7 @@ sodass $\pi$ stetig wird.
|
|||
$\Rightarrow \overline{A} \subseteq A_2$\\
|
||||
$\Rightarrow A_1 = \emptyset$\\
|
||||
$\Rightarrow$ Widerspruch zu $A_1 \neq \emptyset$\\
|
||||
$\Rightarrow A \cap A_1 \neq \emptyset$ und analog
|
||||
$\Rightarrow A \cap A_1 \neq \emptyset$ und analog
|
||||
$A \cap A_2 \neq \emptyset$\\
|
||||
$\Rightarrow$ Widerspruch zu $A$ ist zusammenhängend. $ \qed$
|
||||
\end{beweis}
|
||||
|
@ -614,7 +614,7 @@ sodass $\pi$ stetig wird.
|
|||
|
||||
\begin{definition}\xindex{Zusammenhangskomponente}%
|
||||
Sei $X$ ein topologischer Raum.
|
||||
|
||||
|
||||
Für $x \in X$ sei $Z(x) \subseteq X$ definiert durch
|
||||
\[Z(x) := \bigcup_{\mathclap{\substack{A \subseteq X \text{zhgd.}\\ x \in A}}} A\]
|
||||
|
||||
|
@ -647,7 +647,7 @@ sodass $\pi$ stetig wird.
|
|||
\begin{align*}
|
||||
\Rightarrow Z(x) \cup Z(y) &\subseteq Z(x) \Rightarrow Z(y) \subseteq Z(x)\\
|
||||
&\subseteq Z(y) \Rightarrow Z(x) \subseteq Z(y)
|
||||
\end{align*}
|
||||
\end{align*}
|
||||
\end{enumerate}
|
||||
|
||||
$\qed$
|
||||
|
@ -678,7 +678,7 @@ sodass $\pi$ stetig wird.
|
|||
Ein topologischer Raum $X$ heißt \textbf{kompakt}, wenn jede
|
||||
offene Überdeckung von $X$
|
||||
\[\fU = \Set{U_i}_{i \in I} \text{ mit } U_i \text{ offen in } X\]
|
||||
eine endliche Teilüberdeckung
|
||||
eine endliche Teilüberdeckung
|
||||
\[\bigcup_{\mathclap{i \in J \subseteq I}} U_i = X \text{ mit } |J| \in \mdn\]
|
||||
besitzt.
|
||||
\end{definition}
|
||||
|
@ -688,24 +688,24 @@ sodass $\pi$ stetig wird.
|
|||
%%%%%%%%%%%%%%%%%%%%%%%%%%%%%%%%%%%%%%%%%%%%%%%%%%%%%%%%%%%%%%%%%%%%%
|
||||
|
||||
\begin{bemerkung}\label{abgeschlossen01IstKompakt}
|
||||
Das Einheitsintervall $I := [0,1]$ ist kompakt bezüglich der
|
||||
Das Einheitsintervall $I := [0,1]$ ist kompakt bezüglich der
|
||||
euklidischen Topologie.
|
||||
\end{bemerkung}
|
||||
|
||||
\begin{beweis}
|
||||
Sei $(U_i)_{i \in J}$ eine offene Überdeckung von $I$.
|
||||
|
||||
Es genügt zu zeigen, dass es ein $\delta > 0$ gibt, sodass jedes
|
||||
Es genügt zu zeigen, dass es ein $\delta > 0$ gibt, sodass jedes
|
||||
Teilintervall der Länge $\delta$ von $I$ in einem der $U_i$ enthalten ist.
|
||||
Wenn es ein solches $\delta$ gibt, kann man $I$ in endlich viele
|
||||
Wenn es ein solches $\delta$ gibt, kann man $I$ in endlich viele
|
||||
Intervalle der Länge $\delta$ unterteilen und alle $U_i$ in die endliche
|
||||
Überdeckung aufnehmen, die Teilintervalle enthalten.
|
||||
|
||||
Angenommen, es gibt kein solches $\delta$. Dann gibt es für jedes
|
||||
Angenommen, es gibt kein solches $\delta$. Dann gibt es für jedes
|
||||
$n \in \mdn$ ein Intervall $I_n \subseteq [0,1]$ der Länge $\nicefrac{1}{n}$
|
||||
sodass $I_n \subsetneq U_i$ für alle $i \in J$.
|
||||
|
||||
Sei $x_n$ der Mittelpunkt von $I_n$. Die Folge $(x_n)$ hat einen
|
||||
Sei $x_n$ der Mittelpunkt von $I_n$. Die Folge $(x_n)$ hat einen
|
||||
Häufungspunkt $x \in [0,1]$. Dann gibt es $i \in J$ mit $x \in U_i$.
|
||||
Da $U_i$ offen ist, gibt es ein $\varepsilon > 0$, sodass $(x - \varepsilon, x + \varepsilon) \subseteq U_i$.
|
||||
Dann gibt es $n_0$, sodass gilt:
|
||||
|
@ -713,7 +713,7 @@ $\nicefrac{1}{n_0} < \nicefrac{\varepsilon}{2}$ und für unendlich viele\footnot
|
|||
$n\geq n_0: |x - x_n| < \nicefrac{\varepsilon}{2}$, also $I_n \subseteq (x - \varepsilon, x + \varepsilon) \subseteq U_i$
|
||||
für mindestens ein $n \in \mdn$.\footnote{Sogar für unendlich viele.}
|
||||
|
||||
$\Rightarrow$ Widerspruch
|
||||
$\Rightarrow$ Widerspruch
|
||||
|
||||
Dann überdecke $[0,1]$ mit endlich vielen Intervallen $I_1, \dots, I_d$
|
||||
der Länge $\delta$. Jedes $I_j$ ist in $U_{ij}$ enthalten.
|
||||
|
@ -727,7 +727,7 @@ $\qed$
|
|||
\item $\mdr$ ist nicht kompakt.
|
||||
\item $(0,1)$ ist nicht kompakt.\\
|
||||
$U_n = (\nicefrac{1}{n}, 1-\nicefrac{1}{n}) \Rightarrow \bigcup_{n \in \mdn} U_n = (0,1)$
|
||||
\item $\mdr$ mit der Zariski-Topologie ist kompakt und jede
|
||||
\item $\mdr$ mit der Zariski-Topologie ist kompakt und jede
|
||||
Teilmenge von $\mdr$ ist es auch.\xindex{Topologie!Zariski}
|
||||
\end{bspenum}
|
||||
\end{beispiel}
|
||||
|
@ -770,12 +770,12 @@ $\qed$
|
|||
|
||||
Die offenen Mengen $U_{x_0, y} \times V_{x_0, y}$ für festes $x_0$
|
||||
und alle $y \in Y$ überdecken $\Set{x_0} \times y$. Da $Y$ kompakt
|
||||
ist, ist auch $\Set{x_0} \times Y$ kompakt. Also gibt es
|
||||
$y_1, \dots, y_{m(x_0)}$ mit
|
||||
ist, ist auch $\Set{x_0} \times Y$ kompakt. Also gibt es
|
||||
$y_1, \dots, y_{m(x_0)}$ mit
|
||||
$\bigcup_{i=1}^{m(x_0)} U_{x_0, y_i} \times V_{x_0, y_i} \supseteq \Set{x_0} \times Y$.
|
||||
|
||||
Sei ${\color{blue} U_{x_0}} := \bigcap_{i=1}^{m(x)} U_{x_0, y_i}$.
|
||||
Da $X$ kompakt ist, gibt es $x_1, \dots, x_n \in X$ mit
|
||||
Da $X$ kompakt ist, gibt es $x_1, \dots, x_n \in X$ mit
|
||||
$\bigcup_{j=1}^n U_{x_j} = X$\\
|
||||
$\Rightarrow \bigcup_{j=1}^k \bigcup_{i=1}^{m(x_j)} \underbrace{\left ( U_{x_j, y_i} \times V_{x_j, y_i} \right)}_{\mathclap{\text{Ein grün-oranges Kästchen}}} \supseteq X \times Y$\\
|
||||
$\Rightarrow \bigcup_j \bigcup_i W_i (x_j, y_i) = X \times Y \qed$
|
||||
|
@ -789,7 +789,7 @@ $\qed$
|
|||
\begin{beweis}
|
||||
\underline{z.~Z.:} Komplement ist offen
|
||||
|
||||
Ist $X = K$, so ist $K$ abgeschlossen in $X$. Andernfalls sei
|
||||
Ist $X = K$, so ist $K$ abgeschlossen in $X$. Andernfalls sei
|
||||
$y \in X \setminus K$. Für jedes $x \in K$ seien $U_x$ bzw. $V_y$
|
||||
Umgebungen von $x$ bzw. von $y$, sodass $U_x \cap V_y = \emptyset$.
|
||||
|
||||
|
@ -819,10 +819,10 @@ $\qed$
|
|||
\begin{beweis}
|
||||
Sei $(V_i)_{i \in I}$ offene Überdeckung von $f(K)$\\
|
||||
$\xRightarrow{f \text{ stetig}} (f^{-1}(V_i))_{i \in I}$ ist offene Überdeckung von $K$\\
|
||||
$\xRightarrow{\text{Kompakt}}$ es gibt $i_1, \dots, i_n$,
|
||||
$\xRightarrow{\text{Kompakt}}$ es gibt $i_1, \dots, i_n$,
|
||||
sodass $f^{-1}(V_{i_1}), \dots, f^{-1}(V_{i_n})$ Überdeckung von
|
||||
$K$ ist.\\
|
||||
$\Rightarrow f(f^{-1}( V_{i_1})), \dots, f(f^{-1}(V_{i_n}))$
|
||||
$\Rightarrow f(f^{-1}( V_{i_1})), \dots, f(f^{-1}(V_{i_n}))$
|
||||
überdecken $f(K)$.
|
||||
|
||||
Es gilt: $f(f^{-1}(V)) = V \cap f(X) \qed$
|
||||
|
@ -839,7 +839,7 @@ $\qed$
|
|||
|
||||
Da $\mdr^n$ und $\mdc^n$ hausdorffsch sind, ist $K$ nach
|
||||
\cref{hausdorffraumKompakteTeilmengeAbgeschlossen} abgeschlossen.
|
||||
Nach Voraussetzung kann $K$ mit endlich vielen offenen Kugeln von
|
||||
Nach Voraussetzung kann $K$ mit endlich vielen offenen Kugeln von
|
||||
Radien 1 überdeckt werden $\Rightarrow K$ ist beschränkt.
|
||||
|
||||
\enquote{$\Leftarrow$} Sei $A \subseteq \mdr^n$ (oder $\mdc^n$)
|
||||
|
@ -852,7 +852,7 @@ $\qed$
|
|||
Nach \cref{kompaktTimesKompaktIstKompakt} und
|
||||
\cref{abgeschlossen01IstKompakt} ist $W$ kompakt, also ist $A$
|
||||
nach \cref{abgeschlossenInKomaktIstKompakt} auch kompakt.
|
||||
Genauso ist $Z$ kompakt, weil
|
||||
Genauso ist $Z$ kompakt, weil
|
||||
\[\Set{z \in \mdc | |z| \leq 1}\]
|
||||
homöomorph zu
|
||||
\[\Set{(x,y) \in \mdr^2 | \|(x,y)\| \leq 1}\]
|
||||
|
@ -864,11 +864,11 @@ $\qed$
|
|||
%%%%%%%%%%%%%%%%%%%%%%%%%%%%%%%%%%%%%%%%%%%%%%%%%%%%%%%%%%%%%%%%%%%%%
|
||||
\section{Wege und Knoten}\index{Knoten|(}
|
||||
\begin{definition}\xindex{Weg}\xindex{Weg!geschlossener}\xindex{Weg!einfacher}%
|
||||
Sei $X$ ein topologischer Raum.
|
||||
Sei $X$ ein topologischer Raum.
|
||||
\begin{defenum}
|
||||
\item Ein \textbf{Weg} in $X$ ist eine stetige Abbildung $\gamma:[0,1] \rightarrow X$.
|
||||
\item $\gamma$ heißt \textbf{geschlossen}, wenn $\gamma(1) = \gamma(0)$ gilt.
|
||||
\item $\gamma$ heißt \textbf{einfach}, wenn $\gamma|_{[0,1)}$
|
||||
\item $\gamma$ heißt \textbf{einfach}, wenn $\gamma|_{[0,1)}$
|
||||
injektiv ist.
|
||||
\end{defenum}
|
||||
\end{definition}
|
||||
|
@ -901,11 +901,11 @@ $\qed$
|
|||
$A_1 \cup A_2 = X$. Sei $x \in A_1, y \in A_2, \gamma:[0,1] \rightarrow X$
|
||||
ein Weg von $x$ nach $y$.
|
||||
|
||||
Dann ist $C:= \gamma([0,1]) \subseteq X$ zusammenhängend, weil
|
||||
Dann ist $C:= \gamma([0,1]) \subseteq X$ zusammenhängend, weil
|
||||
$\gamma$ stetig ist.
|
||||
\[C = \underbrace{(C \cap A_1)}_{\ni x} \cup \underbrace{(C \cap A_2)}_{\ni y}\]
|
||||
ist Zerlegung in nichtleere, disjunkte, abgeschlossene Teilmengen
|
||||
$\Rightarrow$ Widerspruch
|
||||
$\Rightarrow$ Widerspruch
|
||||
|
||||
\item Sei $X = \Set{(x,y) \in \mdr^2| x^2 + y^2 = 1 \lor y = 1 +2\cdot e^{-\frac{1}{10} x}}$.
|
||||
|
||||
|
@ -943,7 +943,7 @@ $\qed$
|
|||
\end{beweis}
|
||||
|
||||
\begin{beispiel}[Hilbert-Kurve]\xindex{Hilbert-Kurve}%
|
||||
Es gibt stetige, surjektive Abbildungen
|
||||
Es gibt stetige, surjektive Abbildungen
|
||||
$[0,1] \rightarrow [0,1] \times [0,1]$. Ein Beispiel ist die
|
||||
in \cref{fig:hilbert-curve} dargestellte Hilbert-Kurve.
|
||||
|
||||
|
@ -952,7 +952,7 @@ $\qed$
|
|||
|
||||
\begin{definition}\xindex{Jordankurve}\xindex{Jordankurve!geschlossene}%
|
||||
Sei $X$ ein topologischer Raum. Eine
|
||||
\textbf{Jordankurve} in $X$ ist ein Homöomorphismus
|
||||
\textbf{Jordankurve} in $X$ ist ein Homöomorphismus
|
||||
$\gamma: [0,1] \rightarrow C \subseteq X$ bzw.
|
||||
$\gamma: S^1 \rightarrow C \subseteq X$, wobei $C := \Bild{\gamma}$.
|
||||
\end{definition}
|
||||
|
@ -967,7 +967,7 @@ Jede Jordankurve ist also ein einfacher Weg.
|
|||
|
||||
\begin{figure}[htp]
|
||||
\centering
|
||||
\input{figures/topology-jordan}
|
||||
\input{figures/topology-jordan}
|
||||
\label{fig:jordan-kurvensatz}
|
||||
\caption{Die unbeschränkte Zusammenhangskomponente wird häufig inneres, die beschränkte äußeres genannt.}
|
||||
\end{figure}
|
||||
|
@ -993,15 +993,15 @@ Jede Jordankurve ist also ein einfacher Weg.
|
|||
\label{fig:knot-unknot}
|
||||
}%
|
||||
\subfloat[Kleeblattknoten]{
|
||||
\includegraphics[width=0.2\linewidth, keepaspectratio]{figures/blue-trefoil-knot.png}
|
||||
\includegraphics[width=0.2\linewidth, keepaspectratio]{figures/blue-trefoil-knot.png}
|
||||
\label{fig:knot-trefoil}
|
||||
}%
|
||||
\subfloat[Achterknoten]{
|
||||
\includegraphics[width=0.2\linewidth, keepaspectratio]{figures/blue-eight-knot.png}
|
||||
\includegraphics[width=0.2\linewidth, keepaspectratio]{figures/blue-eight-knot.png}
|
||||
\label{fig:knot-eight-knot}
|
||||
}%
|
||||
\subfloat[$6_2$-Knoten]{
|
||||
\includegraphics[width=0.2\linewidth, keepaspectratio]{figures/blue-6-2-knot.png}
|
||||
\includegraphics[width=0.2\linewidth, keepaspectratio]{figures/blue-6-2-knot.png}
|
||||
\label{fig:knot-6-2}
|
||||
}
|
||||
|
||||
|
@ -1014,20 +1014,20 @@ Jede Jordankurve ist also ein einfacher Weg.
|
|||
Zwei Knoten $\gamma_1, \gamma_2: S^1 \rightarrow \mdr^3$ heißen
|
||||
\textbf{äquivalent}, wenn es eine stetige Abbildung
|
||||
\[H: S^1 \times [0,1] \rightarrow \mdr^3\]
|
||||
gibt mit
|
||||
gibt mit
|
||||
\begin{align*}
|
||||
H(z,0) &= \gamma_1(z) \;\;\;\forall z \in S^1\\
|
||||
H(z,1) &= \gamma_2(z) \;\;\;\forall z \in S^1
|
||||
\end{align*}
|
||||
und für jedes
|
||||
feste $t \in [0,1]$ ist
|
||||
feste $t \in [0,1]$ ist
|
||||
\[H_z: S^1 \rightarrow \mdr^3, z \mapsto H(z,t)\]
|
||||
ein Knoten. Die Abbildung $H$ heißt \textbf{Isotopie} zwischen
|
||||
$\gamma_1$ und $\gamma_2$.
|
||||
\end{definition}
|
||||
|
||||
\begin{definition}\xindex{Knotendiagramm}%
|
||||
Sei $\gamma: [0,1] \rightarrow \mdr^3$ ein Knoten, $E$ eine Ebene und
|
||||
Sei $\gamma: [0,1] \rightarrow \mdr^3$ ein Knoten, $E$ eine Ebene und
|
||||
$\pi: \mdr^3 \rightarrow E$ eine Projektion auf $E$.
|
||||
|
||||
$\pi$ heißt \textbf{Knotendiagramm} von $\gamma$, wenn gilt:
|
||||
|
@ -1047,16 +1047,16 @@ Jede Jordankurve ist also ein einfacher Weg.
|
|||
\begin{figure}[htp]
|
||||
\centering
|
||||
\subfloat[$\Omega_1$]{
|
||||
\includegraphics[height=0.2\linewidth, keepaspectratio]{figures/reidemeister-move-1.png}
|
||||
\includegraphics[height=0.2\linewidth, keepaspectratio]{figures/reidemeister-move-1.png}
|
||||
\label{fig:reidemeister-1}
|
||||
}\qquad\qquad%
|
||||
\subfloat[$\Omega_2$]{
|
||||
\includegraphics[height=0.2\linewidth, keepaspectratio]{figures/reidemeister-move-2.png}
|
||||
\includegraphics[height=0.2\linewidth, keepaspectratio]{figures/reidemeister-move-2.png}
|
||||
\label{fig:reidemeister-2}
|
||||
}
|
||||
|
||||
\subfloat[$\Omega_3$]{
|
||||
\includegraphics[height=0.2\linewidth, keepaspectratio]{figures/reidemeister-move-3.png}
|
||||
\includegraphics[height=0.2\linewidth, keepaspectratio]{figures/reidemeister-move-3.png}
|
||||
\label{fig:reidemeister-3}
|
||||
}
|
||||
|
||||
|
@ -1069,15 +1069,15 @@ Jede Jordankurve ist also ein einfacher Weg.
|
|||
\end{beweis}
|
||||
|
||||
\begin{definition}\xindex{Färbbarkeit}%
|
||||
Ein Knotendiagramm heißt \textbf{3-färbbar},
|
||||
wenn jeder Bogen von $D$ so mit einer Farbe gefärbt werden kann,
|
||||
dass an jeder Kreuzung eine oder 3 Farben auftreten und alle 3
|
||||
Ein Knotendiagramm heißt \textbf{3-färbbar},
|
||||
wenn jeder Bogen von $D$ so mit einer Farbe gefärbt werden kann,
|
||||
dass an jeder Kreuzung eine oder 3 Farben auftreten und alle 3
|
||||
Farben auftreten.
|
||||
\end{definition}
|
||||
|
||||
\begin{figure}[htp]
|
||||
\centering
|
||||
\includegraphics[height=0.3\linewidth, keepaspectratio]{figures/tricoloring.png}
|
||||
\includegraphics[height=0.3\linewidth, keepaspectratio]{figures/tricoloring.png}
|
||||
|
||||
\caption{Ein 3-gefärber Kleeblattknoten}
|
||||
\label{fig:treefoil-knot-three-colors}
|
||||
|
|
|
@ -15,7 +15,7 @@
|
|||
Familie $(U_i, \varphi_i)_{i \in I}$ von Karten auf $X$,
|
||||
sodass $\bigcup_{i \in I} U_i = X$.
|
||||
\item $X$ heißt (topologische) $n$-dimensionale \textbf{Mannigfaltigkeit}\xindex{Mannigfaltigkeit},
|
||||
wenn $X$ hausdorffsch ist, eine abzählbare Basis der
|
||||
wenn $X$ hausdorffsch ist, eine abzählbare Basis der
|
||||
Topologie hat und einen $n$-dimensionalen Atlas besitzt.
|
||||
\end{defenum}
|
||||
\end{definition}
|
||||
|
@ -27,13 +27,13 @@ Anschaulich ist also ein $n$-dimensionale Mannigfaltigkeit lokal dem $\mdr^n$ ä
|
|||
\end{bemerkung}
|
||||
|
||||
\begin{beweis}
|
||||
Sei $(X, \fT)$ ein topologischer Raum und $(U, \varphi)$ mit $U \in \fT$
|
||||
Sei $(X, \fT)$ ein topologischer Raum und $(U, \varphi)$ mit $U \in \fT$
|
||||
und $\varphi:U \rightarrow V \subseteq \mdr^n$,
|
||||
wobei $V$ offen und $\varphi$ ein Homöomorphismus ist, eine Karte auf $X$.
|
||||
|
||||
Da jede offene Teilmenge des $\mdr^n$ genauso mächtig ist wie der $\mdr^n$,
|
||||
$\varphi$ als Homöomorphismus insbesondere bijektiv ist und Mengen, zwischen
|
||||
denen eine Bijektion existiert, gleich mächtig sind, ist $U$ genauso mächtig
|
||||
$\varphi$ als Homöomorphismus insbesondere bijektiv ist und Mengen, zwischen
|
||||
denen eine Bijektion existiert, gleich mächtig sind, ist $U$ genauso mächtig
|
||||
wie der $\mdr^n$. Da jede Mannigfaltigkeit mindestens eine Karte hat, muss
|
||||
jede Mannigfaltigkeit $X$ mindestens so mächtig sein wie der $\mdr^n$. $\qed$
|
||||
\end{beweis}
|
||||
|
@ -59,8 +59,8 @@ Mannigfaltigkeiten können beliebig viele Elemente haben.
|
|||
|
||||
\begin{beispiel}[Mannigfaltigkeiten]
|
||||
\begin{bspenum}
|
||||
\item Jede offene Teilmenge $U \subseteq \mdr^n$ ist eine
|
||||
$n$-dimensionale Mannigfaltigkeit mit einem Atlas aus
|
||||
\item Jede offene Teilmenge $U \subseteq \mdr^n$ ist eine
|
||||
$n$-dimensionale Mannigfaltigkeit mit einem Atlas aus
|
||||
einer Karte.
|
||||
\item $\mdc^n$ ist eine $2n$-dimensionale Mannigfaltigkeit
|
||||
mit einem Atlas aus einer Karte:
|
||||
|
@ -106,7 +106,7 @@ Mannigfaltigkeiten können beliebig viele Elemente haben.
|
|||
Es gibt keine Umgebung von $0$ in $[0,1]$, die homöomorph
|
||||
zu einem offenem Intervall ist.
|
||||
\item $V_1 = \Set{(x,y) \in \mdr^2 | x \cdot y = 0}$ ist
|
||||
keine Mannigfaltigkeit.
|
||||
keine Mannigfaltigkeit.
|
||||
|
||||
Das Problem ist $(0,0)$. Wenn man diesen Punkt entfernt,
|
||||
zerfällt der Raum in 4 Zusammenhangskomponenten.
|
||||
|
@ -116,7 +116,7 @@ Mannigfaltigkeiten können beliebig viele Elemente haben.
|
|||
Mannigfaltigkeit.
|
||||
\item $X = (\mdr \setminus \Set{0}) \cup (0_1, 0_2)$ \label{bsp:mannigfaltigkeit8}
|
||||
|
||||
\[U \subseteq X \text{ offen } \gdw
|
||||
\[U \subseteq X \text{ offen } \gdw
|
||||
\begin{cases}
|
||||
U \text{ offen in } \mdr \setminus \Set{0}, &\text{falls } 0_1 \notin U, 0_2 \in U\\
|
||||
\exists \varepsilon > 0: (-\varepsilon, \varepsilon) \subseteq U &\text{falls } 0_1 \in U, 0_2 \in U
|
||||
|
@ -128,7 +128,7 @@ Mannigfaltigkeiten können beliebig viele Elemente haben.
|
|||
\underline{Aber:} $X$ ist nicht hausdorffsch!
|
||||
Denn es gibt keine disjunkten Umgebungen von $0_1$ und
|
||||
$0_2$.
|
||||
\item \label{bsp:gln-ist-mf}\xindex{Gruppe!allgemeine lineare}$\GL_n(\mdr)$ ist eine Mannigfaltigkeit der Dimension
|
||||
\item \label{bsp:gln-ist-mf}\xindex{Gruppe!allgemeine lineare}$\GL_n(\mdr)$ ist eine Mannigfaltigkeit der Dimension
|
||||
$n^2$, weil offene Teilmengen von $\mdr^{n^2}$ eine
|
||||
Mannigfaltigkeit bilden.
|
||||
\end{bspenum}
|
||||
|
@ -141,12 +141,12 @@ Mannigfaltigkeiten können beliebig viele Elemente haben.
|
|||
Seien $X, Y$ $n$-dimensionale Mannigfaltigkeiten, $U \subseteq X$
|
||||
und $V \subseteq Y$ offen, $\Phi: U \rightarrow V$ ein Homöomorphismus
|
||||
$Z = (X \dcup Y) /_\sim$ mit der von $u \sim \Phi(u)\;\forall{u \in U}$
|
||||
erzeugten Äquivalenzrelation und der von $\sim$ induzierten
|
||||
erzeugten Äquivalenzrelation und der von $\sim$ induzierten
|
||||
Quotiententopologie.
|
||||
|
||||
$Z$ heißt \textbf{Verklebung} von $X$ und $Y$ längs $U$ und $V$.
|
||||
$Z$ besitzt einen Atlas aus $n$-dimensionalen Karten.
|
||||
Falls $Z$ hausdorffsch ist, ist $Z$ eine $n$-dimensionale
|
||||
Falls $Z$ hausdorffsch ist, ist $Z$ eine $n$-dimensionale
|
||||
Mannigfaltigkeit.
|
||||
\end{definition}
|
||||
|
||||
|
@ -210,7 +210,7 @@ Mannigfaltigkeiten können beliebig viele Elemente haben.
|
|||
$G: U \rightarrow \mdr^n, \; u \mapsto (g(u), u)$
|
||||
eine stetige Abbildung auf eine offene Umgebung $V$ von
|
||||
$x$ in $X$ ist.
|
||||
\end{enumerate}
|
||||
\end{enumerate}
|
||||
$\qed$
|
||||
\end{beweis}
|
||||
|
||||
|
@ -244,8 +244,8 @@ Mannigfaltigkeiten können beliebig viele Elemente haben.
|
|||
Sei $X$ ein Hausdorffraum mit abzählbarer Basis der Topologie.
|
||||
$X$ heißt $n$-dimensionale \textbf{Mannigfaltigkeit mit Rand},
|
||||
wenn es einen Atlas $(U_i, \varphi_i)$ gibt, wobei $U_i \subseteq X_i$
|
||||
offen und $\varphi_i$ ein Homöomorphismus auf eine offene
|
||||
Teilmenge von
|
||||
offen und $\varphi_i$ ein Homöomorphismus auf eine offene
|
||||
Teilmenge von
|
||||
\[\mdr_{+,0}^n := \Set{(x_1, \dots, x_n) \in \mdr^n | x_n \geq 0}\]
|
||||
ist.
|
||||
\end{definition}
|
||||
|
@ -275,7 +275,7 @@ $\mdr_{+,0}^n$ ist ein \enquote{Halbraum}\xindex{Halbraum}.
|
|||
|
||||
\begin{definition}\xindex{Rand}%
|
||||
Sei $X$ eine $n$-dimensionale Mannigfaltigkeit mit Rand und
|
||||
Atlas $\atlas$. Dann heißt
|
||||
Atlas $\atlas$. Dann heißt
|
||||
\[\partial X := \bigcup_{(U, \varphi) \in \atlas} \Set{x \in U | \varphi (x) = 0}\]
|
||||
\textbf{Rand} von $X$.
|
||||
\end{definition}
|
||||
|
@ -322,7 +322,7 @@ Differenzierbare Mannigfaltigkeiten der Klasse $C^\infty$ werden auch
|
|||
\textit{glatt} genannt.
|
||||
|
||||
\begin{definition}%
|
||||
Sei $X$ eine differenzierbare Mannigfaltigkeit der Klasse $C^k$
|
||||
Sei $X$ eine differenzierbare Mannigfaltigkeit der Klasse $C^k$
|
||||
($k \in \mdn \cup \Set{\infty}$) mit Atlas $\atlas = (U_i, \varphi_i)_{i \in I}$.
|
||||
|
||||
\begin{defenum}
|
||||
|
@ -330,10 +330,10 @@ Differenzierbare Mannigfaltigkeiten der Klasse $C^\infty$ werden auch
|
|||
mit $\atlas$, wenn alle Kartenwechsel $\varphi \circ \varphi_i^{-1}$
|
||||
und $\varphi_i \circ \varphi^{-1}$ ($i \in I$ mit $U_i \cap U \neq \emptyset$)
|
||||
differenzierbar von Klasse $C^k$ sind.
|
||||
\item Die Menge aller mit $\atlas$ verträglichen Karten auf
|
||||
\item Die Menge aller mit $\atlas$ verträglichen Karten auf
|
||||
$X$ bildet einen maximalen Atlas der Klasse $C^k$. Er
|
||||
heißt \textbf{$C^k$-Struktur}\xindex{Ck-Struktur@$C^k$-Struktur} auf $X$.
|
||||
|
||||
|
||||
Eine $C^\infty$-Struktur heißt auch \textbf{differenzierbare Struktur}\xindex{Struktur!differenzierbare}
|
||||
auf $X$.
|
||||
\end{defenum}
|
||||
|
@ -357,7 +357,7 @@ Differenzierbare Mannigfaltigkeiten der Klasse $C^\infty$ werden auch
|
|||
gibt, sodass $\psi \circ f \circ \varphi^{-1}$ stetig
|
||||
differenzierbar von Klasse $C^k$ in $\varphi(x)$ ist.
|
||||
\item $f$ heißt \textbf{differenzierbar}
|
||||
(von Klasse $C^k$), wenn $f$ in jedem $x \in X$
|
||||
(von Klasse $C^k$), wenn $f$ in jedem $x \in X$
|
||||
differenzierbar ist.
|
||||
\item $f$ heißt \textbf{Diffeomorphismus}\xindex{Diffeomorphismus},
|
||||
wenn $f$ differenzierbar von Klasse $C^\infty$ ist und
|
||||
|
@ -375,7 +375,7 @@ Differenzierbare Mannigfaltigkeiten der Klasse $C^\infty$ werden auch
|
|||
\begin{beweis}
|
||||
Seien $(U', \varphi')$ und $(V', \psi')$ Karten von $X$ bzw. $Y$
|
||||
um $x$ bzw. $f(x)$ mit $f(U') \subseteq V'$.
|
||||
|
||||
|
||||
$\Rightarrow \psi' \circ f \circ (\varphi')^{-1}$\\
|
||||
$= \psi' \circ ( \psi^{-1} \circ \psi) \circ f \circ (\varphi^{-1} \circ \varphi ) \circ (\varphi')^{-1}$
|
||||
|
||||
|
@ -397,8 +397,8 @@ Differenzierbare Mannigfaltigkeiten der Klasse $C^\infty$ werden auch
|
|||
|
||||
\begin{definition}\label{def:8.5}\xindex{Fläche!reguläre}\xindex{Parametrisierung!reguläre}%
|
||||
$S \subseteq \mdr^3$ heißt \textbf{reguläre Fläche} $:\gdw$
|
||||
$\forall s \in S\;\exists $ Umgebung $V(s) \subseteq \mdr^3$ $\exists U \subseteq \mdr^2$ offen:
|
||||
$\exists \text{ differenzierbare Abbildung } F: U \rightarrow V \cap S$:
|
||||
$\forall s \in S\;\exists $ Umgebung $V(s) \subseteq \mdr^3$ $\exists U \subseteq \mdr^2$ offen:
|
||||
$\exists \text{ differenzierbare Abbildung } F: U \rightarrow V \cap S$:
|
||||
$\text{Rg}(J_F(u)) = 2\;\;\;\forall u \in U$.
|
||||
|
||||
$F$ heißt (lokale) \textbf{reguläre Parametrisierung} von $S$.
|
||||
|
@ -440,7 +440,7 @@ Differenzierbare Mannigfaltigkeiten der Klasse $C^\infty$ werden auch
|
|||
%\caption{}
|
||||
\end{figure}
|
||||
|
||||
\[J_F(u,v) =
|
||||
\[J_F(u,v) =
|
||||
\begin{pmatrix}
|
||||
-r(v) \sin u & r'(v) \cos u\\
|
||||
r(v) \cos u & r'(v) \sin u\\
|
||||
|
@ -449,7 +449,7 @@ Differenzierbare Mannigfaltigkeiten der Klasse $C^\infty$ werden auch
|
|||
hat Rang 2 für alle $(u,v) \in \mdr^2$.
|
||||
\item Kugelkoordinaten: $F: \mdr^2 \rightarrow \mdr^3$,\\
|
||||
$(u, v) \mapsto (R \cos v \cos u, R \cos v \sin u, R \sin v)$\\
|
||||
Es gilt: $F(u,v) \in S_R^2$, denn
|
||||
Es gilt: $F(u,v) \in S_R^2$, denn
|
||||
\begin{align*}
|
||||
& R^2 \cos^2(v) \cos^2(u) + R^2 \cos^2(v) \sin^2(u) + R^2 \sin^2(v)\\
|
||||
=& R^2 (\cos^2(v) \cos^2(u) + \cos^2(v) \sin^2(u) + \sin^2(v))\\
|
||||
|
@ -459,7 +459,7 @@ Differenzierbare Mannigfaltigkeiten der Klasse $C^\infty$ werden auch
|
|||
\end{align*}
|
||||
|
||||
Die Jacobi-Matrix
|
||||
\[J_F(u,v) =
|
||||
\[J_F(u,v) =
|
||||
\begin{pmatrix}
|
||||
-R \cos v \sin u & -R \sin v \cos u\\
|
||||
R \cos v \cos u & -R \sin v \sin u\\
|
||||
|
@ -480,10 +480,10 @@ Differenzierbare Mannigfaltigkeiten der Klasse $C^\infty$ werden auch
|
|||
|
||||
\begin{beweis}\leavevmode
|
||||
|
||||
$S \subseteq \mdr^3$ ist als reguläre Fläche eine 2-dimensionale Mannigfaltigkeit.
|
||||
$S \subseteq \mdr^3$ ist als reguläre Fläche eine 2-dimensionale Mannigfaltigkeit.
|
||||
Aus der Definition von regulären Flächen folgt direkt, dass Karten $(U_i, F_i)$ und
|
||||
$(U_j \subseteq \mdr^2, F_j:\mdr^2 \rightarrow \mdr^3)$ von $S$ mit
|
||||
$U_i \cap U_j \neq \emptyset$ existieren, wobei $F_i$ und $F_j$ nach
|
||||
$U_i \cap U_j \neq \emptyset$ existieren, wobei $F_i$ und $F_j$ nach
|
||||
Definition differenzierbare Abbildungen sind.
|
||||
|
||||
\underline{z.Z.:} $F_j^{-1} \circ F_i$ ist ein Diffeomorphismus.
|
||||
|
@ -494,15 +494,15 @@ Differenzierbare Mannigfaltigkeiten der Klasse $C^\infty$ werden auch
|
|||
\caption{Reguläre Fläche $S$ zum Beweis von \cref{kor:regular-surface-mannigfaltigkeit}}
|
||||
\label{fig:parametric-surface-mapping}
|
||||
\end{figure}
|
||||
|
||||
|
||||
|
||||
\underline{Idee:} Finde differenzierbare Funktion $\widetilde{F_j^{-1}}$
|
||||
in Umgebung $W$ von $s$, sodass $\widetilde{F_j^{-1}}|_{S \cap W} = F_j^{-1}$.
|
||||
|
||||
\underline{Ausführung:} Sei $u_0 \in U_i$, $v_0 \in U_j$ mit $F_i(u_0) = s = F_j(v_0)$.
|
||||
|
||||
Da $\rang(J_{F_j}(v_0)) = 2$ ist, ist \obda
|
||||
\[\det
|
||||
Da $\rang(J_{F_j}(v_0)) = 2$ ist, ist \obda
|
||||
\[\det
|
||||
\begin{pmatrix}
|
||||
\frac{\partial x}{\partial u} & \frac{\partial x}{\partial v}\\
|
||||
\frac{\partial y}{\partial u} & \frac{\partial y}{\partial v}
|
||||
|
@ -513,10 +513,10 @@ Differenzierbare Mannigfaltigkeiten der Klasse $C^\infty$ werden auch
|
|||
|
||||
Definiere $\widetilde{F_j}: U_j \times \mdr \rightarrow \mdr^3$ durch
|
||||
\[\widetilde{F_j} (u, v, t) := \left(x(u,v), y(u,v), z(u,v)+t \right )\]
|
||||
|
||||
|
||||
Offensichtlich: $\widetilde{F_j} |_{U_j \times \Set{0}} = F_j$
|
||||
|
||||
\[J_{\widetilde{F_j}} =
|
||||
\[J_{\widetilde{F_j}} =
|
||||
\begin{pmatrix}
|
||||
\frac{\partial x}{\partial u} & \frac{\partial x}{\partial v} & 0\\
|
||||
\frac{\partial y}{\partial u} & \frac{\partial y}{\partial v} & 0\\
|
||||
|
@ -553,7 +553,7 @@ Differenzierbare Mannigfaltigkeiten der Klasse $C^\infty$ werden auch
|
|||
\begin{beispiel}[Lie-Gruppen]
|
||||
\begin{bspenum}
|
||||
\item Alle endlichen Gruppen sind 0-dimensionale Lie-Gruppen.
|
||||
\item $\GL_n(\mdr)$
|
||||
\item $\GL_n(\mdr)$
|
||||
% ist eine Lie-Gruppe, da sie nach \cref{bsp:gln-ist-mf} eine Mannigfaltigkeit ist.
|
||||
% $\det: \GL_n \rightarrow \mdr$ ist eine stetige Abbildung.
|
||||
\item $(\mdr^\times, \cdot)$
|
||||
|
@ -653,10 +653,10 @@ Differenzierbare Mannigfaltigkeiten der Klasse $C^\infty$ werden auch
|
|||
\begin{enumerate}[label=(\roman*),ref=\theenumii.\roman*]
|
||||
\item Für $\Delta \in K$ und $S \subseteq \Delta$ Teilsimplex
|
||||
ist $S \in K$.
|
||||
\item \label{def:simplizialkomplex.ii} Für $\Delta_1, \Delta_2 \in K$ ist
|
||||
$\Delta_1 \cap \Delta_2$ leer oder ein
|
||||
Teilsimplex von $\Delta_1$ und von
|
||||
$\Delta_2$.
|
||||
\item \label{def:simplizialkomplex.ii} Für $\Delta_1, \Delta_2 \in K$ ist
|
||||
$\Delta_1 \cap \Delta_2$ leer oder ein
|
||||
Teilsimplex von $\Delta_1$ und von
|
||||
$\Delta_2$.
|
||||
\end{enumerate}
|
||||
\item $|K| := \bigcup_{\Delta \in K} \Delta$ (mit Teilraumtopologie)
|
||||
heißt \textbf{geometrische Realisierung}\xindex{Realisierung!geometrische}
|
||||
|
@ -723,7 +723,7 @@ Differenzierbare Mannigfaltigkeiten der Klasse $C^\infty$ werden auch
|
|||
|
||||
\input{figures/topology-linear-mapping.tex}
|
||||
|
||||
\item Folgende Abbildung $\varphi: \Delta^n \rightarrow \Delta^{n-1}$
|
||||
\item Folgende Abbildung $\varphi: \Delta^n \rightarrow \Delta^{n-1}$
|
||||
ist simplizial:
|
||||
|
||||
\input{figures/topology-triangle-to-line.tex}
|
||||
|
@ -742,7 +742,7 @@ Differenzierbare Mannigfaltigkeiten der Klasse $C^\infty$ werden auch
|
|||
Sei $K$ ein endlicher Simplizialkomplex. Für $n \geq 0$ sei
|
||||
$a_n(K)$ die Anzahl der $n$-Simplizes in $K$.
|
||||
|
||||
Dann heißt
|
||||
Dann heißt
|
||||
\[\chi(K) := \sum_{n=0}^{\dim K} (-1)^n a_n(K)\]
|
||||
\textbf{Eulerzahl} (oder Euler-Charakteristik\index{Euler-Charakteristik|see{Eulerzahl}})
|
||||
von $K$.
|
||||
|
@ -842,7 +842,7 @@ Differenzierbare Mannigfaltigkeiten der Klasse $C^\infty$ werden auch
|
|||
|
||||
\begin{bemerkung}\label{kor:simplex-unterteilung}
|
||||
Sei $\Delta$ ein $n$-Simplex und $x \in \Delta^\circ \subseteq \mdr^n$.
|
||||
Sei $K$ der Simplizialkomplex, der aus $\Delta$ durch
|
||||
Sei $K$ der Simplizialkomplex, der aus $\Delta$ durch
|
||||
\enquote{Unterteilung} in $x$ entsteht. Dann ist $\chi(K) = \chi(\Delta) = 1$.
|
||||
\end{bemerkung}
|
||||
|
||||
|
@ -920,7 +920,7 @@ Differenzierbare Mannigfaltigkeiten der Klasse $C^\infty$ werden auch
|
|||
$\partial P$ von $0$ aus auf $\partial \fB_1(0) = S^2$.
|
||||
Erhalte Triangulierung von $S^2$.
|
||||
\item Sind $P_1$ und $P_2$ konvexe Polygone und $T_1, T_2$
|
||||
die zugehörigen Triangulierungen von $S^2$, so gibt es
|
||||
die zugehörigen Triangulierungen von $S^2$, so gibt es
|
||||
eine Triangulierung $T$, die sowohl um $T_1$ als
|
||||
auch um $T_2$ Verfeinerung ist (vgl. \cref{fig:topology-3}).
|
||||
|
||||
|
@ -947,7 +947,7 @@ Differenzierbare Mannigfaltigkeiten der Klasse $C^\infty$ werden auch
|
|||
und $C_n(K)$ der $\mdr$-Vektorraum mit Basis $A_n(K)$, d.~h.
|
||||
\[C_n(K) = \Set{\sum_{\sigma \in A_n(K)} c_\sigma \cdot \sigma | c_\sigma \in \mdr}\]
|
||||
|
||||
Sei $\sigma = \Delta(x_0, \dots, x_n) \in A_n(K)$, sodass
|
||||
Sei $\sigma = \Delta(x_0, \dots, x_n) \in A_n(K)$, sodass
|
||||
$x_0 < x_1 < \dots < x_n$.
|
||||
|
||||
Für $i = 0, \dots, n$ sei $\partial_i \sigma := \Delta(x_0, \dots, \hat{x_i}, \dots, x_n)$
|
||||
|
@ -1001,14 +1001,14 @@ Differenzierbare Mannigfaltigkeiten der Klasse $C^\infty$ werden auch
|
|||
\end{beweis}
|
||||
|
||||
\begin{definition}%
|
||||
Sei $K$ ein Simplizialkomplex,
|
||||
$Z_n := \text{Kern}(d_n) \subseteq C_n$ und
|
||||
Sei $K$ ein Simplizialkomplex,
|
||||
$Z_n := \text{Kern}(d_n) \subseteq C_n$ und
|
||||
$B_n := \text{Bild}(d_{n+1}) \subseteq C_n$.
|
||||
|
||||
\begin{defenum}
|
||||
\item $H_n = H_n(K, \mdr) := Z_n / B_n$ heißt $n$-te
|
||||
\item $H_n = H_n(K, \mdr) := Z_n / B_n$ heißt $n$-te
|
||||
\textbf{Homologiegruppe}\xindex{Homologiegruppe} von $K$.
|
||||
\item $b_n(K) := \dim_{\mdr} H_n$ heißt $n$-te
|
||||
\item $b_n(K) := \dim_{\mdr} H_n$ heißt $n$-te
|
||||
\textbf{Betti-Zahl}\xindex{Betti-Zahl} von $K$.
|
||||
\end{defenum}
|
||||
\end{definition}
|
||||
|
|
|
@ -6,7 +6,7 @@
|
|||
\section{Homotopie von Wegen}
|
||||
\begin{figure}[ht]
|
||||
\centering
|
||||
\subfloat[$\gamma_1$ und $\gamma_2$ sind homotop, da man sie
|
||||
\subfloat[$\gamma_1$ und $\gamma_2$ sind homotop, da man sie
|
||||
\enquote{zueinander verschieben} kann.]{
|
||||
\input{figures/topology-homotop-paths.tex}
|
||||
\label{fig:homotope-wege-anschaulich}
|
||||
|
@ -20,7 +20,7 @@
|
|||
\end{figure}
|
||||
|
||||
\begin{definition}%
|
||||
Sei $X$ ein topologischer Raum, $a, b \in X$,
|
||||
Sei $X$ ein topologischer Raum, $a, b \in X$,
|
||||
$\gamma_1, \gamma_2: I \rightarrow X$ Wege von $a$ nach $b$,
|
||||
d.~h. $\gamma_1(0) = \gamma_2(0) = a$, $\gamma_1(1) = \gamma_2(1) = b$
|
||||
|
||||
|
@ -38,7 +38,7 @@
|
|||
\end{definition}
|
||||
|
||||
\begin{bemerkung}
|
||||
Sei $X$ ein topologischer Raum, $a, b \in X$,
|
||||
Sei $X$ ein topologischer Raum, $a, b \in X$,
|
||||
$\gamma_1, \gamma_2: I \rightarrow X$ Wege von $a$ nach $b$
|
||||
und $H$ eine Homotopie zwischen $\gamma_1$ und $\gamma_2$.
|
||||
|
||||
|
@ -70,7 +70,7 @@
|
|||
H'(t, 2s) &\text{falls } 0 \leq s \leq \frac{1}{2}\\
|
||||
H''(t, 2s-1) &\text{falls } \frac{1}{2} \leq s \leq 1\end{cases}$
|
||||
|
||||
$\Rightarrow$ $H$ ist stetig und Homotopie von $\gamma_1$ nach
|
||||
$\Rightarrow$ $H$ ist stetig und Homotopie von $\gamma_1$ nach
|
||||
$\gamma_3$.
|
||||
\end{itemize}
|
||||
$\qed$
|
||||
|
@ -78,12 +78,12 @@
|
|||
|
||||
\begin{beispiel}
|
||||
\begin{bspenum}
|
||||
\item Sei $X = S^1$. $\gamma_1$ und $\gamma_2$ aus
|
||||
\item Sei $X = S^1$. $\gamma_1$ und $\gamma_2$ aus
|
||||
\cref{fig:circle-two-paths} nicht homotop.
|
||||
\item Sei $X = T^2$. $\gamma_1, \gamma_2$ und $\gamma_3$
|
||||
aus \cref{fig:torus-three-paths} sind paarweise
|
||||
nicht homotop.
|
||||
\item Sei $X = \mdr^2$ und $a=b=(0,0)$.
|
||||
\item Sei $X = \mdr^2$ und $a=b=(0,0)$.
|
||||
|
||||
Je zwei Wege im $\mdr^2$ mit Anfangs- und Endpunkt $(0,0)$
|
||||
sind homotop.
|
||||
|
@ -99,7 +99,7 @@
|
|||
$\gamma_0(t) = (0,0) \; \forall t \in I$. Sei
|
||||
$\gamma(0) = \gamma(1) = (0,0)$.
|
||||
|
||||
$H(t,s) := (1-s) \gamma(t)$ ist stetig,
|
||||
$H(t,s) := (1-s) \gamma(t)$ ist stetig,
|
||||
$H(t,0) = \gamma(t)\; \forall t \in I$ und
|
||||
$H(t,1) = (0,0) \; \forall t \in I$.
|
||||
\end{bspenum}
|
||||
|
@ -123,7 +123,7 @@
|
|||
% Mitschrieb vom 05.12.2013 %
|
||||
%%%%%%%%%%%%%%%%%%%%%%%%%%%%%%%%%%%%%%%%%%%%%%%%%%%%%%%%%%%%%%%%%%%%%
|
||||
\begin{bemerkung}\label{kor:homotope-wege}
|
||||
Sei $X$ ein topologischer Raum, $\gamma: I \rightarrow X$ ein
|
||||
Sei $X$ ein topologischer Raum, $\gamma: I \rightarrow X$ ein
|
||||
Weg und $\varphi: I \rightarrow I$ stetig mit $\varphi(0) = 0$,
|
||||
$\varphi(1) = 1$. Dann sind $\gamma$ und $\gamma \circ \varphi$
|
||||
homotop.
|
||||
|
@ -139,7 +139,7 @@
|
|||
|
||||
\begin{definition}\xindex{Weg!zusammengesetzter}%
|
||||
Seien $\gamma_1, \gamma_2$ Wege in $X$ mit $\gamma_1(1) = \gamma_2(0)$.
|
||||
Dann ist
|
||||
Dann ist
|
||||
\[\gamma (t) = \begin{cases}
|
||||
\gamma_1(2t) &\text{falls } 0 \leq t < \frac{1}{2}\\
|
||||
\gamma_2(2t-1) &\text{falls } \frac{1}{2} \leq t \leq 1
|
||||
|
@ -149,7 +149,7 @@
|
|||
\end{definition}
|
||||
|
||||
\begin{bemerkung}\label{kor:assoziativitaet-von-zusammensetzen-von-wegen}
|
||||
Das Zusammensetzen von Wegen ist nur bis auf
|
||||
Das Zusammensetzen von Wegen ist nur bis auf
|
||||
Homotopie assoziativ, d.~h.:
|
||||
\begin{align*}
|
||||
\gamma_1 * (\gamma_2 * \gamma_3) &\neq (\gamma_1 * \gamma_2) * \gamma_3\\
|
||||
|
@ -203,13 +203,13 @@
|
|||
Sei $H_i$ eine Homotopie zwischen $\gamma_i$ und $\gamma_i'$,
|
||||
$i=1,2$.
|
||||
|
||||
Dann ist
|
||||
Dann ist
|
||||
\[H(t,s) := \begin{cases}
|
||||
H_1(2t, s) &\text{falls } 0 \leq t \leq \frac{1}{2}\;\;\;\forall s \in I\\
|
||||
H_2(2t-1,s) &\text{falls } \frac{1}{2} \leq t \leq 1
|
||||
\end{cases}\]
|
||||
|
||||
eine Homotopie zwischen
|
||||
eine Homotopie zwischen
|
||||
$\gamma_1 * \gamma_2$ und $\gamma_1' * \gamma_2 '$.
|
||||
\end{beweis}
|
||||
|
||||
|
@ -219,7 +219,7 @@ Eine spezielle Homotopieäquivalenz sind sog. Deformationsretraktionen:
|
|||
und $\iota = (\id_X)|_A$.
|
||||
|
||||
\begin{defenum}
|
||||
\item $\iota: A \rightarrow X$ mit $\iota(x) = x$ heißt die
|
||||
\item $\iota: A \rightarrow X$ mit $\iota(x) = x$ heißt die
|
||||
\textbf{Inklusionsabbildung}\xindex{Inklusionsabbildung} und
|
||||
man schreibt: $\iota: A \hookrightarrow X$.
|
||||
\item $r$ heißt \textbf{Retraktion}\xindex{Retraktion}, wenn $r|_A = \id_A$ ist.
|
||||
|
@ -264,7 +264,7 @@ Für einen Weg $\gamma$ sei $[\gamma]$ seine \textbf{Homotopieklasse}\xindex{Hom
|
|||
\item Assoziativität folgt aus \cref{kor:assoziativitaet-von-zusammensetzen-von-wegen}
|
||||
\item Neutrales Element $e = [\gamma_0], \gamma_0(t) = x \;\;\; \forall t \in I$.
|
||||
$e * [\gamma] = [\gamma] = [\gamma] * e$, da $\gamma_0 * \gamma \sim \gamma$
|
||||
\item \xindex{Weg!inverser} Inverses Element $[\gamma]^{-1} = [\overline{\gamma}] = [\gamma(1-t)]$,
|
||||
\item \xindex{Weg!inverser} Inverses Element $[\gamma]^{-1} = [\overline{\gamma}] = [\gamma(1-t)]$,
|
||||
denn $\overline{\gamma} * \gamma \sim \gamma_0 \sim \gamma * \overline{\gamma}$
|
||||
\end{enumerate}
|
||||
\end{beweis}
|
||||
|
@ -280,7 +280,7 @@ Für einen Weg $\gamma$ sei $[\gamma]$ seine \textbf{Homotopieklasse}\xindex{Hom
|
|||
$[\gamma^k] \mapsto k$ ist ein Isomorphismus.
|
||||
\item $\pi_1 (\mdr^2, 0) = \pi_1 (\mdr^2, x) = \Set{e}$ für jedes $x \in \mdr^2$
|
||||
\item $\pi_1 (\mdr^n, x) = \Set{e}$ für jedes $x \in \mdr^n$
|
||||
\item $G \subseteq \mdr^n$ heißt \textbf{sternförmig}\xindex{sternförmig} bzgl. $x \in G$,
|
||||
\item $G \subseteq \mdr^n$ heißt \textbf{sternförmig}\xindex{sternförmig} bzgl. $x \in G$,
|
||||
wenn für jedes $y \in G$ auch die Strecke $[x, y] \subseteq G$
|
||||
ist.
|
||||
|
||||
|
@ -338,7 +338,7 @@ Für einen Weg $\gamma$ sei $[\gamma]$ seine \textbf{Homotopieklasse}\xindex{Hom
|
|||
für ein $x \in X$.
|
||||
\end{definition}
|
||||
|
||||
Wenn $\pi_1(X,x) = \Set{e}$ für ein $x \in X$ gilt, dann wegen
|
||||
Wenn $\pi_1(X,x) = \Set{e}$ für ein $x \in X$ gilt, dann wegen
|
||||
\cref{kor:gruppenisomorphismus-wege} sogar für alle $x \in X$.
|
||||
|
||||
\begin{bemerkung}\label{korr:11.5}
|
||||
|
@ -359,7 +359,7 @@ Wenn $\pi_1(X,x) = \Set{e}$ für ein $x \in X$ gilt, dann wegen
|
|||
\item $f_*$ ist wohldefiniert: Seien $\gamma_1, \gamma_2$ homotope
|
||||
Wege von $x$. z.Z.: $f \circ \gamma_1 \sim f \circ \gamma_2$:
|
||||
Nach Voraussetzung gibt es stetige Abbildungen $H:I\times I \rightarrow X$
|
||||
mit
|
||||
mit
|
||||
\begin{align*}
|
||||
H(t,0) &= \gamma_1(t),\\
|
||||
H(t,1) &= \gamma_2(t),\\
|
||||
|
@ -376,7 +376,7 @@ Wenn $\pi_1(X,x) = \Set{e}$ für ein $x \in X$ gilt, dann wegen
|
|||
|
||||
\begin{beispiel}
|
||||
\begin{bspenum}
|
||||
\item $f:S^1 \hookrightarrow \mdr^2$ ist injektiv, aber
|
||||
\item $f:S^1 \hookrightarrow \mdr^2$ ist injektiv, aber
|
||||
$f_*:\pi_1(S^1, 1) \cong \mdz \rightarrow \pi_1(\mdr^2, 1) = \Set{e}$
|
||||
ist nicht injektiv.
|
||||
\item $f: \mdr \rightarrow S^1, t \mapsto (\cos 2 \pi t, \sin 2 \pi t)$
|
||||
|
@ -406,7 +406,7 @@ Wenn $\pi_1(X,x) = \Set{e}$ für ein $x \in X$ gilt, dann wegen
|
|||
stetig mit $f(x_0) = y_0 = g(x_0)$.
|
||||
|
||||
$f$ und $g$ heißen \textbf{homotop} ($f \sim g$), wenn es eine stetige
|
||||
Abbildung $H: X \times I \rightarrow Y$ mit
|
||||
Abbildung $H: X \times I \rightarrow Y$ mit
|
||||
\begin{align*}
|
||||
H(x,0) &= f(x) \; \forall x \in X\\
|
||||
H(x,1) &= g(x) \; \forall x \in X\\
|
||||
|
@ -426,7 +426,7 @@ Wenn $\pi_1(X,x) = \Set{e}$ für ein $x \in X$ gilt, dann wegen
|
|||
Z.~z.: $f \circ \gamma \sim g \circ \gamma$
|
||||
|
||||
Sei dazu $H_\gamma: I \times I \rightarrow Y, (t,s) \mapsto H(\gamma(t), s)$.
|
||||
Dann gilt:
|
||||
Dann gilt:
|
||||
\begin{align*}
|
||||
H_\gamma(t,0) &= H(\gamma(t), 0) = (f \circ \gamma)(t) \;\forall t \in I\\
|
||||
H_\gamma(1,s) &= H(\gamma(1), s) = H(x_0, s) = y_0\;\forall s \in I\\
|
||||
|
@ -450,7 +450,7 @@ Wenn $\pi_1(X,x) = \Set{e}$ für ein $x \in X$ gilt, dann wegen
|
|||
\end{beispiel}
|
||||
|
||||
\begin{satz}[Satz von Seifert und van Kampen \enquote{light}]\label{thm:seifert-van-kampen}
|
||||
Sei $X$ ein topologischer Raum, $U, V \subseteq X$ offen mit
|
||||
Sei $X$ ein topologischer Raum, $U, V \subseteq X$ offen mit
|
||||
$U \cup V = X$ und $U \cap V$ wegzusammenhängend.
|
||||
|
||||
Dann wird $\pi_1(X,x)$ für $x \in U \cap V$ erzeugt von geschlossenen
|
||||
|
@ -460,14 +460,14 @@ Wenn $\pi_1(X,x) = \Set{e}$ für ein $x \in X$ gilt, dann wegen
|
|||
\begin{beweis}
|
||||
Sei $\gamma: I \rightarrow X$ ein geschlossener Weg um $x$.
|
||||
Überdecke $I$ mit endlich vielen offenen Intervallen
|
||||
$I_1, I_2, \dots, I_n$, die ganz in
|
||||
$I_1, I_2, \dots, I_n$, die ganz in
|
||||
$\gamma^{-1}(U)$ oder ganz in $\gamma^{-1}(V)$ liegen.
|
||||
|
||||
\Obda sei $\gamma(I_1) \subseteq U, \gamma(I_2) \subseteq V$, etc.
|
||||
|
||||
Wähle $t_i \in I_i \cap I_{i+1}$, also $\gamma(t_i) \in U \cap V$.
|
||||
Sei $\sigma_i$ Weg in $U \cap V$ von $x_0$ nach $\gamma(t_i) \Rightarrow \gamma$
|
||||
ist homotop zu
|
||||
ist homotop zu
|
||||
\[\underbrace{\gamma_1 * \overline{\sigma_1}}_{\text{in } U} * \underbrace{\sigma_1 * \gamma_2 * \overline{\sigma_2}}_{\text{in } V} * \dots * \sigma_{n-1} * \gamma_2 \text{ mit } \gamma_i := \gamma |_{I_i}\]
|
||||
\end{beweis}
|
||||
|
||||
|
@ -547,17 +547,17 @@ Wenn $\pi_1(X,x) = \Set{e}$ für ein $x \in X$ gilt, dann wegen
|
|||
\begin{beweis}
|
||||
Sei $p: Y \rightarrow X$ eine Überlagerung und $x \in X$ beliebig.
|
||||
Dann existiert eine offene Umgebung $U(x) \subseteq X$ und offene
|
||||
Teilmengen $V_j \subseteq X$ mit
|
||||
Teilmengen $V_j \subseteq X$ mit
|
||||
$p^{-1}(U) = \Dcup V_j$ und
|
||||
$p|_{V_j}: V_j \rightarrow U$ ist Homöomorphismus.
|
||||
|
||||
D.~h. es existiert ein $y \in V_j$, so dass $p|_{V_j}(y) = x$.
|
||||
Da $x \in X$ beliebig war und ein $y \in Y$ existiert, mit
|
||||
Da $x \in X$ beliebig war und ein $y \in Y$ existiert, mit
|
||||
$p(y) = x$, ist $p$ surjektiv. $\qed$
|
||||
\end{beweis}
|
||||
|
||||
\begin{definition}\xindex{Abbildung!offene}%
|
||||
Seien $(X, \fT_X), (Y, \fT_Y)$ topologische Räume und $f:X \rightarrow Y$ eine
|
||||
Seien $(X, \fT_X), (Y, \fT_Y)$ topologische Räume und $f:X \rightarrow Y$ eine
|
||||
Abbildung.
|
||||
|
||||
$f$ heißt \textbf{offen} $:\gdw \forall U \in \fT_X: f(U) \in \fT_Y$.
|
||||
|
@ -600,7 +600,7 @@ Wenn $\pi_1(X,x) = \Set{e}$ für ein $x \in X$ gilt, dann wegen
|
|||
\begin{definition}\xindex{diskret}%
|
||||
Sei $X$ ein topologischer Raum und $M \subseteq X$.
|
||||
|
||||
$M$ heißt \textbf{diskret} in $X$, wenn $M$ in $X$ keinen
|
||||
$M$ heißt \textbf{diskret} in $X$, wenn $M$ in $X$ keinen
|
||||
Häufungspunkt hat.
|
||||
\end{definition}
|
||||
|
||||
|
@ -628,11 +628,11 @@ Wenn $\pi_1(X,x) = \Set{e}$ für ein $x \in X$ gilt, dann wegen
|
|||
$\Rightarrow V_{j_1} \cap V_{j_2} = \emptyset$ nach Voraussetzung.
|
||||
|
||||
\underline{2. Fall}: $p(y_1) \neq p(y_2)$.
|
||||
|
||||
|
||||
Dann seien $U_1$ und $U_2$ disjunkte Umgebungen von $p(y_1)$
|
||||
und $p(y_2)$.
|
||||
|
||||
$\Rightarrow p^{-1}(U_1)$ und $p^{-1}(U_2)$ sind disjunkte
|
||||
$\Rightarrow p^{-1}(U_1)$ und $p^{-1}(U_2)$ sind disjunkte
|
||||
Umgebungen von $y_1$ und $y_2$.
|
||||
|
||||
\item Sei $x \in X$ beliebig, aber fest.
|
||||
|
@ -717,7 +717,7 @@ Wenn $\pi_1(X,x) = \Set{e}$ für ein $x \in X$ gilt, dann wegen
|
|||
|
||||
Sei $q:U \rightarrow V$ die Umkehrabbildung zu $p|_V$.
|
||||
|
||||
Sei $W:= f^{-1}(U) \cap f_0^{-1}(V) \cap f_1^{-1}(V)$. $W$ ist
|
||||
Sei $W:= f^{-1}(U) \cap f_0^{-1}(V) \cap f_1^{-1}(V)$. $W$ ist
|
||||
offene Umgebung in $Z$ von $z$.
|
||||
|
||||
\underline{Behauptung:} $W \subseteq T$
|
||||
|
@ -749,7 +749,7 @@ $p|_{V_j}: V_j \rightarrow U$ Homöomorphismus.
|
|||
\begin{bemerkung}%Bemerkung 12.6 der Vorlesung
|
||||
Wege in $X$ lassen sich zu Wegen in $Y$ liften.
|
||||
|
||||
Zu jedem $y \in p^{-1}(\gamma(0))$ gibt es genau einen Lift von
|
||||
Zu jedem $y \in p^{-1}(\gamma(0))$ gibt es genau einen Lift von
|
||||
$\gamma$.
|
||||
\end{bemerkung}
|
||||
|
||||
|
@ -757,7 +757,7 @@ $p|_{V_j}: V_j \rightarrow U$ Homöomorphismus.
|
|||
Seien $p: Y \rightarrow X$ eine Überlagerung, $a,b \in X$,
|
||||
$\gamma_0, \gamma_1: I \rightarrow X$ homotope Wege von $a$ nach
|
||||
$b$, $\tilde{a} \in p^{-1}(a), \tilde{\gamma_0}, \tilde{\gamma_1}$
|
||||
Liftungen von $\gamma_0$ bzw. $\gamma_1$ mit
|
||||
Liftungen von $\gamma_0$ bzw. $\gamma_1$ mit
|
||||
$\tilde{\gamma_i}(0) = \tilde{a}$.
|
||||
|
||||
Dann ist $\tilde{\gamma_0}(1) = \tilde{\gamma_1}(1)$ und
|
||||
|
@ -784,7 +784,7 @@ $p|_{V_j}: V_j \rightarrow U$ Homöomorphismus.
|
|||
|
||||
Da $p^{-1}(b)$ diskrete Teilmenge von $Y$ ist\\
|
||||
$\Rightarrow \tilde{b_s} = \tilde{H}(1,s) = \tilde{H}(1,0) \;\forall s \in I$\\
|
||||
$\Rightarrow \tilde{b_0} = \tilde{b_1}$ und $\tilde{H}$ ist Homotopie
|
||||
$\Rightarrow \tilde{b_0} = \tilde{b_1}$ und $\tilde{H}$ ist Homotopie
|
||||
zwischen $\tilde{\gamma_0}$ und $\tilde{\gamma_1}$. $\qed$
|
||||
\end{beweis}
|
||||
|
||||
|
@ -801,7 +801,7 @@ $p|_{V_j}: V_j \rightarrow U$ Homöomorphismus.
|
|||
\item Sei $\tilde{\gamma}$ ein Weg in $Y$ um $y_0$ und
|
||||
$p_* ([\tilde{\gamma}]) = e$, also $p \circ \tilde{\gamma} \sim \gamma_{x_0}$
|
||||
|
||||
Nach \cref{proposition:12.7} ist dann
|
||||
Nach \cref{proposition:12.7} ist dann
|
||||
$\tilde{\gamma}$ homotop zum Lift des konstanten Wegs
|
||||
$\gamma_{x_0}$ mit Anfangspunkt $y_0$, also zu
|
||||
$\gamma_{y_0} \Rightarrow [\tilde{\gamma}] = e$
|
||||
|
@ -823,9 +823,9 @@ $p|_{V_j}: V_j \rightarrow U$ Homöomorphismus.
|
|||
|
||||
Zu $i \in \Set{0, \dots, d-1}$ gibt es Weg $\delta_i$ in
|
||||
$Y$ mit $\delta_i(0) = y_0$ und $\delta_i(1) = y_i$\\
|
||||
$\Rightarrow p \cup \delta_i$ ist geschlossener Weg in
|
||||
$\Rightarrow p \cup \delta_i$ ist geschlossener Weg in
|
||||
$X$ um $x_0$.\\
|
||||
$\Rightarrow$ Jedes $y_i$ mit $i=0, \dots, d-1$ ist
|
||||
$\Rightarrow$ Jedes $y_i$ mit $i=0, \dots, d-1$ ist
|
||||
$\tilde{\gamma}(1)$ für ein $[\gamma] \in \pi_1(X,x_0)$.
|
||||
\end{enumerate}
|
||||
\end{beweis}
|
||||
|
@ -885,7 +885,7 @@ $p|_{V_j}: V_j \rightarrow U$ Homöomorphismus.
|
|||
Offensichtlich ist $q(\tilde{p}(z)) = p(z)$.
|
||||
|
||||
\underline{Zu zeigen:} $\tilde{p}$ ist stetig in $z \in \tilde{X}$:
|
||||
|
||||
|
||||
Sei $W \subseteq Y$ offene Umgebung von $\tilde{p}(z)$.
|
||||
|
||||
$\xRightarrow{q \text{ offen}} q(W)$ ist offene Umgebung von $p(z) \cdot d(\tilde{p}(z))$.
|
||||
|
@ -915,8 +915,8 @@ $p|_{V_j}: V_j \rightarrow U$ Homöomorphismus.
|
|||
\end{folgerung}
|
||||
|
||||
\begin{beweis}
|
||||
Seien $x_0 \in X, \tilde{x_0} \in \tilde{X}$ mit
|
||||
$p(\tilde{x_0}) = x_0$ und
|
||||
Seien $x_0 \in X, \tilde{x_0} \in \tilde{X}$ mit
|
||||
$p(\tilde{x_0}) = x_0$ und
|
||||
$\tilde{y_0} \in q^{-1}(x_0) \subseteq \tilde{Y}$.
|
||||
|
||||
Nach \cref{thm:12.11} gibt es genau eine Überlagerung
|
||||
|
@ -925,7 +925,7 @@ $p|_{V_j}: V_j \rightarrow U$ Homöomorphismus.
|
|||
\[g: \tilde{Y} \rightarrow \tilde{X} \text{ mit } g(\tilde{y_0}) = \tilde{x_0} \text{ und } p \circ g = q\]
|
||||
|
||||
Damit gilt: $p \circ q \circ f = q \circ f = p$, $q \circ f \circ g = p \circ g = q$.
|
||||
Also ist $g \circ f: \tilde{X} \rightarrow \tilde{X}$ Lift von
|
||||
Also ist $g \circ f: \tilde{X} \rightarrow \tilde{X}$ Lift von
|
||||
$p:\tilde{X} \rightarrow X$ mit $(g \circ f) (\tilde{x_0}) = \tilde{x_0}$.
|
||||
|
||||
Da auch $\id_{\tilde{x}}$ diese Eigenschaft hat, folgt mit
|
||||
|
@ -961,7 +961,7 @@ der folgende Satz:
|
|||
\[\tilde{U} = \tilde{U}(x, [\gamma]) := \Set{(y, [\gamma * \alpha]) | y \in U, \alpha \text{ Weg in } U \text{ von } x \text{ nach } y} \]
|
||||
|
||||
$p$ ist Überlagerung: $p|_{\tilde{U}} : \tilde{U} \rightarrow U$
|
||||
bijektiv. $p$ ist stetig und damit $p|_{\tilde{U}}$ ein
|
||||
bijektiv. $p$ ist stetig und damit $p|_{\tilde{U}}$ ein
|
||||
Homöomorphismus.
|
||||
|
||||
Sind $\gamma_1, \gamma_2$ Wege von $x_0$ nach $x$ und $\gamma_1 \sim \gamma_2$,
|
||||
|
@ -990,7 +990,7 @@ der folgende Satz:
|
|||
|
||||
\begin{defenum}
|
||||
\item $f$ heißt \textbf{Decktransformation} von $p :\gdw p \circ f = p$.
|
||||
\item Die Decktransformationen von $p: Y \rightarrow X$ bilden mit der Verkettung eine Gruppe,
|
||||
\item Die Decktransformationen von $p: Y \rightarrow X$ bilden mit der Verkettung eine Gruppe,
|
||||
die sog. \textbf{Decktransformationsgruppe}\xindex{Decktransformationsgruppe}.
|
||||
Man schreibt:
|
||||
$\Deck(p)$, $\Deck(Y/X)$ oder $\Deck(Y \rightarrow X)$.
|
||||
|
@ -1023,10 +1023,10 @@ der folgende Satz:
|
|||
\end{itemize}
|
||||
\item Die Menge
|
||||
\[\Fix(f) = \Set{y \in Y | f(y) = y}\]
|
||||
ist abgeschlossen als Urbild der Diagonale
|
||||
ist abgeschlossen als Urbild der Diagonale
|
||||
$\Delta \subseteq Y \times Y$ unter der stetigen
|
||||
Abbildung $y \mapsto (f(y),y)$. Außerdem ist $\Fix(f)$
|
||||
offen, denn ist $y \in \Fix(f)$, so sei $U$ eine
|
||||
offen, denn ist $y \in \Fix(f)$, so sei $U$ eine
|
||||
Umgebung von $p(y) \in X$ wie in \cref{def:12.1}
|
||||
und $U \subseteq p^{-1}(U)$ die Komponente, die $y$
|
||||
enthält; also $p:V \rightarrow U$ ein Homöomorphismus.
|
||||
|
@ -1041,7 +1041,7 @@ der folgende Satz:
|
|||
\item Es sei $x_0 \in X$, $\deg(p) = d$ und $p^{-1}(x_0) = \Set{y_0, \dots, y_{d-1}}$.
|
||||
Für $f \in \Deck(Y/X)$ ist $f(y_0)= \Set{y_0, \dots, y_{d-1}}$.
|
||||
|
||||
Zu $i \in \Set{0, \dots, d-1}$ gibt es höchstens ein
|
||||
Zu $i \in \Set{0, \dots, d-1}$ gibt es höchstens ein
|
||||
$f \in \Deck(Y/X)$ mit $f(y_0) = y_1$, denn ist
|
||||
$f(y_0) = g(y_0)$, so ist $(g^{-1} \circ f)(y_0) = y_0$,
|
||||
also nach \cref{kor:12.14c} $g^{-1} \circ f = \id_Y$.
|
||||
|
@ -1061,7 +1061,7 @@ Nun werden wir eine Verbindung zwischen der Decktransformationsgruppe
|
|||
und der Fundamentalgruppe herstellen:
|
||||
|
||||
\begin{satz}\label{thm:12.15}%In Vorlesung: Satz 12.15
|
||||
Ist $p: \tilde{X} \rightarrow X$ eine universelle Überlagerung,
|
||||
Ist $p: \tilde{X} \rightarrow X$ eine universelle Überlagerung,
|
||||
so gilt:
|
||||
\[\Deck(\tilde{X}/X) \cong \pi_1(X, x_0)\;\;\;\forall x_0 \in X\]
|
||||
\end{satz}
|
||||
|
@ -1074,11 +1074,11 @@ und der Fundamentalgruppe herstellen:
|
|||
eindeutig bestimmt und damit auch $\rho$ wohldefiniert.
|
||||
|
||||
\begin{itemize}
|
||||
\item \underline{$\rho$ ist Gruppenhomomorphismus}: Seien
|
||||
\item \underline{$\rho$ ist Gruppenhomomorphismus}: Seien
|
||||
$f, g \in \Deck(\tilde{X}/ X) \Rightarrow \gamma_{g \circ f} = \gamma_g * g(\gamma_f)$
|
||||
$\Rightarrow p(\gamma_{g \circ f}) = p(\gamma_g) * \underbrace{(p \circ g)}_{=p} (\gamma_f) = \rho(g) \neq \rho(f)$
|
||||
\item \underline{$\rho$ ist injektiv}: $\rho(f) = e \Rightarrow p (\gamma_f) \sim \gamma_{x_0}$
|
||||
$\xRightarrow{\cref{thm:ueberlagerung-weg-satz-12.6}} \gamma_f \sim \gamma_{\tilde{x_0}}$
|
||||
$\xRightarrow{\cref{thm:ueberlagerung-weg-satz-12.6}} \gamma_f \sim \gamma_{\tilde{x_0}}$
|
||||
$\Rightarrow f(x_0) = \tilde{x_0} \xRightarrow{\crefabbr{kor:12.14c}} f = \id_{\tilde{x}}$.
|
||||
\item \underline{$\rho$ ist surjektiv}: Sei $[\gamma] \in \pi_1(X, x_0)$,
|
||||
$\tilde{\gamma}$ Lift von $\gamma$ nach $\tilde{x}$ mit
|
||||
|
@ -1086,7 +1086,7 @@ und der Fundamentalgruppe herstellen:
|
|||
sei $\tilde{x_1}$.
|
||||
|
||||
\underline{$p$ ist reguläre Überlagerung}: Seien
|
||||
$\tilde{x_0}, \tilde{x_1} \in \tilde{X}$ mit
|
||||
$\tilde{x_0}, \tilde{x_1} \in \tilde{X}$ mit
|
||||
$p(\tilde{x_0}) = p(\tilde{x_1})$. Nach \cref{thm:12.11}
|
||||
gibt es genau eine Überlagerung $\tilde{p}: \tilde{X} \rightarrow X$
|
||||
mit $p=p \circ \tilde{p}$ und $\tilde{p}(\tilde{x_0}) = \tilde{x_1}$.
|
||||
|
@ -1163,7 +1163,7 @@ und der Fundamentalgruppe herstellen:
|
|||
\[m_g: X \rightarrow X, x \mapsto g \circ x\]
|
||||
ein Homöomorphismus ist.
|
||||
\item Ist $G$ eine topologische Gruppe, so heißt die Gruppenoperation $\circ$
|
||||
\textbf{stetig}\xindex{Gruppenoperation!stetige}, wenn
|
||||
\textbf{stetig}\xindex{Gruppenoperation!stetige}, wenn
|
||||
\[\forall g \in G: m_g \text{ ist stetig}\]
|
||||
gilt.
|
||||
\end{defenum}
|
||||
|
@ -1175,7 +1175,7 @@ und der Fundamentalgruppe herstellen:
|
|||
\begin{beweis}\leavevmode
|
||||
Nach Voraussetzung ist $m_g := \circ |_{\Set{g} \times X} : X \rightarrow X, x \mapsto g \circ x$ stetig.
|
||||
|
||||
Die Umkehrabbildung zu $m_g$ ist $m_{g^{-1}}$:
|
||||
Die Umkehrabbildung zu $m_g$ ist $m_{g^{-1}}$:
|
||||
\begin{align*}
|
||||
(m_{g^{-1}} \circ m_g)(x) &= m_{g^{-1}} (m_g (x))\\
|
||||
&= m_{g^{-1}} (g \circ x)\\
|
||||
|
@ -1196,7 +1196,7 @@ und der Fundamentalgruppe herstellen:
|
|||
\begin{bemenum}
|
||||
\item Die Gruppenoperation von $G$ auf $X$ entsprechen bijektiv
|
||||
den Gruppenhomomorphismen $\varrho: G \rightarrow \Perm(X) = \Sym(X) = \Set{f: X \rightarrow X | f \text{ ist bijektiv}}$
|
||||
\item Ist $X$ ein topologischer Raum, so entsprechen dabei
|
||||
\item Ist $X$ ein topologischer Raum, so entsprechen dabei
|
||||
die Gruppenoperationen durch Homöomorphismus den Gruppenhomomorphismen
|
||||
$G \rightarrow \Homoo(X)$
|
||||
\end{bemenum}
|
||||
|
@ -1213,7 +1213,7 @@ und der Fundamentalgruppe herstellen:
|
|||
|
||||
Umgekehrt: Sei $\varrho: G \rightarrow \Perm(X)$ Gruppenhomomorphismus. Definiere $\circ: G \times X \rightarrow X$ durch $g \circ x = \varrho (g)(x)$.
|
||||
|
||||
z.~z. \cref{def:gruppenoperation.2}:
|
||||
z.~z. \cref{def:gruppenoperation.2}:
|
||||
\begin{align*}
|
||||
g_1 \circ (g_2 \circ x) &= \varrho (g_1) (g_2 \circ x)\\
|
||||
&= \varrho(g_1) (\varrho(g_2)(x))\\
|
||||
|
@ -1222,8 +1222,8 @@ und der Fundamentalgruppe herstellen:
|
|||
&= (g_1 \cdot g_2) \circ x
|
||||
\end{align*}
|
||||
|
||||
z.~z. \cref{def:gruppenoperation.1}:
|
||||
$1_G \cdot x = \varrho(1_G)(x) = \id_X(x) = x$, weil $\varrho$ ein
|
||||
z.~z. \cref{def:gruppenoperation.1}:
|
||||
$1_G \cdot x = \varrho(1_G)(x) = \id_X(x) = x$, weil $\varrho$ ein
|
||||
Homomorphismus ist.
|
||||
\end{beweis}
|
||||
|
||||
|
@ -1253,7 +1253,7 @@ und der Fundamentalgruppe herstellen:
|
|||
\begin{enumerate}[label=\roman*)]
|
||||
\item $[e] \circ \tilde{x} = \rtilde{e * \delta} = \tilde{x}$
|
||||
\item $\rtilde{\gamma_1 * \gamma_2 * \delta}(1) = [\gamma_1 * \gamma_2] \circ \tilde{x} = ([\gamma_1] * [\gamma_2]) \circ \tilde{x}$\\
|
||||
$\gamma_1 * \gamma_2 * \delta(1) = [\gamma_1] \circ (\tilde{\gamma_2 * \delta})(1) = [\gamma_1] \circ ([\gamma_2] \circ \tilde{x})$
|
||||
$\gamma_1 * \gamma_2 * \delta(1) = [\gamma_1] \circ (\tilde{\gamma_2 * \delta})(1) = [\gamma_1] \circ ([\gamma_2] \circ \tilde{x})$
|
||||
\end{enumerate}
|
||||
\end{beispiel}
|
||||
|
||||
|
|
|
@ -18,7 +18,7 @@
|
|||
\end{aufgabe}
|
||||
|
||||
\begin{aufgabe}\label{ub11:aufg3}
|
||||
Sei $(X, d)$ eine absolute Ebene. Der \textit{Abstand}\xindex{Abstand} eines
|
||||
Sei $(X, d)$ eine absolute Ebene. Der \textit{Abstand}\xindex{Abstand} eines
|
||||
Punktes $P$ zu einer Menge $Y \subseteq X$ von Punkten ist
|
||||
definiert durch $d(P, Y) := \inf{d(P, y) | y \in Y}$.
|
||||
|
||||
|
@ -27,13 +27,13 @@
|
|||
\item \label{ub11:aufg3.a} Ist $\triangle ABC$ ein Dreieck, in dem die Seiten
|
||||
$\overline{AB}$ und $\overline{AC}$ kongruent sind, so
|
||||
sind die Winkel $\angle ABC$ und $\angle BCA$ gleich.
|
||||
\item \label{ub11:aufg3.b} Ist $\triangle ABC$ ein beliebiges Dreieck, so liegt
|
||||
\item \label{ub11:aufg3.b} Ist $\triangle ABC$ ein beliebiges Dreieck, so liegt
|
||||
der längeren Seite der größere Winkel gegenüber und
|
||||
umgekehrt.
|
||||
\item \label{ub11:aufg3.c} Sind $g$ eine Gerade und $P \notin g$ ein Punkt, so gibt
|
||||
es eine eindeutige Gerade $h$ mit $P \in h$ und die
|
||||
$g$ im rechten Winkel schneidet. Diese Grade heißt
|
||||
\textit{Lot}\xindex{Lot} von $P$ auf $g$ und der
|
||||
$g$ im rechten Winkel schneidet. Diese Grade heißt
|
||||
\textit{Lot}\xindex{Lot} von $P$ auf $g$ und der
|
||||
Schnittpunkt des Lots mit $g$ heißt \textit{Lotfußpunkt}\xindex{Lotfußpunkt}.
|
||||
\end{aufgabeenum}
|
||||
\end{aufgabe}
|
||||
|
|
|
@ -12,25 +12,25 @@
|
|||
\section{Axiome für die euklidische Ebene}
|
||||
Axiome\xindex{Axiom} bilden die Grundbausteine jeder mathematischen Theorie. Eine
|
||||
Sammlung aus Axiomen nennt man Axiomensystem\xindex{Axiomensystem}.
|
||||
Da der Begriff des Axiomensystems so grundlegend ist, hat man auch
|
||||
Da der Begriff des Axiomensystems so grundlegend ist, hat man auch
|
||||
ein paar sehr grundlegende Forderungen an ihn: Axiomensysteme sollen
|
||||
\textbf{widerspruchsfrei} sein, die Axiome sollen möglichst
|
||||
\textbf{unabhängig} sein und \textbf{Vollständigkeit} wäre auch toll.
|
||||
Mit Unabhängigkeit ist gemeint, dass kein Axiom sich aus einem anderem
|
||||
herleiten lässt. Dies scheint auf den ersten Blick eine einfache
|
||||
Eigenschaft zu sein. Auf den zweiten Blick muss man jedoch einsehen,
|
||||
dass das Parallelenproblem, also die Frage ob das Parallelenaxiom
|
||||
unabhängig von den restlichen Axiomen ist, über 2000 Jahre nicht
|
||||
Eigenschaft zu sein. Auf den zweiten Blick muss man jedoch einsehen,
|
||||
dass das Parallelenproblem, also die Frage ob das Parallelenaxiom
|
||||
unabhängig von den restlichen Axiomen ist, über 2000 Jahre nicht
|
||||
gelöst wurde. Ein ganz anderes Kaliber ist die Frage nach der
|
||||
Vollständigkeit. Ein Axiomensystem gilt als Vollständig, wenn
|
||||
jede Aussage innerhalb des Systems verifizierbar oder falsifizierbar
|
||||
ist. Interessant ist hierbei der Gödelsche Unvollständigkeitssatz,
|
||||
ist. Interessant ist hierbei der Gödelsche Unvollständigkeitssatz,
|
||||
der z.~B. für die Arithmetik beweist, dass nicht alle Aussagen
|
||||
formal bewiesen oder widerlegt werden können.
|
||||
|
||||
Kehren wir nun jedoch zurück zur Geometrie. Euklid hat in seiner
|
||||
Kehren wir nun jedoch zurück zur Geometrie. Euklid hat in seiner
|
||||
Abhandlung \enquote{Die Elemente} ein Axiomensystem für die Geometrie
|
||||
aufgestellt.
|
||||
aufgestellt.
|
||||
|
||||
\textbf{Euklids Axiome}
|
||||
\begin{itemize}
|
||||
|
@ -39,11 +39,11 @@ aufgestellt.
|
|||
\item \textbf{Kreis} (um jeden Punkt mit jedem Radius)
|
||||
\item Je zwei rechte Winkel sind gleich (Isometrie, Bewegung)
|
||||
\item Parallelenaxiom von Euklid:\xindex{Parallelenaxiom}\\
|
||||
Wird eine Gerade so von zwei Geraden geschnitten, dass die
|
||||
Wird eine Gerade so von zwei Geraden geschnitten, dass die
|
||||
Summe der Innenwinkel kleiner als zwei Rechte ist, dann schneiden sich
|
||||
diese Geraden auf der Seite dieser Winkel.\\
|
||||
\\
|
||||
Man mache sich klar, dass das nur dann nicht der Fall ist,
|
||||
Man mache sich klar, dass das nur dann nicht der Fall ist,
|
||||
wenn beide Geraden parallel sind und senkrecht auf die erste stehen.
|
||||
\end{itemize}
|
||||
|
||||
|
@ -60,7 +60,7 @@ aufgestellt.
|
|||
\end{enumerate}
|
||||
\item \textbf{Abstandsaxiom}\xindex{Abstandsaxiom}: Zu $P, Q, R \in X$ gibt es \label{axiom:2}
|
||||
genau dann ein $g \in G$ mit $\Set{P, Q, R} \subseteq g$,
|
||||
wenn gilt:
|
||||
wenn gilt:
|
||||
\begin{itemize}[]
|
||||
\item $d(P, R) = d(P, Q) + d(Q, R)$ oder
|
||||
\item $d(P, Q) = d(P, R) + d(R, Q)$ oder
|
||||
|
@ -72,7 +72,7 @@ aufgestellt.
|
|||
\begin{definition}
|
||||
Sei $(X, d, G)$ eine Geometrie und seien $P, Q, R \in X$.
|
||||
\begin{defenum}
|
||||
\item $P, Q, R$ liegen \textbf{kollinear}\xindex{kollinear},
|
||||
\item $P, Q, R$ liegen \textbf{kollinear}\xindex{kollinear},
|
||||
wenn es $g \in G$ gibt mit $\Set{P, Q, R} \subseteq g$.
|
||||
\item $Q$ \textbf{liegt zwischen}\xindex{liegt zwischen} $P$
|
||||
und $R$, wenn $d(P, R) = d(P, Q) + d(Q, R)$
|
||||
|
@ -103,10 +103,10 @@ aufgestellt.
|
|||
\begin{beweis}\leavevmode
|
||||
\begin{enumerate}[label=\alph*)]
|
||||
\item \enquote{$\subseteq$} folgt direkt aus der Definition von $PR^+$ und $PR^-$\\
|
||||
\enquote{$\supseteq$}: Sei $Q \in PR \Rightarrow P, Q, R$
|
||||
\enquote{$\supseteq$}: Sei $Q \in PR \Rightarrow P, Q, R$
|
||||
sind kollinear.\\
|
||||
$\overset{\ref{axiom:2}}{\Rightarrow}
|
||||
\begin{cases}
|
||||
\begin{cases}
|
||||
Q \text{ liegt zwischen } P \text{ und } R \Rightarrow Q \in PR\\
|
||||
R \text{ liegt zwischen } P \text{ und } Q \Rightarrow Q \in PR\\
|
||||
P \text{ liegt zwischen } Q \text{ und } R \Rightarrow Q \in PR
|
||||
|
@ -133,21 +133,21 @@ aufgestellt.
|
|||
\begin{enumerate}[label=§\arabic*),ref=§\arabic*,start=3]
|
||||
\item \label{axiom:3}\textbf{Anordnungsaxiome}\xindex{Anordnungsaxiome}
|
||||
\begin{enumerate}[label=(\roman*),ref=\theenumi{} (\roman*)]
|
||||
\item \label{axiom:3.1} Zu jeder
|
||||
Halbgerade $H$ mit Anfangspunkt $P \in X$ und jedem
|
||||
$r \in \mdr_{\geq 0}$ gibt es genau ein
|
||||
\item \label{axiom:3.1} Zu jeder
|
||||
Halbgerade $H$ mit Anfangspunkt $P \in X$ und jedem
|
||||
$r \in \mdr_{\geq 0}$ gibt es genau ein
|
||||
$Q \in H$ mit $d(P,Q) = r$.
|
||||
\item \label{axiom:3.2} Jede Gerade zerlegt
|
||||
$X \setminus g = H_1 \dcup H_2$ in zwei
|
||||
\item \label{axiom:3.2} Jede Gerade zerlegt
|
||||
$X \setminus g = H_1 \dcup H_2$ in zwei
|
||||
nichtleere Teilmengen $H_1, H_2$,
|
||||
sodass für alle $A \in H_i$, $B \in H_j$ mit
|
||||
$i,j \in \Set{1,2}$ gilt:
|
||||
$i,j \in \Set{1,2}$ gilt:
|
||||
$\overline{AB} \cap g \neq \emptyset \Leftrightarrow i \neq j$.\\
|
||||
Diese Teilmengen $H_i$ heißen
|
||||
\textbf{Halbebenen}\xindex{Halbebene} bzgl.
|
||||
Diese Teilmengen $H_i$ heißen
|
||||
\textbf{Halbebenen}\xindex{Halbebene} bzgl.
|
||||
$g$.
|
||||
\end{enumerate}
|
||||
\item \label{axiom:4}\textbf{Bewegungsaxiom}\xindex{Bewegungsaxiom}:
|
||||
\item \label{axiom:4}\textbf{Bewegungsaxiom}\xindex{Bewegungsaxiom}:
|
||||
Zu $P, Q, P', Q' \in X$
|
||||
mit $d(P,Q) = d(P', Q')$ gibt es mindestens 2 Isometrien $\varphi_1, \varphi_2$
|
||||
mit $\varphi_i (P) = P'$ und $\varphi_i(Q) = Q'$ mit $i=1,2$.\footnote{Die \enquote{Verschiebung} von $P'Q'$ nach $PQ$ und die Isometrie, die zusätzlich an der Gerade durch $P$ und $Q$ spiegelt.}
|
||||
|
@ -163,14 +163,14 @@ aufgestellt.
|
|||
%%%%%%%%%%%%%%%%%%%%%%%%%%%%%%%%%%%%%%%%%%%%%%%%%%%%%%%%%%%%%%%%%%%%%
|
||||
\begin{satz}[Satz von Pasch]\label{satz:pasch} %In Vorlesung: Bemerkung 14.5
|
||||
Seien $P$, $Q$, $R$ nicht kollinear, $g \in G$ mit $g \cap \Set{P, Q, R} = \emptyset$
|
||||
und $g \cap \overline{PQ} \neq \emptyset$.
|
||||
und $g \cap \overline{PQ} \neq \emptyset$.
|
||||
|
||||
Dann ist entweder $g \cap \overline{PR} \neq \emptyset$ oder
|
||||
Dann ist entweder $g \cap \overline{PR} \neq \emptyset$ oder
|
||||
$g \cap \overline{QR} \neq \emptyset$.
|
||||
\end{satz}
|
||||
|
||||
Dieser Satz besagt, dass Geraden, die eine Seite eines Dreiecks
|
||||
(also nicht nur eine Ecke) schneiden, auch eine weitere Seite
|
||||
Dieser Satz besagt, dass Geraden, die eine Seite eines Dreiecks
|
||||
(also nicht nur eine Ecke) schneiden, auch eine weitere Seite
|
||||
schneiden.
|
||||
|
||||
\begin{beweis}
|
||||
|
@ -182,7 +182,7 @@ schneiden.
|
|||
\end{beweis}
|
||||
|
||||
\begin{bemerkung}\label{kor:beh3}
|
||||
Sei $P, Q \in X$ mit $P \neq Q$ sowie $A, B \in X \setminus PQ$
|
||||
Sei $P, Q \in X$ mit $P \neq Q$ sowie $A, B \in X \setminus PQ$
|
||||
mit $A \neq B$.
|
||||
Außerdem seien $A$ und $B$ in der selben Halbebene bzgl. $PQ$ sowie
|
||||
$Q$ und $B$ in der selben Halbebene bzgl. $PA$.
|
||||
|
@ -197,8 +197,8 @@ schneiden.
|
|||
\label{fig:geometry-5}
|
||||
\end{figure}
|
||||
|
||||
Auch \cref{kor:beh3} lässt sich umgangssprachlich sehr viel
|
||||
einfacher ausdrücken: Die Diagonalen eines konvexen Vierecks
|
||||
Auch \cref{kor:beh3} lässt sich umgangssprachlich sehr viel
|
||||
einfacher ausdrücken: Die Diagonalen eines konvexen Vierecks
|
||||
schneiden sich.
|
||||
|
||||
\begin{beweis}%In Vorlesung: Behauptung 3
|
||||
|
@ -217,7 +217,7 @@ schneiden sich.
|
|||
$\overline{AP'}$ liegt in der anderen Halbebene
|
||||
bzgl. $PA \Rightarrow C \notin \overline{P'A} \Rightarrow C \in \overline{AQ}$
|
||||
\end{enumerate}
|
||||
Da $C \in PB^+$ und $C \in \overline{AQ}$ folgt nun direkt:
|
||||
Da $C \in PB^+$ und $C \in \overline{AQ}$ folgt nun direkt:
|
||||
$\emptyset \neq \Set{C} \subseteq PB^+ \cap \overline{AQ} \qed$
|
||||
\end{beweis}
|
||||
|
||||
|
@ -286,7 +286,7 @@ schneiden sich.
|
|||
|
||||
\begin{bemerkung}\label{kor:beh2'}
|
||||
Sei $(X, d, G)$ eine Geometrie, die \ref{axiom:1}~-~\ref{axiom:3}
|
||||
erfüllt, $P, Q \in X$ mit $P \neq Q$ und $\varphi$ eine Isometrie mit
|
||||
erfüllt, $P, Q \in X$ mit $P \neq Q$ und $\varphi$ eine Isometrie mit
|
||||
$\varphi(P) = P$ und $\varphi(Q) = Q$.
|
||||
|
||||
Dann gilt $\varphi(S) = S\;\;\;\forall S \in PQ$.
|
||||
|
@ -302,7 +302,7 @@ schneiden sich.
|
|||
&\overset{\mathclap{\ref{axiom:3.1}}}{\Rightarrow} \varphi(S) = S
|
||||
\end{align*}
|
||||
|
||||
$\qed$
|
||||
$\qed$
|
||||
\end{beweis}
|
||||
|
||||
\begin{proposition}\label{satz:14.4}%In Vorlesung: Satz 14.4
|
||||
|
@ -310,7 +310,7 @@ schneiden sich.
|
|||
gibt es zu $P, P', Q, Q'$ mit $d(P, Q) = d(P', Q')$ höchstens
|
||||
zwei Isometrien mit $\varphi(P) = P'$ und $\varphi(Q) = Q'$
|
||||
|
||||
Aus den Axiomen folgt, dass es in
|
||||
Aus den Axiomen folgt, dass es in
|
||||
der Situation von \ref{axiom:4} höchstens zwei Isometrien mit
|
||||
$\varphi_i(P) = P'$ und $\varphi_i(Q) = Q'$ gibt.
|
||||
\end{proposition}
|
||||
|
@ -332,11 +332,11 @@ schneiden sich.
|
|||
|
||||
Nun zu den Beweisen der Teilaussagen:
|
||||
\begin{enumerate}[label=(Teil \roman*),ref=(Teil \roman*)]
|
||||
\item Sei $R \in X \setminus PQ$. Von den drei Punkten
|
||||
\item Sei $R \in X \setminus PQ$. Von den drei Punkten
|
||||
$\varphi_1(R), \varphi_2(R), \varphi_3(R)$ liegen zwei
|
||||
in der selben Halbebene bzgl. $P'Q' = \varphi_i(PQ)$.
|
||||
|
||||
\Obda seien $\varphi_1(R)$ und $\varphi_2(R)$ in der
|
||||
\Obda seien $\varphi_1(R)$ und $\varphi_2(R)$ in der
|
||||
selben Halbebene.
|
||||
|
||||
Es gilt: $\begin{aligned}[t]
|
||||
|
@ -392,7 +392,7 @@ schneiden sich.
|
|||
|
||||
Also gilt insbesondere $\varphi(\triangle A'B'C') = \triangle ABC$. $\qed$
|
||||
\end{beweis}
|
||||
|
||||
|
||||
\begin{bemerkung}[WSW-Kongruenzsatz]\xindex{Kongruenzsatz!WSW}%
|
||||
Sei $(X, d, G)$ eine Geometrie, die \ref{axiom:1}~-~\ref{axiom:4} erfüllt.
|
||||
Seien außerdem $\triangle ABC$ und $\triangle A'B'C'$ Dreiecke, für die gilt:
|
||||
|
@ -424,15 +424,15 @@ schneiden sich.
|
|||
|
||||
\begin{definition}\label{def:14.8}%In Vorlesung: 14.8
|
||||
\begin{defenum}
|
||||
\item \label{def:14.8a} Ein \textbf{Winkel}\xindex{Winkel} ist ein Punkt $P \in X$
|
||||
\item \label{def:14.8a} Ein \textbf{Winkel}\xindex{Winkel} ist ein Punkt $P \in X$
|
||||
zusammen mit $2$ Halbgeraden mit Anfangspunkt $P$.\\
|
||||
Man schreibt: $\angle R_1 P R_2$ bzw. $\angle R_2 P R_1$\footnote{Für dieses Skript gilt: $\angle R_1 P R_2 = \angle R_2 P R_1$. Also sind insbesondere alle Winkel $ \leq 180^\circ$.}
|
||||
\item Zwei Winkel sind \textbf{gleich}, wenn es eine Isometrie gibt,
|
||||
\item Zwei Winkel sind \textbf{gleich}, wenn es eine Isometrie gibt,
|
||||
die den einen Winkel auf den anderen abbildet.
|
||||
\item \label{def:14.8c} $\angle R_1' P' R_2'$ heißt \textbf{kleiner} als
|
||||
$\angle R_1 P R_2$, wenn es eine Isometrie $\varphi$
|
||||
gibt, mit $\varphi(P') = P$, $\varphi(P'R'^{+}_{1}) = PR_{1}^{+}$
|
||||
und $\varphi(R_2')$ liegt in der gleichen Halbebene
|
||||
und $\varphi(R_2')$ liegt in der gleichen Halbebene
|
||||
bzgl. $PR_1$ wie $R_2$ und in der gleichen Halbebene
|
||||
bzgl. $PR_2$ wie $R_1$
|
||||
\item \label{def:14.8d} Im Dreieck $\triangle PQR$ gibt es \textbf{Innenwinkel}\xindex{Innenwinkel} und
|
||||
|
@ -455,7 +455,7 @@ schneiden sich.
|
|||
\end{figure}
|
||||
|
||||
\begin{bemerkung}\label{bem:14.9}%In Vorlesung: Bemerkung 14.9
|
||||
In einem Dreieck ist jeder Innenwinkel kleiner als jeder nicht
|
||||
In einem Dreieck ist jeder Innenwinkel kleiner als jeder nicht
|
||||
anliegende Außenwinkel.
|
||||
\end{bemerkung}
|
||||
|
||||
|
@ -481,7 +481,7 @@ schneiden sich.
|
|||
\caption{Situation aus \cref{bem:14.9}}
|
||||
\end{figure}
|
||||
|
||||
Es gilt: $d(Q,M) = d(M,R)$ und $d(P,M) = d(M,A)$ sowie
|
||||
Es gilt: $d(Q,M) = d(M,R)$ und $d(P,M) = d(M,A)$ sowie
|
||||
$\angle PMR = \angle AMQ \Rightarrow \triangle MRQ$ ist
|
||||
kongruent zu $\triangle AMQ$, denn eine der beiden Isometrien, die
|
||||
$\angle PMR$ auf $\angle AMQ$ abbildet, bildet $R$ auf $Q$ und
|
||||
|
@ -496,7 +496,7 @@ schneiden sich.
|
|||
\begin{proposition}[Existenz der Parallelen]\label{prop:14.7}%In Vorlesung: Proposition 14.7
|
||||
Sei $(X, d, G)$ eine Geometrie mit den Axiomen \ref{axiom:1}~-~\ref{axiom:4}.
|
||||
|
||||
Dann gibt es zu jeder Geraden $g \in G$ und jedem Punkt $P \in X \setminus g$
|
||||
Dann gibt es zu jeder Geraden $g \in G$ und jedem Punkt $P \in X \setminus g$
|
||||
mindestens eine Parallele $h \in G$ mit $P \in h$ und $g \cap h = \emptyset$.
|
||||
\end{proposition}
|
||||
|
||||
|
@ -535,7 +535,7 @@ Halbebene bzgl. $PQ$ liegt wie $R$.
|
|||
|
||||
\begin{figure}[htp]
|
||||
\centering
|
||||
\includegraphics[width=0.4\linewidth, keepaspectratio]{figures/Spherical_triangle_3d_opti.png}
|
||||
\includegraphics[width=0.4\linewidth, keepaspectratio]{figures/Spherical_triangle_3d_opti.png}
|
||||
\caption{In der sphärischen Geometrie gibt es, im Gegensatz zur euklidischen Geometrie, Dreiecke mit drei $90^\circ$-Winkeln.}
|
||||
\label{fig:spherical-triangle}
|
||||
\end{figure}
|
||||
|
@ -567,7 +567,7 @@ Sei im Folgenden \enquote{$\IWS$} die \enquote{Innenwinkelsumme}.
|
|||
Sei $\alpha$ ein Innenwinkel von $\triangle$.
|
||||
|
||||
\begin{behauptung}
|
||||
Es gibt ein Dreieck $\triangle'$ mit
|
||||
Es gibt ein Dreieck $\triangle'$ mit
|
||||
$\IWS(\triangle') = \IWS(\triangle)$ und einem Innenwinkel
|
||||
$\alpha' \leq \frac{\alpha}{2}$.
|
||||
|
||||
|
@ -579,8 +579,8 @@ Sei im Folgenden \enquote{$\IWS$} die \enquote{Innenwinkelsumme}.
|
|||
\end{behauptung}
|
||||
|
||||
\begin{beweis}
|
||||
Es seien $A, B, C \in X$ und $\triangle $ das Dreieck mit den
|
||||
Eckpunkten $A, B, C$ und $\alpha$ sei der Innenwinkel bei $A$,
|
||||
Es seien $A, B, C \in X$ und $\triangle $ das Dreieck mit den
|
||||
Eckpunkten $A, B, C$ und $\alpha$ sei der Innenwinkel bei $A$,
|
||||
$\beta$ der Innenwinkel bei $B$ und $\gamma$ der Innenwinkel bei $C$.
|
||||
|
||||
Sei $M$ der Mittelpunkt der Strecke $\overline{BC}$. Sei außerdem
|
||||
|
@ -613,7 +613,7 @@ Sei im Folgenden \enquote{$\IWS$} die \enquote{Innenwinkelsumme}.
|
|||
\end{figure}
|
||||
|
||||
\begin{beweis}
|
||||
Sei $g$ eine Parallele von $AB$ durch $C$.
|
||||
Sei $g$ eine Parallele von $AB$ durch $C$.
|
||||
|
||||
\begin{itemize}
|
||||
\item Es gilt $\alpha' = \alpha$ wegen \cref{prop:14.7}.
|
||||
|
@ -639,7 +639,7 @@ WSW.\xindex{Kongruenzsatz!SWW}
|
|||
\label{fig:hyperbolische-geometrie-2}
|
||||
\end{figure}
|
||||
|
||||
Der Beweis wird hier nicht geführt. Für Beweisvorschläge wäre ich
|
||||
Der Beweis wird hier nicht geführt. Für Beweisvorschläge wäre ich
|
||||
dankbar.
|
||||
|
||||
\begin{figure}[htp]
|
||||
|
@ -730,10 +730,10 @@ $\xRightarrow{\text{Strahlensatz}} \frac{a}{h_c} = \frac{c}{h_a} \rightarrow a \
|
|||
\item $(\mdr^2, d_\text{Euklid})$ ist offensichtlich eine euklidische Ebene.
|
||||
\item Sei $(X,d)$ eine euklidische Ebene und $g_1, g_2$ Geraden
|
||||
in $X$, die sich in einem Punkt $0$ im rechten Winkel
|
||||
schneiden.
|
||||
schneiden.
|
||||
|
||||
Sei $P \in X \setminus (g_1 \cup g_2)$ ein Punkt und $P_X$ der
|
||||
Fußpunkt des Lots von $P$ auf $g_1$ (vgl. \cref{ub11:aufg3.c})
|
||||
Sei $P \in X \setminus (g_1 \cup g_2)$ ein Punkt und $P_X$ der
|
||||
Fußpunkt des Lots von $P$ auf $g_1$ (vgl. \cref{ub11:aufg3.c})
|
||||
und $P_Y$ der Fußpunkt des Lots von $P$ auf $g_2$.
|
||||
|
||||
Sei $x_P := d(P_X, 0)$ und $y_P := d(P_Y, 0)$.
|
||||
|
@ -754,11 +754,11 @@ $\xRightarrow{\text{Strahlensatz}} \frac{a}{h_c} = \frac{c}{h_a} \rightarrow a \
|
|||
\label{fig:14.13.0.1}
|
||||
\end{figure}
|
||||
|
||||
|
||||
|
||||
Sei $h:X \rightarrow \mdr^2$ eine Abbildung mit
|
||||
$h(P) := (x_P, y_P)$
|
||||
Dadurch wird $h$ auf dem Quadranten
|
||||
definiert, in dem $P$ liegt, d.~h.
|
||||
definiert, in dem $P$ liegt, d.~h.
|
||||
\[\forall Q \in X \text{ mit } \overline{PQ} \cap g_1 = \emptyset = \overline{PQ} \cap g_2\]
|
||||
|
||||
Fortsetzung auf ganz $X$ durch konsistente Vorzeichenwahl.
|
||||
|
@ -833,7 +833,7 @@ $\xRightarrow{\text{Strahlensatz}} \frac{a}{h_c} = \frac{c}{h_a} \rightarrow a \
|
|||
Betrachte nun $z_1$ und $z_2$ als Punkte in der
|
||||
euklidischen Ebene. Die Mittelsenkrechte zu diesen
|
||||
Punkten schneidet die $x$-Achse. Alle Punkte auf
|
||||
der Mittelsenkrechten zu $z_1$ und $z_2$ sind gleich
|
||||
der Mittelsenkrechten zu $z_1$ und $z_2$ sind gleich
|
||||
weit von $z_1$ und $z_2$ entfernt. Daher ist
|
||||
der Schnittpunkt mit der $x$-Achse der Mittelpunkt
|
||||
eines Kreises durch $z_1$ und $z_2$ (vgl. \cref{fig:hyperbolische-geometrie-axiom-1-2})
|
||||
|
@ -867,7 +867,7 @@ $\xRightarrow{\text{Strahlensatz}} \frac{a}{h_c} = \frac{c}{h_a} \rightarrow a \
|
|||
|
||||
\underline{Zu zeigen:}
|
||||
$\forall A \in H_i$, $B \in H_j$ mit
|
||||
$i,j \in \Set{1,2}$ gilt:
|
||||
$i,j \in \Set{1,2}$ gilt:
|
||||
$\overline{AB} \cap g \neq \emptyset \Leftrightarrow i \neq j$\\
|
||||
\enquote{$\Leftarrow$}: $A \in H_1, B \in H_2: \overline{AB} \cap g \neq \emptyset$
|
||||
|
||||
|
@ -882,10 +882,10 @@ $\xRightarrow{\text{Strahlensatz}} \frac{a}{h_c} = \frac{c}{h_a} \rightarrow a \
|
|||
|
||||
Sei $h$ die Gerade, die durch $A$ und $B$ geht.
|
||||
|
||||
Da $A,B \notin g$, aber $A, B \in h$ gilt, haben $g$ und $h$
|
||||
Da $A,B \notin g$, aber $A, B \in h$ gilt, haben $g$ und $h$
|
||||
insbesondere
|
||||
mindestens einen unterschiedlichen Punkt. Aus \ref{axiom:1.1} folgt, dass sich
|
||||
$g$ und $h$ in höchstens einen Punkt schneiden. Sei $C$ dieser
|
||||
$g$ und $h$ in höchstens einen Punkt schneiden. Sei $C$ dieser
|
||||
Punkt.
|
||||
|
||||
Aus $A,B \notin g$ folgt: $C \neq A$ und $C \neq B$. Also liegt
|
||||
|
@ -903,8 +903,8 @@ $\xRightarrow{\text{Strahlensatz}} \frac{a}{h_c} = \frac{c}{h_a} \rightarrow a \
|
|||
\end{beweis}
|
||||
|
||||
\begin{definition}\xindex{Möbiustransformation}%
|
||||
Es seien $a,b,c,d \in \mdr$ mit $ad - bc \neq 0$ und
|
||||
$\sigma: \mdc \rightarrow \mdc$ eine Abbildung definiert durch
|
||||
Es seien $a,b,c,d \in \mdr$ mit $ad - bc \neq 0$ und
|
||||
$\sigma: \mdc \rightarrow \mdc$ eine Abbildung definiert durch
|
||||
\[\sigma(z) := \frac{az + b}{cz+d}\]
|
||||
|
||||
$\sigma$ heißt \textbf{Möbiustransformation}.
|
||||
|
@ -953,7 +953,7 @@ $\xRightarrow{\text{Strahlensatz}} \frac{a}{h_c} = \frac{c}{h_a} \rightarrow a \
|
|||
&= \frac{a(a'z+b')+b(c'z+d')}{c(a'z+b') + d(c'z+d')}\\
|
||||
&= \frac{(aa'+bc')z + ab' + bd'}{(ca'+db')z + cb' + dd'}\\
|
||||
&= \begin{pmatrix}aa'+bc'&ab'+bd'\\ca'+db'&cb'+dd'\end{pmatrix} \circ z\\
|
||||
&= \left ( \begin{pmatrix}a&b\\c&d\end{pmatrix} \cdot \begin{pmatrix}a'&b'\\c'&d'\end{pmatrix} \right ) \circ z
|
||||
&= \left ( \begin{pmatrix}a&b\\c&d\end{pmatrix} \cdot \begin{pmatrix}a'&b'\\c'&d'\end{pmatrix} \right ) \circ z
|
||||
\end{align*}
|
||||
\item Es gilt $\sigma(z) = (-\sigma)(z)$ für alle $\sigma \in \SL_2(\mdr)$
|
||||
und $z \in \mdh$.
|
||||
|
@ -975,7 +975,7 @@ $\xRightarrow{\text{Strahlensatz}} \frac{a}{h_c} = \frac{c}{h_a} \rightarrow a \
|
|||
|
||||
Daher genügt es zu zeigen, dass man mit $A_{\lambda}$, $B_t$ und $C$ alle Matrizen
|
||||
aus $\SL_2(\mdr)$ erzeugen kann, genügt es also von einer beliebigen
|
||||
Matrix durch Multiplikation mit Matrizen der Form $A_{\lambda}$,
|
||||
Matrix durch Multiplikation mit Matrizen der Form $A_{\lambda}$,
|
||||
$B_t$ und $C$ die Einheitsmatrix zu generieren.
|
||||
|
||||
Sei also
|
||||
|
@ -1056,9 +1056,9 @@ $\xRightarrow{\text{Strahlensatz}} \frac{a}{h_c} = \frac{c}{h_a} \rightarrow a \
|
|||
\begin{definition}\xindex{Doppelverhältnis}%In Vorlesung: Def+Prop 15.4
|
||||
Seien $z_1, z_2, z_3, z_4 \in \mdc$ paarweise verschieden.
|
||||
|
||||
Dann heißt
|
||||
Dann heißt
|
||||
\[\DV(z_1, z_2, z_3, z_4) := \frac{\frac{z_1 - z_4}{z_1 - z_2}}{\frac{z_3 - z_4}{z_3 - z_2}} = \frac{(z_1 - z_4) \cdot (z_3 - z_2)}{(z_1 - z_2) \cdot (z_3 - z_4)}\]
|
||||
\textbf{Doppelverhältnis} von
|
||||
\textbf{Doppelverhältnis} von
|
||||
$z_1, \dots, z_4$.
|
||||
\end{definition}
|
||||
|
||||
|
@ -1071,7 +1071,7 @@ $\xRightarrow{\text{Strahlensatz}} \frac{a}{h_c} = \frac{c}{h_a} \rightarrow a \
|
|||
oder wenn zwei der $z_i$ gleich sind.
|
||||
\item $\DV(0, 1, \infty, z_4) = z_4$ (Der Fall $z_4 \in \Set{0, 1, \infty}$ ist zugelassen).
|
||||
\item \label{bem:15.4d} Für $\sigma \in \PSL_2(\mdc)$ und $z_1, \dots, z_4 \in \mdc \cup \Set{\infty}$
|
||||
ist
|
||||
ist
|
||||
\[\DV(\sigma(z_1), \sigma(z_2), \sigma(z_3), \sigma(z_4)) = \DV(z_1, z_2, z_3, z_4)\]
|
||||
und für $\sigma(z) = \frac{1}{\overline{z}}$ gilt
|
||||
\[\DV(\sigma(z_1), \sigma(z_2), \sigma(z_3), \sigma(z_4)) = \overline{\DV(z_1, z_2, z_3, z_4)}\]
|
||||
|
@ -1110,7 +1110,7 @@ $\xRightarrow{\text{Strahlensatz}} \frac{a}{h_c} = \frac{c}{h_a} \rightarrow a \
|
|||
Durch Einsetzen ergibt sich $\DV(z_1, \dots, z_4)=1$.
|
||||
\end{itemize}
|
||||
|
||||
Im Fall, dass ein $z_i = \infty$ ist, ist
|
||||
Im Fall, dass ein $z_i = \infty$ ist, ist
|
||||
entweder $\DV(0, 1, \infty, z_4) = 0$ oder $\DV(0, 1, \infty, z_4) \pm \infty$
|
||||
\item $\DV(0, 1, \infty, z_4) = \frac{(0- z_4) \cdot (\infty - 1)}{(0 -1) \cdot (\infty - z_4)} = \frac{z_4 \cdot (\infty - 1)}{\infty - z_4} = z_4$
|
||||
\item Wenn jemand diesen Beweis führt, bitte an info@martin-thoma.de schicken.%TODO
|
||||
|
|
|
@ -12,16 +12,16 @@
|
|||
\section{Krümmung von Kurven}\label{sec:Kurvenkrümmung}
|
||||
\begin{definition}%In Vorlesung: Def.+Bem. 16.1
|
||||
Sei $\gamma: I = [a, b] \rightarrow \mdr^n$ eine Kurve.
|
||||
|
||||
|
||||
\begin{defenum}
|
||||
\item Die Kurve $\gamma$ heißt
|
||||
\item Die Kurve $\gamma$ heißt
|
||||
\textbf{durch Bogenlänge parametrisiert}\xindex{parametrisiert!durch Bogenlänge},
|
||||
wenn gilt:
|
||||
\[\|\gamma'(t)\|_2 = 1 \;\;\; \forall t \in I\]
|
||||
Dabei ist $\gamma'(t) = \left (\gamma_1'(t), \gamma_2'(t), \dots, \gamma_n'(t) \right)$.
|
||||
\item $l(\gamma) = \int_a^b \|\gamma'(t)\| \mathrm{d} t$ heißt
|
||||
\textbf{Länge von $\gamma$}\xindex{Kurve!Länge einer}.
|
||||
\end{defenum}
|
||||
\end{defenum}
|
||||
\end{definition}
|
||||
|
||||
\begin{bemerkung}[Eigenschaften von Kurven I]%In Vorlesung: Def.+Bem. 16.1
|
||||
|
@ -29,7 +29,7 @@
|
|||
|
||||
\begin{bemenum}
|
||||
\item Ist $\gamma$ durch Bogenlänge parametrisiert, so ist $l(\gamma) = b-a$.
|
||||
\item \label{bem:16.1d} Ist $\gamma$ durch Bogenlänge parametrisiert, so ist
|
||||
\item \label{bem:16.1d} Ist $\gamma$ durch Bogenlänge parametrisiert, so ist
|
||||
$\gamma'(t)$ orthogonal zu $\gamma''(t)$ für alle $t \in I$.
|
||||
\end{bemenum}
|
||||
\end{bemerkung}
|
||||
|
@ -85,8 +85,8 @@ Da $n(t)$ und $\gamma''(t)$ nach \cref{bem:16.1d} linear
|
|||
da gilt:
|
||||
|
||||
\begin{align*}
|
||||
\langle n(t), \gamma'(t) \rangle &=
|
||||
\left \langle
|
||||
\langle n(t), \gamma'(t) \rangle &=
|
||||
\left \langle
|
||||
\begin{pmatrix}- \cos \frac{t}{r}\\ - \sin \frac{t}{r}\end{pmatrix},
|
||||
\begin{pmatrix}- \sin \frac{t}{r}\\ \cos \frac{t}{r}\end{pmatrix}
|
||||
\right \rangle\\
|
||||
|
@ -128,7 +128,7 @@ Da $n(t)$ und $\gamma''(t)$ nach \cref{bem:16.1d} linear
|
|||
Also gilt:
|
||||
\[\det(\gamma'(t), n(t), b(t)) = 1\]
|
||||
$b(t)$ heißt \textbf{Binormalenvektor}\xindex{Binormalenvektor},
|
||||
die Orthonormalbasis
|
||||
die Orthonormalbasis
|
||||
\[\Set{\gamma'(t), n(t), b(t)}\]
|
||||
heißt \textbf{begleitendes Dreibein}\xindex{Dreibein!begleitendes}.
|
||||
\end{defenum}
|
||||
|
@ -185,16 +185,16 @@ für eine $C^\infty$-Funktion $f: \mdr^3 \rightarrow \mdr$.
|
|||
\begin{beweis}\leavevmode
|
||||
\begin{enumerate}[label=\alph*)]
|
||||
\item \label{bew:tangentialebene.a} $J_F$ ist eine $3 \times 2$-Matrix, die mit einem $2 \times 1$-Vektor
|
||||
multipliziert wird. Das ist eine lineare Abbildung und aus der
|
||||
multipliziert wird. Das ist eine lineare Abbildung und aus der
|
||||
linearen Algebra ist bekannt, das das Bild ein Vektorraum ist.
|
||||
Da $\rang(J_F) = 2$, ist auch $\dim (T_s S) = 2$.
|
||||
\item Hier kann man wie in \cref{bew:tangentialebene.a} argumentieren
|
||||
\item $T_s S = \{x \in \mdr^3 | \exists \text{parametrisierte Kurve }
|
||||
\gamma:[- \varepsilon, + \varepsilon] \rightarrow S
|
||||
\text{ für ein } \varepsilon > 0
|
||||
\item $T_s S = \{x \in \mdr^3 | \exists \text{parametrisierte Kurve }
|
||||
\gamma:[- \varepsilon, + \varepsilon] \rightarrow S
|
||||
\text{ für ein } \varepsilon > 0
|
||||
\text{ mit } \gamma(0) = s \text{ und } \gamma'(0) = x
|
||||
\}$\\
|
||||
Wenn jemand diesen Beweis führt, bitte an info@martin-thoma.de
|
||||
Wenn jemand diesen Beweis führt, bitte an info@martin-thoma.de
|
||||
schicken.%TODO
|
||||
\item Sei $x \in T_s S, \gamma:[-\varepsilon, +\varepsilon] \rightarrow S$
|
||||
eine parametrisierte Kurve mit $\varepsilon > 0$ und $\gamma'(0) = s$,
|
||||
|
@ -231,7 +231,7 @@ Im Folgenden werden diese Begriffe jedoch synonym benutzt.
|
|||
von $s$ und eine lokale Parametrisierung $F: U \rightarrow V$
|
||||
von $S$ um $s$, sodass auf $F(U) = V \cap S$
|
||||
ein stetiges Normalenfeld existiert.
|
||||
\item $S$ ist genau dann orientierbar, wenn es einen
|
||||
\item $S$ ist genau dann orientierbar, wenn es einen
|
||||
differenzierbaren Atlas von $S$ aus lokalen Parametrisierungen
|
||||
$F_i: U_i \rightarrow V_i,\;i \in I$ gibt, sodass
|
||||
für alle $i, j \in F$ und alle $s \in V_i \cap V_j \cap S$
|
||||
|
@ -256,7 +256,7 @@ Im Folgenden werden diese Begriffe jedoch synonym benutzt.
|
|||
|
||||
\begin{figure}[htp]\xindex{Möbiusband}
|
||||
\centering
|
||||
\includegraphics[width=0.5\linewidth, keepaspectratio]{figures/moebius-strip.pdf}
|
||||
\includegraphics[width=0.5\linewidth, keepaspectratio]{figures/moebius-strip.pdf}
|
||||
\caption{Möbiusband}
|
||||
\label{fig:moebius-strip}
|
||||
\end{figure}
|
||||
|
@ -266,7 +266,7 @@ Im Folgenden werden diese Begriffe jedoch synonym benutzt.
|
|||
Sei $S$ eine reguläre Fläche, $s \in S$, $n(s)$ ist ein Normalenvektor
|
||||
in $s$, $x \in T_s S$, $\|x\| = 1$.
|
||||
|
||||
Sei $E$ der von $x$ und $n(s)$ aufgespannte 2-dimensionale
|
||||
Sei $E$ der von $x$ und $n(s)$ aufgespannte 2-dimensionale
|
||||
Untervektorraum von $\mdr^3$.
|
||||
|
||||
Dann gibt es eine Umgebung $V \subseteq \mdr^3$ von $s$, sodass
|
||||
|
@ -277,7 +277,7 @@ Im Folgenden werden diese Begriffe jedoch synonym benutzt.
|
|||
\end{bemerkung}
|
||||
|
||||
\begin{beweis}
|
||||
\enquote{Satz über implizite Funktionen}\footnote{Siehe z.~B.
|
||||
\enquote{Satz über implizite Funktionen}\footnote{Siehe z.~B.
|
||||
\url{https://github.com/MartinThoma/LaTeX-examples/tree/master/documents/Analysis\%20II}}
|
||||
\end{beweis}
|
||||
|
||||
|
@ -362,18 +362,18 @@ Im Folgenden werden diese Begriffe jedoch synonym benutzt.
|
|||
\end{beweis}
|
||||
|
||||
\begin{bemerkung}%In Vorlesung: Bem.+Def. 18.6
|
||||
Sei $S$ eine reguläre Fläche und $n=n(s)$ ein Normalenvektor an
|
||||
Sei $S$ eine reguläre Fläche und $n=n(s)$ ein Normalenvektor an
|
||||
$S$ in $s$.
|
||||
|
||||
Sei $T_{s}^{1} S = \Set{x \in T_s S | \|x\| = 1} \cong S^1$.
|
||||
Dann ist
|
||||
Dann ist
|
||||
\[ \kappanor^n(s): T^1_s S \rightarrow \mdr, \;\;\; x \mapsto \kappanor(s,x)\]
|
||||
eine glatte Funktion und
|
||||
eine glatte Funktion und
|
||||
$\Bild \kappanor^n(s)$ ist ein abgeschlossenes Intervall.
|
||||
\end{bemerkung}
|
||||
|
||||
\begin{definition}\xindex{Hauptkrümmung}\xindex{Gauß-Krümmung}%In Vorlesung: Bem.+Def. 18.6
|
||||
Sei $S$ eine reguläre Fläche und $n=n(s)$ ein Normalenvektor an
|
||||
Sei $S$ eine reguläre Fläche und $n=n(s)$ ein Normalenvektor an
|
||||
$S$ in $s$.
|
||||
|
||||
\begin{defenum}
|
||||
|
@ -388,7 +388,7 @@ Im Folgenden werden diese Begriffe jedoch synonym benutzt.
|
|||
\end{definition}
|
||||
|
||||
\begin{bemerkung}%In Vorlesung: Bem.+Def. 18.6
|
||||
Ersetzt man $n$ durch $-n$, so gilt:
|
||||
Ersetzt man $n$ durch $-n$, so gilt:
|
||||
|
||||
\begin{align*}
|
||||
\kappanor^{-n}(s, x) &= - \kappanor^n(x)\; \forall x \in T_s^1 S\\
|
||||
|
@ -457,7 +457,7 @@ $s$. Weiter sei $p := F^{-1}(s)$.
|
|||
\item \label{bem:19.1a} Die Einschränkung des Standardskalarproduktes des $\mdr^3$ auf
|
||||
$T_s S$ macht $T_s S$ zu einem euklidischen Vektorraum.
|
||||
\item $\Set{D_p F(e_1), D_p F(e_2)}$ ist eine Basis von $T_s S$.
|
||||
\item Bzgl. der Basis $\Set{D_p F(e_1), D_p F(e_2)}$ hat das
|
||||
\item Bzgl. der Basis $\Set{D_p F(e_1), D_p F(e_2)}$ hat das
|
||||
Standardskalarprodukt aus \cref{bem:19.1a} die Darstellungsmatrix
|
||||
$I_S$.
|
||||
\item $g_{i,j}(s)$ ist eine differenzierbare Funktion von $s$.
|
||||
|
@ -465,7 +465,7 @@ $s$. Weiter sei $p := F^{-1}(s)$.
|
|||
\end{bemerkung}
|
||||
|
||||
\begin{bemerkung}
|
||||
\[\det(I_S) = \left \| \frac{\partial F}{\partial u_1}(p) \times \frac{\partial F}{\partial u_2}(p) \right \|^2\]
|
||||
\[\det(I_S) = \left \| \frac{\partial F}{\partial u_1}(p) \times \frac{\partial F}{\partial u_2}(p) \right \|^2\]
|
||||
\end{bemerkung}
|
||||
|
||||
\begin{beweis}\leavevmode
|
||||
|
@ -495,7 +495,7 @@ $s$. Weiter sei $p := F^{-1}(s)$.
|
|||
\begin{defenum}
|
||||
\item Das Differential $\mathrm{d} A = \sqrt{\det (I)} \mathrm{d} u_1 \mathrm{d} u_2$
|
||||
heißt \textbf{Flächenelement} von $S$ bzgl. der Parametrisierung $F$.
|
||||
\item \label{def:berechenbares-integral}Für eine Funktion $f: V \rightarrow \mdr$ heißt
|
||||
\item \label{def:berechenbares-integral}Für eine Funktion $f: V \rightarrow \mdr$ heißt
|
||||
\[\int_V f \mathrm{d} A := \int_U f(\underbrace{F(u_1, u_2)}_{=: s}) \sqrt{\det I(s)} \mathrm{d} u_1 \mathrm{d} u_2\]
|
||||
der \textbf{Wert des Integrals} von $f$ über $V$, falls das Integral rechts
|
||||
existiert.
|
||||
|
@ -535,7 +535,7 @@ $s$. Weiter sei $p := F^{-1}(s)$.
|
|||
|
||||
\begin{propenum}
|
||||
\item \label{prop:5.1a} $n$ induziert für jedes $s \in S$ eine lineare Abbildung $d_s n: T_s S \rightarrow T_{n(s)} S^2$
|
||||
durch
|
||||
durch
|
||||
\[d_s n(x) = \frac{\mathrm{d}}{\mathrm{d} t} n (\underbrace{s \text{\enquote{+}} tx}_{\mathclap{\text{Soll auf Fläche $S$ bleiben}}}) \Bigr |_{t=0}\]
|
||||
Die Abbildung $d_s n$ heißt \textbf{Weingarten-Abbildung}
|
||||
\item $T_{n(s)} S^2 = T_s S$.
|
||||
|
@ -560,7 +560,7 @@ $s$. Weiter sei $p := F^{-1}(s)$.
|
|||
|
||||
Sei $x_i = D_p F(e_i) = \frac{\partial F}{\partial u_i} (p)\;\;\; i = 1,2$
|
||||
|
||||
\underline{Beh.:}
|
||||
\underline{Beh.:}
|
||||
$\langle x_i, d_s n(x_j) \rangle = \langle \frac{\partial^2 F}{\partial u_i \partial u_j} (p), d_s n (x_i) \rangle$
|
||||
|
||||
$\Rightarrow \langle \frac{\partial^2 F}{\partial u_i \partial u_j} (p), d_s n (x_i) \rangle = \langle x_j, d_s n (x_i) \rangle$
|
||||
|
@ -597,9 +597,9 @@ $s$. Weiter sei $p := F^{-1}(s)$.
|
|||
|
||||
\begin{beweis}
|
||||
Nach \cref{def:18.4} ist $\kappanor(s, \gamma) = \langle \gamma''(0), n(s) \rangle$.
|
||||
Nach Voraussetzung gilt
|
||||
Nach Voraussetzung gilt
|
||||
\[n(\gamma(t)) \perp \gamma'(t) \Leftrightarrow \langle \gamma''(0), n(s) \rangle = 0\]
|
||||
Die Ableitung nach $t$ ergibt
|
||||
Die Ableitung nach $t$ ergibt
|
||||
\begin{align*}
|
||||
0 &= \frac{\mathrm{d}}{\mathrm{d}t}(\langle n (\gamma(t)), \gamma'(t))\\
|
||||
&= \left \langle \frac{\mathrm{d}}{\mathrm{d}t} n(\gamma(t)) \Bigr |_{t=0}, \gamma'(0) \right \rangle + \langle n(s), \gamma''(0) \rangle\\
|
||||
|
@ -629,7 +629,7 @@ $s$. Weiter sei $p := F^{-1}(s)$.
|
|||
Eigenvektoren $y_1, y_2$ von $II_s$. Ist $x \in T_s S$, $\|x\| = 1$,
|
||||
so gibt es $\varphi \in [0,2\pi)$ mit $x = \cos \varphi \cdot y_1 + \sin \varphi \cdot y_2$.
|
||||
|
||||
Seien $\lambda_1, \lambda_2$ die Eigenwerte von $II_s$, also
|
||||
Seien $\lambda_1, \lambda_2$ die Eigenwerte von $II_s$, also
|
||||
$II_s(y_i, y_i) = \lambda_i$. Dann gilt:
|
||||
\begin{align*}
|
||||
II_s (x,x) &= \cos^2 \varphi \lambda_1 + \sin^2 \varphi \lambda_2\\
|
||||
|
|
|
@ -20,13 +20,13 @@
|
|||
$(X, \fT_X)$ ist also nicht hausdorffsch.
|
||||
|
||||
\textbf{Teilaufgabe c)} Nach Bemerkung \ref{Trennungseigenschaft}
|
||||
sind metrische Räume hausdorffsch. Da $(X, \fT_X)$ nach (b) nicht
|
||||
sind metrische Räume hausdorffsch. Da $(X, \fT_X)$ nach (b) nicht
|
||||
hausdorffsch ist, liefert die Kontraposition der Trennungseigenschaft,
|
||||
dass $(X, \fT_X)$ kein metrischer Raum sein kann.
|
||||
\end{solution}
|
||||
|
||||
\begin{solution}[\ref{ub1:aufg4}]
|
||||
\textbf{Teilaufgabe a)}
|
||||
\textbf{Teilaufgabe a)}
|
||||
|
||||
\textbf{Beh.:} $\forall a \in \mdz: \Set{a}$ ist abgeschlossen.
|
||||
|
||||
|
@ -35,7 +35,7 @@
|
|||
Wenn jemand diese Aufgabe gemacht hat, bitte die Lösung an info@martin-thoma.de
|
||||
schicken.%TODO
|
||||
|
||||
\textbf{Teilaufgabe b)}
|
||||
\textbf{Teilaufgabe b)}
|
||||
|
||||
\textbf{Beh.:} $\Set{-1, 1}$ ist nicht offen
|
||||
|
||||
|
@ -50,7 +50,7 @@
|
|||
in dieser Topologie offen sein $\Rightarrow \Set{-1,1}$ ist
|
||||
nicht offen. $\qed$
|
||||
|
||||
\textbf{Teilaufgabe c)}
|
||||
\textbf{Teilaufgabe c)}
|
||||
|
||||
\textbf{Beh.:} Es gibt unendlich viele Primzahlen.
|
||||
|
||||
|
@ -58,7 +58,7 @@
|
|||
|
||||
Annahme: Es gibt nur endlich viele Primzahlen $p \in \mdp$
|
||||
|
||||
Dann ist
|
||||
Dann ist
|
||||
\[\mdz \setminus \Set{-1, +1} \overset{\text{FS d. Arithmetik}}= \bigcup_{p \in \mdp} U_{0,p}\]
|
||||
endlich. Das ist ein Widerspruch zu $|\mdz|$ ist unendlich und
|
||||
$|\Set{-1,1}|$ ist endlich. $\qed$
|
||||
|
@ -67,7 +67,7 @@
|
|||
\begin{solution}[\ref{ub2:aufg4}]
|
||||
\begin{enumerate}[label=(\alph*)]
|
||||
\item \textbf{Beh.:} Die offenen Mengen von $P$ sind
|
||||
Vereinigungen von Mengen der Form
|
||||
Vereinigungen von Mengen der Form
|
||||
\[\prod_{j \in J} U_j \times \prod_{i \in \mdn, i \neq j} P_i\]
|
||||
wobei $J \subseteq \mdn$ endlich und $U_j \subseteq P_j$
|
||||
offen ist.
|
||||
|
@ -79,7 +79,7 @@
|
|||
$\forall{j \in J}$
|
||||
eine Basis der Topologie.
|
||||
|
||||
Damit sind die offenen
|
||||
Damit sind die offenen
|
||||
Mengen von $P$ Vereinigungen von Mengen der obigen
|
||||
Form. $\qed$
|
||||
\end{beweis}
|
||||
|
@ -95,10 +95,10 @@
|
|||
für alle $i \in \mdn$ gilt entweder $p_i(Z) \subseteq \Set{0}$
|
||||
oder $p_i(Z) \subseteq \Set{1}$. Es sei $z_i \in \Set{0,1}$
|
||||
so, dass $p_i(Z) \subseteq \Set{z_i}$ für alle $i \in \mdn$.
|
||||
Dann gilt also:
|
||||
Dann gilt also:
|
||||
\[\underbrace{p_i(x)}_{= x_i} = z_i = \underbrace{p_i(y)}_{= y_i} \forall i \in \mdn\]
|
||||
Somit folgt: $x = y \qed$
|
||||
|
||||
|
||||
\end{beweis}
|
||||
\end{enumerate}
|
||||
\end{solution}
|
||||
|
@ -107,9 +107,9 @@
|
|||
\begin{enumerate}[label=(\alph*)]
|
||||
\item \textbf{Beh.:} $\GL_n(\mdr)$ ist nicht kompakt.\\
|
||||
\textbf{Bew.:} $\det: \GL_n(\mdr) \rightarrow \mdr \setminus \Set{0}$
|
||||
ist stetig. Außerdem ist
|
||||
$\det(\GL_n(\mdr)) = \mdr \setminus \Set{0}$ nicht
|
||||
kompakt. $\overset{\ref{kor:5.6}}{\Rightarrow}$
|
||||
ist stetig. Außerdem ist
|
||||
$\det(\GL_n(\mdr)) = \mdr \setminus \Set{0}$ nicht
|
||||
kompakt. $\overset{\ref{kor:5.6}}{\Rightarrow}$
|
||||
$\GL_n(\mdr)$ ist nicht kompakt. $\qed$
|
||||
\item \textbf{Beh.:} $\SL_1(\mdr)$ ist nicht kompakt, für $n > 1$ ist $\SL_n(\mdr)$ kompakt.\\
|
||||
\textbf{Bew.:} Für $\SL_1(\mdr)$ gilt:
|
||||
|
@ -119,7 +119,7 @@
|
|||
|
||||
$\SL_n(\mdr) \subseteq \GL_n(\mdr)$ lässt sich mit einer
|
||||
Teilmenge des $\mdr^{n^2}$ identifizieren. Nach \cref{satz:heine-borel}
|
||||
sind diese genau dann kompakt, wenn sie beschränkt und
|
||||
sind diese genau dann kompakt, wenn sie beschränkt und
|
||||
abgeschlossen sind. Definiere nun für für $n \in \mdn_{\geq 2}, m \in \mdn$:
|
||||
\[A_m = \text{diag}_n(m, \frac{1}{m}, \dots, 1)\]
|
||||
Dann gilt: $\det A_m = 1$, d.~h. $A_m \in \SL_n(\mdr)$,
|
||||
|
@ -138,7 +138,7 @@
|
|||
nachgelesen werden.
|
||||
|
||||
\begin{definition}\xindex{Homomorphismus}%
|
||||
Seien $(G, *)$ und $(H, \circ)$ Gruppen und
|
||||
Seien $(G, *)$ und $(H, \circ)$ Gruppen und
|
||||
$\varphi:G \rightarrow H$ eine Abbildung.
|
||||
|
||||
$\varphi$ heißt \textbf{Homomorphismus}, wenn
|
||||
|
@ -161,16 +161,16 @@
|
|||
\begin{solution}[\ref{ub3:meinsExtra2}]
|
||||
Die Definition einer Isotopie kann auf \cpageref{def:Isotopie} nachgelesen
|
||||
werden, die einer Isometrie auf \cpageref{def:Isometrie}.
|
||||
|
||||
|
||||
\begin{definition}\xindex{Isomorphismus}%
|
||||
Seien $(G, *)$ und $(H, \circ)$ Gruppen und
|
||||
Seien $(G, *)$ und $(H, \circ)$ Gruppen und
|
||||
$\varphi:G \rightarrow H$ eine Abbildung.
|
||||
|
||||
$\varphi$ heißt \textbf{Isomorphismus}, wenn $\varphi$ ein bijektiver
|
||||
Homomorphismus ist.
|
||||
\end{definition}
|
||||
|
||||
Eine Isotopie ist also für Knoten definiert, Isometrien machen nur in
|
||||
Eine Isotopie ist also für Knoten definiert, Isometrien machen nur in
|
||||
metrischen Räumen Sinn und ein Isomorphismus benötigt eine Gruppenstruktur.
|
||||
\end{solution}
|
||||
|
||||
|
@ -180,7 +180,7 @@
|
|||
\textbf{Beh.:} $M$ ist wegzusammehängend $\gdw M$ ist zusammenhängend
|
||||
\begin{beweis}
|
||||
\enquote{$\Rightarrow$}: Da $M$ insbesondere ein
|
||||
topologischer Raum ist folgt diese Richtung direkt
|
||||
topologischer Raum ist folgt diese Richtung direkt
|
||||
aus \cref{kor:wegzusammehang-impliziert-zusammenhang}.
|
||||
|
||||
\enquote{$\Leftarrow$}: Seien $x,y \in M$ und
|
||||
|
@ -192,7 +192,7 @@
|
|||
\item $Z^C := \Set{\tilde{z} \in M | \nexists \text{Weg von } x \text{ nach } \tilde{z}}$ ist offen
|
||||
|
||||
Da $M$ eine Mannigfaltigkeit ist, existiert zu jedem
|
||||
$\tilde{z} \in Z^C$ eine offene und wegzusammenhängende Umgebung
|
||||
$\tilde{z} \in Z^C$ eine offene und wegzusammenhängende Umgebung
|
||||
$U_{\tilde{z}} \subseteq M$.
|
||||
|
||||
Es gilt sogar $U_{\tilde{z}} \subseteq Z^C$, denn
|
||||
|
@ -226,7 +226,7 @@
|
|||
Da $(\mdr \setminus \Set{0}) \cup \Set{0_1}$ homöomorph
|
||||
zu $\mdr$ ist, exisitert ein Weg $\gamma_1$ von $0_1$
|
||||
zu einem beliebigen Punkt $a \in \mdr \setminus \Set{0}$.
|
||||
|
||||
|
||||
Da $(\mdr \setminus \Set{0}) \cup \Set{0_2}$ ebenfalls
|
||||
homöomorph zu $\mdr$ ist, existiert außerdem ein Weg
|
||||
$\gamma_2$ von $a$ nach $0_2$. Damit existiert ein
|
||||
|
@ -241,18 +241,18 @@
|
|||
% \textbf{Beh.:} $H_k = \begin{cases}\mdr &\text{für } k\in \Set{0,1}\\
|
||||
% 0 &\text{für } k \geq 2$
|
||||
% \newcommand{\triangleSimplizialkomplex}{\mathord{\includegraphics[height=5ex]{figures/triangleSimplizialkomplex.pdf}}}
|
||||
% \textbf{Bew.:} $S^1$ ist homöomorph zum Simplizialkomplex
|
||||
% \textbf{Bew.:} $S^1$ ist homöomorph zum Simplizialkomplex
|
||||
% $X = \triangleSimplizialkomplex$, d.~h. dem Rand
|
||||
% von $\Delta^2$. Es gilt:
|
||||
% \[X = \Set{\underbrace{v_0, v_1, v_2}_{A_0(X)}, \underbrace{\Delta (v_1, v_2)}_{=: a_0}, \underbrace{\underbrace{\Delta (v_0, v_2)}_{=: a_1}, \underbrace{\Delta(v_0, v_1)}_{=: a_2}}_{A_1(X)}}\]
|
||||
% Damit folgt:
|
||||
% Damit folgt:
|
||||
% \begin{enumerate}
|
||||
% \item Für $k \geq 2$ ist $C_k(X) \cong 0$, da es in diesen
|
||||
% \item Für $k \geq 2$ ist $C_k(X) \cong 0$, da es in diesen
|
||||
% Dimensionen keine Simplizes gibt, d.~h. $A_k(X) = \emptyset$ gilt.\\
|
||||
% Also: $H_k(X) \cong 0 \; \forall k \geq 2$
|
||||
% \item $C_0(X) = \Set{\sum_{i=0}^2 c_i v_i | c_i \in \mdr}$, da
|
||||
% \item $C_0(X) = \Set{\sum_{i=0}^2 c_i v_i | c_i \in \mdr}$, da
|
||||
% $A_0(x)$ Basis von $C_0(X)$ ist;\\
|
||||
% $C_1(X) = \Set{\sum_{i=0}^2 c_i a_i | c_i \in \mdr}$, da
|
||||
% $C_1(X) = \Set{\sum_{i=0}^2 c_i a_i | c_i \in \mdr}$, da
|
||||
% $A_1(X)$ Basis von $C_1(X)$ ist.
|
||||
% \item Für die Randabbildungen $d_i: C_i(X) \rightarrow C_{i-1}(X)$ gilt:
|
||||
% $d_0 \equiv 0$, $d_1: C_1(X) \rightarrow C_0(X)$ ist definiert durch
|
||||
|
@ -287,7 +287,7 @@
|
|||
$\angle BCA \xRightarrow{\crefabbr{bem:14.9}} \measuredangle BC' A > \measuredangle BCA$\\
|
||||
$\Rightarrow \measuredangle BCA < \measuredangle BC' A = \measuredangle ABC' < \measuredangle ABC $
|
||||
Sei umgekehrt $\measuredangle ABC > \measuredangle BCA$,
|
||||
kann wegen 1. Teil von \cref{ub11:aufg3.b} nicht
|
||||
kann wegen 1. Teil von \cref{ub11:aufg3.b} nicht
|
||||
$d(A,B) > d(A,C)$ gelten.\\
|
||||
Wegen \cref{ub11:aufg3.a} kann nicht $d(A,B) = d(A,C)$
|
||||
gelten.\\
|
||||
|
@ -343,7 +343,7 @@
|
|||
d(B, C) &= d(B', C')
|
||||
\end{align*}
|
||||
|
||||
Sei $\varphi$ die Isometrie mit $\varphi(A) = A'$, $\varphi(B) = B'$ und
|
||||
Sei $\varphi$ die Isometrie mit $\varphi(A) = A'$, $\varphi(B) = B'$ und
|
||||
$\varphi(C')$ liegt in der selben Halbebene bzgl. $AB$ wie $C$. Diese
|
||||
Isometrie existiert wegen \ref{axiom:4}.
|
||||
|
||||
|
|
|
@ -5,23 +5,23 @@ wird, die die Inhalte noch lernen, sind sehr wahrscheinlich einige
|
|||
Fehler im Skript. Das können Übertragungsfehler, Tippfehler oder
|
||||
Verständnisprobleme sein.
|
||||
|
||||
Verbesserungsvorschläge (auch wenn es nur einzelne Textsetzungsprobleme oder
|
||||
Verbesserungsvorschläge (auch wenn es nur einzelne Textsetzungsprobleme oder
|
||||
Rechtschreibfehler sind) bitte immer direkt melden oder verbessern!
|
||||
|
||||
Den Verbesserungsvorschlag kann man
|
||||
* entweder direkt selbst umsetzen und einen pull request machen oder
|
||||
* mir per Email (info@martin-thoma.de) schicken.
|
||||
Den Verbesserungsvorschlag kann man
|
||||
* entweder direkt selbst umsetzen und einen pull request machen oder
|
||||
* mir per Email (info@martin-thoma.de) schicken.
|
||||
|
||||
Ich werde dann versuchen die Verbesserungsvorschläge zeitnah einzuarbeiten.
|
||||
|
||||
Zeichnungen
|
||||
===========
|
||||
Das erstellen der Zeichnungen ist sehr zeitaufwendig. Das ist der
|
||||
Das erstellen der Zeichnungen ist sehr zeitaufwendig. Das ist der
|
||||
Grund, warum manchmal nur ein "TODO" im Dokument steht.
|
||||
|
||||
Ihr könnt mir gerne Zeichnungen schicken (entweder schön auf Papier
|
||||
Zeichnen und abfotographieren / einscannen oder schon mit Inscape /
|
||||
Gimp / ... oder sogar mit TikZ erstellen).
|
||||
Gimp / ... oder sogar mit TikZ erstellen).
|
||||
|
||||
Akzeptable Formate sind: .jpg, .pdf, .svg, .png, .gif, .tex, .sketch
|
||||
Alles andere kann ich vermutlich nicht einbinden.
|
||||
|
@ -37,8 +37,8 @@ Rechtliches
|
|||
===========
|
||||
Die Autoren kann man über Git ermitteln. Ich schreibe meist nur den
|
||||
Tafelanschrieb der Vorlesung ab; eventuell noch mit ein paar
|
||||
Notizen meinerseits. Wenn mir Verbesserungsvorschläge per Email
|
||||
geschickt werden, ist der Autor sowie das Datum der Email in der
|
||||
Notizen meinerseits. Wenn mir Verbesserungsvorschläge per Email
|
||||
geschickt werden, ist der Autor sowie das Datum der Email in der
|
||||
Commit-Nachricht von Git zu sehen.
|
||||
|
||||
Bilder habe ich entweder selbst erstellt oder von tex.stackexchange.com.
|
||||
|
|
|
@ -6,19 +6,19 @@ der Vorlesung von Prof.~Dr.~Herrlich sowie die Mitschriften einiger
|
|||
Übungen und Tutorien.
|
||||
|
||||
Das Skript ist kostenlos über \href{http://martin-thoma.com/geotopo/}{martin-thoma.com/geotopo}
|
||||
verfügbar. Wer es gerne in A5 (Schwarz-Weiß, Ringbindung) für 10 Euro hätte,
|
||||
verfügbar. Wer es gerne in A5 (Schwarz-Weiß, Ringbindung) für 10 Euro hätte,
|
||||
kann mir eine E-Mail schicken (info@martin-thoma.de).
|
||||
|
||||
\section*{Danksagungen}
|
||||
An dieser Stelle möchte ich Herrn~Prof.~Dr.~Herrlich für einige
|
||||
Korrekturvorschläge und einen gut strukturierten Tafelanschrieb
|
||||
An dieser Stelle möchte ich Herrn~Prof.~Dr.~Herrlich für einige
|
||||
Korrekturvorschläge und einen gut strukturierten Tafelanschrieb
|
||||
danken, der als Vorlage für dieses Skript diente. Tatsächlich basiert
|
||||
die Struktur dieses Skripts auf der Vorlesung von Herrn~Prof.~Dr.~Herrlich
|
||||
und ganze Abschnitte konnten direkt mit \LaTeX{} umgesetzt werden.
|
||||
Vielen Dank für die Erlaubnis, Ihre Inhalte in diesem Skript einbauen
|
||||
zu dürfen!
|
||||
|
||||
Vielen Dank auch an Frau Lenz und Frau Randecker, die es mir erlaubt
|
||||
Vielen Dank auch an Frau Lenz und Frau Randecker, die es mir erlaubt
|
||||
haben, ihre Übungsaufgaben und Lösungen zu benutzen.
|
||||
|
||||
Jérôme Urhausen hat durch viele Verbesserungsvorschläge und Beweise zu einer erheblichen
|
||||
|
@ -70,11 +70,11 @@ der Umgang mit komplexen Zahlen $\mdc$, deren Betrag, Folgen und
|
|||
Häufungspunkten nicht weiter schwer fallen.
|
||||
Diese Vorkenntnisse werden vor allem in \enquote{Analysis I} vermittelt.
|
||||
|
||||
Außerdem wird vorausgesetzt, dass (affine) Vektorräume, Faktorräume,
|
||||
Außerdem wird vorausgesetzt, dass (affine) Vektorräume, Faktorräume,
|
||||
lineare Unabhängigkeit, der Spektralsatz und der projektive Raum $\praum(\mdr)$ aus
|
||||
\enquote{Lineare Algebra I} bekannt sind. In \enquote{Lineare Algebra II}
|
||||
wird der Begriff der Orthonormalbasis eingeführt.
|
||||
|
||||
Obwohl es nicht vorausgesetzt wird, könnte es von Vorteil sein
|
||||
\enquote{Einführung in die Algebra und Zahlentheorie} gehört zu
|
||||
\enquote{Einführung in die Algebra und Zahlentheorie} gehört zu
|
||||
haben.
|
||||
|
|
|
@ -46,10 +46,10 @@
|
|||
%%
|
||||
\usepackage{../shortcuts}
|
||||
|
||||
\hypersetup{
|
||||
pdfauthor = {Martin Thoma},
|
||||
pdfkeywords = {Geometrie und Topologie},
|
||||
pdftitle = {Geometrie und Topologie - Definitionen}
|
||||
\hypersetup{
|
||||
pdfauthor = {Martin Thoma},
|
||||
pdfkeywords = {Geometrie und Topologie},
|
||||
pdftitle = {Geometrie und Topologie - Definitionen}
|
||||
}
|
||||
\allowdisplaybreaks
|
||||
|
||||
|
|
|
@ -38,10 +38,10 @@
|
|||
\usepackage[left=10mm,right=10mm, top=2mm, bottom=10mm]{geometry}
|
||||
\usepackage{../shortcuts}
|
||||
|
||||
\hypersetup{
|
||||
pdfauthor = {Martin Thoma},
|
||||
pdfkeywords = {Geometrie und Topologie},
|
||||
pdftitle = {Geometrie und Topologie - Definitionen}
|
||||
\hypersetup{
|
||||
pdfauthor = {Martin Thoma},
|
||||
pdfkeywords = {Geometrie und Topologie},
|
||||
pdftitle = {Geometrie und Topologie - Definitionen}
|
||||
}
|
||||
\allowdisplaybreaks
|
||||
|
||||
|
|
|
@ -18,15 +18,15 @@
|
|||
tick align=outside,
|
||||
%minor tick num=-3,
|
||||
enlargelimits=true]
|
||||
\addplot[domain=0:12, red, thick,samples=500] {1/3*x^1.5};
|
||||
\addplot[domain=0:12, dotted, orange, thick,samples=500] {1*x^1.5};
|
||||
\addplot[domain=0:12, dashed, blue, thick,samples=500] {2*x^1.5};
|
||||
\addplot[domain=0:12, red, thick,samples=500] {1/3*x^1.5};
|
||||
\addplot[domain=0:12, dotted, orange, thick,samples=500] {1*x^1.5};
|
||||
\addplot[domain=0:12, dashed, blue, thick,samples=500] {2*x^1.5};
|
||||
|
||||
\addplot[domain=0:12, red, thick,samples=500] {-1/3*x^1.5};
|
||||
\addplot[domain=0:12, dotted, orange, thick,samples=500] {-1*x^1.5};
|
||||
\addplot[domain=0:12, dashed, blue, thick,samples=500] {-2*x^1.5};
|
||||
\addplot[domain=0:12, red, thick,samples=500] {-1/3*x^1.5};
|
||||
\addplot[domain=0:12, dotted, orange, thick,samples=500] {-1*x^1.5};
|
||||
\addplot[domain=0:12, dashed, blue, thick,samples=500] {-2*x^1.5};
|
||||
\addlegendentry{$a=\frac{1}{3}$}
|
||||
\addlegendentry{$a=1$}
|
||||
\addlegendentry{$a=2$}
|
||||
\end{axis}
|
||||
\end{axis}
|
||||
\end{tikzpicture}
|
||||
|
|
|
@ -27,5 +27,5 @@
|
|||
}
|
||||
]
|
||||
\addplot3[surf] {y*y-x*x*x};
|
||||
\end{axis}
|
||||
\end{axis}
|
||||
\end{tikzpicture}
|
||||
|
|
|
@ -14,13 +14,13 @@
|
|||
\tkzDefLine[orthogonal=through P,/tikz/overlay](O,X) \tkzGetPoint{helper}
|
||||
\tkzInterLL(O,X)(P,helper) \tkzGetPoint{xp}
|
||||
\draw [decorate,decoration={brace,amplitude=4pt,mirror}]
|
||||
(O) -- (xp) node [black,midway,xshift=0cm, yshift=-0.3cm]
|
||||
(O) -- (xp) node [black,midway,xshift=0cm, yshift=-0.3cm]
|
||||
{\footnotesize $x_P$};
|
||||
|
||||
\tkzDefLine[orthogonal=through P,/tikz/overlay](O,Y) \tkzGetPoint{helper}
|
||||
\tkzInterLL(O,Y)(P,helper) \tkzGetPoint{yp}
|
||||
\draw [decorate,decoration={brace,amplitude=4pt}]
|
||||
(O) -- (yp) node [black,midway,xshift=-0.4cm]
|
||||
(O) -- (yp) node [black,midway,xshift=-0.4cm]
|
||||
{\footnotesize $y_P$};
|
||||
|
||||
\tkzDrawPolygon(O,xp,P,yp)
|
||||
|
|
|
@ -10,13 +10,13 @@
|
|||
\tkzDefLine[orthogonal=through P,/tikz/overlay](O,X) \tkzGetPoint{helper}
|
||||
\tkzInterLL(O,X)(P,helper) \tkzGetPoint{xp}
|
||||
\draw [decorate,decoration={brace,amplitude=4pt,mirror}]
|
||||
(O) -- (xp) node [black,midway,xshift=0cm, yshift=-0.3cm]
|
||||
(O) -- (xp) node [black,midway,xshift=0cm, yshift=-0.3cm]
|
||||
{\footnotesize $x_P$};
|
||||
|
||||
\tkzDefLine[orthogonal=through P,/tikz/overlay](O,Y) \tkzGetPoint{helper}
|
||||
\tkzInterLL(O,Y)(P,helper) \tkzGetPoint{yp}
|
||||
\draw [decorate,decoration={brace,amplitude=4pt}]
|
||||
(O) -- (yp) node [black,midway,xshift=-0.4cm]
|
||||
(O) -- (yp) node [black,midway,xshift=-0.4cm]
|
||||
{\footnotesize $y_P$};
|
||||
|
||||
\tkzDrawPolygon(O,xp,P,yp)
|
||||
|
|
|
@ -18,9 +18,9 @@
|
|||
\path[name path=trpath] (tl) -- (fr);
|
||||
\path[name path=tlpath] (tr) -- (fl);
|
||||
|
||||
\draw[name intersections={of=brpath and rbpath}] (intersection-1)coordinate (br){};
|
||||
\draw[name intersections={of=blpath and lbpath}] (intersection-1)coordinate (bl){};
|
||||
\draw[name intersections={of=trpath and tlpath}] (intersection-1)coordinate (tb){};
|
||||
\draw[name intersections={of=brpath and rbpath}] (intersection-1)coordinate (br){};
|
||||
\draw[name intersections={of=blpath and lbpath}] (intersection-1)coordinate (bl){};
|
||||
\draw[name intersections={of=trpath and tlpath}] (intersection-1)coordinate (tb){};
|
||||
|
||||
\shade[right color=gray!10, left color=black!50, shading angle=105] (tf) -- (bf) -- (bl) -- (tl) -- cycle;
|
||||
\shade[left color=gray!10, right color=black!50, shading angle=75] (tf) -- (bf) -- (br) -- (tr) -- cycle;
|
||||
|
|
|
@ -63,5 +63,5 @@
|
|||
\node at (axis cs:0,3) [anchor=east] {$y$};
|
||||
\node at (axis cs:2,0) [anchor=north] {$x$};
|
||||
|
||||
\end{axis}
|
||||
\end{axis}
|
||||
\end{tikzpicture}
|
||||
|
|
|
@ -43,5 +43,5 @@
|
|||
\node[blue] at (axis cs:0,3) [anchor=east] {$x_2$};
|
||||
\node[blue] at (axis cs:2,0) [anchor=north] {$x_1$};
|
||||
|
||||
\end{axis}
|
||||
\end{axis}
|
||||
\end{tikzpicture}
|
||||
|
|
|
@ -9,7 +9,7 @@
|
|||
},
|
||||
}
|
||||
\begin{tikzpicture}
|
||||
|
||||
|
||||
\draw[->] (-1.5,0) -- (5.5,0) node [below] {$\mathbb{R}$};
|
||||
|
||||
\foreach \x in {-1,...,5}
|
||||
|
|
|
@ -23,5 +23,5 @@
|
|||
% Draw axis text
|
||||
\node at (axis cs:-1,0.5) [anchor=east] {$\mathfrak{B}_r(0) = $};
|
||||
|
||||
\end{axis}
|
||||
\end{axis}
|
||||
\end{tikzpicture}
|
||||
|
|
|
@ -12,10 +12,10 @@
|
|||
\draw[grid] (\x,-0.5) -- (\x,2.5);
|
||||
\draw[grid] (-0.5,\y) -- (2.5,\y);
|
||||
}
|
||||
|
||||
|
||||
|
||||
%draw the axes
|
||||
\draw[axis] (-1,0,0) -- (3,0,0) node[anchor=west]{$y$};
|
||||
\draw[axis] (0,-1,0) -- (0,3,0) node[anchor=west]{$x$};
|
||||
|
||||
|
||||
\end{tikzpicture}
|
||||
|
|
|
@ -1,7 +1,7 @@
|
|||
\begin{tikzpicture}[scale=.5, z={(.707,.3)}]
|
||||
\draw (2,3,2) -- (0,0,0) -- (4,0,0) -- (4,0,4) -- (2,3,2)
|
||||
\draw (2,3,2) -- (0,0,0) -- (4,0,0) -- (4,0,4) -- (2,3,2)
|
||||
-- (4,0,0);
|
||||
\draw[color=gray, style=dashed] (2,3,2) -- (0,0,4)
|
||||
\draw[color=gray, style=dashed] (2,3,2) -- (0,0,4)
|
||||
-- (0,0,0);
|
||||
\draw[color=gray, style=dashed] (0,0,4) -- (4,0,4);
|
||||
\end{tikzpicture}
|
||||
|
|
|
@ -28,14 +28,14 @@
|
|||
enlargelimits=true,
|
||||
tension=0.08]
|
||||
% plot the stirling-formulae
|
||||
\addplot[domain=-4:8, red, thick,samples=500] {0.5*x};
|
||||
\addplot[domain=-2:2, red, thick,samples=500] {2*x};
|
||||
\addplot[domain=-4:4, red, thick,samples=500] {x};
|
||||
\addplot[domain=-4:8, red, thick,samples=500] {-0.5*x};
|
||||
\addplot[domain=-4:8, red, thick,samples=500] {0.5*x};
|
||||
\addplot[domain=-2:2, red, thick,samples=500] {2*x};
|
||||
\addplot[domain=-4:4, red, thick,samples=500] {x};
|
||||
\addplot[domain=-4:8, red, thick,samples=500] {-0.5*x};
|
||||
\addplot[color=red,only marks,mark=o]
|
||||
plot coordinates {
|
||||
(1.5,3)
|
||||
(1.5,1.5)
|
||||
};
|
||||
\end{axis}
|
||||
\end{axis}
|
||||
\end{tikzpicture}
|
||||
|
|
|
@ -6,7 +6,7 @@
|
|||
\node (Rright) at (6,0) {};
|
||||
\draw[dashed,very thick] (Pleft) -- (P);
|
||||
\draw[dotted,very thick] (P) -- (R) -- (Rright);
|
||||
\draw [thick,decoration={brace,mirror,raise=0.2cm},decorate] (Pleft) -- (P) node [pos=0.5,anchor=north,yshift=-0.25cm] {$PR^-$};
|
||||
\draw [thick,decoration={brace,mirror,raise=0.2cm},decorate] (P) -- (R) node [pos=0.5,anchor=north,yshift=-0.25cm] {$\overline{PR}$};
|
||||
\draw [thick,decoration={brace,mirror,raise=0.8cm},decorate] (P) -- (Rright) node [pos=0.5,anchor=north,yshift=-0.85cm] {$PR^+$};
|
||||
\draw [thick,decoration={brace,mirror,raise=0.2cm},decorate] (Pleft) -- (P) node [pos=0.5,anchor=north,yshift=-0.25cm] {$PR^-$};
|
||||
\draw [thick,decoration={brace,mirror,raise=0.2cm},decorate] (P) -- (R) node [pos=0.5,anchor=north,yshift=-0.25cm] {$\overline{PR}$};
|
||||
\draw [thick,decoration={brace,mirror,raise=0.8cm},decorate] (P) -- (Rright) node [pos=0.5,anchor=north,yshift=-0.85cm] {$PR^+$};
|
||||
\end{tikzpicture}
|
||||
|
|
|
@ -29,5 +29,5 @@
|
|||
\node at (axis cs:0.8,1.2) [anchor=-90] {$y$};
|
||||
\draw (axis cs:0.8,1.2) circle[radius=0.6];
|
||||
\addplot[mark=*] coordinates {(0.8,1.2)};
|
||||
\end{axis}
|
||||
\end{axis}
|
||||
\end{tikzpicture}
|
||||
|
|
|
@ -9,7 +9,7 @@
|
|||
},
|
||||
}
|
||||
\begin{tikzpicture}
|
||||
|
||||
|
||||
\draw[->] (-0.5,0) -- (1.5,0) node [below] {$\mathbb{R}$};
|
||||
|
||||
\foreach \x in {0,...,1}
|
||||
|
|
|
@ -63,7 +63,7 @@
|
|||
\addplot[hatchcolor=red,mark=none, pattern=custom north west lines, draw=none] coordinates {(4.5, 0) (4.5,5) (5.5,5) (5.5,0) };
|
||||
\addplot[red,mark=none, thick] coordinates {(4.5, 0) (4.5,5)};
|
||||
\addplot[red,mark=none, thick] coordinates {(5.5, 0) (5.5,5)};
|
||||
|
||||
|
||||
|
||||
\addplot[mark=none, dashed] coordinates {(1, 0) (1,3)};
|
||||
\addplot[mark=none, dashed] coordinates {(5, 0) (5,3)};
|
||||
|
@ -78,5 +78,5 @@
|
|||
|
||||
\node[red] at (axis cs:1,-0.3) [anchor=north] {$U_1 \times X_2$};
|
||||
\node[red] at (axis cs:5,-0.3) [anchor=north] {$U_2 \times X_2$};
|
||||
\end{axis}
|
||||
\end{axis}
|
||||
\end{tikzpicture}
|
||||
|
|
|
@ -23,5 +23,5 @@
|
|||
(0,0) (-1,1) (-2,2) (-1,3) (0, 3) (1, 4)};
|
||||
\addplot[mark=none, blue, smooth cycle, thick, tension=3] coordinates {%
|
||||
(0,0) (-1,1) (-2,2) (-1,3) (0, 3) (1, 4)};
|
||||
\end{axis}
|
||||
\end{axis}
|
||||
\end{tikzpicture}
|
||||
|
|
|
@ -21,5 +21,5 @@
|
|||
\addplot[domain=0:2.5, red, thick,samples=20] {-x+2.5};
|
||||
\node[point,label={[label distance=0cm]45:$e_0$}] at (axis cs:2.5,0) {};
|
||||
\node[point,label={[label distance=0cm]0:$e_1$}] at (axis cs:0,2.5) {};
|
||||
\end{axis}
|
||||
\end{axis}
|
||||
\end{tikzpicture}
|
||||
|
|
|
@ -20,5 +20,5 @@
|
|||
\node (b)[point,label={[label distance=0cm]5:$e_1$}] at (axis cs:0,2.5) {};
|
||||
\node (c)[point,label={[label distance=0cm]0:$e_2$}] at (axis cs:2,2) {};
|
||||
\draw[thick,fill=orange!50] (a.center) -- (b.center) -- (c.center) -- cycle;
|
||||
\end{axis}
|
||||
\end{axis}
|
||||
\end{tikzpicture}
|
||||
|
|
|
@ -21,6 +21,6 @@
|
|||
\addplot[domain=0.0105:0.011, blue, thick,samples=20] {10};
|
||||
\addlegendentry{$\{(x, \sin(\frac{1}{x})) \in X \times Y\}$}
|
||||
\addlegendentry{$(-1,1) \subseteq Y$}
|
||||
\end{axis}
|
||||
\end{axis}
|
||||
\draw[ultra thick,blue] (0,0.5) -- (0,4);
|
||||
\end{tikzpicture}
|
||||
|
|
|
@ -9,7 +9,7 @@
|
|||
\node (e)[point] at (0,2) {};
|
||||
\node (f)[point] at (4,2) {};
|
||||
\end{scope}
|
||||
|
||||
|
||||
\node (p)[point,label={[label distance=0cm]5:$P$}] at (1.5,0.5) {};
|
||||
|
||||
\draw[pattern=north east lines] (a.center) -- (b.center) -- (c.center) -- cycle;
|
||||
|
|
|
@ -9,7 +9,7 @@
|
|||
\node (e)[point] at (0,2) {};
|
||||
\node (f)[point] at (4,2) {};
|
||||
\end{scope}
|
||||
|
||||
|
||||
\node (p)[point,label={[label distance=0cm]5:$P$}] at (1.5,0.5) {};
|
||||
|
||||
\draw[pattern=north east lines] (a.center) -- (p.center) -- (b.center) -- cycle;
|
||||
|
|
|
@ -14,7 +14,7 @@
|
|||
\begin{scope}[decoration={
|
||||
markings,
|
||||
mark=at position 0.6 with {\arrow{>}}}
|
||||
]
|
||||
]
|
||||
\draw[postaction={decorate}] (a.center) -- (b.center);
|
||||
\draw[postaction={decorate}] (d.center) -- (c.center);
|
||||
\end{scope}
|
||||
|
@ -22,7 +22,7 @@
|
|||
\begin{scope}[decoration={
|
||||
markings,
|
||||
mark=at position 0.55 with {\arrow{>>}}}
|
||||
]
|
||||
]
|
||||
\draw[postaction={decorate}] (b.center) -- (c.center);
|
||||
\draw[postaction={decorate}] (a.center) -- (d.center);
|
||||
\end{scope}
|
||||
|
|
|
@ -21,7 +21,7 @@
|
|||
\begin{scope}[decoration={
|
||||
markings,
|
||||
mark=at position 0.6 with {\arrow{>}}}
|
||||
]
|
||||
]
|
||||
\draw[postaction={decorate}] (a.center) -- (b.center);
|
||||
\draw[postaction={decorate}] (d.center) -- (c.center);
|
||||
\end{scope}
|
||||
|
@ -29,7 +29,7 @@
|
|||
\begin{scope}[decoration={
|
||||
markings,
|
||||
mark=at position 0.55 with {\arrow{>>}}}
|
||||
]
|
||||
]
|
||||
\draw[postaction={decorate}] (b.center) -- (c.center);
|
||||
\draw[postaction={decorate}] (a.center) -- (d.center);
|
||||
\end{scope}
|
||||
|
|
|
@ -38,7 +38,7 @@
|
|||
\begin{scope}[decoration={
|
||||
markings,
|
||||
mark=at position 0.6 with {\arrow{>}}}
|
||||
]
|
||||
]
|
||||
\draw[postaction={decorate}] (a.center) -- (b.center);
|
||||
\draw[postaction={decorate}] (d.center) -- (c.center);
|
||||
\end{scope}
|
||||
|
@ -46,7 +46,7 @@
|
|||
\begin{scope}[decoration={
|
||||
markings,
|
||||
mark=at position 0.55 with {\arrow{>>}}}
|
||||
]
|
||||
]
|
||||
\draw[postaction={decorate}] (b.center) -- (c.center);
|
||||
\draw[postaction={decorate}] (a.center) -- (d.center);
|
||||
\end{scope}
|
||||
|
|
|
@ -36,7 +36,7 @@
|
|||
\begin{scope}[decoration={
|
||||
markings,
|
||||
mark=at position 0.6 with {\arrow{>}}}
|
||||
]
|
||||
]
|
||||
\draw[postaction={decorate}] (a.center) -- (b.center);
|
||||
\draw[postaction={decorate}] (d.center) -- (c.center);
|
||||
\end{scope}
|
||||
|
@ -44,7 +44,7 @@
|
|||
\begin{scope}[decoration={
|
||||
markings,
|
||||
mark=at position 0.55 with {\arrow{>>}}}
|
||||
]
|
||||
]
|
||||
\draw[postaction={decorate}] (b.center) -- (c.center);
|
||||
\draw[postaction={decorate}] (a.center) -- (d.center);
|
||||
\end{scope}
|
||||
|
|
|
@ -13,8 +13,8 @@
|
|||
\node at ($(C')+(0,0.4)$) {$C'$};
|
||||
\node at ($(B)+(0.2,-0.2)$) {$B$};
|
||||
\node at ($(C)+(0.28,0.5)$) {$C$};
|
||||
\tkzDrawPolygon[ultra thick,color=blue,fill=blue!20](A,B',C')
|
||||
\tkzDrawPolygon[line width=0.3pt,color=red,fill=red!20](A,B,C)
|
||||
\tkzDrawPolygon[ultra thick,color=blue,fill=blue!20](A,B',C')
|
||||
\tkzDrawPolygon[line width=0.3pt,color=red,fill=red!20](A,B,C)
|
||||
\tkzDrawPoints(A,B',C',B,C)
|
||||
\tkzLabelSegment[below,red](A,B){$c$}
|
||||
\tkzLabelSegment[left,red](A,C){$b$}
|
||||
|
|
|
@ -18,7 +18,7 @@
|
|||
\tkzDrawLines(A,B)
|
||||
\tkzDrawLine[dashed,color=orange,add=0.5 and 0.2](F,P)
|
||||
\tkzDrawLine[dashed,color=blue,add=0.5 and 0.2](G,P)
|
||||
%
|
||||
%
|
||||
\tkzLabelPoint[below left](A){$A$}
|
||||
\tkzLabelPoint[below left](G){$G$}
|
||||
\tkzLabelPoint[above left](P){$P$}
|
||||
|
|
|
@ -18,8 +18,8 @@
|
|||
enlargelimits=true,
|
||||
tension=0.08]
|
||||
% plot the stirling-formulae
|
||||
\addplot[domain=-4:8, red, thick,samples=500] {0.5*x};
|
||||
\addplot[domain=-2:2, red, thick,samples=500] {2*x};
|
||||
\addplot[domain=-4:8, red, thick,samples=500] {-0.5*x};
|
||||
\end{axis}
|
||||
\addplot[domain=-4:8, red, thick,samples=500] {0.5*x};
|
||||
\addplot[domain=-2:2, red, thick,samples=500] {2*x};
|
||||
\addplot[domain=-4:8, red, thick,samples=500] {-0.5*x};
|
||||
\end{axis}
|
||||
\end{tikzpicture}
|
||||
|
|
|
@ -29,5 +29,5 @@
|
|||
\addplot[mark=none] coordinates {(\i,-0.2) (\i,5.2)};
|
||||
}
|
||||
\addplot[mark=none] coordinates {(0,2) (5,2)};
|
||||
\end{axis}
|
||||
\end{axis}
|
||||
\end{tikzpicture}
|
||||
|
|
|
@ -68,7 +68,7 @@ in dem Erstellen dieses Skripts steckt:
|
|||
|07.02.2014 | 11:15 - 11:20 | 5 | Definitionen vereinfacht
|
||||
|07.02.2014 | 11:35 - 11:45 | 10 | Definition "operiert durch Homöomorphismen" korrigiert
|
||||
|07.02.2014 | 15:00 - 15:30 | 30 | Verbesserungsvorschläge von Jérôme Urhausen, Email vom 08.02.2014, umgesetzt.
|
||||
|07.02.2014 | 15:30 - 15:45 | 15 | Verbesserungen
|
||||
|07.02.2014 | 15:30 - 15:45 | 15 | Verbesserungen
|
||||
|07.02.2014 | 19:30 - 21:20 | 110 | Textsetzung, kleine Fehler und Verbesserung eines Bildes
|
||||
|10.02.2014 | 10:30 - 11:05 | 35 | Formulierung in Definitionen vereinfacht; Textsetzung
|
||||
|10.02.2014 | 11:05 - 11:20 | 15 | Verbesserungsvorschläge von Marco, Email 1 vom 10.02.2014, umgesetzt.
|
||||
|
@ -92,6 +92,6 @@ in dem Erstellen dieses Skripts steckt:
|
|||
|20.02.2014 | 12:00 - 13:00 | 60 | Verbesserungsvorschläge von Jonathan (Facebook, 20.02.2014) eingearbeitet.
|
||||
|20.02.2014 | 13:00 - 13:45 | 45 | Verbesserungsvorschläge von Jérôme Urhausen, Email 1 vom 20.02.2014, umgesetzt.
|
||||
|20.02.2014 | 19:30 - 20:15 | 45 | Verbesserungsvorschläge von Jérôme Urhausen, Email 2 vom 20.02.2014, umgesetzt.
|
||||
| Zwischenstand | --- | --- | 6081 Minuten => Über 100 Stunden!
|
||||
| Zwischenstand | --- | --- | 6081 Minuten => Über 100 Stunden!
|
||||
|17.03.2014 | 16:00 - 18:00 | 120 | Textsetzung
|
||||
|19.03.2014 | 08:00 - 10:00 | 120 | Verbesserung des Symbolverzeichnisses
|
|
@ -1,4 +1,4 @@
|
|||
Zu diesem Skript haben einige Leute beigetragen. Die Personen, die am
|
||||
Zu diesem Skript haben einige Leute beigetragen. Die Personen, die am
|
||||
meisten beigetragen haben, stehen direkt im Skript unter "Danksagungen".
|
||||
|
||||
Hier ist eine (hoffentlich bald) vollständige Liste der Mitwirkenden (alphabetisch geordnet) mit
|
||||
|
|
|
@ -4,7 +4,7 @@ bitte ich um eine Email.
|
|||
|
||||
Konventionen
|
||||
============
|
||||
* `\mathbb{N}` sollte vermieden werden. Stattdessen wird
|
||||
* `\mathbb{N}` sollte vermieden werden. Stattdessen wird
|
||||
`\mathbb{N}_0` und `\mathbb{N}_+` verwendet.
|
||||
* `\subset` sollte vermieden werden. Stattdessen wird
|
||||
`\subseteq` bzw. `\subsetneq` verwendet.
|
||||
|
|
|
@ -2,7 +2,7 @@
|
|||
\subsection*{Teilaufgabe a}
|
||||
\textbf{Gegeben:}
|
||||
|
||||
\[A =
|
||||
\[A =
|
||||
\begin{pmatrix}
|
||||
3 & 15 & 13 \\
|
||||
6 & 6 & 6 \\
|
||||
|
@ -11,7 +11,7 @@
|
|||
|
||||
\textbf{Aufgabe:} LR-Zerlegung von $A$ mit Spaltenpivotwahl
|
||||
|
||||
\textbf{Lösung:}
|
||||
\textbf{Lösung:}
|
||||
|
||||
\begin{align*}
|
||||
&
|
||||
|
@ -85,7 +85,7 @@ Nun gilt: $P A = L R = A^{(1)}$ (Kontrolle mit \href{http://www.wolframalpha.com
|
|||
|
||||
\textbf{Gegeben:}
|
||||
|
||||
\[A =
|
||||
\[A =
|
||||
\begin{pmatrix}
|
||||
9 & 4 & 12 \\
|
||||
4 & 1 & 4 \\
|
||||
|
@ -111,7 +111,7 @@ Falls $A$ symmetrisch ist, gilt:
|
|||
|
||||
\begin{align}
|
||||
\det(A_1) &= 9 > 0\\
|
||||
\det(A_2) &=
|
||||
\det(A_2) &=
|
||||
\begin{vmatrix}
|
||||
9 & 4 \\
|
||||
4 & 1 \\
|
||||
|
|
|
@ -7,7 +7,7 @@ wobei $L$ eine invertierbare, untere Dreiecksmatrix ist.
|
|||
|
||||
Geben Sie die Formel zur Berechnung von $y_i$ an.
|
||||
|
||||
\textbf{Lösung:}
|
||||
\textbf{Lösung:}
|
||||
\[y_i = \frac{b_i - \sum_{k=1}^{i-1} l_{ik} \cdot y_k}{l_{ii}}\]
|
||||
|
||||
\begin{algorithm}[H]
|
||||
|
|
|
@ -33,7 +33,7 @@ mit Hilfe der LR Zerlegung nach $\Delta x$ auf.
|
|||
\overbrace{\begin{pmatrix}
|
||||
1 & 0\\
|
||||
\frac{1}{9} & 1
|
||||
\end{pmatrix}}^{=: L} \cdot
|
||||
\end{pmatrix}}^{=: L} \cdot
|
||||
\overbrace{\begin{pmatrix}
|
||||
3 & 1\\
|
||||
0 & \frac{8}{9}
|
||||
|
@ -81,7 +81,7 @@ Anschließend berechnen wir
|
|||
\begin{pmatrix}
|
||||
x_1\\
|
||||
y_1
|
||||
\end{pmatrix} &=
|
||||
\end{pmatrix} &=
|
||||
\begin{pmatrix}
|
||||
x_0\\
|
||||
y_0
|
||||
|
@ -89,7 +89,7 @@ Anschließend berechnen wir
|
|||
\Leftrightarrow\begin{pmatrix}
|
||||
x_1\\
|
||||
y_1
|
||||
\end{pmatrix} &=
|
||||
\end{pmatrix} &=
|
||||
\begin{pmatrix}
|
||||
-\frac{1}{3}\\
|
||||
0
|
||||
|
@ -102,7 +102,7 @@ Anschließend berechnen wir
|
|||
\Leftrightarrow\begin{pmatrix}
|
||||
x_1\\
|
||||
y_1
|
||||
\end{pmatrix} &=
|
||||
\end{pmatrix} &=
|
||||
\begin{pmatrix}
|
||||
-\nicefrac{13}{18}\\
|
||||
-\nicefrac{15}{18}
|
||||
|
@ -124,7 +124,7 @@ also ausführlich:
|
|||
\overbrace{\begin{pmatrix}
|
||||
1 & 0\\
|
||||
l_{12} & 1
|
||||
\end{pmatrix}}^L \cdot
|
||||
\end{pmatrix}}^L \cdot
|
||||
\overbrace{\begin{pmatrix}
|
||||
r_{11} & r_{12}\\
|
||||
0 & r_{22}
|
||||
|
@ -139,7 +139,7 @@ also ausführlich:
|
|||
\begin{pmatrix}
|
||||
1 & 0\\
|
||||
l_{12} & 1
|
||||
\end{pmatrix} \cdot
|
||||
\end{pmatrix} \cdot
|
||||
\begin{pmatrix}
|
||||
3 & \cos y\\
|
||||
0 & r_{22}
|
||||
|
@ -156,7 +156,7 @@ also ausführlich:
|
|||
\begin{pmatrix}
|
||||
1 & 0\\
|
||||
x^2 & 1
|
||||
\end{pmatrix} \cdot
|
||||
\end{pmatrix} \cdot
|
||||
\begin{pmatrix}
|
||||
3 & \cos y\\
|
||||
0 & -x^2 \cdot \cos y + e^y
|
||||
|
|
|
@ -1,5 +1,5 @@
|
|||
\section*{Aufgabe 4}
|
||||
\textbf{Aufgabe}:
|
||||
\textbf{Aufgabe}:
|
||||
|
||||
\[I(f) = \int_a^b f(x) \mathrm{d}x \]
|
||||
|
||||
|
|
|
@ -15,7 +15,7 @@ maximaler Ordnung zu erhalten. Wie hoch ist die Ordnung?
|
|||
|
||||
\paragraph{Lösung}
|
||||
|
||||
Nach VL kann bei Vorgabe von $s$ Knoten auch die Ordnung $s$ durch
|
||||
Nach VL kann bei Vorgabe von $s$ Knoten auch die Ordnung $s$ durch
|
||||
geschickte Wahl der Gewichte erreicht werden. Nach Satz 27 ist diese
|
||||
Wahl eindeutig.
|
||||
Also berechnen wir die Gewichte, um die Ordnung $p=2$ zu sichern.
|
||||
|
@ -56,7 +56,7 @@ LGS lösen können:
|
|||
\begin{align}
|
||||
\begin{pmatrix}
|
||||
c_1^0 & c_2^0\\
|
||||
c_1^1 & c_2^1
|
||||
c_1^1 & c_2^1
|
||||
\end{pmatrix}
|
||||
\cdot x
|
||||
=
|
||||
|
|
|
@ -26,10 +26,10 @@
|
|||
\title{Numerik Klausur 1 - Musterlösung}
|
||||
\makeatletter
|
||||
\AtBeginDocument{
|
||||
\hypersetup{
|
||||
\hypersetup{
|
||||
pdfauthor = {Martin Thoma, Peter, Felix},
|
||||
pdfkeywords = {Numerik, KIT, Klausur},
|
||||
pdftitle = {\@title}
|
||||
pdfkeywords = {Numerik, KIT, Klausur},
|
||||
pdftitle = {\@title}
|
||||
}
|
||||
\pagestyle{fancy}
|
||||
\lhead{\@title}
|
||||
|
|
|
@ -83,5 +83,5 @@ Das Verfahren ist also:
|
|||
|
||||
Alternativ kann man auch in einer angepassten LR-Zerlegung direkt die
|
||||
Anzahl an Zeilenvertauschungen zählen. Dann benötigt man $P$ nicht mehr.
|
||||
Ist die Anzahl der Zeilenvertauschungen ungerade, muss das Produkt
|
||||
Ist die Anzahl der Zeilenvertauschungen ungerade, muss das Produkt
|
||||
der $r_ii$ negiert werden.
|
||||
|
|
|
@ -1,6 +1,6 @@
|
|||
\section*{Aufgabe 2}
|
||||
|
||||
\paragraph{Voraussetzung:}
|
||||
\paragraph{Voraussetzung:}
|
||||
Gegeben sei eine Funktion $F$:
|
||||
\begin{align*}
|
||||
F: \mathbb{R} &\rightarrow [-1, 1]\\
|
||||
|
@ -32,23 +32,23 @@ Sei $0 \leq x < y \leq 1$. Dann folgt:\marginnote{Teil 2: $F$ ist auf $[0,1]$ ei
|
|||
\end{align}
|
||||
|
||||
Da $F|_{[0,1]}$ eine Selbstabbildung und eine Kontraktion ist und
|
||||
offensichtlich $[0,1]$ abgeschlossen ist, greift der
|
||||
offensichtlich $[0,1]$ abgeschlossen ist, greift der
|
||||
Banachsche Fixpunktsatz. Es folgt direkt, dass auch für alle $x \in [0,1]$
|
||||
die Folge $(x)_k$ gegen den einzigen Fixpunkt $x^*$ konvergiert.
|
||||
\end{proof}
|
||||
|
||||
\subsection*{Anmerkung}
|
||||
Um zu zeigen, dass es genau einen Fixpunkt $x^*$ in $(0,1)$ gibt,
|
||||
braucht man den Banachschen Fixpunktsatz nicht. Nur um zu zeigen,
|
||||
Um zu zeigen, dass es genau einen Fixpunkt $x^*$ in $(0,1)$ gibt,
|
||||
braucht man den Banachschen Fixpunktsatz nicht. Nur um zu zeigen,
|
||||
dass die Fixpunktiteration auf für jedes $x \in \mathbb{R}$ gegen
|
||||
diesen Fixpunkt $x^*$ konvergiert, braucht man ihn.
|
||||
|
||||
So kann man die existenz eines Fixpunktes zeigen:
|
||||
|
||||
Offensichtlich ist $F(0) \neq 0$ und $F(1) \neq 1$, also ist der
|
||||
Offensichtlich ist $F(0) \neq 0$ und $F(1) \neq 1$, also ist der
|
||||
Fixpunkt - falls vorhanden - in $(0,1)$. $F$ ist in $(0,1)$ stetig
|
||||
und streng monoton fallend. Da auch $-x$ in $(0,1)$ streng monoton
|
||||
fallend ist, folgt, dass $\cos(x) - x$ in $(0,1)$ streng monoton
|
||||
fallend ist, folgt, dass $\cos(x) - x$ in $(0,1)$ streng monoton
|
||||
fallend ist.
|
||||
|
||||
$x=0 \Rightarrow \cos(x) - x = \cos(0) - 0 = 1$
|
||||
|
|
|
@ -36,11 +36,11 @@ ist. Wenn diese nun auf $N$ Intervalle aufgepflittet wird gilt folgendes:
|
|||
\end{align}
|
||||
|
||||
$\sum_{i=1}^{N-1} f(a + i \cdot h)$ steht für die Grenzknoten
|
||||
(deshalb werden sie doppelt gezählt). Von den Grenzknoten gibt es
|
||||
insgesamt $N-2$ Stück, da die tatsächlichen Integralgrenzen $a$ und $b$
|
||||
(deshalb werden sie doppelt gezählt). Von den Grenzknoten gibt es
|
||||
insgesamt $N-2$ Stück, da die tatsächlichen Integralgrenzen $a$ und $b$
|
||||
nur einmal in die Berechnung mit einfließen.
|
||||
|
||||
$\sum_{l=0}^{N-1} f(a + \frac{1}{2} \cdot h + l \cdot h)$ sind die jeweiligen
|
||||
$\sum_{l=0}^{N-1} f(a + \frac{1}{2} \cdot h + l \cdot h)$ sind die jeweiligen
|
||||
mittleren Knoten der Intervalle. Davon gibt es $N$ Stück.
|
||||
|
||||
\subsection*{Teilaufgabe c)}
|
||||
|
|
|
@ -18,12 +18,12 @@ $c_1 = 0$ nicht die Gauss-Quadratur sein kann (Satz 31), kommt nur Ordnung $p=4$
|
|||
und $p=5$ in Frage.
|
||||
|
||||
In dieser Aufgabe sind nur die symmetrischen QF, also die von Ordnung
|
||||
$p=4$ explizit anzugeben. Für die QF von Ordnung $p=5$ hätte man nur
|
||||
$p=4$ explizit anzugeben. Für die QF von Ordnung $p=5$ hätte man nur
|
||||
die Gewichte in Abhängigkeit der Knoten darstellen müssen und
|
||||
eine Bedinung nur an die Knoten herleiten müssen.
|
||||
|
||||
\subsection*{Ordnung 4}
|
||||
Es gilt $g(x) = c$ für eine Konstante $c$, da $\text{Grad}(g(x))=0$ ist.
|
||||
Es gilt $g(x) = c$ für eine Konstante $c$, da $\text{Grad}(g(x))=0$ ist.
|
||||
Also ist \ref{a3} gleichbedeutend mit:
|
||||
\begin{align}
|
||||
\int_0^1 M(x) \cdot c \mathrm{d}x &= 0 \\
|
||||
|
@ -56,7 +56,7 @@ Aus $c_1 = 0 $ folgt, dass $c_3 = 1$ ist. Außerdem muss $c_2 = \frac{1}{2} $ se
|
|||
Es handelt sich um die aus der Vorlesung bekannte Simpsonregel.
|
||||
|
||||
\subsection*{Ordnung 5}
|
||||
Es gilt $g(x) = ax+c$ für Konstanten $a \neq 0, c$, da $\text{Grad}(g(x))=1$ ist.
|
||||
Es gilt $g(x) = ax+c$ für Konstanten $a \neq 0, c$, da $\text{Grad}(g(x))=1$ ist.
|
||||
Also ist \ref{a3} gleichbedeutend mit:
|
||||
\begin{align}
|
||||
\int_0^1 M(x) \cdot (ax+c) \mathrm{d}x &= 0 \\
|
||||
|
|
|
@ -28,10 +28,10 @@
|
|||
\title{Numerik Klausur 2{} - Musterlösung}
|
||||
\makeatletter
|
||||
\AtBeginDocument{
|
||||
\hypersetup{
|
||||
\hypersetup{
|
||||
pdfauthor = {Felix Benz-Baldas, Martin Thoma, Peter Merkert},
|
||||
pdfkeywords = {Numerik, KIT, Klausur},
|
||||
pdftitle = {\@title}
|
||||
pdfkeywords = {Numerik, KIT, Klausur},
|
||||
pdftitle = {\@title}
|
||||
}
|
||||
\pagestyle{fancy}
|
||||
\lhead{\@title}
|
||||
|
|
|
@ -15,7 +15,7 @@
|
|||
\end{align}
|
||||
|
||||
\item Abgeschlossenheit: $D$ ist offentsichtlich abgeschlossen.
|
||||
\item Kontraktion: \\ %TODO:
|
||||
\item Kontraktion: \\ %TODO:
|
||||
%\textbf{Behauptung:} $F(x)$ ist auf $A$ eine Kontraktion.
|
||||
%\textbf{Beweis:}
|
||||
%z.Z.: $\exists L \in [0, 1): \forall x,y \in A: || F(x) - F(y) || \leq L \cdot || x - y||$
|
||||
|
|
|
@ -14,7 +14,7 @@ Für die ersten 3. Ordnungsbedingungen gilt:
|
|||
\end{align*}
|
||||
|
||||
\subsection*{Teilaufgabe c}
|
||||
Wähle die Simpson-Regel, also $c_1=0, c_2 = \frac{1}{2}, c_3 = 1$ und
|
||||
Wähle die Simpson-Regel, also $c_1=0, c_2 = \frac{1}{2}, c_3 = 1$ und
|
||||
$b_1 = b_3 = \frac{1}{6}$ und $b_2 = \frac{4}{6}$.
|
||||
|
||||
Überprüfe nun Ordnungsbedingungen 1-4 $\Rightarrow$ Simpson-Regel hat Ordnung 4
|
||||
|
|
|
@ -25,10 +25,10 @@
|
|||
\title{Numerik Klausur 3 - Musterlösung}
|
||||
\makeatletter
|
||||
\AtBeginDocument{
|
||||
\hypersetup{
|
||||
\hypersetup{
|
||||
pdfauthor = {Felix Benz-Baldas, Martin Thoma, Peter Merkert},
|
||||
pdfkeywords = {Numerik, KIT, Klausur},
|
||||
pdftitle = {\@title}
|
||||
pdfkeywords = {Numerik, KIT, Klausur},
|
||||
pdftitle = {\@title}
|
||||
}
|
||||
\pagestyle{fancy}
|
||||
\lhead{\@title}
|
||||
|
|
|
@ -1,7 +1,7 @@
|
|||
\section*{Aufgabe 1}
|
||||
\textbf{Gegeben:}
|
||||
|
||||
\[A =
|
||||
\[A =
|
||||
\begin{pmatrix}
|
||||
6 & -6 & 0 \\
|
||||
-3 & 7 & 2 \\
|
||||
|
@ -15,7 +15,7 @@ b =\begin{pmatrix}
|
|||
|
||||
\textbf{Aufgabe:} $Ax = b$ mit Gaußelimination und Spaltenpivotwahl lösen
|
||||
|
||||
\textbf{Lösung:}
|
||||
\textbf{Lösung:}
|
||||
|
||||
\begin{align}
|
||||
\begin{gmatrix}[p]
|
||||
|
|
|
@ -25,10 +25,10 @@
|
|||
\title{Numerik Klausur 4 - Musterlösung}
|
||||
\makeatletter
|
||||
\AtBeginDocument{
|
||||
\hypersetup{
|
||||
\hypersetup{
|
||||
pdfauthor = {Martin Thoma, Peter, Felix},
|
||||
pdfkeywords = {Numerik, KIT, Klausur},
|
||||
pdftitle = {\@title}
|
||||
pdfkeywords = {Numerik, KIT, Klausur},
|
||||
pdftitle = {\@title}
|
||||
}
|
||||
\pagestyle{fancy}
|
||||
\lhead{\@title}
|
||||
|
|
|
@ -25,7 +25,7 @@ Für alle tridiagonalen Matrizen gilt:
|
|||
|
||||
Offensichtlich ändert diese Operation nur Zeile 2. $a_{21}$ wird zu 0,
|
||||
$a_{22}$ ändert sich irgendwie, alles andere bleibt unverändert.
|
||||
Die gesammte Matrix ist keine tridiagonale Matrix mehr, aber die
|
||||
Die gesammte Matrix ist keine tridiagonale Matrix mehr, aber die
|
||||
um Submatrix in $R^{(n-1) \times (n-1)}$ ist noch eine.
|
||||
|
||||
Muss man zuvor Zeile 1 und 2 tauschen (andere Zeilen kommen nicht in
|
||||
|
@ -35,7 +35,7 @@ an der tridiagonalen Struktur der Submatrix.
|
|||
|
||||
\paragraph{Teil 2: (ii) für $A \in \mathbb{R}^{2 \times 2}$}
|
||||
Sei $\begin{pmatrix}a_{11} & a_{12}\\a_{21} & a_{22} \end{pmatrix} \in \mathbb{R}^{2 \times 2}$
|
||||
beliebig.
|
||||
beliebig.
|
||||
|
||||
O.B.d.A sei die Spaltenpivotwahl bereits durchgeführt, also $|a_{11}| \geq |a_{21}|$.
|
||||
|
||||
|
@ -55,7 +55,7 @@ Nun folgt:
|
|||
\end{gmatrix}
|
||||
\end{align}
|
||||
|
||||
Wegen $|a_{11}| \geq |a_{21}|$ gilt:
|
||||
Wegen $|a_{11}| \geq |a_{21}|$ gilt:
|
||||
\begin{align}
|
||||
\|\frac{a_{21}}{a_{11}}\| \leq 1
|
||||
\end{align}
|
||||
|
@ -71,7 +71,7 @@ Damit ist Aussage (ii) für $A \in \mathbb{R}^{2 \times 2}$ gezeigt.
|
|||
\paragraph{Teil 3: (ii) für allgemeinen Fall}
|
||||
|
||||
Aus Teil 2 folgt die Aussage auch direkt für größere Matrizen.
|
||||
Der worst case ist, wenn man beim Addieren einer Zeile auf eine
|
||||
Der worst case ist, wenn man beim Addieren einer Zeile auf eine
|
||||
andere mit $\max_{i,j}|a_{ij}|$ multiplizieren muss um das erste nicht-0-Element
|
||||
der Zeile zu entfernen und das zweite auch $\max_{i,j}|a_{ij}|$ ist.
|
||||
Dann muss man aber im nächsten schritt mit einem Faktor $\leq \frac{1}{2}$
|
||||
|
|
|
@ -4,14 +4,14 @@ Bestimme alle Quadraturformeln mit $s=3$ und Knoten
|
|||
$0 = c_1 < c_2, c_3$ und Ordnung $p \geq 4$.
|
||||
|
||||
Schreiben Sie ein Programm in Pseudocode, welches zu vorgegebenem
|
||||
$c_2$ den Knoten $c_3$ und die Gewichte $b_i$ möglichst effizient
|
||||
$c_2$ den Knoten $c_3$ und die Gewichte $b_i$ möglichst effizient
|
||||
berechnet.
|
||||
|
||||
Wie viele symmetrische Quadraturformeln gibt es mit diesen Eigneschaften?
|
||||
|
||||
\subsection*{Lösung}
|
||||
Da $c_1 = 0$ kann es keine Gauß-Quadraturformel sein. Daher kann
|
||||
die Ordnung nicht $2 \cdot s = 6$ sein. Interessant sind also
|
||||
Da $c_1 = 0$ kann es keine Gauß-Quadraturformel sein. Daher kann
|
||||
die Ordnung nicht $2 \cdot s = 6$ sein. Interessant sind also
|
||||
\begin{itemize}
|
||||
\item[(A)] Symmetrische Quadraturformeln der Ordnung 4
|
||||
\item[(B)] Unsymmetrische Quadraturformeln der Ordnung 4
|
||||
|
|
|
@ -26,10 +26,10 @@
|
|||
\title{Numerik Klausur 5 - Musterlösung}
|
||||
\makeatletter
|
||||
\AtBeginDocument{
|
||||
\hypersetup{
|
||||
\hypersetup{
|
||||
pdfauthor = {Martin Thoma, Peter, Felix},
|
||||
pdfkeywords = {Numerik, KIT, Klausur},
|
||||
pdftitle = {\@title}
|
||||
pdfkeywords = {Numerik, KIT, Klausur},
|
||||
pdftitle = {\@title}
|
||||
}
|
||||
\pagestyle{fancy}
|
||||
\lhead{\@title}
|
||||
|
|
|
@ -34,10 +34,10 @@ und diese liegt in $[0,1]$.
|
|||
\Leftrightarrow x &= - \ln(2x) = F_2(x)\label{a2iif2}
|
||||
\end{align}
|
||||
|
||||
Gleichung \ref{a2iif1} zeigt, dass der Fixpunkt von $F_1$ mit der
|
||||
Gleichung \ref{a2iif1} zeigt, dass der Fixpunkt von $F_1$ mit der
|
||||
Nullstelle von $f$ übereinstimmt.
|
||||
|
||||
Gleichung \ref{a2iif2} zeigt, dass der Fixpunkt von $F_1$ mit der
|
||||
Gleichung \ref{a2iif2} zeigt, dass der Fixpunkt von $F_1$ mit der
|
||||
Nullstelle von $f$ übereinstimmt. Da es nur in $[0,1]$ eine Nullstelle
|
||||
gibt (vgl. Teilaufgabe i), ist die Einschränkung von $x$ auf $\mathbb{R}^+$
|
||||
irrelevant.
|
||||
|
@ -49,7 +49,7 @@ Rechenungenauigkeit)
|
|||
|
||||
$F_1$ ist auf $[0,1]$ eine Kontraktion mit Kontraktionszahl $\theta = \frac{1}{2}$:
|
||||
|
||||
Nach dem Mittelwertsatz der Differentialrechnung ex. ein $\xi \in (a,b)$ mit $ 0 \leq a < b \leq 1$, sodass
|
||||
Nach dem Mittelwertsatz der Differentialrechnung ex. ein $\xi \in (a,b)$ mit $ 0 \leq a < b \leq 1$, sodass
|
||||
gilt:
|
||||
|
||||
|
||||
|
|
|
@ -28,10 +28,10 @@
|
|||
\title{Numerik Klausur 6 - Musterlösung}
|
||||
\makeatletter
|
||||
\AtBeginDocument{
|
||||
\hypersetup{
|
||||
\hypersetup{
|
||||
pdfauthor = {Martin Thoma, Peter, Felix},
|
||||
pdfkeywords = {Numerik, KIT, Klausur},
|
||||
pdftitle = {\@title}
|
||||
pdfkeywords = {Numerik, KIT, Klausur},
|
||||
pdftitle = {\@title}
|
||||
}
|
||||
\pagestyle{fancy}
|
||||
\lhead{\@title}
|
||||
|
|
|
@ -1,7 +1,7 @@
|
|||
Diese Lösungen sind noch im Aufbau.
|
||||
|
||||
Wenn du einen Fehler findest (auch Textsetzungs- und Rechtschreibfehler
|
||||
oder missverständliche Stellen)
|
||||
oder missverständliche Stellen)
|
||||
oder selbst eine Lösung geschrieben hast, kannst du mir eine Email
|
||||
schreiben (info@martin-thoma.de). Oder du machst direkt einen Pull-Request.
|
||||
|
||||
|
|
|
@ -53,7 +53,7 @@ sie auf jeden Fall für $a=1, b=0$ sowie für $a=1, b=1$ gelten. Aber:
|
|||
\frac{2\cdot1+5\cdot0}{5\cdot1+10\cdot0} = \frac{3}{5} &\neq \frac{8}{15} = \frac{3\cdot1+5\cdot1}{5\cdot1+10\cdot1}
|
||||
\end{align}
|
||||
|
||||
Offensichtlich gibt also es kein $c_2$, dass diese Bedingung für jedes $a,b \in \mathbb{R}$
|
||||
Offensichtlich gibt also es kein $c_2$, dass diese Bedingung für jedes $a,b \in \mathbb{R}$
|
||||
erfüllt. Daher kann es keine Quadraturformel der Ordnung $5$ mit den Knoten
|
||||
$0$ und $1$ geben.
|
||||
|
||||
|
@ -83,7 +83,7 @@ Und damit:
|
|||
\end{align}
|
||||
|
||||
Nun könnte man das ganze in die 4. Ordnungsbedinung einsetzen \dots aber ich
|
||||
glaube nicht, dass das schön wird. Mache das, wer will.
|
||||
glaube nicht, dass das schön wird. Mache das, wer will.
|
||||
|
||||
\subsubsection*{Ordnung 4}
|
||||
Die Simpson-Regel erfüllt offensichtlich alle Bedinungen und hat
|
||||
|
@ -97,4 +97,4 @@ Ordnung 4:
|
|||
\end{align}
|
||||
|
||||
Dass die Simpson-Regel Ordnung 4 hat, lässt sich schnell über
|
||||
die Ordnungsbedingungen zeigen.
|
||||
die Ordnungsbedingungen zeigen.
|
||||
|
|
Some files were not shown because too many files have changed in this diff Show more
Loading…
Add table
Reference in a new issue